+ All Categories
Home > Documents > lf1.czlf1.cz/wp-content/uploads/228-vypracovane_otazky_interna... · Web viewprognoza – 50 %...

lf1.czlf1.cz/wp-content/uploads/228-vypracovane_otazky_interna... · Web viewprognoza – 50 %...

Date post: 20-Jan-2020
Category:
Upload: others
View: 4 times
Download: 0 times
Share this document with a friend
220
Vypracované otázky na INTERNU made by Edita Homolková, Monika Ambroziová, Adam Král, Petr Vermach a Vítek Křehnáč Otázky z interny pro zkoušku v 5. ročníku 1 a Synkopa, kolaps b Hypovitaminózy 2 a Respirační insuficience b Reaktivní artritida a psoriatická artritida 3 a Kardiomyopatie b Adrenokortikální insuficience; zásady léčby hypokortikálních stavů 4 a Portální hypertenze; krvácení z jícnových varixů b Dnavá artritida 5 a Ischemická choroba srdeční - patogeneze, klinické formy b Autoimunní trombocytopenie 6 a Akutní infarkt myokardu s ST elevacemi b Febrilní neutropenie 7 a Chronické srdeční selhání b Ankylozující spondylartritida 8 a Fibrilace síní, její komplikace a léčba b Hypertyreóza 9 a Mitrální stenóza a mitrální regurgitace b Akutní komplikace diabetu 10 a Jaterní cirhóza b Léčba zánětlivých revmatických onemocnění; biologická léčba 11 a Chronická ischemická choroba srdeční (ICHS) b Monoklonální gamapatie a chorobné stavy spojené s MG; amyloidóza 12 a Bronchogenní karcinom b Dřeňové útlumy 13 a Plicní embolie b Poruchy metabolismu draslíku 14 a Zánětlivá onemocnění plic - pneumonie, plicní absces b Anemie hemolytické, vrozené 15 a Ischemická choroba dolních končetin b Hemaferezy v klinické hematologii a transfuzním lékařství 16 a Nestabilní angina pectoris, akutní infarkt myokardu bez ST elevací b Primární a sekundární imunodeficience 17 a Nespecifické střevní záněty b Principy podpůrné léčby v hematologii 18 a Astma bronchiale b Krvácivé stavy z destičkových příčin; trombocytopatie, trombocytopenie (mimo ITP) 19 a Ikterus b Rychle progredující glomerulonefritidy 20 a Aortální stenóza a aortální regurgitace b Infekční artritidy 21 a Onemocnění hlubokého žilního systému; trombóza hlubokých žil b Onemocnění kůry nadledvin 1
Transcript
Page 1: lf1.czlf1.cz/wp-content/uploads/228-vypracovane_otazky_interna... · Web viewprognoza – 50 % dosáhne kompletního uzdravení do 6 měsíců, 20 % má prodloužený průběh –

Vypracované otázky na INTERNU made by Edita Homolková, Monika Ambroziová, Adam Král, Petr Vermach a Vítek Křehnáč

Otázky z interny pro zkoušku v 5. ročníku1 a Synkopa, kolapsb Hypovitaminózy2 a Respirační insuficienceb Reaktivní artritida a psoriatická artritida3 a Kardiomyopatieb Adrenokortikální insuficience; zásady léčby hypokortikálních stavů4 a Portální hypertenze; krvácení z jícnových varixůb Dnavá artritida5 a Ischemická choroba srdeční - patogeneze, klinické formyb Autoimunní trombocytopenie6 a Akutní infarkt myokardu s ST elevacemib Febrilní neutropenie7 a Chronické srdeční selháníb Ankylozující spondylartritida8 a Fibrilace síní, její komplikace a léčbab Hypertyreóza9 a Mitrální stenóza a mitrální regurgitaceb Akutní komplikace diabetu10 a Jaterní cirhózab Léčba zánětlivých revmatických onemocnění; biologická léčba11 a Chronická ischemická choroba srdeční (ICHS)b Monoklonální gamapatie a chorobné stavy spojené s MG; amyloidóza12 a Bronchogenní karcinomb Dřeňové útlumy13 a Plicní embolieb Poruchy metabolismu draslíku14 a Zánětlivá onemocnění plic - pneumonie, plicní abscesb Anemie hemolytické, vrozené15 a Ischemická choroba dolních končetinb Hemaferezy v klinické hematologii a transfuzním lékařství16 a Nestabilní angina pectoris, akutní infarkt myokardu bez ST elevacíb Primární a sekundární imunodeficience17 a Nespecifické střevní zánětyb Principy podpůrné léčby v hematologii18 a Astma bronchialeb Krvácivé stavy z destičkových příčin; trombocytopatie, trombocytopenie (mimo ITP)19 a Ikterusb Rychle progredující glomerulonefritidy20 a Aortální stenóza a aortální regurgitaceb Infekční artritidy21 a Onemocnění hlubokého žilního systému; trombóza hlubokých žilb Onemocnění kůry nadledvin22 a Rizikové faktory aterosklerózy; posouzení celkového kardiovaskulárního rizikab Nehodgkinské maligní lymfomy23 a Hematemeza, meléna, enteroragieb Metody náhrady funkce ledvin24 a Virové hepatitidyb Onemocnění příštítných tělísek25 a Arteriální hypertenzeb Glomerulonefritidy26 a Tuberkulóza plicb Systémová sklerodermie, polymyozitida, dermatomyozitida

1

Page 2: lf1.czlf1.cz/wp-content/uploads/228-vypracovane_otazky_interna... · Web viewprognoza – 50 % dosáhne kompletního uzdravení do 6 měsíců, 20 % má prodloužený průběh –

Vypracované otázky na INTERNU made by Edita Homolková, Monika Ambroziová, Adam Král, Petr Vermach a Vítek Křehnáč

27 a Cholelitiáza a chronická cholecystitidab Vrozené nemoci ledvin28 a Vředová choroba žaludku a duodenab Sjögrenův syndrom29 a Chronická obstrukční plicní nemocb Sekundární glomerulopatie30 a Akutní a chronická pankreatitidab Hodgkinova choroba31 a Kolorektální karcinomb Anemie hemolytické, získané32 a Synkopa, kolapsb Ledviny a hypertenze33 a Respirační insuficienceb Tubulointersticiální nefritidy34 a Kardiomyopatieb Metabolický syndrom35 a Portální hypertenze; krvácení z jícnových varixůb Syndrom diseminované intravaskulární koagulace36 a Ischemická choroba srdeční - patogeneze, klinické formyb Myeloproliferativní choroby (mimo CML)37 a Akutní infarkt myokardu s ST elevacemib Poruchy fosfokalciového metabolismu38 a Chronické srdeční selháníb Principy účelné hemoterapie; transfuzní přípravky a krevní deriváty. Předpoklady darování krve, vyšetření

dárce39 a Fibrilace síní, její komplikace a léčbab Struma, záněty štítné žlázy, nádory štítné žlázy40 a Mitrální stenóza a mitrální regurgitaceb Diabetes mellitus 1. typu41 a Jaterní cirhózab Onemocnění gonád; polyglandulární syndromy (syndromy mnohočetné endokrinní neoplazie, autoimunitní

polyglandulární syndromy)42 a Chronická ischemická choroba srdeční (ICHS)b Hypotalamo-hypofyzární systém a jeho onemocnění; diabetes insipidus centralis43 a Bronchogenní karcinomb Urolitiáza44 a Plicní embolieb Akutní leukémie45 a Zánětlivá onemocnění plic - pneumonie, plicní abscesb Umělá výživa46 a Ischemická choroba dolních končetinb Alergická onemocnění; lékové alergie47 a Nestabilní angina pectoris, akutní infarkt myokardu bez ST elevacíb Onemocnění adenohypofýzy48 a Nespecifické střevní zánětyb Mimokloubní revmatismus49 a Dyslipidémieb Onemocnění pleury50 a Diabetes mellitus 2. typub Myokarditidy51 a Trombofilní stavyb Onemocnění perikardu; srdeční tamponáda52 a Metabolické osteopatie. Pagetova chorobab Bradyarytmie a raménkové blokády53 a Krvácivé stavy z plazmatických příčin

2

Page 3: lf1.czlf1.cz/wp-content/uploads/228-vypracovane_otazky_interna... · Web viewprognoza – 50 % dosáhne kompletního uzdravení do 6 měsíců, 20 % má prodloužený průběh –

Vypracované otázky na INTERNU made by Edita Homolková, Monika Ambroziová, Adam Král, Petr Vermach a Vítek Křehnáč

b Tumory jater a podjaterní krajiny54 a Poruchy metabolismu sodíku a vodyb Nemoci jícnu55 a Chronická lymfatická leukémie a příbuzné chorobné stavyb Onemocnění mediastina56 a Akutní selhání ledvinb Šok57 a Chronické komplikace diabetub Urgentní stavy v pneumologii58 a Systémové vaskulitidyb Varixy dolních končetnin; chronická žilní insuficience59 a Poruchy acidobazické rovnováhyb Fibrotizující plicní procesy60 a Plazmocytomb Komplikace infarktu myokardu61 a Osteoporózab Akutní záněty žlučníku a žlučových cest62 a Nádory ledvinb Hemoptýza63 a Infekce močových cest a akutní pyelonefritidab Metabolické nemoci jater (nealkoholická steatohepatitida, porfyrie, Wilsonova choroba, hemochromatóza)64 a Osteoartróza a spondylartrózab Infekční endokarditida65 a Chronická myeloidní leukémieb Zácpa a průjem66 a Revmatoidní artritida a juvenilní idiopatická artritidab Nemoci konečníku a anu67 a Onemocnění štítné žlázyb Onemocnění povrchového žilního systému68 a Obezitab Pneumotorax69 a Ledviny a diabetesb Jaterní selhání70 a Chronická renální insuficienceb Polypy trávicího traktu; hereditární polypózní syndromy71 a Systémový lupus erytematodesb Chronická bronchitida; bronchiektázie72 a Nefrotický syndromb Divertikly trávicího traktu, divertikulární choroba rektosigmatu73 a Malnutriceb Sekundární arteriální hypertenze74 a Myelodysplatický syndromb Nádory trávicího traktu včetně endokrinních75 a Anémie sideropenická; anémie chronických onemocněníb Nádory žaludku a duodena76 a Anemie megaloblastové, perniciozní anemieb Akutní zánětlivá onemocnění dolních dýchacích cest77 a Cushingův syndromb Karcinom slinivky břišní78 a Hypotyreózab Horní a dolní funkční dyspeptický syndrom79 a Poruchy acidobazické rovnováhyb Supraventrikulární arytmie80 a Infekce močových cest a akutní pyelonefritidab Akutní srdeční selhání; plicní edém

3

Page 4: lf1.czlf1.cz/wp-content/uploads/228-vypracovane_otazky_interna... · Web viewprognoza – 50 % dosáhne kompletního uzdravení do 6 měsíců, 20 % má prodloužený průběh –

Vypracované otázky na INTERNU made by Edita Homolková, Monika Ambroziová, Adam Král, Petr Vermach a Vítek Křehnáč

81 a Dyslipidémieb Akutní a chronická gastritida a gastropatie82 a Trombofilní stavyb Nauzea, zvracení, singultus83 a Obezitab Onemocnění chlopní pravého srdce84 a Revmatoidní artritida a juvenilní idiopatická artritidab Plicní hypertenze; cor pulmonale85 a Osteoartróza a spondylartrózab Akutní formy tepenné ischemie86 a Chronická renální insuficienceb Malabsorpční syndrom87 a Plazmocytomb Onemocnění mozkových cév; cévní mozkové příhody88 a Diabetes mellitus 2. typub Komorové arytmie89 a Anémie sideropenická; anémie chronických onemocněníb Granulomatózní plicní procesy90 a Poruchy metabolismu sodíku a vodyb Oběhová zástava a kardiopulmonální resuscitace91 a Hypotyreózab Aneuryzma a disekce aorty92 a Ledviny a diabetesb Léčba arteriální hypertenze - základní třídy antihypertenzív, hlavní vlastnosti93 a Cushingův syndromb Léčba chronické ICHS94 a Anémie sideropenická; anémie chronických onemocněníb Léčba akutních koronárních syndromů95 a Chronická myeloidní leukémieb Léčba chronického srdečního selhání96 a Onemocnění štítné žlázyb Principy léčby arytmií97 a Astma bronchialeb Léčba diabetu

1.A SYNKOPA, KOLAPS

Synkopa- krátkodobá ztráta vědomí způsobená globální hypoperfuzí mozku se ztrátou posturálního tonu rychlým

nástupem, s prodromy či bez nich a rychlou spontánní úpravou (typicky do 20 vteřin až několik minut)- pokud vzniká na základě strukturálního onemocnění srdce, znamená 2násobné zvýšení mortality- presynkopální stav – nástup prodromů bez ztráty vědomíKO – prodromy: slabost, závrať, nauzea, palpitace, pocení

- může být doprovázeno pomočením, ktřečovýmí záškuby svalů, pád, retrográdní amnézieReflexní – vazovagální – 40 %, hl. u mladších lidí, reflexní aktivace vagu a inhibice sympatiku s následou

vazodilatacíDg - na nákladě prodromů slabosti, vertiga a nauzey při delším stání/ fyzické a psychické

zátěži/ po jídle a vyloučení jiných příčin- vyhýbat se spouštěcím mechanismům, hydratace

- situační – neadekvátní reakce autonomní neuroregulace, i zhoršené plnění LK při ↑ tlaku v plicním řečišti

- syn. dráždivého karotického sinu – vede k bradykardii vagovou reakcí, stačí límeček/ poloha hlavyDg - masáž karotického sinu

Ortostatická – 10-15 %, po postavení (za několik vteřin), nedostatečná vazokonstrikce

4

Page 5: lf1.czlf1.cz/wp-content/uploads/228-vypracovane_otazky_interna... · Web viewprognoza – 50 % dosáhne kompletního uzdravení do 6 měsíců, 20 % má prodloužený průběh –

Vypracované otázky na INTERNU made by Edita Homolková, Monika Ambroziová, Adam Král, Petr Vermach a Vítek Křehnáč

- ve vyšším věku, antihypertenziva, alkohol, po fyzické zátěži a jídle, dehydrataceDg – posturální hypotenze doprovázená slabostí = ↓ systol TK >20 mm Hg a diastolického >10 mm

Hg, nebo ↓ systolického TK <90 mm HgKardiální, kardiovaskulární – 10 %, náhlé ↓ srdečního výdeje/ relativní insuficience srdečního výdeje při zátěži

- podezření při vzniku synkopy vleýe/ přo zátěži, je-li s palpitacemi/ bolestí na hrudi/ v přítomnosti strukturálního postižení srdce, rodinná anamnéza náhlé smrti

- arytmie, stenózy chlopní, hypertrofická obstrukční kardiomyopatie, plicní embolizace a hypertenze, srdeční tamponáda, disekce aorty

Dg - pečlivé vyš. vylučující závažné oněm. Anamnéza (nástup záchvatu, opakování, okolnosti předcházející záchvatu (alkohol, léky, močeni), průvodní symptomy (nauzea, zvracení, dušnost), potenciální predispozice (DM, poranění páteře, ztráta krve...)

- hodnota TK v leže a po rychlém postavení, hodnota TK na obou HK a DK (obstrukční onem. tepen), šelest na a.carotis, a.subclavia, a.temporalis (stenóza), cyanóza, paličkovité prsty, vrozené srdeční vady

- hodnota tepové frekvence a TK při masáži karotického sinu- elektrolyty a dehydratace, glykémie, hematokrit a Hb- EKG – arytmie, IM, myokarditida…- echokardiografie- Holter –při podezření na arytmie- test na nakloněné rovině – při často recidivujících synkopách nejasné etio, ověření vazovagální příčiny- neurologické vyšetření, USG karotid 5) tenze arteriálních plynů 6) průkaz léků v moči

Th - léčba dle vyvolávající etiologie- vazovagální synkopa - režimová opatření (vyhýbání se provokujícím faktorům, při prodromech položit na

záda a elevovat končetiny)- implantace kardiostimulátoru - syndrom karotického sinu, rekurentní synkopy (>5/ rok), synkopa se

zraněním + dokumentace kardioinhibiční reakce při tilt testuKolaps

- mdloba, provázen pádem a ztrátou vědomí/ stavem slabosti

1.B HYPOVITAMINÓZY

Vit. A (karotenoidy) – získáváme z potravy živočiš. původu (játra, žloutky, máslo, mléko) nebo se tvoří z provitaminu β-karotenu, který je v zelenině a ovoci

- antioxidační působení – prevence tumorů (ca plic)- Fce: tvorba a fce očního barviva, správná fce a diferenciace epiteliál. bb. (uplatňuje se v prevenci

promoční fáze karcinogenního mechanismu), podpora imunit. reakcí, antioxidač. efekt- Nedostatek: xeroftalmie, šeroslepost, xeroza spojivek, keratomalacie, změny na kůži- Nadbytek: hepatotoxicita, teratogeneze, pruritus, ataxie, bolesti, suchost sliznic- DDD: 1 mg

Vit. D (kalciferol) – sk. steroidních látek, z nichž ergokalciferol (D2) a cholekalciferol (D3) mají antirachitické účinky- syntéza regulovaná zpětnovazebně a PTH- Fce: metabolismus ukládání Ca a P, podporuje růst, zvyšuje resorpci Ca a snižuje jeho vyluč.

stolicí- Nedost: rachitis a osteomalacie- Nadb: vyplavování Ca z kostí a jeho ukládání v ledvinách, srdci a cévách, poruchy GITu- DDD: do 10 g- Zdroj: rybí tuk, játra, mořs. ryby, žloutek, máslo, v organismu se tvoří z provitaminu fotoaktivací

slunečním světlemVit. E (tokoferol) – Fce: antioxidační, chrání buněč. membr. před oxidačním pošk. v důsledku peroxidace lipidů,

spolu s vit. C blokuje nitrosační reakci a tedy endogenní vznik nitrosaminů, synergické působení se selenem

- Nedost: nejsou známy - DDD: 15 mg - Zdroj: obilné klíčky, slunečnicový či řepkový olej, ovesné vločky, semena, ořechy, žloutky

5

Page 6: lf1.czlf1.cz/wp-content/uploads/228-vypracovane_otazky_interna... · Web viewprognoza – 50 % dosáhne kompletního uzdravení do 6 měsíců, 20 % má prodloužený průběh –

Vypracované otázky na INTERNU made by Edita Homolková, Monika Ambroziová, Adam Král, Petr Vermach a Vítek Křehnáč

Vit. K – jde o deriváty naftochinonu, syntezovány jsou hl. v zelených rostlinách vlivem světla, v organismu působením střevní flóry

- Fce: antihemoragická a hemokoagulační (podílí se na tvorbě protrombinu v játrech)- Nedost: hemoragie u novorozenců, sklon ke krvácení a prodlouž. protrombin. času. K jeho nedostatku

dochází spíš při poruchách jeho resorpce (nedostatek žluče) či ↓ syntézou ve střevě v důsl. potlačení střevní mikroflory ATB

- DDD: 0,5-1 mg- Zdroj: zelené rostl., luštěniny, sýry, játra, činnost střev. flóry

Vitamíny Skupiny B - tento komplex zahrnuje více než 20 chem. heterogenních složek s významným podílem na metab. procesech v organismu

Thiamin – B1 - fce: jako koenzym karboxyláz je důležitý pro mtb glukosy a energet. zásobení nerv. a sval. bb. - Nedost: beri – beri ( vlhká forma s otoky a srdeční insuficiencí anebo suchá neuropatická),

vyčerpanost,anorexie- alkoholici, sepse, dialyzovaní, kriticky nemocní- DDD: 1-1,4 mg- Zdroj: kvasnice, hrách, sojová mouka, obilné klíčky, vnitřnosti, vepřové maso

Riboflavin – B2 - fce: je součást flavinových enzymů zprostředkujících oxidoredukč. děje, podílí se na detoxik.ačních procesech

- Nedost: změny na kůži a sliznicích, ragády ústních koutků, dermatitida, neuropsychické změny-DDD: 1,6 – 2 mg- Zdroj: droždí, obilné klíčky, játra, ledviny, vejce, ořechy, ryby, mléko

Niacin – kys. nikotinová, nikotinamid – B3 - fce: součást koenzymů dehydrogenáz účastnících se na přenosu vodíku v organismu (NAD, NADP, NADPH), je označován také jako PP faktor (antipelagrický)

- Nedost: pelagra (z italštiny: pelle- kůže, agra– hrubá) – nemoc 3d = dermatitis, diarrhoe, demence- kys. nikotinová je zčásti syntetizována z tryptofanu, proto se příznaky z nedostatku (kožní změny, GIT

obtíže, neurolog.příznaky) objevovaly hl. při nedostatku tryptofanu v potravě, tj. při převaze kukuřičné stravy

- DDD: 15-20 mg- Zdroj: většina rostl. a živočišných potravin, hl. maso, ryby, kvasnice, obiloviny, luštěniny

Kys. pantotenová – B5: - fce: jako součást koenzymu A se podílí na řadě metabol. fcí. - Nedost: atrofie vlasových folikulů, ztráta pigmentu, dermatitis. - Den. potř: 8 mg. - Zdroj: kvasnice, játra, maso, soja, žloutek, cereálie.

Pyridoxin – B6: - tato skupina zahrnuje 3 chemicky příbuzné látky – pyridoxol, pyridoxal, pyridoxamin. - Fce: souč. dekarboxyláz a transamináz, podílí se na syntéze kys. nikotinové z tryptofanu a na

přeměně kys. linolové na arachidonovou. - Nedost: kožní a sliznič. změny, ragády úst.koutků, periferní neuropatie. - Den.potř: 2 mg. - Zdroj: velké zastoupení v rostl. i živočiš. Potravinách obilné klíčky, celozrnné produkty, játra,

vepřové, ryby. Kys. listová, folacin – B9: - fce: spolu s vit.B12 se podílí na metabolismu nukleoproteinů, účastní se přenosu

jednouhlíkových radikálů a má význam.roli ve všech procesech buněč. dělení, hl. v hematopoeze. Chemicky obsahuje pterin, kys. p – aminobenzoovou a glutamovou.

- Nedost: při nedostatč. přívodu, vstřebávání nebo zvýš. potřebou v těhotenství, anemiích. Projevuje se makrocytární anemií.

- Den.potř: 200 g, v těhotenství min.400 g – prevence kongenitál. malformací. - Zdroj: játra, kvasnice, listová zelenina, mouka, rýže, luštěniny., vejce, zčásti je produkována

mikroorg. střevního traktu. Cyankobalamin – B12: - fce: role v hematopoeze, ve fci perifer. nerv. systému, účast na tvorbě nukl. kys.,

transmetylačních pochodech, působí anabolicky. - Nedost: perniciozní anemie, neuropatie. - Den.potř: 3 g. - Zdroj: játra, ledviny, maso, žloutky, je tvořen střevní flórou, jeho příjem závisí na zastoupení

živočiš. potravin ve stravě a na správně fungující žaludeč. sekreci. Kys. pangamová – B15: - fce: lipotropní faktor (podílí se na přenosu metylových skupin).

6

Page 7: lf1.czlf1.cz/wp-content/uploads/228-vypracovane_otazky_interna... · Web viewprognoza – 50 % dosáhne kompletního uzdravení do 6 měsíců, 20 % má prodloužený průběh –

Vypracované otázky na INTERNU made by Edita Homolková, Monika Ambroziová, Adam Král, Petr Vermach a Vítek Křehnáč

- Zdroj: jako ostat.vit.B. Kys. orotová – B13: nemá vitamínový charakter, v optimální dávce snižuje tvorbu cholesterolu. Biotin – H: - fce: je kofaktorem karboxyláz.

- Nedost: kožní změny, dermatitis, únava, hypercholesterolémie, cévní poruchy. - Den. potř: 50-200 g. - Zdroj: všude, hl. droždí, zčásti tvořen střevní florou.

VITAMÍN C kys. askorbová: - fce: význačný antioxidant, podporuje resorpci Fe, jako souč. hydroxylačního enzymu se podílí na

tvorbě kolagenu a tím udržuje integritu buněč. membrán, účastní se tvorby karnitinu a tím nepřímo ovlivňuje beta – oxidaci MK, zvyšuje aktivitu mikrosomál. enzymů a urychluje detoxikaci cizorodých látek, blokuje nitrosační reakce a tím tvorbu karcinogenních nitrosaminů, podporuje imunitní procesy, podporuje fce CNS.

- Nedost: skorbut (těmto příznakům zabrání denní dávka 20-30 mg), vyšší potřeba je u DM (při hyperglykémii vázne zpětná reaktivace kys. dehydroaskorbové na askorbovou v bb.), u kuřáků (kouření snižuje askorbémii a kuřáci potřebují o 50% vit.C denně víc), při prevenci aterosklerózy (kys. askorbová snižuje oxidované formy LDL – chol.)

- Zdroj: citrusové ovoce, kiwi, černý rybíz, maliny, zelí, nové brambory, rajčata, šípky, fortifikované džusy.

- Den. potř: 75-100 mg.

2.A RESPIRAČNÍ INSUFICIENCE

=stav nedostatečného zevního dýchání- arbitrárně PaO2 <8 kPa (60 mm Hg) a pCO2 >6,5 kPa (48,9 mm Hg)- syndrom, který je důsledkem řady plicních i mimoplicních patologických stavů (ARDS, CHOPN, těžké astma,

intersticiální plicní procesy)Klasifikace RI - vliv zátěže - latentní – v klidu jsou hodnoty krevních plynů normální, zhoršují se při zátěži

- manifestní – hodnoty jsou patologické i v klidu - průběh - akutní – aspirace, astmatický záchvat, pneumotorax

- chronická – CHOPN, plicní fibrózy- chronická s akutním zhoršením – exacerbace CHOPN

- hodnota tlaků arteriál. krev. plynů - parciální (hypoxemie)- globální (hypoxemie + hyperkapnie) - kompenzovaná (pH

normální, bikarbonát)- dekompenzovaná (respirační acidóza)

- u akutní tachypnoe, hyperventilace, tachykardie- u chronické kompenzace ledvinami (retence Na+ a hydrogenkarbonátů, sekterce H+, K+ a Cl-), polycytémieEtio - alveolární hypoventilace (obstrukce DC, postižení CNS)

- nesprávný poměr ventilace/ perfuze (emfyzém, embolie)- porucha difuze (IPP)- zkratová cirkulace (AV pravolevý zkrat, funkční zkrat – krev obtéká alveolus)- postižení dýchacího centra (léky), nervosvalové poruchy, anémie, plicní embolie…

KO - příznaky základního onemocnění - příznaky hypoxemie – dušnost a cyanóza - příznaky hyperkapnie – spavost, neklid, třes, bolest hlavy, nitrolební hypertenze, vazodilatace, kóma- postižení CNS – neklid, úzkost, poruchy vědomí - postižení KVS – cor pulmonale, ischemie myokardu, arytmie - orgánové selhávání, respirační kachexie

Dg – fyzikálně – cyanóza, vlhké chrůpky, krepitus, pískoty, tiché dýchání…- rtg/ HRCT – rozsáhlejší poškozaní (pneumonie, interstic. Edém, pneumotorax…)-fční vyšetření – obstrukční/ restrikční porucha, postižení plicní difuze- vyšetření krevních plynů a acidobazické rovnováhy z arteriální krve

Th – kyslík, podpora dýchání invazivní/ neinvazivní, stabilizace vnitřního prostředí

7

Page 8: lf1.czlf1.cz/wp-content/uploads/228-vypracovane_otazky_interna... · Web viewprognoza – 50 % dosáhne kompletního uzdravení do 6 měsíců, 20 % má prodloužený průběh –

Vypracované otázky na INTERNU made by Edita Homolková, Monika Ambroziová, Adam Král, Petr Vermach a Vítek Křehnáč

- u chronické respirační insuficience (CHOPN) je ↓ citlivost dýchacího centra na pCO2 a při podání O2 se začne metabolizovat laktát → ↑ hydrogenkarbonáty a pH,ale dýchací centrum se utlumí → hypoventilace → hyperkapnie → tkáňová hypoxie

- lze se tomu vyhnout umělou ventilací, ale rychlé zrušení laktátové acidózy povede k rozvoji mtb alkalózy (přetrvávají ↑ hydrogenkarbonáty) → přesnu K+ do bb. → alkalóza s hypokalémií (arytmie, oběhové slehání)

Syndrom akutní dechové tísně (ARDS)- akutní hypoxemická respirační insuficience s přítomností bilaterálních plicních infiltrátů a je vyloučeno

levostranné kardiální selhání, PaO2/ FiO2 = <200 mm Hg- úmrtnost 25-70 %- riziko ARDS: sepse, polytraumata, šok, opakované transfuze, akutní pankreatitida, polékové reakce,

předávkování, transplantace kostní dřeně, eklampsie, embolizace plodové vody, popáleninyEtio – plicní infekce (miliární tbc, CMV, herpes virus, SARS…), idiopatické pneumonie (akutní intersticiální

pneumonie…), nádory, vaskulopatie, srpkovitá anémie- poškození endotelu v plicním řečišti a epitelu v alveolech → zánětlivé a koagulační mediátory- difuzní poškození sklípků – vznik exudátu v alveolech → intersticiální edém, hyalinní membrány, ztluštění

alveolárních sept → proliferace pneumocytů II → ztluštění cév a ↑ vaskulární rezistence → mikroatelektázy → intersticiální fibróza → dušnost, tachypnoe, hypoxemie

- pokud se nepodaří zastavit ARDS → SIRS (syn. systémové zánětlivé odpovědi) → šok/ MODS (multiorgánové selhání)

KO – náhlé zhoršení dušnosti u lidí s plic. chorobou/ nádorem/ infekcí….Dg – dušnost, hypoxemie nereagující na podání O2, krepitus v plicích = obraz nekardiálního plicního edému

- rtg –bilaterální plicní infiltráty- BAL, CRP, albumin, prokalcitonin, amylázy

Th – oxygenoterapie, umělá plicní ventilace (pokud hypoxemie při podání kyslíku maskou, únava dýchacích svalů…)

- nejnižší možné množství tekutin k udržení tlaku a perfuze (tzv. suché plíce) – furosemid + albumin- kortikoidy- inhalace NO

2.B REAKTIVNÍ ARTRITIDA A PSORIATICKÁ ARTRITIDA

Reaktivní artritida- reaktivní aseptická artritida, která se vyvine v intervalu 2-6 týdnů od infekce urogenitálu/ GIT - Chlamydia trachomatis, Shigella flexneri, salmonely, yersinie- postiženi jsou mladí jedinci 20 – 40 let, častěji u HLA B27= prezentuje se triádou (Reiterův syndrom): artritis, uretritis, konjuktivitis- incidence je 10-30/100000, i když je to často přehlédnuto pro mírnost příznakůdg kritéria - artritida (2 z následujících) - asymetrická

- monoartritida- postiženy převážně dolníkončetiny

- předcházející infekce (alespoň 1) - enteritis (alespoň 1denní průjem 3 – 6týdnů před artritis)- uretritis (alespoň 1denní dysurie 3 – 6týdnů před artritidou)

KO - iniciální infekce:enteropatické ReA (Shigella, Campylobacter, Salmonella, Yersinia)/ sex. přenosné urogenitálního traktu (Chalmydia, Neisseria)

- muskuloskeletální - obvykle první manifestací (s časovým odstupem od infekce) je monoartritida kolene, nebo asymetrická oligoartritida dolních končetin, různé intenzity, která je většinou přechodná

- kožní a slizniční - keratoderma blennorhagica na plosách nohou a rukou, a jiné psoriatiformní leze, u enteropatických erythema nodosum, balanitis circinata

- urogenitál – sterilní uretritis, prostatitid, cystitis, všechny uretrální příznaky jsou mírné a je nutno po nich pátrat

- GIT - Enteritis, Colitis (ať už vyvovalatel, nebo nezávisle na první infekci – při probíhající ReA), podobné změny ulcerózní kolitidě, nezávisle na vyvolválajícím agens

- oční - konjunktivitida, uveitida (zčervenání, slzení fotofobie)- teploty, únavnost, malátnost, hubnutí

8

Page 9: lf1.czlf1.cz/wp-content/uploads/228-vypracovane_otazky_interna... · Web viewprognoza – 50 % dosáhne kompletního uzdravení do 6 měsíců, 20 % má prodloužený průběh –

Vypracované otázky na INTERNU made by Edita Homolková, Monika Ambroziová, Adam Král, Petr Vermach a Vítek Křehnáč

Dg - anamnéza- laboratorní průkaz infekce (moč, stěr z uretry, cervixu, PCR (Chlamydie), kultivace stolice, sérologie

(IgG,IgA,IgM)Th - nesteroidními antirevmatiky

- při výraznějších bolestech Prednison 20-40mg denně, intraartikulárně při monoartritidě, Sulfasalazin účinkuje hl. u chronických forem

prognoza – 50 % dosáhne kompletního uzdravení do 6 měsíců, 20 % má prodloužený průběh – déle je to bolí, 2-18 %pacientů přechází do chronicity, znich až u 50 % vzniklo sakroilitida a u 25 % vznikla ankylozující spondylitida

Psoriatická artritida= PsA je zánětlivé revmatické onemocnění, projevující se artritidou při psoriáze, při negativitě revmatoidních

faktorů- patogeneze není zcela jasná, je asociována s geny, které jsou asociovány s psoriázou (HLA Cw6, HLA B27), role

zevních faktorů (infekce, trauma stres je velká neznámá) významnou roli hraíjí různé skupiny T lymfomů- prevalence PsA je 1 %, postihuje nejčastěji lidi mezi 30 a 50 lety, PsA má 15%lidí s psoriázou (kontrolní otázka:

„jaká je prevalence psoriázy?“) Vznik psoriázy zpravidla předchází PsA, ale mohou vzniknout i najednouKO - heterogenní onemocnění a je rozdělováno do 5 podskupin, které mezi sebou mohou přecházet

- hlavními projevy jsou: psoriáza, periferní artritida, axiální postižení, enteritida, daktylitida- dominující postižení DIP kllubů- nesymetrická oligoartritis- mutilitující artritis (eroze --- subluxace --- „teleskopicé prsty“)- symetrická oligoartritis – pseudorevmatoidní- psoriatická spondylitida- mimoskeletální projevy - psoriáza, postižení nehtů, oční, střevní, kardiální

Dg - ↑ reaktanty akutní fáze, hypergamglobulinémie, hypalbuminémie, anémi chronických chorob- zobrazovací metody - postižení DIP

- kombinované změny: eroze + proliferativní změny – periostózy, kostní ankylózaTh - je to systémové onemocnění a může způsobit chronickou bolest, deformity

-systémové glukokortikoidy způsobí relaps psoriázy po vysazení – NEBRAT- intraartikulární kortikoid u oligoartritid- MTX, Cyklosporin, Leflunomid- Infliximab působí až u 70% na všechny projevy nemoci, kteří nereagují na konvenční léčbu – KOMBINACE

S MTX NEZVYŠUJE ÚČINNOST (na rozdíl od RA)Prognoza - u 40% pacientů dochází k závažným deformitám kloubů

3.A KARDIOMYOPATIE

= skupina onemocnění myokardu vedoucí k poruše mechanické/ elektrické funkce- projevuje se neadekvátní hypertrofií/ dilatací mokardu, často genetické příčiny- samostatně/ součást systém. onem, vedou k srdečnímu selhání- primární – postihuje jen srdce - sekundární – při Fabryho ch. (deficit α-galaktosidázy, amyloidóza hemochromatóza, sarkoidóza, akromegalie…Dilatační kardiomyopatie

- postupná dilatace všech srdečních oddílů, nejvíce LK při zachovalé tloušťce stěny a s postupnou poruchou systolické funkce → srdečná selhání, arytmie, poruchy převodního sys. (↓ EF, později je i porucha diastolické funkce (compliance))

Etio – postinfekční – coxsackie, adenoviry, HIV, difterie, Chagasova ch. (Tripanosoma cruzii)- alkoholická - přímý tox. účinek, deficit thiaminu, toxicita aditiv, abstinence zastaví progresi- medikamentózně - toxická (antracykliny, litium, fenothiaziny)- těhotenská (v posledním měsíci těhotenství až 5 měsíců po porodu), tachykardií indukovaná- idiopatická - nejčastější (pravděpodobně po myokarditidě)

KO - progresivní levostranná insuf. (námahová dušnost, tachykardie), později globální insuf. (↑ náplň krčních žil, hepatomegalie, otoky), poruchy komorového rytmu (fibrilace síní, komorové extrasystoly), embolie

- nejčastěji u lidí středního věku (ale i mladí a staří)Dg – fyzikálně – možné šelesty mitrální/ trikuspidální regurgitace (dilatace anulů, dysfce papilární svalů)

9

Page 10: lf1.czlf1.cz/wp-content/uploads/228-vypracovane_otazky_interna... · Web viewprognoza – 50 % dosáhne kompletního uzdravení do 6 měsíců, 20 % má prodloužený průběh –

Vypracované otázky na INTERNU made by Edita Homolková, Monika Ambroziová, Adam Král, Petr Vermach a Vítek Křehnáč

- rtg (rozšířený srdeční stín = kardiomegalie), EKG (nespecifické změny, často blok levého Tawarova raménka), echo (často rel. mitrální insuf.), MR (vyloučení jiné příčiny – chlopenní vady/ těžká ICHS) koronarografie (u podezření na aterosklerózu)

Th – režimová opatření – vyloučit zátěž (jen aerobní), pozor na infekce (atb, vakcinace proti chřipce)- kauzální - abstinence, vysazení kardiotoxických léků, léčba infekce- farm – ACEI (omezení symptomů a mortality)

- β-blokátory – carvediol, bisoprolol, metaprolol- warfarin - hl u vyššího stupně dilatace LK a fibrilace síní- diuretikum + digitoxin – symptomaticky- amiodaron – u maligních arytmií, udržení sinusového rytmu u fibrilace síní- biventrikulární kardiostimulátor – u dyssynchronie kontrakce- kardioverter-defibrilátor – u rizika maligních arytmíí- transplantace srdce

Hypertrofická kardiomyopatie - často náhlá úmrtí mladých sportovců- výrazná hypertrofie myokardu (dezorganizací myofibril) s malou dutinou LK → diastolická dysfce, pak i

systolická dysfce- hypertrofie může postihnout různé části komory → může být obstrukce výtokového traktu LK/ septum, zadní

stěna…- obstrukce výtokového traktu LK – dynamicky se chová jako aortální stenóza

- díky dopřenému pohbu předního cípu mitrální chopně směrem k vyboulenému hypertrofickému septu v průběhu systoly

- díky prodloužení cípu mitrální chlopně a ventrální posun papilárního svalu→ systola má 2 vrcholy = bifázický puls (cíp se na chvilinku připleskne a pak se zase oddálí a odteče

zbytek krve z LK)Etio – nejčastěji AD (mutace různých částí sarkomery - β-myosin, troponin, aktin…)KO – přivedou je příznaky srdeční selhání/ arytmie/ angina pectoris (tlustý sval trpí ischemií), v ětšinou

asymptomatický (pak naráz náhlá srdeční smrt a šlus)- systolický šelest v auskultačním místě aortální chopně/ 4. Mezižebří parasternálně a ten se zesiluje při

manévrech vedoucí k poklesu tepového objemu (Valsalův manévr, vertikalizme pacienta)Dg - ECHO, MR a katetrizace (tlakový gradient mezi LK a kořenem aorty, event. biopsie)

- EKG (známky hypertrofie levé komory, obraz pseudoinfarktu s hlubokými Q a neg. T vlevo prekordiálně = známka hypertrofie septa)

- Echo (asymetrická hypertrofie septa, dopředný pohyb předního cípu mitrální chlopně v systole), katetrizaceTh - symptomatická - omezení fyz. námahy (riziko náhlé smrti), zákaz inotropik (digitalis, sympatomimetika) a

nitrátů a diuretik (snižují preload)- farm - verapamil/ β-blokátory - ↓ frekvenci → prodlužují diastolu a zlepšují plnění LK

- amiodaron – u komorových arytmií- kardioverter-defibrilátor (nejúčinější prevence náhlé smrti)- chir – klínová resekce části mezikomorového septa, alkoholová abkace

Restriktivní kardiomyopatie- porucha diastolické funkce levé komory (porucha plnění) → ↓ pokles tepového objemu → aktivace sympatiku

→ kompenzatorní tachykardie- velice vzácná- primární – endokardiální fibroelastóza- sekundární – sarkoidózy, hemochromatóza, amyloidóza, glykogenózaKO – příznaky srdečního selhání, tachykardie, ↑ náplň krčních žil s zvýrazněním v inspiriu, hepatomegalie,Dg - viz předešléTh – léčba příčiny u sekundárních

- symptomaticky srdeční selhání (diuretika, profylaxe trombembolie...), event. transplantaceArytmogenní kardiomyopatie pravé komory

- velice vzácná lipomatózní degenerace myokardu pravé komory s komorovými arytmiemi- často familiárně, synkopy u mladách lidí u nichž někdo v rodině zemřel v mladém věku náhlou smrtí

Tako-tsubo kardiomyopatie- dilatace hrou LK

10

Page 11: lf1.czlf1.cz/wp-content/uploads/228-vypracovane_otazky_interna... · Web viewprognoza – 50 % dosáhne kompletního uzdravení do 6 měsíců, 20 % má prodloužený průběh –

Vypracované otázky na INTERNU made by Edita Homolková, Monika Ambroziová, Adam Král, Petr Vermach a Vítek Křehnáč

- ST elevace na EKG, akineze hrotu, není postižení koronárních tepen- je katecholamin-dependentní (u feochromocytomu, stres)

3.B ADRENOKORTIKÁLNÍ INSUFICIENCE; ZÁSADY LÉČENÍ HYPOKORTIKÁLNÍCH STAVŮ

- stav vzniklý nepoměrem mezi sekrecí a potřebou steroidních hormonů kůry nadledvin- nejčastěji autoimunitní postižení (80 %), samostatně/ v rámci polyglandulárního syndromu- 5/ 100 000 obyvatelPrimární (periferní) adrenokortikální insuficience = Addisonova choroba

- nedostatek glukokortikoidů, mineralokortikoidů a androgenů- postižení obou nadledvin (funkce 1 je dostatečná)- příčiny: autoimunitní postižení, metastázy, lymfom, hemoragie (Waterhousenův-Friderichsenův syn.), infekce

(TBC), amyloidóza, hemochromatóza, kongenitální adrenální hyperplazie, léky (ketokonazol, etomidat…)

KO – ze začátku nespecifické příznaky, únavnost, nechutenství, hubnutí, GIT příznaky (průjem, zvracení), hypotenze (ortostatická), sklon k hypoglykémii, artralgie, myalgie, amenorea

- hyperpigmentace – díky ↑ POMC → ↑ MSH - místa exponovaná slunci, dlaňové rýhy, linea alba, periareolárně, perigenitálně,

grafitové skvrny na bukální slizniciLab – hyperkalcémie, možná acidóza, hyperkalcémie, hypoglykémie, ↑ ACTH

- normocytární normochromní anémie, neutropenie, lymfocytóza nebo eosinofilieSekundární (centrální) adrenokortikální insuficience

- nedostatek glukokortikoidů a androgenů, mineralokortikoidy obvykle produkovány dostatečně (renin)- příčiny: cysty, tumory, traumata (chir.), záněty, vaskulární léze, kongenitální defekty, malformace, hamartomyKO – není hyperpigmentace, menší sklon k hypotenzi a hyperkalémii

Akutní adrenokortikální insuficience a hypokortikální (addisonská) krize- život ohrožující stav- Waterhousenův-Friedrichsenův syn. (např. meningoková sepse), nedostatečné zvýšení substituční dávkyKO – slabost, bolest břicha nebo zad, nauzea, zvracení, šok s hypotenzí, tachykardie, hypoglykémie,

hyperkalémie, zmatenost, hyperpyrexie, komaDg – bazální sérová konc. kortizolu ráno nalačno - >150 nmol/l dg potvrzuje, >550 nmol/l dg vyvrací (co je mezi –

parciální insuf., sekrece může stačit za bazálních podmínek)- dynamický stimulační test – inzulinový toleranční test – indukce hypoglykémie pod 2,2 mmol/l, hodnocení

nástupu koncentrace kortizolu- ne u DM, ICHS, neurolog. záchvatovitých onem.

- kortizonový (krátký ACTH) test - UZ nadledvin, MR/ (CT) nadledvin, MR hypotalamo-hypofyzární oblasti

Th – hydrokortizon – bazální substituční dávka 15-30 mg/ den (nápodoba cirkadial. rytmu – ½ až 2/3 ráno a zbytek napůl do polední a večerní dávky)

- při zátěži zvyšovat (až na 300-400 mgú den)- fludrokortizon – mineralokortikoidní substituce, 0,05-0,2 mg/ den

4.A PORTÁLNÍ HYPERTENZE, KRVÁCENÍ Z JÍCNOVÝCH VARIXŮ

Portální hypertenze= ↑ portosystémového gradientu nad 5 mm Hg, klinicky závažné nad 10 mm HgEtio – trombóza v. portae, trombóza jaterních žil (Budd-Chiariho syn.), jaterní cirhóza, chronická jaterní

onemocnění- intrahepatální cévní rezistence – aktivní ovlivnění průsvitu sinusoid hvězdicovitými bb., pasivně otokem

hepatocytů a regenerativními změnami- ↑ přítok krve do splanchniku – vznik kolaterál (jícnové varixy, hemoroidy, mezi levou ledvinou a slezinou)

→ možné ruptury, zhoršení průtoku játry, ↑ vazodilatačních substancích v systém. oběhu

- dilatace splanchniku → hypotenze, aktivace renin-angiotenzin-aldosteron. sys., ↑ ADH (retense Na+ a vody)

11

Page 12: lf1.czlf1.cz/wp-content/uploads/228-vypracovane_otazky_interna... · Web viewprognoza – 50 % dosáhne kompletního uzdravení do 6 měsíců, 20 % má prodloužený průběh –

Vypracované otázky na INTERNU made by Edita Homolková, Monika Ambroziová, Adam Král, Petr Vermach a Vítek Křehnáč

Dg – UZ zvětšení sleziny, UZ-Doppler směr a rychlost toku v portální žíle- HVPG (hepatovenózní tlakový tlakový gradient) norma 3-4 mm Hg

KO – pavoučkovité névy, dilatace žil břišní stěny, palmární erytém (dík hyperkinetické cirkulaci), splenomegalie- ascites, hepatorenální syndrom, hepatosplenomegalie, krvácení

Krvácení jícnových varixů- u 30-60 % lidí s cirhózou- nebezpečí krvácení hl. u portálního tlaku >12 mm Hg, velké varixy nad 5 mm- recidiva do 2 let u 60-100 %KO – hemateméza, meléna (často i bez zvracení krve), hemoragický šokDg – endoskopieTh – na JIP, kanylace centrální žíly, erymasa (hematokrit ne víc než 30 %!, jinak recidiva)

- laváž žaludku, atb- endoskopická zástava krvácení – ligace, sklerotizace- vazokonstrikce splanchniku terlipresinem/ somatostatinu- 2balónková nazogastrická sonda při nemožnosti endoskopické zástavy (ne víc než 24 h)- transjugulární intrahepatální portosystémová spojka (TIPS) – kanylace v. hepatica transjugulární punkcí a

probodnutí jaterního parenchymu stentem do v. portae- zhoršení encefalopatie

- β-blokátory neselektivní (carvedilol, dostatečná dávka když pulsy klesnou o 25 % nebo dosažení 55 pulsů/min)

- vysoká mortalita (do 6 týdnů 30-50 %)Ascites

- volná tekutina v břiše (fyziologické je 150 ml), i při karcinomatóze peritonea, pankreatitida, tbc, trombóza portální žíly)

Etio – při portální hypertenzi se ↑ tlak v jaterních sinusoidech → přestup albuminu do extravaskulárního prostoru spolu s tekutinou → drenáž do lymfatických cest která pak nestačí (norma 1 l/den se může zvýšit na 10 l/den) a přebytek uniká povrchem jater do peritoneální dutiny

KO – špatné prognostické znamení, při větším je dyspepsie, dušnost, umbilikální kýlaDg – při malém množství SONO, poklepem je zjistitelné od 1500-3000 ml, undulace při > 10 l

- punkce – spontánní bakteriální peritonitida, albuminový gradient sérum-ascites (SAAG), který koreluje s velikostí portálním tlakem (pro PH svědčí rozdíl >11 g/l)

Th – klid na lůžku, vysazení neurotoxické medikace (aminoglykosidy, antiflogistika), omezení soli na 3 g/den- diuretika – spironolakton (antagonista aldosteronu) 400 mg/den + furosemid 160 mg/den, - paracentéza – u ascitu refrakterního k diuretikům

- i.v. albumin jako prevence hypovolemie, je lepší najednou vypustit co nejvíce (až 5 l), nebo za hospitalizace celý

- punkce ascitu s jeho extrakorporální koncentrací a zpětnou infuzí i.v. nebo intraperitoneálně- umělá peritoneovenózní spojka drénující ascites přímo do centrální žíly

Spontánní bakteriální peritonitida= bakteriální infekce ascitu bez zjistitelného a chirurgicky léčitelného zdroje ascitu- vysoká mortalita (30 % u neléčení), u 20 % ascitůEtio – přestup bakterií translokací přes neporušenou střevní stěnu, hlavně u ascitu s ↓ obsahem bílkovin (není

taková ozonizační aktivita)- hlavně anaerobní bakterie, 75 % G- (e. coli, klebsiela, enterobacter, proteus)

KO – většinou nevýrazné, např. jen refrakterní ascites- subfebrilie, difuzní bolest břicha,leukocytóza

Dg – kultivace, granulocytů v ascitu >25*109/lTh – cefalosporin III. generace (ceflotaxim 2 g/ 8/ 12 h)

Jaterní encefalopatie- neuropsychiatrický syndrom s poruchami vědomí a chování, neurologickými poruchami, přítomností flapping

tremoru a se změnami na EEGEtio – není jasné, transport toxinů přímo ze střeva do systémového oběhu, amoniakKO – kvalitativní i kvantitativní změny vědomí, flapping tremor (natažení HK se zavřenýma očima), kontrují

apraxie (neschopnost se podepsat, namalovat hvězdu…), foetor hepaticus

12

Page 13: lf1.czlf1.cz/wp-content/uploads/228-vypracovane_otazky_interna... · Web viewprognoza – 50 % dosáhne kompletního uzdravení do 6 měsíců, 20 % má prodloužený průběh –

Vypracované otázky na INTERNU made by Edita Homolková, Monika Ambroziová, Adam Král, Petr Vermach a Vítek Křehnáč

- minimální encefalopatie – klinický nález normální, ale abnormální psychometrické testy (spojování číslic od 1 do 25 za čas)

- nejčastěji se projeví po vyvolávajícím faktoru (infekce, krvácení z jícnových varixů, dehydratace, elektrolyt. dysbalance)

Dg – klinika, ↑ amoniak (ale nemusí nijak korelovat s projevy), změny EEG (nejsou specifické)Th – omezení bílkovin (záměna za rostlinné), nevstřebatelná atb působící na střevní bakterie (rifaximin),

nevstřebatelný disacharid laktulóza…Hepatorenální syndrom

- fční selhání ledvin při jaterním onemocnění s portální hypertenzíEtio – portální hypertenze → ↓ periferního odporu, centrální hypovolemie, aktivace sympatiku) → renální

arteriální vazokonstrikce s hypoperfuzí hl. kůry ledvinKO – výhradně u lidí s ascitem

- rychle progredující – zdvojnásobení sérového kreatinu během 2 týdnů, špatná prognóza- pomalu progredující

Dg – není specifický test, ↓ glomerulární filtrace <40 ml/min, nebo ↑ kreatininu >135 μmol/l, konc Na+ bývá snížená pod 10 mmol/l a močová osmolalita vyšší než plazmatická

Th – vyloučit neurotoxickou medikaci, diuretika, NSA, léčit infekci- korekce hypovolemie (albumin)- systémová vazokonstrikce terlipresinem

5.A ISCHEMICKÁ CHOROBA SRDEČNÍ – PATOGENEZE, KLINICKÉ FORMY

- choroby srdce s postižením koronárních tepen především aterosklerózou a v důsledku toho reverzibilní/ ireverzibilní ischemie části myokardu

Ateroskleróza- většina případů ICHS je způsobená aterosklerózou věnčitých tepen (podle oblasti převažujícího postižení : ICHS,

ICHDK, ICHM)= určité patologické vystupňování ochranné odpovědi na poškození endoteliálních a hladkých svalových buněk

cévní stěny1. endoteliální dysfce – může vzniknout mech. poškozením, zánětem, toxickým poškozením oxidovanými LDL,

homocysteinem, nikotinem- fce endotelu - tvoří netrombogenní vnitřní povrch cévního řečiště; působí jako selektivně

propustná bariéra pro transport látek- vytváří a secernuje vazoaktivní látky (NO, prostacyklin, a vazokonstričně

působící endotelin)- produkuje růstové faktory a cytokiny, má schopnost oxidovat lipoproteiny

v průběhu jejich transportu do cévní stěny2. tvorba lipoidních proužků – agregáty makrofágů bohatých na lipidy a T-lymfocyty uvnitř cévní stěny

- nezužují průsvit tepen, mohou progredovat, nebo regredovat3. tvorba fibrózních plátů – vedou ke zúžení průsvitu tepny

- obsahují makrofágy, pěnové bunky (makrofágy s nitrobuněčnou akumulací oxidovaných LDL), proliferující bb. hladké svaloviny, aktivované T-lymfocyty, vazivo

4. vznik komplexního aterosklerotického plátu = fibrózní pláty komplikované kalcifikací, trombózou, krvácením- stabilní aterosklerotický plát – vysoký obsah vaziva, malé lipidové jádro, často je kalcifikovaný a

má silný fibrózní kryt, zvětšuje se jen zvolna, způsobí fixní zúžení lumen- obraz námahové anginy pectoris nebo němé ischémie myokardu

- nestabilní plát – bohaté lipidové jadro, tenký fibrózní kryt (čepička), náchylný ke vzniku figurací- na místě defektu adherují a agregují trombocyty a vzniká bílý trombus

- současně s aktivací trombogenních procesů se aktivují i trombolytické děje- převáží-li trombogenní pochody → narůstání trombu → úplný uzávěr tepny → AIM- převáží-li trombolytické pochody → uzávěr jen částečný → nestabilní AP, nebo non-Q-infarkt

ICHS – ischémie myokardu vzniklá v důsledku nepoměru mezi potřebou a dodávkou O2 myokardu- může být jak koronární, tak nekoronární

13

Page 14: lf1.czlf1.cz/wp-content/uploads/228-vypracovane_otazky_interna... · Web viewprognoza – 50 % dosáhne kompletního uzdravení do 6 měsíců, 20 % má prodloužený průběh –

Vypracované otázky na INTERNU made by Edita Homolková, Monika Ambroziová, Adam Král, Petr Vermach a Vítek Křehnáč

- nedostatek kyslíku způsobuje: 1) nedostatečný přívod (↓transportní kapacita krve, spasmus koronárních cév, stenóza koron.cév)

2) ↑ spotřeba (↑ SF, ↑ kontraktilita, ↑ napětí stěny v systole)- nejčastěji je projevem arterosklerózy koronárních artérií, stenóza způsobí omezení průtoku krve a tím ischémii

myokardu, nejčastější příčina úmrtí v rozvinutých zemíchPtg - postupné zužování průsvitu koronámí artérie, při stenóze >75% (kritická stenóza) projevy stabilní angíny

pectoris (stenokardie závislá na zátěži), průsvit se může rychle zmenšit rupturou aterosklerotického plátu nebo koronárním spazmem

- koronární rezerva - rozdíl mezi koronárním průtokem v klidu a při maximální zátěžiRizkové faktory - neovlivnitelné - věk (muži nad 45, ženy postmenopauzální), mužské pohlaví, genetika

- ovlivnitelné - dyslipoproteinémie (>LDL, <HDL, >TGL), AHT, DM, metabolický syndrom (obezita, DM2, >TGC, AHT), kouření

- nedostatek pohybu, stres, hyperhomocysteinémie, hyperfibrinogenémie, protilátky proti fosfolipidům

formy - akutní – bezprostřední ohrožení života- nestabilní angina pectoris- IM- náhlá srdeční smrt

- chronické - asymptomatická ICHS (ot. 11.A)- angina pectoris (námahová, smíšená, variantní)- stav po IM- dysrytmická forma ICHS- chronické srdeční selhání

5.B AUTOIMUNNÍ TROMBOCYTOPENIE

Idiopatická trombocytopenická purpura (ITP) = morbus maculosus Werlhofi- 3 – 5/ 100 000 obyvatel, často recidivuje- primární – 80 % případů-sekundární – 20 % případů, při infekcích (HIV, HVC, Helicobacter pylori), sys. Onemocnění (SLE), nádor. choroby

(lymfoproliferace z B bb.)- trombocytopenické purpury, u kt. z neznámé příčiny dochází za účasti imunit. mechanismů k urychlenému

rozpadu trombocytů- protilátky třídy IgG → fagocytóza- trombopoetin v norměAkutní forma - dětský věk, prudký průběh s častou spontánní úpravou, navazuje na banální (často virovou)

infekci- cirkulující imunokomplexy s afinitou k trombocytům/ zkřížená reakce virového antigenu s

trombocyty- rozvoj krvácivých projevů je prudký (hodiny), generalizovaná purpura ( extenzorové plochy

předloktí, holeně, trup v místě otlaku, pas), na kůži rozsáhlé hematomy i krvácení ze sliznic- celkový stav je nedotčený, krvácení do orgánů a CNS velmi vzácná

Chronická forma - dospělí, plíživý počátek a chronický průběh, krvácivé projevy se stupňují, častěji ženy než muži- autoprotilátky proti antigenům trombocytů → urychleně vychytány ve slezině, ve slezině i

tvorba protilátek- častější závažná orgánová krvácení, do CNS-fatální

Dg - klinická; autoprotilátky,ITP může být jedním z projevů SLE či může provázet B-lymfoproliferace- vyloučení hl. dřeňového útlumu (akut. Leukemie, myelodysplastický sy.)

Lab – těžké formy 10 * 109/l destiček, vše ostatní v normě- biopsie spíš u starších – megakaryocyty přítomny (často zmnoženy, i sníženy), ostatní v normě- antitrombocytární protilátky - obtížně prokazatelné, protože Ig jsou na povrchu destiček fyziologicky

přítomné- průkaz protilátek vázajících se na panel izolovaných a imobilizovaných destičkových

antigenů, úspěšné u 60 %

14

Page 15: lf1.czlf1.cz/wp-content/uploads/228-vypracovane_otazky_interna... · Web viewprognoza – 50 % dosáhne kompletního uzdravení do 6 měsíců, 20 % má prodloužený průběh –

Vypracované otázky na INTERNU made by Edita Homolková, Monika Ambroziová, Adam Král, Petr Vermach a Vítek Křehnáč

- mohou pronikat přes placentu → příznaky u novorozence odezní spontánně (týdny)Th – u hodnot mezi 20 – 50 * 109/l

- glukokortikoidy – u akut. forem zkrácení doby spontánní remise- u chron. forem úprava stavu v 50 %

- i.v. imunoglobuliny – u život ohrožující situace, spolu s glukokortikoidy- rychlá úprava, ale krátkodobá

- trombocyty- anti CD20 monoklonal. protilátky (rituximab), cyklofosfamid, cyklosporin A – po selhání iniciální léčby- splenektomie – u refrakterních forem, při trombocytokinetickém průkazu převažujícího zániku ve slezině

6.A AKUTNÍ INFARKT MYOKARDU ST ELEVACEMI

→ emergentní perkutánní koronární intervence, kompletní uzávěr tepny= STEMI – ST elevace alespoň ve 2 sousedních svodech (např. III + aVF, V2 + V3…)

- ST min 1 mm nad izoelektrickou linií (pokud ve svodech V1 – V3 tak o 2 mm)- elevace ST ve aVR – až v 65 % významné postižení kmene levé koronární tepny (superhrozné!)- blok levého Tawarova raménka a bifascikulární blok znemožňuje dobrou diagnostiku IM

Infarkt myokardu = nekróza myokardu v důsledku náhlého uzávěru koronární arterie prasklým aterosklerotickým plátem a nasedlou

trombózou- hemodynamicky významná stenóza při zúžení o 60-70 %Ptg - ateroskleróza koronární artérie → stabilní ateromový plát → nestabilní plát → ruptura plátu s tvorbou trombu

uzavírajícího lumen artérie → nestabilní angina pectoris/ IM/ náhlá srdeční smrt- vyvolávající faktory - námaha, stres, nejčastěji ráno (6:00 - 12:00), riziko IM při nestabilní AP až 20%- nekróza začíná už 20-30 min po uzávěru (subendokard - nejcitlivější) → supepikardiální → transmurální do 6 h (Q-

infarkt); při neúplném uzavření/ časné rekanalizaci je nekróza pouze subendokardiální (non-Q-infarkt)KO - bolest - intenzivní dlouhotrvající (alespoň 30 min) na hrudi lokalizovaná retrosternálně/ prekordiálně, která neustupuje

v klidu (ani po podání nitroglycerinu)- vyzařování bolesti do paží po ulnární straně, ramen, krku, dolní čelisti, může vyzařovat i do epigastria (při

spodním infarktu)- nemusí být u diabetiků se senzitivní neuropatií

- pocit slabosti, strach, úzkost, vegetativní doprovod (bledost, pocení, nauzea, zvracení)- dušnost (vlhké chrůpky, plicní edém)

- arytmie (hlavně komorové), často pokles TK, porucha motility myokardu během vteřin po uzávěruDg - biochemie – průkaz látek typických pro myokard a fyziologicky uložených pouze intracelulárně

- diagnostické okno až 120 min!!!- myoglobin - stoupá v 90-120 min, normalizace 24h, nespecifický, v časné diagnostice- kreatinkináza - CK-MB frakce, možný odhad velikosti infarktového ložiska

- stoupá za 2-4 h, max za 24h, normalizace za 2-3 dny- troponin I a T – 3-5 h – 10 d po IM, nejspecifičtější, nejcitlivější metoda

- zobrazovací metody – rtg – stupeň městnání- echokardiografie – hypokineze v akutní fázi, direkce aorty jako dif.dg.- selektivní koronarografie

- nespecifické: leukocytóza, ↑ FW- EKG - transmurální (Q infarkt) - akutní (první hodiny) - vysoké špičaté T (gotické T), patologické hluboké Q (šířka >0,04

s, výška >25% R) v kombinaci s elevací ST, která plynule přechází do vlny T (Pardeeho vlna); při rekanalizaci se ST normalizuje téměř okamžitě

- subakutní - postupný pokles elevace ST, tím se snižuje R, až může vznikat komplex QS, negativní vlny T

- chronické - přetrvávající negativní T nebo jeho normalizace, R se postupně normalizuje, většinou celoživotně přetrvává hluboké Q

- pokud déle než 6 měsíců elevace ST, myslet na aneuryzma srdce- netransmurální infarkt (non-Q infarkt) - deprese ST + negativní T, jinak žádné změny QRS komplexu

15

Page 16: lf1.czlf1.cz/wp-content/uploads/228-vypracovane_otazky_interna... · Web viewprognoza – 50 % dosáhne kompletního uzdravení do 6 měsíců, 20 % má prodloužený průběh –

Vypracované otázky na INTERNU made by Edita Homolková, Monika Ambroziová, Adam Král, Petr Vermach a Vítek Křehnáč

LOKALIZACE INFARKTU ZMĚNY V EKG SVODECHAnteroseptální V1-V3, IApikální V4-V6Laterální V5-V6, aVLDiafragmatický II, III, aVFZadní Kontralat. deprese ST v V1-V3

Komplikace – ot 60.BTh – ot. 94.B

6.B FEBRILNÍ NEUTROPENIE

= stav, kdy u nemocného s počtem neutrofilních leukocytů pod 0,5*109/l naměříme 2 x v průběhu 12 h vyšší tělesnou teplotu než 38 °C, nebo jednou vyšší než 38,5 °C

- vzniká jako infekční komplikace imunosuprimovaných pacientů (onkologická léčba, hematolog. malignity...)- průběh infekce v podmínkách neutropenie a imunodeficience je velmi těžký, může se vyvinout sepsi s možností

dalších komplikací (endotoxinovým šokem, DIC)- je nutné neprodleně zahájit empirickou ATB léčbu (bez čekání na lab. výsledek, G+ i G-)!!!- odběr hemokultury při vzestupu teplot z periferní žíly a každého lumen centrálního žilního katetru (mikrobiologie

a mykologie) + skiagram hrudníku (vyloučení plic. infiltrátu)- profylaxe ATB a antimykotiky není rutinně používánaPříčiny - neznámá příčina

- důsledek toxického poškození dřeně (th. cytostatiky, radioterapie, infiltrace k. dřeně nádory)- neefektivní proliferace neutrofilů

Th - kombinace ATB pokrývající G- i G+ flóru (cefalosporin - ceftazidim + aminoglykosid - amikacin), popř. kombinace s vankomycinem

- po určení agens se léčba modifikuje dle zjištěné citlivosti na ATBProfylaxe - snížení expozice nemocného vůči infekci (dodržování hygieny, izolace, profylaxe mykotické

superinfekce)- úprava granulocytopenie (hematopetické růstové faktory)- posílení imunity (imunomodulátory, i.v. imunoglobuliny)

7.A CHRONICKÉ SRDEČNÍ SELHÁNÍ

- srdeční selhání je klinický sydrom, který je součástí vývoje mnoha srdečních chorob (ICHS, hypertenze, chlopenní vady, kardiomyopatie…)

- dušnost, únava, snížená tolerance zátěže, tachykardie, cvalový rytmus, venostatické chrůpky, periferní otoky, systolická/ diastolická dysfce…

Srdeční selhání – akutní/ akutně zhoršené/ chronické- levostranné/ pravostranné/ oboustranné- systolické/ diastolické/ smíšené

= stav, při kterém není srdce schopno bez zvýšení plnícího tlaku (tlak na konci diastoly) udržet takový srdeční výdej, který by pokryl potřebu organismu

Ptg (při low-output failure) - nejčastěji hypertenze (HT → ICHS → IM → srd. ins.)- ↓kontraktility - porucha systol. fce - snížená kontraktilita (IM, myokarditida, dilat. kardiomyopatie) - tlaková zátěž - ↑ afterload (arteriální HT, aortální stenóza)- objemová zátěž - ↑preload (insuf. chlopní - musí vypuzovat nejen norm. objem, ale i to, co regurgituje,

anémie a tyreotoxikóza - celkový srdeční výdej je zvýšen)- porucha diast. fce - porucha plnění komor, porucha distenzibility (konstr. perikarditida, srdeční

tamponáda)- arytmie

Kompenzační mechanismy - Neuroendokrinní - jsou prospěšné v akutní situaci, při dlouhém trvání situaci zhoršují a vzniká circulus vitiosus

16

Page 17: lf1.czlf1.cz/wp-content/uploads/228-vypracovane_otazky_interna... · Web viewprognoza – 50 % dosáhne kompletního uzdravení do 6 měsíců, 20 % má prodloužený průběh –

Vypracované otázky na INTERNU made by Edita Homolková, Monika Ambroziová, Adam Král, Petr Vermach a Vítek Křehnáč

- aktivace sympatiku + vyplavení katecholaminů – stimulace α rec → vazokonstrikce žil → zmenšení řečiště (> afterload). Selhávající komora však na zvýšení preloadu odpoví jen nevelkým zvýšením výdeje a vysoký plnící tlak levé komory ohrožuje kapilárním městnáním funkci plic a stimulací βrec → ↑kontraktilitu, ↑frekvenci, vede k redistribuci oběhu do srdce, mozku

- aktivace systému renin-angiotenzin-aldosteron-angiotenzin II (> afterload, působí jako růstový faktor hladké svaloviny cév a myokardu), aldosteron (> preload)

- aktivace ADH - retence vody (> preload)- aktivace atriálnlho a mozkového natriuretického peptidu - působí natriureticky a vazodilatačně (<

preload)- Změna tloušťky myokardu - z objemového přetížení - při akutní srd. insuf. dilatace komor, při chron. srd.

insuf. (insuf. chlopní) excentrická hypertrofie (hypertrofie + dilatace) - z tlakového přetížení (AHT, stenózy chlopní) koncentrická hypertrofie (hypertrofie bez dilatace)

Pro dg - přítomnost subjektivních potíží (dušnost, únavnost) a objektivní známky (periferní otoky, tachykardie, cvalový rytmus, venostatické chrůpky)

- objektivní průkaz srdeční dysfce v klidu (systolická/ diastolická)- ve sporných případech odpověď na léčbu (např. ústup dušnosti po diureticích)- stanovení natriuretických peptidů, kdy jejich normální hodnoty vylučují srdeční selhání (BNP <100pg/ml,

NT-proBNP <120 pg/ ml)NYHA: I. – bez omezení tělesné aktivity

II. – menší tělesná aktivita ok, potíže při těžší aktivitěIII. – značné omezení tělesné aktivity, potíže při základních činostechIV. – obtíže invalidizují, potíže v klidu

Asymptomatická dysfce LK – stav, kdy je objektivně prokázána porušená fce LK, ale nemocný je asymptomatický i bez léčby

Epidemiologie – výrazně roste s věkem, díky ↓ úmrtnosti na KV onemocnění- 1-2 % populace (do 60 let 0,8 %, mezi 60-69 2,3 %, nad 70 let 9,1 %), výskyt asymptomatické

systolické dysfce je asi stejný- riziko je 20 % (každý 5!)

KO – levostranné – nejčastěji IM, arteriální hypertenze, aortální cholemií vady, mitrální chlopenní vady, kardiomyopatie, mitrální stenóza, myxom levé síně

- může být dlouho asymptomatické pouze se ↑ plnícím tlakem nebo ↓ minutového výdeje v klidu i při zátěži

- ↓ srdeční výdej a zhoršení perfuze periferních tkání (selhání dopředu) → únava, fyzická nevýkonnost, pocení, nykturie, oligurie, při těžších formách kachexie, somnolence a změny chování

- ↑ tlaku a městnání krve před PK a plicní kongesce → námahou dušnost, paroxysmální noční dušnost (kardiální astma, úleva po posazení, spí s několika polštáři pod zády), plicní edém (tachypnoe, ortopnoe, úzkost, neklid, bledost, periferní vazokonstrikce, cyanóza, vykašlávání zpěněného sputa s krví pak růžové), chrůpky na plicích (bilaterální, inspirační, vlhké, nemění se po zakašlání, pohyb dle gravitace)

- kompenzační mechanismy → tachykardie (aktivace sympatiku, může chybět při léčbě β-blokátory, digoxinem, verapamilem a diltiazemem), cvalový rytmus, pulsus alterans, zvedavý úder hrotu

- primární onemocnění → stenokardie při ICHS/ aortální stenóze, poruchy srdečního rytmu, hypertenze, tyreotoxikóza

- pravostranné – nejčastěji levostranné srdeční selhání s postkapilární plicní hypertenzí, akutní infarkt PK, plicní choroby (CHOPN), plicní hypertenze jakékoli etio, vady chlopně trikuspidální a plicnice (v dospělosti velmi vzácné), obstrukce pravostranného atrioventrikulárního ústí a trikuspidální stenóza

- slabost a únava (při levostranném selhání, i hypokalémie z diuretické léčby), dyspepsie, nechutenství, pocit plnosti v epigastriu, zvracení, zácpa (městnání v v GITu), bolest v pravém podžebří při rozpětí jaterního pouzdra, nykturie,dušnost

- ↑ náplň krčních žil (fyziologicky ne víc než 2 cm nad sternokostální skloubení)- hepatojugulární reflux (zatlačení na ½ min v pravém podžebří)- hepatomegalie (i porucha jater. fcí), se subikterem, až kardiální cirhóza

17

Page 18: lf1.czlf1.cz/wp-content/uploads/228-vypracovane_otazky_interna... · Web viewprognoza – 50 % dosáhne kompletního uzdravení do 6 měsíců, 20 % má prodloužený průběh –

Vypracované otázky na INTERNU made by Edita Homolková, Monika Ambroziová, Adam Král, Petr Vermach a Vítek Křehnáč

- periferní otoky dle hydrostatického tlaku (v době objevení již retinace 3-5 l, přírůstek hmotnosti)

- ascites- trikuspidální insuficience při těžším a dlouhotrvajícím pravostranném selhání, kdy dilatace

pravé komory vede k realtivní insuficienci chlopněDg – symptomy srdečního selhání + objektivní průkaz systolické/ diastolické dysfce + příznivá odpověď na léčbu

- EKG – patologické (jakkoli, spíš poukazuje na etio), pokud je pořádku, je dg nepravděpodobná- rtg hrudníku – plicní kongesce, zmnožení plicní kresby (hl. v horních plicních polích, rozšíření hilů, alveolární/

intersticiální plicní edém)- velikost srdečního stínu, kardiotorakální index (max šířka srdce/ vnitřní šířka hrudníku, KTI

>0,5 je patologické), tekutina v pleurální dutině, aortální/ mitrální tvar srdce, plicní patologie- echokardiografie – nutné pro dg, objektivní průkaz dysfce LK (systol./ diastol.), většinou určí i etio

- hodnocení srdečních oddílů (systol. dysfce EF <50 %, lehká 40-50 %, střední 30-40 %, těžká <30 %)

- dopplerem lzd měři diastolická fce LK – poměr časně diastolické (E) a pozdně diastolické (A) rychlosti – norma >1 (E:A se ↓ při zhoršené relaxaci LK, ale také např. s ↑ věkem a ↑ s ↑ tlakem v LS

- při zhoršujícím se selhávání ↑ plnící tlak v LK ai tlak v LS a poměr E:A se ↑ do normálních hodnot = pseudonormalizace dopplerovské křivky (lze prokázat pak hypertrofie LK nebo dilatace LS…) (diastolická fce lze měřit přesně jen invazivně)

- dopplerovské zobrazení – diastolickou fci LK lze posoudit měřením pohybu mitrálního prstence (vrcholová rychlost v systole a v časné diastole a v pozdní diastole)

- BNP a NT-proBNP – natriuretické peptidy z kardiomyocytů srdečních komor při jejich ↑ napětí- normální hladiny u neléčeného člověka prakticky vylučují srdeční selhání

s pravděpodobností 90 %, prognostická hodnota- norma je BNP <100pg/ml, NT-proBNP <120 pg/ ml)

- renální fce (kreatinin, urea), iontogram (K, Na, Al), normocytární normochromní anémie

- náhrada chlopně - nutná doživotní antikoagulační léčba u mechanických chlopní

7.B ANKYLOZUJÍCÍ SPONDYLITIDA

- zánětlivé revmatické onemocnění klinicky charakterizované bolestmi v dolní části zad a později v celých zádech způsobené zánětem sakroiliakálních kloubů a dlouhých páteřních ligament

- rozeznáváme 3 formy onemocnění: Axiální – onemocnění páteřeRhizomelická – onemocnění páteře + kořenové kloubyPeriferní – onemocnění páteře + periferní artritida (asymetrická

oligoartritida)- patogeneze není zcela objasněna, u 95% nemocných přítomný antigen HLA B27, z exogenních vlivů se mohou

uplatnit střevní bakterie vyvolávající imunopatologické změny. KO - axiální příznaky - zánětlivá bolest dolních zad – plíživý začátek, po několika měsících přechod do chronicity

- často se objevuje ve 2. 1/2 noci, zlepšuje se po rozcvičení bývá spojena s ranní ztuhlostí

- bolest a ztuhlost krční a hrudní páteře - bolest na přední hrudní stěně- artritida ramenního a kyčelního kloubu

- Periferní artritida – často jako oligoartritida nesymetrická, přechodná a migratorní s predilekcí DK (často jako daktylitida – párkovitý prst). Může být i jako symetrická polyartritida připomínající RA

- Mimokloubní projevy - Akutní přední uveitida - Kardiální komplikace – aortitida, aortální insuficience, převodní poruchy - GIT manifestace – zánětlivé změny ve stěně (kolitida)- Plicní postižení – fibróza - Renální postižení – sekundární AA amyloidóza, IgA nefropatie

- osteoporóza obratlů - je metodou DEXA obtížně zjistitelná pro maskování snížené kostní denzity přítomností syndesmofytů

18

Page 19: lf1.czlf1.cz/wp-content/uploads/228-vypracovane_otazky_interna... · Web viewprognoza – 50 % dosáhne kompletního uzdravení do 6 měsíců, 20 % má prodloužený průběh –

Vypracované otázky na INTERNU made by Edita Homolková, Monika Ambroziová, Adam Král, Petr Vermach a Vítek Křehnáč

- Postihuje mladé muže, rychle se rozvíjí a je lokalizovaná v krční a bederní páteři (osteoporotické fraktury s neuro komplikacemi)

Dg - klinické vyšetření - vyšetření sakroiliakálních kloubů –Mennelův manévr - rozvíjení páteře ve třech rovinách –Schoberův test (min. 5cm)- vzdálenost okciput – kolmá stěna (max 2 cm)- expanze hrudníku (min 5 cm u starších pokles ale ne víc než pod 2.5cm)- postupné zvětšování hrudní kyfózy

- zobrazovací metody- RTG – základní kritérium AS je sakroiliitida (často bilat. + sym.) + charakteristické eroze, nepravidelnosti kloubní štěrbiny a osteosklerotické změny

- Finálním stádiem postižení v SI kloubu je ankylóza- Iniciální změnou jsou malé erozivní změny předních rohů obratlů

(zečtvercovatění obratle), dále vznik syndesmofytů způsobujících ankylózu (začátek na LS páteři a postupují výše).

- dle lokalizace změn rozlišujeme stádia: 1. Stádium – unilat. postižení SI kloubu2. Stádium – bilat. postižení SI kloubu3. Stádium – postižení LS páteře4. Stádium – postižení Th páteře 5. Stádium – postižení krční páteře

- Lab - Reaktanty akutní fáze většinou ↑ (aktivitu hodnotíme pomocí CRP)- Normocytární, normochromní anémie- Přítomnost antigenu HLA B27 - V synoviálním výpotku > 2000 buněk

Klasifikace - New Yorská klasifikace z roku 1984 (RTG nález sakroiliitidy + klinická kritéria)- Sakroiliitida na RTG/MRI + 1 specifický klinický příznak - PozitivitaHLA B27 + 2 specifické klinické příznaky

Th - Rehabilitace + fyzikální terapie (1 ročně i lázeňská léčba)- Nesteroidní antirevmatika - TNF blokující látky

8.A FIBRILACE SÍNÍ, JEJÍ KOMPLIKACE A LÉČBA

- patří mezi tachyarytmie- nejčastější klinicky významnou arytmií, prevalence 1 %- chaotická neuspořádaná elektrická aktivita v síních se ztrátou mechanické kontrakce- převod přes AV uzel je náhodný, RR intervaly nepravidelné- ↑ riziko tvorby trombů (levé ouško)Ptg – vzniká díky rychlým salvám síňových extrasystol vycházejících ze svalových vláken v plicních žilách a udržují

se díky degenerativním změnám ve svalovině síní- často u ICHS, srdečního selhání a chlopenních vad

Klinické formy – paroxysmální – záchvaty fibrilací spontánně začínají a končí (do 48 h)- perzistující – taková, která se ukončí antiarytmikem/ elektrickou kardioverzí- chronická, permanentní – nelze ukončit

KO – asymptomatický/ oligosymptomatický – častěji u starších pacientů- uspokojivá komorový frekvence při ↓ převodní kapacitě AV uzlu

- symptomatický – palpitace, mladší jedinci s rychlou komorovou odpovědí- celková slabost, presynkopy- trombembolie – do CNS, dolních končetin…- u pacientů s mitrální stenózou nebo špatnou funkci levé komory může zhoršit srdeční

selhávání; neefektivní kontrakce síní → městnání v síních- nepravidelný a nestejnoměrný puls

Dg – EKG – nepravidelné RR, chybí vlna P a místo ní fibrilační vlnky, QRS většinou štíhlé, frekvence dle kapacity AV uzlu

Th – kontrola frekvence – u perzistující fibrilace se bradykardizujícími léky udržuje komorová frekvence- volby u starých s minimálními symptomy- zlepšení plnění komor

19

Page 20: lf1.czlf1.cz/wp-content/uploads/228-vypracovane_otazky_interna... · Web viewprognoza – 50 % dosáhne kompletního uzdravení do 6 měsíců, 20 % má prodloužený průběh –

Vypracované otázky na INTERNU made by Edita Homolková, Monika Ambroziová, Adam Král, Petr Vermach a Vítek Křehnáč

- 60-80/ min v klidu a 90-115/min při střední zátěži- β-blokátory/ veramapril, dioxin při nedostatečném efektu- pokud farmaka nefungují – ablace AV uzlu a trvalá kardiostimulace

- kontrola rytmu – snaha udržet sinusový rytmus díky profylaktické antiarytmické léčby a opakovaných elektrických kardioverzí

- u mladých a symptomatických bez přidruženého organického postižení srdce- propafenon – u paroxysmů- sotalol- amiodaron

- prevence tromboembolií - u pacientů bez rizikových faktorů ASA, u ostatních heparin/ warfarin (INR 2-3)- katétrová ablace fibrilace síní – elektrická izolace plicních žil

8.B HYPERTYREÓZA

- syndrom ze ↑ sekrece a působení hormonů štítné žlázy- cca 1 % populace (všechny etio)Primární (periferní)

Graves—Basedowova choroba – nejčastější, ženy 30.-60 rok, autoimunní (stimulační protilátky proti TSH-R)- difúzní struma, endokrinní orbitopatie (infiltrace vaziva a svalů orbity →

exoftalmus, tyreoidální dermopatie (3 %, akumulace glykosaminoglykanů v podkoží pretibiálně),

Toxický adenom – autonomní nadprodukceToxická polynodózní struma – 1 i více autonomních adenomů v polynodózní struměAmiodaronem indukovaná toxická struma – antiarytmikum III. třídy (blokace K+ kanálu, prodloužení AP,

komorové i supraventrikulární arytmie)Tranzitorní destrukční hypertyreóza u tyreoiditid – uvolnění hormonů z poškozené tkáně

Sekundární Tyreotropin produkující adenom – přítomná/ i bez strumy, při makroadenomu příznaky z expanzivního chování

- ↑/ nesuprimované TSH, ↑ free-T4 (i T3)- Th – jako u primární hypertyreózy + neurochirurgické odstranění (reoperace/

radioterapie)Hypofyzární rezistence na T4 a T3

KO - ↑ metabolismu, hubnutí, teplá kůže, ↑ pocení, křehké nehty, ↑ motilita střeva, nauzea, zvracení, poruchy menstruačního cyklu, ↑ TF (nejčastěji sinusová tachykardie), arytmie (supraventrikulární extrasystoly, flutter síní, fibrilace síní), hyperkinetická cirkulace, anémie, granulocytóza, osteopenie/ osteoporóza, klidový třes, ↑ psychomotorické tempo, neklid, agitovanost, svalová slabost, snadná unavitelnost, zvýšený a třesoucí se hlas, Dalrymplův příznak (retrakce horního víčka), Graefoho příznak (zaostávání souhybu horního víčka při pohledu dolů), Kocherův příznak (zírání), hypokalemická periodická paralýza (10 % mužů, po fyzické zátěži/ p.o. cukry, ataky trvají hodiny až dny)

Dg – tyreostatika – blokace syntézy hormonů, iniciální léčbě (normalizace fce)- β blokátory – tlumení palpitací, TF…- tyreoidektomie – totální u G-B nemoci a polynodózní tox. strumě- radioaktivní 131I – při vysokém riziku chirurgického výkonu (normalizace do 2-6 měsíců)

Tyreotoxická krize – vystupňovaná tyreotoxikóza, hypermetabolismus, horečka až hyperpyrexie, tachykardie, arytmie, srdeční selhání, neklid, profúzní pocení, delirium až koma

- JIP, vodní, elektrolytová a acidobazická rovnováha, tyreostatika parenterálně, β-blokátory, glukokortikoidy

9.A MITRÁLNÍ STENÓZA A MITRÁLNÍ REGURGITACE

Steóza mitrálky- norma 4-6 cm2

- symptomatická <1,5 cm2 a kritická <1 cm2

Ptg - ↑ tlakový gradient LS – LK → ↑ tlak v LS → fibrilace síní, trombóza

20

Page 21: lf1.czlf1.cz/wp-content/uploads/228-vypracovane_otazky_interna... · Web viewprognoza – 50 % dosáhne kompletního uzdravení do 6 měsíců, 20 % má prodloužený průběh –

Vypracované otázky na INTERNU made by Edita Homolková, Monika Ambroziová, Adam Král, Petr Vermach a Vítek Křehnáč

→ plicní postkapilární hypertenze → plicní edém, dušnost→ zatížení PK → trikuspidalizace vady,

pravostranné selháníEtio – porevmatická – za 10tky let po revmat. horečce (Streptokok)

- postupné srůstání komisur nebo i cípů a kalcifikaci, LS hypertrofuje a časem dilatuje- v LS může být až 25 mmHg → exsudace do alveolů, dušnost- při rozvoji fibrilace síní klesá srdeční výdej až o 20 %- trikuspidalizace vady → ustoupí dušnost a rozvine se pravostranné městnání (otoky…)

- karcinoid, SLE, mukopolysacharidózyKO – první je námahová dušnost, doprovázená kašlem

- hemoptýza (překrvení bronchů, ruptura bronchiální žíly)- systémové embolizace z LS- infekční endokarditida

Dg – facies mitralis - začervenalé tváře s venektáziemi na nápadně bledém obličeji s cyanózou rtů- auskultace – diastolický šelest poslechovém místě mitrálky, otevírací zvuk daný napnutím závěsného

aparátu (mitrální 3zvuk)- ECHO – morfologie chlopně, kalcifikace, dilatace LS, jícnovou sondou průkaz trombu v levém oušku, tlakový

síňokomorový gradient- katetrizace – posouzení plicní hypertenze, nález na koronárkách před plánovaným kardiochirurgickým

zákrokem- EKG – P mitrale – rozšířená vlna P (>3 čtverečky), fibrilace síní, Sokolovův index > 10,5 mm (projev

hypertrofie pravé komory)- RTG - trojúhelníkový srdeční stín, známky plicní venostázy

Th – balónková valvuloplastika - přes venózní řečiště → PS → transseptálně → LS a tam se balónek v mitrálním ústí nafoukne a srostlé cípy roztrhne

- náhrada - mechanická protéza- farm – diuretika, antiarytmika pro udržení sinusového rytmu, antikoagulace

Insuficience mitrálkyEtio - primární – degenerativní – prolaps při myxomatózní degeneraci

- Marfanuv syn. - kalcifikace anulu

- pozánětlivá – porevmatická, SLE, sklerodermie- infekční endokarditida- kongenitální postižení

- sekundární – ischemická- dilatační, hypertrofická KMP

- postižení cípů – prolaps, revmatické postižení, infekční endokarditida, trauma, sklerodermie, SLE- postižení mirtárlního anulu – angulární kalcifikace, dilatace, absces- postižení závěsného aparátu – ruptuta/ elongace šlašinek, ruptura papilárního svalu (ICHS, trauma)- abnormality LK – ICHS, dilatační kardiomyopatie, hypertrofická kardiomyopatie

Ptg – insuf. → během systoly se krev vrací z komory do síně → objemové přetížení levé síně + levé komory (do té jde jak krev z oběhu, tak krev do síně regurgitovaná) → dilatace levé síně + excentrická hypertrofie a současná dilatace levé komory (velká síň i komora) → systolická dysfce, srdeční selhání , plicní hypertenze

KO – akutně – kardiální plicní edém (ruptura šlašinky)- chronická - velice dlouho asymptomatický průběh (zrádné), až později při dekompenzaci levé komory

obraz podobný mitrální stenózeDg - auskultace - holosystolický šelest nasedající na 1. ozvu s max. nad hrotem a s propagací do axily)

- při prolapsu chlopně bývá šelest slyšet mezosystolický - poklepově srdce zvětšeno, vlhké chrůpky- RTG - zvětšení stínu srdce doleva, možná kalcifikace v chlopni- EKG - P mitrale, fibrilace síní, hypertrofíe levé komory- ECHO – etiologie, dilatace a dysfce LK, určení typu chir. řešení

Th – farm – prevence endokarditidy- léčba fibrilace síní- léčba srdečního selhání – ACEI, sartany

21

Page 22: lf1.czlf1.cz/wp-content/uploads/228-vypracovane_otazky_interna... · Web viewprognoza – 50 % dosáhne kompletního uzdravení do 6 měsíců, 20 % má prodloužený průběh –

Vypracované otázky na INTERNU made by Edita Homolková, Monika Ambroziová, Adam Král, Petr Vermach a Vítek Křehnáč

- chir - valvuloplastika – preferována, implantace umělého prstence, resekce prolabujícího cípu, sešití prasklých šlašinek

9.B AKUTNÍ KOMPLIKACE DIABETU

Hypoglykemie- glc <3,8 mmol/l, při léčbě inzulinem a inzulinovými sekretagogy, fyzická zátěž, velké množství alkoholu,

inzulinom- 3,8 – 3,6 glukagon, 3,5 – 3,2 katecholaminy, 3,1 – 2,7 růstový hormon, 2,8 – 2,6 kortisolKO – autonomní (neurogenní) příznaky – tachykardie, palpitace, třes, nervozita, úzkost, strach, pocení, hlad,

parestezie- neuroglykopenické příznaky – porucha soustředění/ uvažování/ chování/ řeči, zmatenost, diplopie, křeče,

komaTh – sacharidy (slazené nápoje) při lehčí hypoglykemii

- injekční glukagon + i.v. glc (20 % do resuscitace, pak 5/10 % kontinuální infuze až do 5,5 mmol/l)Následky – Somogyiho fenomén – ↑ hodnoty glykemie objevující se jako následek předchozí hypoglykemie při

léčbě inzulinem (důsledkem ↑ odpovědi kontraregulačních hormonů na hypoglykemii)- hypokalémie- konvulze- iktus- ICHS

HyperglykémieDiabetické hyperosmolární hyperglykemické koma – vzácné, spíše u starších s DM II, pozvolný rozvoj, letalita

60%- ↑ gly → polyurie a žízeň, dehydratace, hypotenze, zmatenost, poruchy vědomí- díky relativnímu nedostatku inzulínu- bez Kussmaulova dýcháníLab – gly často >40 mmol/l, osmolalita plasmy >320 mmol/kg, Na+ ↑/↓/ norma, ↑ urea a kreatinin a

hematokrit (dehydratace), bikarbonát a baze v norměTh – JIP, rehydratace (1000 ml 0,9 % NaCl v první hodině), bolus inzulinu 0,1 – 0,15 UI/kg/hod (pokles

pozvolný o -4 mmol/hod), substituce K+ při <5 mmol/l- riziko trombembolie

Diabetická ketoacidóza – u DM I i II,- absolutní nedostatek inzulínu → ketogenezeKO – dehydratace (žízeň, suché sliznice, hypotenze, tachykardie), nauzea, zvracení, slabost, bolest

břicha (diabetická pseudoperitonitida), Kussmaulovo dýchání, komaLab – metabolická acidóza, hyperglykémie, hyperketonémie (>5 mmol/l), ↓ bikarbonátu, přebytek

bazí, může být ↑ amyláza, hypertriglyceridémie, hypercholesterolémie, možná leukocytózaTh – zajistit žilní vstup, EKG

- rehydratace – 5 -10 l/24 hod- 0,9 % (150 mmol/l) NaCl (rychlost dle kliniky)- při gly 15 -18 mmol/l vyměnit za 5 % glc, glykemii udržen na 10 mmol/l, ne rychle

→ osmotický edém mozku!- 0,45 % (75 mmol/l) NaCl jen při Na+ >155 mmol/l

- inzulin - úvodní bolus 6 – 10 UI, kontinuální infúze 2 – 6 UI/hod, úprava dle gly- přechod na s.c. inzulin + 1-2 hod ponechat infúzi

- K+ - 7,5 % KCl (cca 20 mmol/hod) hradí se 2/3 → zbytek 13,6 % KH2PO4 (1/3)- deplece při acidóze 300 – 1000 mmol

- ABR – úprava při pH < 7- CAVE: metabolická alkalóza (argininhydrochlorid)- odhad dávky NaHCO3: 0,3 * hmotnost * base deficit/ 3

Laktátová acidóza – díky tkáňové hypoxii (šok, srdeční selhání, hypoxemie, anémie, CO)/ porucha metabolismu laktátu (sepse, selhání jater, metformin)

KO – Kussmaulovo dýchání, nespecifické bolesti břicha, nauzea, zvracení, koma

22

Page 23: lf1.czlf1.cz/wp-content/uploads/228-vypracovane_otazky_interna... · Web viewprognoza – 50 % dosáhne kompletního uzdravení do 6 měsíců, 20 % má prodloužený průběh –

Vypracované otázky na INTERNU made by Edita Homolková, Monika Ambroziová, Adam Král, Petr Vermach a Vítek Křehnáč

Lab – pH < 7,2, laktát >5 mmol/l (norma 0,4 – 1,2), anion gap nad -18 mmol/lTh – JIP, ventilační podpora

- NaHCO3

10.A JATERNÍ CIRHÓZA

= difuzní proces se zánětlivými a nekrotickými změnami s následnou fibrózou a tvorbou regeneračních uzlůEtio – reakce na nekrózu

- alkohol, chronická hep B/ C, paracetamol, brucelóza, autoimunitní hepatitidy, vrozené metabolické choroby, cholestáza při biliární obstrukci primární/ sekundární, granulomatózní hep

KO - až 40 % je asymptomatická- dekompenzace = objevení příznaků poruchy fce bb (ikterus, otoky, krvácivé projevy), portální hypertenze

(ascites, encefalopatie, krvácení jícnových varixů, hepatorenální syndrom, spontánní bakteriální peritonitida)- Child-Pughova klasifikace – pro kvantifikaci dekompenzace

- stupeň encefalopatie, bilirubin (< 34, 34-51, >51 μmol/l), albumin (>35, 28-35, <28), protrombinový čas (1-3, 4-6, >6 s), INR (<1,7, 1,7-2,3, >2,3)

Th – dle vyvolávající příčiny- důležitá je dostatečná výživa (1-1,5 g/kg/den proteinů), vitamíny, transplantace jater

Alkoholická cirhózaEtio – dlouhodobý abusus, při excesivním pití už za 18 měsícůKO – při přechodu z alkoholické hepatitidy (steatotické změny, cholestáza, proliferace žlučových duktů) se objeví

anorexie, subfebrilie, ikterusDg – hepatomegalie, pavoučkovité névy, palmární erytém, krvácivé projevy na kůži, otoky

- ↑ aminotransferáz – AST 2x ↑ než ALT, ↑ GMT, později i zámky poruchy fce jater, ascitesTh – abstinence, multivitaminy, kys. listová, léčba komplikací

- transplantace po 6 měsících abstinencePostvirová cirhóza

- nález podobný jako u alkoholické, zvýšení transamináz nevýrazné- pro potvrzení přechodu chronické hep do cirhózy a prognózy je biopsie- léčba symptomatologická, řešením je transplantace

Biliární cirhóza- extra/ intrahepatální obstrukce s cholestázou a vznikem modulární fibrózyPrimární biliární cirhóza – chronické cholestatické, granulomatózní a destruktivní onemocnění interlobulárních

a septálních žlučovodů- autoimunní poškození intrahepatálních žlučovodůKO – častěji ženy středního věku, pruritus, často symptomy jiných autoimunitních

onemocnění- sucho v ústech, kostní změny (osteoporóza), ikterus, suchá kůže, xantelesmata

Dg – 2-3 x ↑ ALP často i GMT, ↑ transamináz, antimitochondriální protilátky (AMA), biopsie

Th – kys. ursodeoxycholová 15 mg/kg/den (↓ konc. tox. žluč. kys.), substituce vitamínů rozpustných v tucích

- někdy imunosuprese cyklosporinem, azathioprinem, prednisonem- cholecystramin na pruritus (plazmaferéza)- transplantace jater

Prognóza – progreduje do selhání do 10 letSekundární biliární cirhóza – u chronické (alespoň 6 měsíců) obstrukce (cholelithiáza, iatrogenní striktury,

chron. pankreatitida, biliární ca, ca hlavy pankreatu, cystická fibróza)KO – příznaky primárního onemocnění, často pruritus…Dg – dg základního onemocnění, ↑ bilirubin a ALP, hypercholesterolémie, deficit A/ D/

E/ KTh – dekomprese žluč. cest, cholecystramin, vitamíny

Primární sklerozující cholangoitida (PSC) – progresivní, často fatální, cholestáza- samostatně/ v 70 % společně s nespecifickými střevními záněty (i sklerodermie a

chronická pankreatitida), častěji cholangiokarcinom

23

Page 24: lf1.czlf1.cz/wp-content/uploads/228-vypracovane_otazky_interna... · Web viewprognoza – 50 % dosáhne kompletního uzdravení do 6 měsíců, 20 % má prodloužený průběh –

Vypracované otázky na INTERNU made by Edita Homolková, Monika Ambroziová, Adam Král, Petr Vermach a Vítek Křehnáč

- subepitelová fibróza uzavírá cesty, periduktální fibróza v játrechKO – únava, pruritus, bolest v pravém hypochondriu, cholestatická ikterus, zimice a

teploty, nechutenstvíDg – cholestáza spojená s UK/ CD, není dilatace cest na USG, ↑ obstrukční enzymy

(ALP, GMT, IgM, bilirubin)Th – překlenutí stenty, kys. ursodeoxycholová, hepatoprotektiva, vitamíny,

cefalosporin při cholangitidě, transplantace jaterVaskulární choroby vyvolávající postižení jater

- při městnání krve (spíš ale fibrotizace než cirhóza)Městnavé srdeční selhání a plicní hypertenze – poškození hl. centrilobulárních zónBudd-Chiariho syndrom – ztížený odtok z velkých/ malých jaterních žil

- často první projev protrombotického stavu (polycytaemia vera, poruchy koagulace, trombocytóza)

- akutní i chronické selhání jater s dominancí ascitu a portální hypertenze- dopplerSONO, kontrastní CT (zvětšený lobus caudatus)Th - TIPS

Veno-okluzivní nemoc – akutně po transplantaci kostní dřeněTrombóza portální žíly – při jaterní cirhóze, malignitě v hepatobiliární oblast, pankreatitidě, břišních

chirurgických výkonech, trombofilních stavech- dopplerSONO

Onemocnění jaterních žil – aneurysmata (ohrožení krvácením)

11.A CHRONICKÁ ISCHEMICKÁ CHOROBA SRDEČNÍ

- patogenenze viz ot. 5.AStav po IM

- bolesti na hrudi u těchto pacientů jsou brány závažněji- horší prognóza než dravá populace

Stabilní angina pectoris- opakované krátkodobí ischemie myokardu vznikající při námaze v důsledku hemodynamicky významné stenózy

věnčitých tepen (>60-70 %)KO - bolest na hrudi (stenokardie) při fyzické námaze, emočním stresu nebo chladu

- i dušnost bez stenokardií- bolest je svíravá/ tlaková, retrosternální, plošná, vyzařuje do levéno ramene (ale i čalist, krk, záda)- po zastavení se/ nitroglycerinu bolest odeznívá

Dg - anamnéza- klidové EKG – často beze změn, v aktivním záchvatu (deprese ST >1mm, inverze T, které mizí po ústupu

bolesti)- zátěžové EKG – bicyklový ergometr, pozitivní při angin. bolesti nebo EKG známky ischemie ¨

- deprese ST- zátěžová echokardiografie – pozorování porušené kontraktility myokardu po zátěži (bicykl, dobutamin)- perfuzní thaliová scintigrafie myokardu – Th 201 i.v.; vychytáváno pouze perfundovanými oblastmi,

snímáme gama kamerouTh – prognózu zlepšují ASA, statiny, ACEI a β-blokátory, léky symptomatické jsou blokátory Ca kanálů,

trimetazidin (ovlivňuje mtb myokardu) a ivabradin- sekundámí prevence - zákaz kouření, úprava hyperlipoproteinémie, DM a AHT, léčba obezity, odstranění

stresu, tělesný pohyb, dieta (↓ tuků a cholesterolu, dostatek vlákniny, ovoce, zeleniny, olivového oleje), při homocysteinémii kys. listová, vitamin B6 a B12 (snižují hladinu homocysteinu), střídmá konzumace alkoholu (červené víno), vitamin E

- antiagregancia - ASA - inhib. cyklooxygenázy tvořící tromboxan A2, 100 mg denně- clopidogrel 75 mg/ den

- β-blokátory - ↓ celkovou mortalitu i riziko IM a maligních arytmií, antianginózní léky první volby- nesmí se vysadit náhle- kardioselektivní metoprolol/ atenolol, neselektivní carvedilol (hl. u chron. srdeč. selhání)

- statiny - ↓ celkovou i KV mortalitu, u všech s ICHS, prevence progrese aterosklerózy

24

Page 25: lf1.czlf1.cz/wp-content/uploads/228-vypracovane_otazky_interna... · Web viewprognoza – 50 % dosáhne kompletního uzdravení do 6 měsíců, 20 % má prodloužený průběh –

Vypracované otázky na INTERNU made by Edita Homolková, Monika Ambroziová, Adam Král, Petr Vermach a Vítek Křehnáč

- artrovastatin, simvastatin- ACEI - ↓ celkovou i KV mortalitu u ICHS

- pokud po antianginózní léčbě přetrvává hypertenze, lepší u diabetiků- nitráty - ↓frekvenci záchvatů, ale neovlivní mortalitu ani morbiditu

- uvolňují NO, systémová venodilatace → ↓preload → ↓ metab. nároky myokardu, vzniká rezistence (nitroglycerin již po 10 h) → část dne nepodáváme, aby nevznikla tolerance

- vymizení stenokardií do 2 min po sublingválním podání- isosorbit monomononitrát – delší biolog. Poločas (12-16 h)- molsidomin – donor NO, nevzniká na něj tolerance

- blokátory Ca kanálů - blok L-typu Ca2+ kanálů vstupu Ca2+ do bb. → cévní vazodilatace, ↓ kontraktilita a frekvence myokardu

- amlodipin (NÚ: otoky DK)- trimetazidin – modulátor energetického mtb myokardu- ivabradin – ovlivňuje činnost SA uzle- invazivní terapie – PCI- karidiochir – když selže PCI, aortokoronární bypass (aa. mammaria int., štěpy z DK)

Variantní angina pectoris = Prinzmetalova, vazospastická- intermitentní ischemii myokardu ze spasmu koronárních tepen- při fyzické námaze, emoční zátěži, kokain- asi důsledek těžké/ atypické endoteliální dysfce- hl. ženy středního věkuDg – záchyt přechodných ST elevací při Holter EKG/ zátěžovém EKGTh – jako u klasické AP

- β-blokátory KONTRAINDIKOVÁNY !!!!Smíšená forma anginy pectoris

- fixní stenóza + spasmusMikrovaskulární angina pectoris (small vessel disease)

- klinické projevy AP a pozitivním zátěžovým testem a normálním koronarogramemSrdeční selhání v důsledku ICHS

11.B MONOKLONÁLNÍ GAMAPATIE A CHOROBNÉ STAVY SPOJENÉ S MG; AMYLOIDÓZA

= lymfoproliferativní choroby s tvorbou paraproteinu; proliferace klonu plazmatických bb., které produkují patologický monoklonální protein = M-komponenta s elektroforetickou pohyblivostí γ-globulinů (jakýkoli plnohodnotný Ig/ izolovaný těžký či lehký řetězec)

- 1 cm3 nádorové tkáně (cca 109 bb) → 5 g M-komponenty/l plasmy- lehké řetězce Ig prochází glomerulární membránou → Bence-Jonesova bílkovina- hyperviskózní syn. – poruchy zraku, neurologické příznaky, trombózy- Raynaudův syn. – při produkci kryoglobulinů- provázeno infekcemi1. Plazmocytom - viz. Ot. 60 A2. Monoklonál. gamapatie nejasného významu = MGUS (monoclonal gammopathies of undetermined

significance)-stavy, při nichž je paraprotein v séru do 30 g/l, v dřeni není víc než 10 % plazmat. bb. a není nalezeno orgánové

poškození CRAB (= hyperkalcémie, renální insuficience, anémie, kostní léze), nejsou známky lymfoproliferace- benigní monoklonální gamapatie, léčba není nutná, pravidelné sledování- riziko rozvoje mnohočetného myelomu

3. Waldenströmova makroglobulinémie - lymfoproliferativní onem. vyššího věku, podobá se plazmocytomu (tvorba paraproteinu) i CLL- hepatosplenomegalie, lymfadenopatie- nebývá postižen kostní metabolismus- M-komponenta je tvořena IgM (nebývá proteinurie), část může být kryoglobulinyKO - únava, slabost, mírná adenomegalie a hepatosplenomegalie

- v závislosti na produkci a konc. IgM (makroglobulin) → viskózní syndrom → porucha zraku při dilataci retinálních cév, neuropatie, porucha sluchu, asi 10 % tvoří kryoglobuliny (Raynaudův fenomén)

25

Page 26: lf1.czlf1.cz/wp-content/uploads/228-vypracovane_otazky_interna... · Web viewprognoza – 50 % dosáhne kompletního uzdravení do 6 měsíců, 20 % má prodloužený průběh –

Vypracované otázky na INTERNU made by Edita Homolková, Monika Ambroziová, Adam Král, Petr Vermach a Vítek Křehnáč

Dg - ↑FW, imunoELFO = paraprotein IgMTh - chlorambucil, při hyperviskozitě - plazmaferéza, rituximab

4. Nemoci těžkých řetězců (HCD)- paraprotein z těžkých řetězců, přítomen v plasmě i moči- může provázet lymfom i jiné lymfoproliferativní onemocněníγ HDC – postižení hlavně uzlin, Waldayerův okruh, dřeň, játra, slezinaα HDC – při lymfoplazmocytoidním procesu tenkého střeva, mezi 20. – 30- rokem

- etio u chron. střevní infekce (Campylobacter pylori)μ HDC – spíše hepatomegalie bez lymfadenopatie

Amyloidóza - nepatří mezi nádorová onem., ale její primární forma AL (light chain) často provází plazmocytom- etio?. porucha degradace proteinů s jejich systémovým nebo lokálním ukládáním ve stěnách malých

cév a extracelulárním prostoru orgánů5. Primární amyloidóza (AL)

- bez zjevné příčiny, fibrilární složka = lehké řetězce Ig, progresivní rázKO - slabost, hubnutí, parestezie, závratě, kardiální selhání (amyloidóza srdce)

- hepatosplenomegalie, adenomegalie, makroglosie, kožní purpura, postižení GITu amyloidem → průjmy, malabsorpční sy, hypotenze - renální selhání (nefrotický syndrom), sy karpálního tunelu, neuropatie (parestesie), postižení srdce (echo, ↑ natriuretický peptid)

Lab – proteinurie (Bence-Jonesova bílkovina), prim. amyloidóza AL často předchází plazmocytomu- u většiny v séru/ moči paraprotein

Dg - biopsie kůže, jazyka... konžská červeň, fluorescenční barvení - thioflavin; odlišení prim. a sek. formy A pomocí KMnO4 (fibrily amyloidu A ztrácejí po inkubaci s KMnO4 vázat konžskou červeň)

Th - málo účinná, zkouší se etanercept (inhibitor TNFα)- prognóza závažná, vede k selhání postiž. orgánu (přežití 2-5 roky)Sekundární amyloidóza (AA) - na podkladě jiného chron. zánětlivého onem. (osteomyelitida, RA, TBC, Chronova

choroba, nádory…)- KO - proteinurie, nefrotický sy, městnavá srd. slabost, malabsorpce, poruchy motility, kožní léze,

neuropatie, koagulopatie, v anamnéze =chron. hnisavé onem. (bronchiektázie, TBC, RA)- Th - protizánětlivá léčba zákl. onem., imunosuprese, inhibitory THFα (infliximab, etanercept)

12.A BRONCHOGENNÍ KARCINOM

-označení pro nádory průdušek i nádory v plicním parenchymu- nejčastější ca u mužů- rozdělení podle morfologie 4 typy/ biologického chování 2 typyMalobuněčný karcinom - rostou rychle, brzy vzdálené metastázy, chemo- a radioterapieNemalobuněčné karcinomy - spinocelulární ca

- adenokarcinom - velkobuněčný karcinom

Etio - kouření (20x vyšší riziko), profesionální expozice (azbest), expozice škodlivin z prostředí (222Rn), genetika - působením karcinogenů vznikají nejprve metaplastické a dysplastické změny, poté carcinoma in situ →

invazivně rostoucí karcinomKO - časné symptomy - teplota, kašel, bolest na hrudi, hemoptýza

- pozdní manifestace - nespecifické projevy – hubnutí, nauzea, zvracení- intratorakální propagace – Hornerův syndrom, sy komprese HDŽ, dysfagie, dysfonie,

pleuritidy, iritace pl. brachialis- extratorakální propagace – lymfatické uzliny, játra, nadledviny, kosti- paraneoplastické syndromy

Dg – fyzikální nález na plicích v drtivé většině normální, někdy oslabené dýchání a zkrácený poklep- mohou být zvětšené nadklíčkové uzliny a zvětšená a hrbolatá játra- PET – zobrazení nádarů a etastáz- kostní scintigrafie a CT mozku u malobuň. ca- biopsie (bronchoskopie + další metody – laváž, punkce l.u.)- u nádoru stanovujeme: typing, grading, staging- nádorové markery – CEA, TPA, SCC, NSE

26

Page 27: lf1.czlf1.cz/wp-content/uploads/228-vypracovane_otazky_interna... · Web viewprognoza – 50 % dosáhne kompletního uzdravení do 6 měsíců, 20 % má prodloužený průběh –

Vypracované otázky na INTERNU made by Edita Homolková, Monika Ambroziová, Adam Král, Petr Vermach a Vítek Křehnáč

- růstové faktory nádoru – TGF α, IL-1- staging -Stádium I – T1N0M0, T2N0M0

- Stádium II – T1N1M0, T2N1M0- Stádium IIIa – T3N0M0, T1-3N2M0- Stádium IIIb – T4N1-3M0, T1-4N3M0 (od IIIb inoperabilní)- Stádium IV – T1-3N1-3M1

Th – malobuň. ca – u metastáz jen paliativně- radioterapie – hlavně u limitovaného stádia

- preventivní ozáření mozku- chemoterapie – cisplatina a etoposid

- nemalobuň. ca – dle klinického stádia- chirurgické odstranění u malého bez metastáz- chir + adjuvantní chemoterapie- chemoterapie + radioterapie

- biologická léčba – protilátky proti VEGRF, inhibitory tyrosinkinázy

12.B DŘEŇOVÉ ÚTLUMY

= anémie z poruchy tvorby prekurzorů erytrocytů = dřeňové útlumy: aplastická anémiečistá aplázie červené krevní řadyanémie v rámci MDS (viz. tam)paroxyzmální noční hemoglobinurie

→ abnormální rce IS na noxu poškozující kmenovou krvetvornou b.Aplastická anémie (AA) - dřeňový útlum

- poškození kmenové bb vedoucí k jejímu zániku Etio – infekce (hepatitidy, EBV, CMV, parvovirus B19, HIV), ionizující záření (celotělová expozice 1,3-2,4 Gy aplazie

je reverzibilní, při ≥ 2,5 Gy nevratné poškození), léky (chloramfenikol, přípravky zlata, nesteroidní antirevmatika, sulfonamidy), chem. látky (benzen, pesticidy), idiopatická 40-70%

- imunitní reakce poškozující krvetvornou b. - zánik cestou indukce apoptózy- poškození hematopoet. stromatu

KO - SAA (severe aplastic anemia) = akutně vzniklý dřeň. útlum,infekční a krvácivé projevy na podkladě agranulocytózy a trombocytopenie (septické horečky, nekrotizující stomatitis, kožní a slizniční krvácení, epistaxe, gynek. krvácení)

- Chronická forma - horečky, známky krvácení, výraznější projevy anémie Lab - těžká AA- buněčnost dřeně <30 % normy + 2 z násl. kritérií v perf. krvi: retikulocyty <0,1%

neutrofilní segmenty <0,5.109/ltrombocyty <20.109/l

- středně těžká AA - pancytopenie nesplňující kritéria pro těžkou apl. anémii- anémie makrocytární

Th – alogenní transplantace krvetvorných bb. - u mladších pac. s těžkou formou (medián přežití 3-6 měsíců)- kombinovaná imunosuprese –u pacientů neindikovaných k transplantaci

- antithymocytární globulin 4 dny + methylprednisolon 14 dnů a následně cyclosporin A 6 měsíců, podpůrná léčba (transfúze ery a trombocytů)

Čistá aplazie červené řady = Pure red cell aplasia (PRCA)- incidence 4,23 případů/ 10 000 pacientů léčených ESA (erythropoesis-stimulating agent)- vrozená (Diamond-Blackfanův syndrom)- idiopatická- asociace s: infekce (parvovirus B19, HIV, EBV), systémové poruchy imunity (SLE, revmatoidní artritida),

leukemie, lymfom, thymom, léky (protilátky proti rekombinantnímu EPO, azathioprine)KO - těžká normocytární anemie (Hb méně než 120 g/l u žen a 135 g/l u mužů), retikulocytopenie (< 0,1% nebo

40*109/l), absence erytroblastů v jinak normálně celulární kostní dřeni (norma megakaryocytů a myelocytů).

- anti-EPO protilátky – neutralizační protilátky zkříženě reagují s epoetinem alfa, beta, s dabapoetinem a s endogenním erytropoetinem

27

Page 28: lf1.czlf1.cz/wp-content/uploads/228-vypracovane_otazky_interna... · Web viewprognoza – 50 % dosáhne kompletního uzdravení do 6 měsíců, 20 % má prodloužený průběh –

Vypracované otázky na INTERNU made by Edita Homolková, Monika Ambroziová, Adam Král, Petr Vermach a Vítek Křehnáč

- protilátky často IgG1 nebo IgG4, hlavně u pacientů léčených rekombinantním EPO (může být i primární tvorba protilátek)

Th - Transfúze – erymasa akutně- Imunoglobulin – při asociaci s infekcí parvovirem B19 (spíše při imunodeficienci – řešení chronické infekce,

akutní infekce B19 způsobuje aplastickou krizi, která se sama upraví)- Cyclosporin A + kortikosteroidy – nejúčinnější (CyA: 2,5 – 3 mg/kg 2 krát denně → 150 – 250 ng/ml za 3 až 4

měsíce)- relaps za 1,5 – 40 měsíců

- Cyklofosfamid – při refrakci k CyA, hlavně při asociaci s LGL (large granular lymfocytes = velký granulární lymfocyt, funkčně odpovídající NK buňce), 50 mg/den a pak 20 – 30 mg/den

- Anti-thymocytární globulin – 15 mg/den- Hematide – agonista EPO receptoru, není zkřížená reakce s protilátkami proti rekombinantnímu EPO

Parvovirus B19 – DNA virus (1 vlákno), odolný- vazba na hemaglutinin – P antigen (sys. krevních skupin)- přenos kapénkami, krevními deriváty, vertikálně z matky na dítě- asymptomatická infekce/ erythrema infectiosum (pátá nemoc)/ atritidy/ hydrops fetalis (hl. 2

trimestr)/ PRCA/ leukopenie/ trombocytopenie…Paroxysmální noční hemoglobinurie (PNH)

- získaná mutace PIG-A genu kódující tvorbu glykosyl-fosfatidyl-inositolu (GPI) → váže některé antigeny k buněčné membráně

- deficit membránových bílkovin, kt. jsou poutány GPI (CD59 a CD55, inhibitory terminálního aktivovaného komplexu komplementu) na povrchu ery → intravaskulární hemolýza s hemoglobinurií + současná aktivace koagul. systému → ↑ incidence žilních trombóz (portální, mezenterické, plicní a CNS řečiště)

- uvolněný Hb působí konzumpci NO → vazokonstrikce v renálním řečišti, rozvoj plicní hypertenzeKO - hemoglobinurie s maximem ráno (stresové podněty, infekce, zátěž, menses)

- bolesti břicha, pocit tlaku na prsou (protrahovaná vazokonstrikce), anemické příznaky, splenomegalie (subklinická trombóza v portálním řečišti)

- závažné komplikace - trombóza jater. žil (Buddův-Chiariho sy = hepatomegalie, porucha jater fcí, ascites)Lab – cytopenie, ↑ přímý i nepřímý bilirubin a LD

- průkaz deficitu CD 59 a CD55 antigenů na povrchu ery a antigenu průtokovou cytometrií- Hamův test = hemolýza ery nemocného aktivovaným čerstvým sérem (obs. komplement) v kyselém pH

Th - transfúze promytých, deleukotizovaných ery (bez komplementu)- kortikoidy + antikoagulační profilaxe- hlídat Fe (moc při transfúzích, ale i ztráty močí)- transplantace u mladších, protilátky proti C5 komplementu

13.A PLICNÍ EMBOLIE

- 2.- 3. nejčastější (potenciálně smrtelné) KV onemocnění po ICHS, nárůst incidence s věkem (↑ rizik. fakt.) - celková mortalita 10 - 20 %, i u diagnostikovaných a léčených 2 - 8%, klinicky němá až v 70% - postoperačně TEN u 20-50% (bez profylaxe) = uzávěr plicnice embolem (trombus, tuk, vzduch (300 ml), plodová voda, nádorové hmoty), nejčastěji z povodí v.

cava inf. (v. femoralis, pánevní žíly, lýtkové žíly), vzácně z v. cava sup. nebo z pravého srdce (kabel kardiostimulátoru), nejčastěji je postižen pravý dolní lalok

Predisponující faktory - vrozené trombofilní stavy - nedost. ATIII, PC, PS, APC mutace V.f., mutace genu pro protrombin, hyperhomocysteinémie, ↑ f.VIII, dysfibrinogenémie,…

- získané trombofilie - malignity, antifosf. Syn., autoimunní on., polycytémie, trombocytóza, HIT,….

- stáza - imobilizace, varixy, žilní insuf,….- poranění žilní stěny - katétry, operace,…- věk, těhotenství-horm. léčba, obezita, CHSS, respir.insuf., traumata,…

Etio - hluboká žilní trombóza, nejčastěji embolii vyvolá ranní vstávání, ↑ nitrobřišního tlaku, tělesná námahaDle hemodynamického dopadu – akutní malá/ akutní masivní/ akutní submasivní/ chronická tromboembolická

plicní hypertenze

28

Page 29: lf1.czlf1.cz/wp-content/uploads/228-vypracovane_otazky_interna... · Web viewprognoza – 50 % dosáhne kompletního uzdravení do 6 měsíců, 20 % má prodloužený průběh –

Vypracované otázky na INTERNU made by Edita Homolková, Monika Ambroziová, Adam Král, Petr Vermach a Vítek Křehnáč

Markery rizika – klinické (šok, hypotenze), průkaz dysfce PK (ECHO, BNP), poškození myokardu (troponin)Ptg - obstrukce plicnice vyvolá náhlé ↑ aftedoadu, což vede k dilataci pravé komory, ↓ srdečního výdeje a

hypotenzi (= cor pulmonale acutum), následuje hypoxie z přesunu proudění krve v plicích přes AV zkraty, uvolnění mediátorů z trombocytů (tromboxan) vyvolá další vazokonstrikci a tím ještě více ↑ afterload

KO - dušnost, tachypnoe, bolest na hrudi (tlaková),hypoventilace, hypoxie, hypokapnie, tachykardie, kašel, strach- akutní malá – asymptomatická, nově vzniklá námahová dušnost, tachypnoe, tachykardie, bolest vázaná na

respiraci- akutní masivní – hemodynamická nestabilita díky tlakovému přetížení PK → dilatace a selhání = šokový stav

se systolickým TK <90 mm Hg, klidová dušnost, centrální i periferní cyanóza- akutní submisivní – po opakované embolizaci, postupně narůstající dušnost- chronická tromboembolická plicní hypertenze – střední tlak v plicnici >25 mm Hg

- opakovaná malá embolizace s nedostatečnou fibrinolytickou aktivitou organismuDg - D-dimery – fibrin-degradační produkt, vyloučení plicní embolie při neg. nálezu

- troponin – senzitivní marker poškození myokardu při hemodynamicky významné embolie se zatížením PK- perfúzní a ventilační scintigrafie plic – pokud je výpadek při perfúzním scinty a zároveň ventilačním- CT – metoda volby, s kontrastní látkou do periferní žíly- angiografie plic – nástřik kontrastní látky do plicnice, umožňuje endovaskulární léčbu při neúčinnosti

antikoagulace- ECHO – hlavně zhodnocení velikosti PK a stanovení tlaku v plicnici

- dilatace PK a kmene plicnice, dysfce PK, přesun mezikomorového septa doleva, otevřené foramen ovale…

- EKG – S kmiit v I svodu, Q kmit ve III svodu s inverzí vlny T, negativní vlny T ve V1 – V3- rtg – atelektáza, vysoká stav bránice, pleurální výpotek, infiltrace plicní tkáně

Th - malá a střední embolie – antikoagulační (heparin LMWH) + warfarin (na 6 měsíců min.) + oxygenoterapie- masivní pl. embolizace – trombolitika tPA (altepláza) nebo streptokináza a urokináza

- podpůrná léčba: i.v. noradrenalin,/ dopamin, kylík (i intubace a mechanická ventilace)

- indikace k trombolýze – kardiogenní šok/ hypotenze, větčí PE při snížené kardiopulmonální rezervě, průkaz trombu v pravém srdci, foramen ovale apertum, výrazná neustupující hypoxémie, submasivní PE s dysfcí PK a pozitivitou troponinů

Prevence – cvičení, chůze, bandáže, hydratace, intermitentní pneumatické komprese, rehabilitace- farmaka – LMWH s.c.

- digabatran – inhibitor protrombinu- fondaparinux

- kavální filtr

13.B PORUCHY METABOLISMU DRASLÍKU

Hyperkalemie- mírná 5,4-6 mmol/l, těžká >6,5 mmol/l- nutno vyloučit hemolýzuEtio – chronické renální selhání, oligoanurie

- léky: K štřící diuretika, ACEI, NSA, heparin, cyklosporin, betablokátory- katabolismus – acidóza, deficit inzulinu (diabetická ketoacidóza), rabdomyolýzy, cvičení- Addisonova ch.- hypoaldosteronismus- vzácné tubulární syndromy

EKG – hrotnatá vlana T, vymizelá P vlna, prodloužení PQ, rozšířený QRS, komorová arytmie- důležité pro diagnostiku je sledování vylučování kalia močí:

a) Při ↑ přívodu draslíku nebo velikém katabolizmu stoupá koncentrace kalia v moči nad 100 mmol/l b) Při renálním selhání dochází k retenci kalia v organismu a ↓ vylučování – koncentrace kalia v moči

pod 30 mmol/lTh - podání 10 ml kalciumglukonátu i.v (působí rychle a krátce, antagonizme membránové účinky K+)

- podání 50 – 100 ml NaHCO3 ve formě 0.5 M roztoku při acidóze- podání infuze glukóza + inzulin (podmíní vstup K do bb., účinek za 15 min a trvá po dobu podávání)

29

Page 30: lf1.czlf1.cz/wp-content/uploads/228-vypracovane_otazky_interna... · Web viewprognoza – 50 % dosáhne kompletního uzdravení do 6 měsíců, 20 % má prodloužený průběh –

Vypracované otázky na INTERNU made by Edita Homolková, Monika Ambroziová, Adam Král, Petr Vermach a Vítek Křehnáč

- kličkové diuretikum (furosemid) u převoněných a se zachovalou diurézou- podání 15 – 30 g iontoměniče (kalcium-/ natriumoikystyren v malém klyzmatu (účinek během desítek min a

přetrvává hodiny)- hemodialýza nebo peritoneální dialýza (rychlý účinek, u anurie)- přerušení přívodu kalia

Hypokalemie- mírná 3-3,5 mmol/l, těžká <2,5 mmol/l- nejčastěji při léčbě diuretikyNedostatečný příjem – malnutrice či zvýšený katabolismus, hladovění, nesprávná dieta, při parenterální výživě

(při dobré funkci ledvin je snížený odpad kalia v moči)Nadměrné ztráty do střeva – průjem, laxativa, zvracení, gastrinom, VIPom, ileostomie, střevní píštělPřesku K+ z ECT do bb. - ↑ sympatická aktivita (stres), β-sympatomimetika, inzulin, kofein, alkalózaPrimární hyperaldosteronismusSekundární hyperaldosteronismus – srdeční selnání, ascitická jaterní cirhózaZtráty ledvinami – diuretika, tubulární syndromy, tubulotoxicé léky, glukokortikoidyKO – únava, zácpa, slabost, snížený svalový tonus

- EKG - nízká T, vlna U, prodloužený QT, extrasystoly, sklon k rozvíjení toxických účinků digitalisu- ↓koncentrační schopnosti ledvin, polyurii a deficitu ECT → renální tubulární insuficience a retence

dusíkatých katabolitů- dochází ke ztrátám H+ → rozvoj alkalózy

Th – mírná hypok – substituce KCl p.o. (50-150 mmol/den, KCl acidifikuje - kontrolovat), podání kalium šetřících diuretik

- těžká hypok – monitorace EKG, i.v. KCl (20-40 mmol v 1 l fyziolog. Roztoku rychlostí 10-20 mmol/ h

14.A ZÁNĚTLIVÁ ONEMOCNĚNÍ PLIC – PNEUMONIE, PLICNÍ ABSCES

Pneumonie- akutní zánět v oblasti respiračních bronchiolů, alveolárních struktur a plicního intersticia- klinická definice: přítomnost čerstvého infiltrátu na RTG + 2 známky respirační infekce (kašel, dušnost, bolest

na hrudi, poslechový nález)Etio – St. Pneumoniae, H. influenzae, Cl. pneumonie, Legionella sp., G- , chřipka, St. aureus…Klasifikace I. - dle etiologie - infekční – bakteriální x virové x parazitární x mykotické

- neinfekční – aspirační x inhalační x hypersenzitivní x alergické- dle RTG obrazu – alární x lobární x lobulární x intersticiální- dle epidemiologie – CAP = komunitní, HAP = nozokomiální, VAP = ventilátorová (díky

mikroaspiraci), imunokompromitovaných, v ústavech sociální péčeKlasifikace II. - Katarální - bronchopneumonie, lobulární pneumonie

- Krupózní - lobární nebo alární pneumonie- Atypická - virové, mykoplazmové,chlamydiové, pneumocystové- Neinfekční - aspirační, inhalační, postradiační, hypersensitivní

Dle závažnosti – lehká – bez vážných příznaků a přidružených onem., lze léčit empiricky a ambulantně- středně těžká - ↑ teplota, mírná dušnost, schvácenost, vyšší věk, přidružená onemocnění

- hospitalizace krátkodobá, možné ambulantní doléčení dle aktuálního stavu- atb urychleně a pak popř změnit

- těžká – alterace základ. život. fcí, i.v. atb v nemocnici (JIP), KO – kašel – z počátku suchý, pak produktivní, sputum purulentní, reazvé/ drobná příměs krve

- dušnost, tachypnoe, horečka nad 38 °C, poty, zimnice, bolesti na hrudníku- schvácenost, bolesti hlavy, kloubů, svalů…, dyspepsie, splenomegalie, anémie- začíná pozvolna, trvá dlouho - komplikace empyémem, abscesem, sepsí

Dg – anamnéza (cestování?, inkubační doba…)- fyzikálně - lokalizovaný zkrácený polep, trubicovité/ sklípkové dýchání s přízvučnými vlhkými chrůpky/

krepitus- u výpotku oslabené dýchání a bubínkový poklep

30

Page 31: lf1.czlf1.cz/wp-content/uploads/228-vypracovane_otazky_interna... · Web viewprognoza – 50 % dosáhne kompletního uzdravení do 6 měsíců, 20 % má prodloužený průběh –

Vypracované otázky na INTERNU made by Edita Homolková, Monika Ambroziová, Adam Král, Petr Vermach a Vítek Křehnáč

- lab – CRP, prokalcitonin, leukocytóza >10*109/ l nebo >15 % nesegmentovaných leukocytů, pH, krevní plyny, natremie, urea, kreatinin, gly, albumin…

- rtg – zadopřední i boční projekce!, alární/ lobární/ segmentární, bronchopneumonie u nehomogenního infiltrátu, retikulonodulace u intersticiální

- bronchoskopie – při podezření na jíné onemocnění, k odběrům- mikrobiologie – sputum, hemokultura, sérologie, antigeny v moči

Th – atb racionálně dle epidemiologické situace, mělo by zabrat za 2-3 dny- atb pronikající do plicní tkáně – makrolidy, fluorochinolony (norfloxacin)- lehká – pneumokoky a hemofily - aminopeniciliny (ampicilin), cefalosporin II (cefuroxim)

- i pro atypické patogeny - makrolidy (klaritromycin, azitromycin), tetracykliny (doxycyklin), fluorochinolony (ofloxacin)

- středně těžká – na začátek se doporučuje i.v. a pak p.o.- kombinace makrolidy s aminopeniciliny/ cefalosporin II a III, monoterapii fluorochinolony

- těžká – i.v. kombinace aminopeniciliny s inhibitory β-laktamáz s makrolidy a s aminoglykosid- u nozokomiální dle oddělení- podpůrná - mukolytika, antitusika, analgetika, antipyretika, inhalační terapie, rehydratace, osygenoreapie,

LMWH…Plicní absces

- lokalizovaný hnisavý proces s nekrózou plicní tkáně (velikost dutiny >2 cm, jinak spíš abscedující pneumonie s mnohačetnými nekrotickými lézemi)

Etio – aspirace patogena z orofaryngu/ žaludku, komplikace pneumonie, projev septikémie, za bronchiální obstrukcí (nádor, cizí těleso), v plicním infarktu po embolizaci, u bronchiektázií

- často imunodeficity, dlouhodobě ležící, špatná ústní hygiena, narkomaniKO – febrilie, produktivní kašel s hnisavým sputem (drobná hemoptýza), může vykašlat velké množství

purulentního obsahu při vzniku komunikace abscesu s průduškou- empyém při komunikaci s pleurální dutinou, hemoptýza při nahlodání cévy

Dg – fyzikálně – chrůpky, poklepové ztemění a oslabené dýchání při větším procesu- rtg – homogenní zastínění → vznik dutinky s hladinkou, pleurální výpotek častý- lab – ↑ CRP, leukocytóza..

Th – vysoká dávka širokospektrých atb – anaeroby! → klindamycin/ penicilin s metronidazolem- drenáž, pokud vznikne chronický absces, resekuje se

14.B ANEMIE HEMOLYTICKÉ, VROZENÉ

1. Porucha struktury ery membrány:Dědičná sférocytóza (AD)- nedostatečná tvorba některého ze strukturálních proteinů skeletu ery membrány (α, β řetězec spektrinu či

ankyrin) → oslabení kontaktu mezi bílkovinným skeletem a lipidovou dvojvrstvou membrány → zmenšení povrchu ery, vzniká kulatá krvinka = sférocyt

- ↓ deformovatelnost, ↑rigidita, ↑ propustnost pro Na+ → zánik ve slezinných sinusoidech (extravask. hemolýza)

KO - mírná až střední anémie, splenomegalie, ikterus, komplikací je hemolytická krize (při infekci-prohloubení zákl. příznaků + bolesti břicha), vážnější je aplastická krize (prudká anemizace, často po virózách); často cholelithiáza

Dg – normocytární, normochromní anemie, ↑ retikulocyty (až 7-10 %), sférocyty- ↑ přímý i nepřímý bilirubin, Pink test = hodnotí stupeň hemolýzy, vystavení ery kritické konc. glycerolového

činidla - nespecif. testTh – splenektomie (u dětí aspoň parciální při opakování krizí, od 15 u všech s klinickými příznaky)

- přeočkovat proti pneumokokům, meningokokům, záškrtu a tetanu!Dědičná eliptocytóza- deficit cytoskeletu- > 25 % eliptocytů v periferní krvi- klinika a léčba podobně jako u sférocytózy

2. Porucha enzymatické výbavy eryDeficit pyruvátkinázy (AR)

31

Page 32: lf1.czlf1.cz/wp-content/uploads/228-vypracovane_otazky_interna... · Web viewprognoza – 50 % dosáhne kompletního uzdravení do 6 měsíců, 20 % má prodloužený průběh –

Vypracované otázky na INTERNU made by Edita Homolková, Monika Ambroziová, Adam Král, Petr Vermach a Vítek Křehnáč

- nejč. porucha postihující enzymy anaerobní glykolýzy (90 % energie pro ery) → porucha plasticity a deformability, extravask. hemolýza

KO - homozygoti (těžká hemolyt. anémie), heterozygoti (chron. hemolyt. anémie, ikterus, hepatosplenomegalie)- normochromní, normo až makrocytární, retikulocytóza, ↑ bilirubin

DG - průkaz ↓ aktivity pyruvátkinázyTh - splenektomie, převody ery a trvalá chelatační léčba - zabrání hemosiderózeDeficit G-6-PD-nejčastější porucha enzymu pentosového cyklu (produkce NADPH) - aerobní glykolýza (10 % E pro ery)- význam pro udržení sulfhydrylových sk. Hb, enzymů a membr. proteinů ery v redukovaném (funkčním stavu)- pokles G6PD pod 2 % → denaturace Hb za přítomnosti O2 a hemolýza.KO - zhoršuje se během infekce, oxidační l. (akut. hemolyt. anémii s intravas. hemolýzou, ikterus,

hemoglobinurie, bolesti hlavy a břicha, teplota- favismus = přecitlivělost na sviňský bob (Faba vulgaris) - obs. divicin, kt. oxiduje glutathion, chinoliny - ox.

úč.Dg - průkaz ↓aktivity G6PD, Heinzova tělíska v období hemolýzy (po inkubaci ery s oxid. činidlem)Th - prevence před oxid. činidly, prevence infekce

- u akut. Hemolýzy hydratace, forsírovaná diuréza, transfúze ery- transplantace krve

3. Porucha tvorby HbBeta-thalasémie (AR)- defektní syntéza β-globinového řetězce Hb (2 alely), Středomoří- porucha tvorby β řetězce → zvýšení syntézy řetězců γ a δ + nadbytku α → ↓ rozpustnost, poškození membrány

erytroidních bb → rozpad již ve dřeni = neefektivní erytropoesaKO - kompenzační hyperplazie erytropoézy → usurace kortikalis, deformity, fraktury kosti, extramedulární

erytropoéza; variabilní-dle exprese genu- thalassemia minima - bez anémie a klin. příznaků- thalassemia minor - lehká anémie, splenomegalie, heterozygoti β+- thalassemia intermedia - středně těžká anémie (nutné transfúze), splenomegalie, přetížení Fe- thalassemia major - homozygoti β0, mikrocytární anémie, hepatosplenomegalie, retardace růstu, deformity

skeletu, nutné intenzivní transfúze již u dětíLab – major: těžká mikrocyt. hypochromní anémie, střední ↑ retikulocytů, ↑ přímý i nepřímý bilirubin,↑ Fe a

feritin, ↑HbA2 a HbF, tenké krvinky (leptocyty), terčovité ery, hyperplastická dřeňTh - cheláty, transfúze

- transplantace před organ. přetížením železem- prenatal. dg.!

Alfa-thalasémie (AR)-porucha tvorby řetězce α (4 alely) → ↑syntézy řetězce γ vytvářející u novorozenců tetramery γ4- Hb Barts, u

dospělých je zvýšena tvorba β řetězců - tetramery β4-HbH (je velmi nestabilní); delece všech 4 genů je neslučitelná se životem, delece 3 genů = těžká anémie

KO – heterozygoti - hypochromní mikrocytární anémie s hemolyt. složkou, HbH do 5 %- homozygoti - hlubší anémie s hepatosplenomegalií (HbH onemocnění)

- hypochromií mikrocyty s basofilním tečkováním a tečkovité ery, variabilní počet retikulocytůTh - transfúze ery a cheláty, prenatal. dg.!

- transplantaceSrpkovitá anémie (AD)- nejčastější hemoglobinopatie- patologický HbS vzniká substitucí kys. glutamové valinem v 6. pozici β-řetězce globinu. HbS → ↓ rozpustnost

Hb, polymerizuje v redukovaném stavu, v oxidovaném stavu brání jeho polymerizaci vazby kyslíku- ↓ deformability →fixace při průchodu kapilárami do tvaru srpku- černoši - Afrika, AmerikaKO – homozygoti s > 50 % HbS - hemolytické anémie, hemolytické krize (infekce, prochlazení – při ↑ nároku na

dodávku O2, ikterus, ↑ retikulocytů), splenomegalie (částečně extravaskulární hemolýza), intravask. hemolýza v oblastech se ↓ tenzí O2 a poškození endotelu

32

Page 33: lf1.czlf1.cz/wp-content/uploads/228-vypracovane_otazky_interna... · Web viewprognoza – 50 % dosáhne kompletního uzdravení do 6 měsíců, 20 % má prodloužený průběh –

Vypracované otázky na INTERNU made by Edita Homolková, Monika Ambroziová, Adam Král, Petr Vermach a Vítek Křehnáč

cév peroxidací → uzávěr cév (vazookluzivní krize = bolestivá infarzace dlouhých kostí, prstů končetin, sleziny, plicní infarkty)

- opakované infarkty sleziny - její zmenšení „autosplenektomie“Dg – u homozygotů Hb 60-70 g/l a ↑ retikulocyty

- HbS v ELFO HbTh - prevence snížené dodávky O2 (prochlazení, včas léčit infekty, hydroxyurea (stimuluje tvorbu HbF - zlepšuje

stav nemocných)- při helmolytické krizi výměnná erytrocytoferésa, transfúze, antikoagulace, analgetika- transplantace krvetvorných bb.

15.A ISCHEMICKÁ CHOROBA DOLNÍCH KONČETIN

= soubor příznaků končetinové ischemie, akutní či chronické, vznikající většinou na podkladě aterosklerózy- výskyt roste s věkem, více u mužů,- stehenní (50%), pánevní (35%), periferní (15%, hlavně při DM)Definice - chronická klidová ischemická bolest vyžadující pravidelnou analgetickou léčbu >2 týdny

- kožními defekty a gangrénou s prokázanou obliterující chorobou tepen- objektivní kriteria : kotníkové tlaky <50 mm Hg

nebo prstové tlaky <30 mm Hg a nebo hodnoty transkut. tlaku kyslíku <30 mmH g.

Rizikové faktory – mužské pohlaví, věk, diabetes, kouření, hypertenze, dyslipidemie, renální insuficienceEtio – ateroskleróza – v 90 %, stenoticko-okluzivní typ

- u kuřáků a hyperlipidemiků v pánevním a stehenním řečišti, u diabetiků častěji na bérci- Lerishův syn. – izolované postižení bifurkace aorty, postihuje mladší muže, silné kuřáky

- hýžďové klaudikace, impotence- dilatační arteriopatie – difúzní ektatické úseky až aneurysmata vyplněné tromby

- trombangiitis obliterans - mladí silní kuřáci- aneurysmata (Blue Toe syn.), cystická degenerace adventicie, kompresivní syndromy, iatrogenní,

hematologické stavy, poúrazové, nádorové postiženíPrognóza – klaudikující, léčení – po 5 letech 75 % se zlepší/ stabilizují, ostatní se zhorší/ amputaceKO – 20-50 % asymptomatických

- chronická – klaudikace - svíravá svalová bolest při chůzi a odezní do 10 min po zastavení (klaudikační interval = vzdálenost do vniku bolesti)

- klidová bolest - při hlubší ischemii, zpravidla na plantě a prstech DK a šíří se ke kotníkům a dál- ustupuje při svěšení končetiny- atrofie kůže, vymizení ochlupení, trofické změny na nehtech, mykóza

- defekty – nehojí se, vysoké riziko pro ztrátu končetiny- Fontainova klasifikace: I. stádium - bez obtíží (diagnostikujeme ↓periferních tlaků, šelesty)

II. stádium - klaudikační bolest (ischemická bolest svalů) při zátěži, propaguje se distálně, nutí pacienta, aby se zastavil (claudicatio intermitens)

a) ujde >200 mb) ujde <200 m

lII. stádium - klidové bolesti, hlavně v noci, zesilují se při zvednutí končetiny

IV. stádium - nekróza akrálních části končetiny, vředy (vyvíjí se z předchozí oděrky, otlaku...)

Dg – anamnéza, klinické vyšetření, polohové testy- kotníkové/ prstové tlaky, zátěžový test na běhátku (treadmill), duplexní SONO (charakter plátu, kalcifikace,

jiná etio)- při stenóze > 90% není distálně hmatný tep, při dostatečném kolaterálnlm oběhu nemusí být příznaky- CT/ MR angiografie u podezření na postižení břišní aorty/ pánve/ femorálního řečistě- DSA (digitální subtrakční angiografie) – postižení bércového řečiště- dopplerovské měření tlaku v klidu - normálně je sTK na kotníku o 10 mm Hg ↑než na paži → index

kotník/paže >1 - index 0,9-0,75 (= lehčí forma), 0,75-0,5 (= středně těžká forma), < 0,5 (- riziko amputace)

33

Page 34: lf1.czlf1.cz/wp-content/uploads/228-vypracovane_otazky_interna... · Web viewprognoza – 50 % dosáhne kompletního uzdravení do 6 měsíců, 20 % má prodloužený průběh –

Vypracované otázky na INTERNU made by Edita Homolková, Monika Ambroziová, Adam Král, Petr Vermach a Vítek Křehnáč

Th – léčba aterosklerózy – hypolipidemika, antiagregancia (ASA 100mg/den,/clopidogrel k profylaxi trombózy), ACEI, sartany, CCB (blokátory kalciových kanálů)

- léčba diabetu- pohyb (denně chodit alespoň 1-2 hodiny (podpora tvorby kolaterálního oběhu), pouze do stádia III)- warfarin – u ICHDK na podkladě embolizací do periferie- léčba klaudikací – naftiidrofuryl → vazodilatace, pentoxifilin (Agapurin) –vazodilatační a reologické účinky

- prostanoidy - prostaglandin E1 - vazodilatace, ↑klaudikační vzdálenost, pokud není možný revaskularizační zákrok

- revaskularizační zákroky – perkutánní transluminální angioplastika (dilatace balónkem/ stentem)- operace (trombendarterektomie, bypass)

15.B HEMAFEREZY V KLINICKÉ HEMATOLOGII A TRANSFUZNÍM LÉKAŘSTVÍ

Terapeutické hemaferezy- léčebné eliminační výkony s cílem zredukovat obsah patogenní substance nebo krevních elementů v cirkulaci

nemocného, a tím ovlivnit průběh onemocnění (hematologie, nefrologie, neurologie, revmatologie, onkologie)-za použití separátorů krevních komponent, na specializovaných transfuzních či hematol. pracovištích- centrifugační princip, v průběhu výkonu se nemocnému odebírá krev, kt. se mísí s antikoagulačním roztokem a

poté vstupuje do přístroje → oddělení a sběr separované komponenty, zbývající složky krve se vracejí zpět do cirkulace

- nesmí se odebrat více než 20% odebíraných složek krveDepleční výkony

- snížení obsahu zmnožených krevních elementů v cirkulaci nemocného- leukocytaferéza - ALL, AML, CLL, CCL mající leukocytózu >100*109/l- erytrocytaferéza - primární polycytémia, při přetížení železem (hemochromatóza, sekundární hemosideróza)- trombocytaferéza - trombocyty >1500*109/l, CML, esenciální trombocytémie, myelofibróza, primární

polycytémieVýměnná plazmaferéza a výměnná erytrocytaferéze

- z cirkulace se odstraní větší část plazmy či krevních elementů a nahradí se roztoky krystaloidů, koloidů, plazmou

- TPE (terapeutic plasma exchange) = neselektivní odst. plazmy s autoprotilátkami IgG nebo IgM, alloprotilátek cirkulujících imunokomplexů, inhibitorů enzymů a koagulačních faktorů, paraproteinu

- při výměně 1 objemu plasmy lze odstranit až 60 % patogenní komponenty- imunoadsorpční kolona

- výměnná erytrocytaferéza - u pacientů se srpkovitou anémií či malárií odstranění patol. ery Sběr buněk PBC, DLI

- cílem je příprava produktu pro terapii nemocného, např. pro zajištění transplantace pomocí- PBC = peripheral blood progenitor cell - před sběrem je nutná mobilizace prekurzorů do perif. krve (růst.

faktory), u nemocných se provádí pomocí kombinace chemoterapie a růst. f.)- DLI = donor lymphocyte infusion

16.A NESTABILNÍ ANGINA PECTORIS, AKTUNÍ INFARKT MYOKARDU BEZ ST ELEVACÍ

Nestabilní angina pectoris- charakteristické recidivující bolesti na hrudi v důsledku ICHSEtio - ruptura nestabilního aterosklerotického plátu → nasedající intraluminální trombóza věnčité tepny

→ částečný uzávěr → nestabilní AP (nedojde k nekróze, nejsou kardiomarkery – troponin a kreatinkináza)→ IM s ST depresemi (NSTEMI)

→ úplný uzávěr → IM s ST elevacemi (STEMI)KO - nově vzniklá klidová /námahová AP

- zhoršení stávající stabilní AP- klidová stenokardie s ústupem spontánním nebo po podání nitroglycerinu- stenokardie silnější trvající do 20 minut

34

Page 35: lf1.czlf1.cz/wp-content/uploads/228-vypracovane_otazky_interna... · Web viewprognoza – 50 % dosáhne kompletního uzdravení do 6 měsíců, 20 % má prodloužený průběh –

Vypracované otázky na INTERNU made by Edita Homolková, Monika Ambroziová, Adam Král, Petr Vermach a Vítek Křehnáč

Dg – může být hypertenze a tachykardie jako důsledek aktivace sympatiku- EKG - deprese ST nebo inverze T jako projev subendokardiální ischemie, jen přechodné- negativní biochemické markery myokardiální nekrózy- nitroglycerinový test – ústup bolestí do 2 min

Th – monitorace na koronární jednotce- ASA, heparin, β-blokátory, kontinuální podávání izosorbiddinitrátu, statiny- chronicky – CCB (amlodipin),

NSTEMI- AIM – viz 6.A- jako nestabilní AP, ale s pozitivními kardiomarkeryDg – EKG – deprese ST, inverze vlny T, ale může být i zcela OK- !!! elevace ST v aVR!!! = nemoc levého kmene

17.A NESPECIFICKÉ STŘEVNÍ ZÁNĚTY

Crohnova choroba= granulomatózní, transmurální zánět postihující segmentárně kteroukoli část trávicí trubice- chronický průběh se střídáním relapsů a remisí- hlavně v mladém věku (medián 25-28 let)- nejčastěji postihuje ileocékální oblast (40 %), tlusté střevo (30 %), tenké střevo proximálně od terminálního ilea

(15-20 %), jícen + žaludek a duodenum (5-10 %)- u 1/3 mimostřevní projevyKO – obvykle není příliš korelace mezi klinikou a zánětem

- může probíhat s malými GIT symptomy a výraznými extraintestinálními (artritida, episkleritida) a nespecifickými (↓ váhy, sideropenická anémie, ↑ teploty)

- ileocekální lokalizace – bolest břicha (zpočátku tlak, v pravém podbřišku, pak křečovité kolem pupku), průjem, hubnutí, zvýšená teplota, nadýmání, flatulence

- při vyšší aktivitě bolesti kolem řiti (infiltrace papil, anální fisury, perianální absces/ píštěl)

- tlusté střevo – samostatné postižení jen u cca 30 %, může být kontinuální a zasahovat i rektu (jako UC)- vředovité defekty orientované v podélné ose- průjmy, bolest břicha křečovitá a v závislosti na vyprazdňování (často pod pupkem), i tenesmy

a krvácení z konečníku- tenké střevo – postiženo orální ileum a jejunum, nejčastěji různě dlouhá těsná stenóza

- typicky těžký dyspeptický syndrom (nechutenství, nadýmání, říhání, škoukání, flatulence)- možný subileóní až ileózní stav- proteino-energetická malnutrice- častěji metabolické komplikace

- rektum a anální kanál – izolovaně vzácně, spíš spolu s tlustým střevem a ileocekálním přechodem- eroze, vředy, hypertrofické řitní papily, typicky tvorba abscesů a píštělí- častěji u mladších a žen- nepříznivý a vleklý průběh

- atypická lokalizace – jícen – stenózy s vředy (dysfagie) - žaludek – poruchy evakuace, pocit plnosti a bolest po jídle, nauzea, zvracení- duodenum – stenózy, malabsorpce…

Střevní komplikace – perforace – může být krytá se vznikem abscesu, nebo volná s peritonitidou- píštěle – slepé/ komunikují (s abscesem, dutým org, povrchem těla)- fibrózní stenózy – subileózní až ileózní stavy

Extraintestinální projevy – u 1/3- kožní projevy (pyoderma gangrenosum, erythrema nodosum), artritida (akutní

synovitida velkých kloubů), iridocyklitida/ episkleritida- koinciduje primární sklerotizující cholangiitida, skroileitida, artralgie drobných kloubů- léčba střevních projevů na ně nemá vliv

Th – k navození remise – mesalazin/ sulfasalazin, busedonid (topický kortikosteroid), při vysoké aktivitě prednison, biologická léčba (anti TNFα infliximab)

35

Page 36: lf1.czlf1.cz/wp-content/uploads/228-vypracovane_otazky_interna... · Web viewprognoza – 50 % dosáhne kompletního uzdravení do 6 měsíců, 20 % má prodloužený průběh –

Vypracované otázky na INTERNU made by Edita Homolková, Monika Ambroziová, Adam Král, Petr Vermach a Vítek Křehnáč

- k udržení – mesalazin/ sulfasalazin, azathioprine, MTX- atb (metronidazol, fluorochinolony) při infekcích/ sepsích- chirurgie – nejčastěji ileocékální resekce, strikturoplastiky (do 5 cm), drenáže abscesů, řešení píštělí

Prognóza – většinou léčitelné, u 5-10 % trvalá invaliditaUlcerózní kolitida

= hemoragicko-katarální zánět postihující sliznici tlustého střeva – vždy rektum a šíří se kontinuálně- chronické, s relapsy a remisemi- medián kolem 25. roku- proktitida 25 %, levostranná kolitida 45 %, extenzivní kolitida (přesahuje ileální flexoru) 30 %KO – tenesmy (bolestivé nucení na stolici, s hlenem a krví eventuelně s hnisem)

- krvavé průjmy u levostranného nebo difuzního postižení- hubnutí, zvýšená teplota, bolesti břicha při extenzivním postižení- indeterminovaná kolitida – akutní, až fulminantně probíhající extenzivní kolitida s endoskopickými a

histologickými rysy UC i CD (možná přítomnost i perianálních hnisavých komplikací)

Střevní komplikace – toxické megakolon - u fulminantní formy, paralytický ileus s dilatací tlustého střeva na >6 cm, hl u extenzivní a levostranné kolitidy

- riziko perforace a peritonitidy → mortalita 50 %- masivní krvácení – narušení stěny vředem, chir řešení- kolorektální ca – podezření při nálezu stenózy (pro UC netypické), dispenzarizace

Extraintestinální projevy – jako u CD - primární sklerózující cholangitida vedoucí u cca ½ k biliární cirhóze, cholangiogenní ca

spojen s ↑ rizikem kolorektálního caDg – příznaky, koloskopie, histologie

- ↑ zánětlivé parametry, sideropenická anémie, hypoalbuminémie, pozitivita protilátek pANCA a nepřítomnost ASCA

Th – k navození remise aminosalicyláty, u těžších forem kortikoidy- udržování aminosylicyláty a azathioprin, 6-merkaptopurin, infliximab- chirurgie = definitivní vyléčení proktokolektomií (s ileo-pouch-anální anastomózou)

- komplikace, selhání léčiv, dysplazie/ karcinomPrognóza – dobrá, formuje se v prvních 2 letech, u 10-15 % chirurgické řešení

17.B PRINCIPY PODPŮRNÉ LÉČBY V HEMATOLOGII

- podpůrná a doplňková léčba je nepostradatelnou součástí komplexní protinádorové terapie, umožňuje nemocnému překonat komplikace, jimiž je provázena zákl. léčba nádorového onem.

1. Prevence a léčba infekcí- zvláštnosti infekcí u onkol. pacientů: poškození sliznic při CHT/RT, neutropenie, riziko reaktivace vir. infekcí,

sekun. infekce, nozokom. infekce- hlavní zásady léčby - MÝT SI RUCE!, rozdělení pacientů do rizik. skupin

- nízké riziko - možná ambl. péče,pohyblivý, bez neutropenie nebo předpokládaná neutropenie < 500 /mm3 méně než 5 dní, věk pod 60 let, nepřítomnost ložiskové infekce, ochota lékaře kontrolovat pacienta ambulantně á 24-48 h, p.o. ATB chinolon ve vysokých dávkách n. chinolon + penicilin s inhibitorem β-laktamázy

- vysoké riziko – nutná hospitalizace, špatný performans status, aktivita nádor. onem. léčba pro ak. leukémii, dehydratace, tachypnoe, zmatenost, oběh. nestabilita, neutropenie déle než 7 dní, parenterální antipseudomonádový penicilin + aminoglykosid n. karbapenem v monoterapii nebo cefalosporin 4. generace v monoterapii

- snaha o získání diagnostického materiálu (hemokultury…)- výměna ATB při neúčinnosti nebo podle citlivosti vykultivovaného agens- zvážit empirická antimykotika u pacientů s febrilní neutropenií > 7 dní (flukonazol)- úprava empirického ATB protokolu na základě dlouhodobých profilů citlivosti- předpoklad, že každá teplota je infekčního původu

36

Page 37: lf1.czlf1.cz/wp-content/uploads/228-vypracovane_otazky_interna... · Web viewprognoza – 50 % dosáhne kompletního uzdravení do 6 měsíců, 20 % má prodloužený průběh –

Vypracované otázky na INTERNU made by Edita Homolková, Monika Ambroziová, Adam Král, Petr Vermach a Vítek Křehnáč

- oportunní infekce – relevantní – herpes simplex/ zoster (acyclovir i.v., valacyclovir p.o.), kandidóza, aspergilus (flukonazol - profylaxe, amfotericin B - syst. mykóza)

- irelevantní – CMV, EBV- problematika růstových faktorů- snižují riziko infekcí

2. Substituční léčba krevními deriváty- Erytrocyty – podáváme u pacientů s Hb < 80 nebo Hb < 100 u pacientů s přidruženým srdeč. nebo plic. onem.- Trombokoncentráty u pacientů s trombocyty: < 10 tis při absenci krvácení

< 20 tis v sepsi n. s mírnými projevy krvácení< 50 tis před středně závažným chir. zákrokem< 100 tis před zásadním chir. zákrokem

3. Léčba bolesti- WHO zásady - pokud možno p.o.

- pravidelně v přiměřených intervalech a dostatečných dávkách- používej žebřík- individualizuj – různá metabolizace analgetik- ověřuj účinnost – zeptáme se pacienta

- analgetický žebřík: 1. stupeň: mírná bolest – neopiátové analgetikum, dostatečná dávka = do 24 h v pohodě2. stupeň: střední bolest – neopiátové analgetikum (nevysazovat) + slabý opiát (kodein)3. stupeň: silná bolest – neopiátové analgetikum + silný opiát (morfin) + adjuvans4. stupeň: epidurální anestezie – nepomáhá-li ani p.o. ani parenterální opiát v max.

dávkách, funguje na bolesti v pánvi a v oblasti břišní- adjuvans – steroidy (moz. meta), antikonvulziva, antidepresiva (neuropatická bolest), antihistaminika,

psychostimulancia4. Antiemetická léčba- cíl – kompletní úleva od nauzey a zvracení- zvracení – časné (do 24 h), opožděné (po 24 h), anticipační (dané strachem z CHT)- rizikové faktory pro zvracení - věk (mladí zvrací více), ženy, abstinence

- emetogenní RT – celotělové ozáření, plášť. pole, ozáření horní břišní etáže- emetogenní CHT - cisplatina, cyklofosfamid, antracykliny, etopsid, vinkristin

- léky – setrony (serotoninoví antagonisté), kortikoidy, dopaminové agonisté

18.A ASTMA BRONCHIALE

- chronické zánětlivé onemocnění dýchacích cest spojené s jejich strukturálními změnami- zánět je spojen s bronchiální hyperreaktivitou, obstrukcí a příznaky (pískoty, dušnost, kašel, tíže na hrudi)- onemocnění s velikou interindividuální variabilitou – hovoříme proto o astmatickém syndromu- prevalence v ČR 8 %Klasifikace - dle úrovně kontroly - kontrolované

- částečná kontrola - nedostatečná kontrola

- dle tíže - intermitentní astma - 2-agonisté podle potřeby- lehké perzistující astma – nízká dávka inhalačních KS / antileukotrienů, teofylinů- středně těžké perzistující astma – střední dávka inhalačních kotikosteroidů/ 2-agonisté

s dlouhým účinkem- těžké perzistující astma – vysoká intenzita léčby

- dle léčby - snadno léčitelné – astma je pod kontrolou- obtížně léčitelné – adekvátní léčbou do 6 měsíců nezvládnutelné,snaha dále dosáhnout

kompromisní kontroly nad astmatem Exacerbace astmatu – stav postupně se zhoršující dušnosti, zkráceného dechu, kašle, hvízdavého dechu, pocitů

tíhy na hrudníku, častá je kombinace těchto příznaků + dechová tíseň- Lehká – nelze sofistikovaně definovat - Středně těžká – příhoda trvající 2 dny, je nutné změnit léčbu (není ale nutná hospitalizace

ani systémová léčba)- Těžká – je příhoda vyžadující urgentní řešení

37

Page 38: lf1.czlf1.cz/wp-content/uploads/228-vypracovane_otazky_interna... · Web viewprognoza – 50 % dosáhne kompletního uzdravení do 6 měsíců, 20 % má prodloužený průběh –

Vypracované otázky na INTERNU made by Edita Homolková, Monika Ambroziová, Adam Král, Petr Vermach a Vítek Křehnáč

- musí splňovat kritérium - podání sys. léčby kortikosteroidy nebo ↑ její udržovací dávky na 3 dny

- hospitalizace či urgentní vyhledání lékaře vyžadující podání systémové léčby

Etio - na vzniku astmatu se podílí dědičné faktory + negativní vlivy zevního prostředí- jedná se o multifaktoriální, polygenně dědičné onemocnění (identifikováno přes 100 genů se vztahem k

astmatu)- významný genetický faktor je atopie – výrazná nadprodukce IgE jako odpověď na obecné alergeny zevního

prostředí (průkazné u víc jak poloviny nemocných)- astma s prokazatelným alergenem označujeme jako „extrinsic astma“, protikladem je „intrinsic astma“, bez

prokazatelného alergenu- kontakt se spouštěči vede k akutnímu astmatu s projevy bronchokonstrikce, edému, mukózní sekrece, kašle

a k amplifikaci zánětuPtg - mechanismus přecitlivělosti I. typu zprostředkovaný IgE + neimulogické mechanismy → zánět

zprostředkovaný Th2-lymfocyty, eozinofily a žírnými buňkami, který ↑ reaktivitu bronchů, zpočátku jsou obstrukce reverzibilní, později ale dochází k přestavbě jejich stěn

- edém a hyperémie sliznice, hypersekrece vazkého bronchiálního sekretu, spasmus hladké svaloviny bronchůKO - záchvatovitá dušnost (po inhalaci alergenu, po námaze, po rozrušení)

- při záchvatu slyšíme exspirační pískoty, přítomna plicní hyperinflace s hypersonorním poklepem, obstrukční ventilační porucha, malé dechové exkurze, zapojení pomocných dýchacích svalů, zatahování jugula, mezižebří a epigastria

- ↓ PEF (vrcholová expirační rychlost), ↓ FEV1 (maximální výdechový objem za 1 sekundu), stoupá RV (reziduální objem)

Dg – anamnéza, typický průběh- spirometrie – průkaz bronchiální obstrukce a její reverzibility - lab – eozinofily, ECP (eozinofilní kationický protein)

Th – režimová opatření – nekouřit, vyhnout se alergenu- farm úlevová – β2-mimetika s rychlým nástupem účinku – fenoterol, salbutamol

- inhalační anticholinergika s krátkým účinkem – ipratropium bromid- teofylin i.v.- systémové kortikosteroidy

- farm kontrolující asthma – inhalacní kortikosteroidy- inhalační β2-mimetika s dlouhodobým účinkem, dlouhodobý teofylin- systémové kortikosteroidy-anti-IgE – omalizumab

- 5ti stupňová, dle úrovně kontroly astmatu

18.B KRVÁCIVÉ STAVY Z DESTIČKOVÝCH PŘÍČIN; TROMBOCYTOPATIE, TROMBOCYTOPENIE (MIMO ITP)

TROMBOCYTOPENIE = nedostatečný počet trombocytů- vzniká nepoměrem mezi novotvorbou a zánikem trombocytů (selhání tvorby v k. dřeni či urychlený zánik,

zadržování destiček mimo cirkulaci)KO - krvácení do kůže (v podobě petechií s rozvojem purpury, rozsáhlé hematomy - neúměrné inzultu, vznikají i

spontánně)- krvácení do sliznic (epistaxe, krvácení z dásní, meno-metroragie)- nejzávažnější - krvácení do sítnice, CNS (tříštivý charakter)- drobná poranění → obtížně stavitelná krvácení, spontánní krvácení při poklesu trombocytů pod 30.109/l - spontánní krvácení při trombocytech 50 - 100.109/l → podezření i na poruchu funkce

1. Trombocytopenie ze snížené tvorby trombocytů- Trombocytopenie amegakaryocytární - útlum novotvorby destiček, megakaryocyty v k. dření chybí nebo jsou

snížené- izolovaný útlum megakaryocytární řady/ častěji v rámci útlumu celé myeloidní řady- vrozené – vzácné, v rámci širšího postižení krvetvorby

38

Page 39: lf1.czlf1.cz/wp-content/uploads/228-vypracovane_otazky_interna... · Web viewprognoza – 50 % dosáhne kompletního uzdravení do 6 měsíců, 20 % má prodloužený průběh –

Vypracované otázky na INTERNU made by Edita Homolková, Monika Ambroziová, Adam Král, Petr Vermach a Vítek Křehnáč

- získané - po léčbě myelotoxickými l. a ioniz. zářením, virové inf., infiltrace k. dřeně nádory, fibrózní přestavba u myeloprolif. onem., předpokládaná účast imunologických mechanismů u idiopatických forem

- Dg - trepanobiopsie- Th – imusuprese, allotransplantace, inhibitory fibrinlolýzy (PAMBA, EACA), převod destiček

- Trombocytopenie megakaryocytární - neefektivní trombopoéza, normální či zvýšený počet megakaryocyty s různými tvarovými odchylkami

- do této skupiny patří: trombocytopenie u megaloblastických anémií, PNH, MDS, hereditární trombocytopenické trombocytopatie (sy Bernardův-Soulierův)

2. Trombocytopenie ze zvýšeného zániku- formy megakaryocytární s normálním nebo ↑počtem megakaryocytů v k. dřeni (nadměrný rozpad ve slezině,

intravask. konzumpce v koagulač. procesu či ztráty organismu, imunitní mechanismy)- Autoimunitní trombocytopenická purpura (ITP) - viz. ot. 5 B- Konsumpční trombocytopenie - při procesu intravaskulární mikrotrombotizace = DIC (ot. 35 B)

- Trombotická trombocytopenická purpura (TTP) = syndrom Moschcowitzův - vzácné, závažné, 4/100 000, více ženy

- centrální febrilie, hemolytická anémie, trombocytopenie s krvácivými projevy, pestrá neurologická sympt., orgánové postižení = ledviny!!

- schistocyty v krevním nátěru, ↑ LDH- mnohočetná trombocytární mikrotrombotizace s minimálním podílem fibrinu

(hyalinní tromby) s predilekcí v CNS- vrozené familiární formy (AD) – vysokomolekulární formy von Willebrandova f.

(HMWvWF) se schopností indukovat intravask. trombocytární mikrotrombotizaci

- důsledek deficitu specif. plazmat. metaloproteinázy - manifestace v jakémkoli věku

- získané formy TTP - protilátky proti proteinázám- Th – plazmaferéza (alternativně zmražená plasma), imunosuprese

- hemolyticko-uremický syndrom (HUS) – orgánově omezená purpura- epidemická forma - děti, předchází ji prodromální enterokolitis E. coli

verotoxin+ (kapilarotoxický účinek, hl renální kapiláry) → hemoragická enterokolitida, anémie, trombocytopenie, renální postižení

- sporadická forma - chybí střevní symptomatologie – plazmaferéza- často se rozvíjí chron. renální selhání

- Polékové trombocytopenie – vnímaví jedinci, prudký rozvoj po podání provokující látky- sulfonamidy, chinidin, složky potravy…

- Potransfúzní trombocytopenie – alloprotilátky při antigenní inkompatibilitě- rozvoj do 7-10 dnů

- Neonatální alloimunní trombocytopenie – protilátky od matky proti destičkovým antigenům - Heparinem indukovaná trombocytopenie (HIT) – u 1-3 %

- I. typ – důsledek nespecifické schopnosti mukopolysacharidové molekuly heparinu aglutinovat destičky, pokles destiček nedosahuje nebezpečných hodnot

- II. typ – tvorba protilátek proti komplexu heparinu a destičkového faktoru 4 na povrchu destiček + indukce periferní trombocytární trombotizace

- větší trombocytopenie, ischemické komplikace3. Trombocytopenie ze ↑ ztrát a sekvestrace

- ztráty – velká krváceni, polytraumata, operace při mimotělním oběhu- sekvestrace – u splenomegalie z jakékoli příčiny

TROMBOCYTOPATIE = porucha destičkových funkcí; prodloužení krvácivosti při normálním počtu trombocytů1. Vrozené

- Bernardův-Soulierův syndrom - porucha adheze trombocytů, vrozené odchylky povrchové membr.- Destičkový typ vWF choroby - ↑ afinita defektního GPIb pro vWF, urychlená destrukce těchto tromboc.

39

Page 40: lf1.czlf1.cz/wp-content/uploads/228-vypracovane_otazky_interna... · Web viewprognoza – 50 % dosáhne kompletního uzdravení do 6 měsíců, 20 % má prodloužený průběh –

Vypracované otázky na INTERNU made by Edita Homolková, Monika Ambroziová, Adam Král, Petr Vermach a Vítek Křehnáč

- Glanzmannova trombastenie - porucha prim. agregace, porucha povrchové membrány trombocytů, AR- Heřmanského-Pudlákův sydrom - chybí skladovací granula trombocytů → agregační porucha, albíni

2. Získané - myeloproliferativní onem., chron. renální onem., monokonální gamapatie, léky (NSA)- th - zákl. onem., hemostyptika (PAMBA, EACA-kys. ε-aminokapronová)

19.A IKTERUS

- nejčastější příznak jaterních onemocnění, žluté zbarvení kůže, sklér a sliznic- při ↑ bilirubinu 2-3x (80 % vzniká rozpadem Hb)- nekonjugovaný bilirubin je vázán na albumin a transportován do jater, kde je konjugován a vyloučen do žluče,

v dolní části ilea a kolon je bakteriemi částečně měněn na sterkobilinogen a ten je zpětně vychytáván a při jaterní nedostatečnosti vylučován do moče jak urobilin

- když není v moči = nekonjugovaná hyperbilirubinémie, + když v séru je >50% celkového bilirubinu nekonjugovaného

Premikrozomální hyperbilirubinemie (s převážně nekonjugovaným bilirubinem)- ↑ produkce - hemolýza- ↓ vychytávání buňkou – dlouhodobé hladovění- ↓ konjugace buňkou – vrozená – syn Gilbertův, Criglerův-Najarův

- získaná – hepatitida, cirhóza (s více poškozenou exkreční fcí)Postmikrozomální hyperbilirubinemie (s převážně konjugovaným bilirubinem)

- porucha intrahepatální exkrece na úrovni buňky – vrozená – syn Dubinův-Jahnsonův, Rotorův, cholestatická žloutenka v dětství

- získaná – hepatitidy, cirhóza- extrahepatální biliární obstrukce

- obojí mohou způsobit sepse a lékyDg – při akutní hepatocelulární lézi nutné markery virových hepatitid, abúzus alkoholu, léky…Familiární nekonjugované hyperbilirubinemie

Gilbertův syndrom – nejčastější, 5-10 % populace (hl. mladí muži), AR, defekt glukuronizace (i porucha jaterního transportu bilirubinu), většinou benigní

KO – subikterus až ikterus sklér, bez subjektivních obtíží, střídání období hyperbilirubinemieDg - ↑ nekonjug. bilirubin, možné lehké ↑ alkalické fosfatázy (uvolňuje se z kanalikulárních

membrán hepatocytů)- hladový test – za 24 hod dojde ke 100% ↑ bilirubinu

Th – benigní, neléčí se- varovat před paracetamolem

Criglerův-Najjanův syndrom – těžké, vzácné, AR, deficit konjugačního enzymu- typ I (enzym zcela chybí, úmrtí do 1 roku), typ II (aktivita enzymu <10 %)Dg – extrémní hyperbilirubinemieTh – u typu I plasmaferézy a transplantace jater, u typu II fenobarbital (indukce

enzymu) a fototerapieFamiliární konjugované hyperbilirubinemie

Dubin-Johnsonův syndrom – vzácné, benigní, AR, intermitentní ikterus, bez ThRotorův syndrom – benigní, bez terapie

Cholestáza - intrahepatální (akutní i chronické hepatitidy, polékové poškození, HAK, cirhóza, primární sklerotizující

cholangitida) a extrahepatální (konkrement, striktury žlučovodů, malignity slinivky, cholangiokarcinom)KO – ikterus, pruritus (žlučové kys.), při delším trvání hypercholesterolémie (xantelazmata, acholická stolice,

tmavá močDg - ↑ bilirubin, ALP, GMT, hypercholersterolemie

- při ↑ hladiny Fe spíše intrahepatální postižení- SONO, ERCP, CT,PCT

Th – dle příčiny- cholestyramin na pruritus, suplementace vit A, D, E, K, prevence kostní nemoci

40

Page 41: lf1.czlf1.cz/wp-content/uploads/228-vypracovane_otazky_interna... · Web viewprognoza – 50 % dosáhne kompletního uzdravení do 6 měsíců, 20 % má prodloužený průběh –

Vypracované otázky na INTERNU made by Edita Homolková, Monika Ambroziová, Adam Král, Petr Vermach a Vítek Křehnáč

19.B RYCHLE PROGREDUJÍCÍ GLOMERULONEFRITIDY (RPGN)

- závažné postižení ledvin s rychlým zhoršováním renálních funkcí (během dnů, max.týdnů),které vyústí v nutnost dialýzy

- výskyt u 2-5% bioptovaných- život ohrožující stavy, rychlá terapieDefinice - smíšený močový nález, nefritický sy (otoky, proteinurie do 3g/l), hematurie -erytrocyt. válce,

hypertenze, oligurie +/- , ↓ funkce ledvin- aspoň 2 příznaky- patolog. kriterium – renální biopsie (RB) - postižení srpky většiny glomerulů (>70%)

Etio - srpky - extrakapilární proliferace, vznik v důsledku ruptury kapilár, průnik fibrinogenu a monocytů- zprvu jsou epitelové (makrofágy a epitelie, mohou se ještě zhojit), postupně náhrada vazivem až

zánik glom.- renální intersticium – zánětlivý infiltrát, až intersticiální fibróza a tubulární atrofie

Dg - renální biopsie + imunofluorescence (IF)Klasifikace:

1. ANCA asociované renální vaskulitidy (80% RP GN):- protilátky proti cytoplazmě neutrofilů- Granulomatóza s polyangiitidou (dřív Wegenerova granulomatóza) cANCA

- přítomnost Ab proti cytoplazmě neutrofilů, na IF v celé cytoplazmě – cAnca- generalizovaná vaskulitida- typický nález – granulomy – obrovské jaderné bb. se nalepují na kapilární stěnu (u jiné

GN to není)- Anca + Ab- IF – negativní nebo jen slabě pozitivní (pauciimunitní GN)- srpky

- výskyt kolem 50 let, i v 30 letech nebo 80 (průběh není tak závažný)- RB-celulární srpek v Bowman. pouzdru až ischemický kolaps glom., kolem

glom.periglomerulitida- extrarenální manifestace – ORL (epistaxe, chron. rinitida, sedlovitý nos, postižení

středního ucha)- plíce – infiltráty, difuzní (intraalveolární krvácení –

hemoptýza) nebo ložiskové infiltráty- episkleritidy, retinální vaskulitidy, krvácení do Git,

artralgie,myalgie,kožní vaskulitidy- Th – indukční – kortikoidy 1mg/ kg/ den + cyklofosfamid (CFA) 10-15 mg/ kg v

intervalech 2-4 týdny- v pulzech je lepší nástup účinku při nižší kumulativní dávce, min. 3

měsíce- udržovací – po remisi, azatioprim, mykofenolát mofetil- plazmaferéza

- prognóza – u časné dg a th je možné obnovení renálních fcí (RF)- Mikroskopická polyangiitida (vaskulitida drobných cév bez glom.) pANCA, proti myelo peroxidáze- Churg – Strausové syndrom pANCA (astma bromchiale, eosinofilie >1,5.109, nekrotizující

vaskulitida)2. GN s pozitivními protilátkami proti bazální memnráně glom. (anti-GBM nefritida)

- Goodpasturova choroba - antirenální syndrom- tvorba Ab (IgG1) proti glomerulární bazální membráně (kolagen IV), může i proti

membráne plicních sklípků pak je to Goodpastureův syndrom- u nás ročně 10-20 pacientů (vzácná), výskyt -muži 20-30 let, kuřáci- KO – jako RP GN, může být i krvácení do plic, vývoj perakutní (dny), i pozvolnější, často

těžká anémie (hypochromní mikrocytární, subklinické plicní hemoragie)- hemoptýza- ze začátku spíš mikroskopická hematurie a malá proteinurie, pak oligurie s

hypertenzí

41

Page 42: lf1.czlf1.cz/wp-content/uploads/228-vypracovane_otazky_interna... · Web viewprognoza – 50 % dosáhne kompletního uzdravení do 6 měsíců, 20 % má prodloužený průběh –

Vypracované otázky na INTERNU made by Edita Homolková, Monika Ambroziová, Adam Král, Petr Vermach a Vítek Křehnáč

- Dg - IgG antiGBM (titr koreluje se závažností- histologie - lineární IF i IgG, celulární srpek

- Th – indukční KS + pulzy CFA, vždycky plazmaferéza- udržovací – KS + azatioprim- léčba lze po 6 měsících vysadit, relaps jen vyjímečně

- prognóza – renální fce stabilizujeme, obnovit se nedají, plicní krvácení ustupuje- (pro jistotu pulmorenální sy není to samé co RP GN, pr sy je součastné akutní renální

selhání a plicní postižení)3. Imunokomplexové RP GN

- Akutní poststreptokoková GN (↑ ASLO, ↓ C3 komplement)- Parainfekční GN – hantaviróya, leptoviróza (v průbehu onemocnění)- Henoch-Schomleinova nefritida (↑ IgA, norm C3)- vrámci SLE - často s antifosfolipidovým syndromem – + protilátky (Ab) proti fosfolipidům v cévním

epitelu, deštičkách,...- rekurentní (opakující se) trombózy, opakované aborty, konzumpční trombocytopenie

- Kryoglobulinémie – precipitace Ab za chladu (poměrně často membranoproliferativní i srpky)

20.A AORTÁLNÍ STENÓZA A AORTÁLNÍ REGURGITACE

Aortální stenóza- nejšastěji operovaná vada v dospělosti (50 % skleroticko-kalcifikační změny 3-cípé chlopně, 35 % kalcifikovaná

bikuspidální chlopeň, 10% porevmaticky změněná)- normálně je 3cípá s plochou aortálního ústí 2,5-4 cm2

- symptomy se začínají projevovat při ploše ústí <1 cm2 (individuální!)- zvýšené riziko infekční endokarditidy- stenóza je překážkou při vypuzení krve = ↑ afterloadu → tlakové přetížení LK → koncentrická hypertrofie →

špatné prokrvování svaloviny → difuzní ischemie sval. bb. → nahrazení vazivem → diastolická dysfce LK, později dilatuje a objeví se i systolická dysfce

Etio – degenerace – proces jako u aterosklerózy, v 60-80 letech, dříve u bikuspidální chlopně- porevmatická – srůsty cípů v oblasti komisur, často postižení i mitrální chlopně-kongenitální, Fabryho ch., hyperlipoproteinémie…

KO - velice dlouho asymptomatický, pří příznacích se již jedná o stenózu vyššího stupně- angina pectoris – u 2/3 nemocných s těsnou AoS, ↑ potřeba O2 hypertrofického myokardu- synkopy – při zátěži nedostatečně roste srdeční výdej, vazodilatací v namáhaných orgánech se dostaví

hypotenze → nedostatečné prokrvení mozku- i v důsledku paroxysmu tachyarytmie

- srdeční selhání – nejtěžší projev AoS, průměrné přežití je pak 2 roky- netolerance námahy, později projevy postkapilármí plicní hypertenze

- náhlá smrt – maligní arytmieDg – fyzikálně – hlučný systolický šelest nad aortální chlopní s propagací do karotid

- EKG - známky hypertrofie levého srdce (Sokolovův index = S ve V1 + R ve V6 >35 mm), negativizace vlny T (typické pro tlakové přetížení)

- rtg – hypertrofie LK, prominence aortálního knoflíku, kalcifikace v aortálním ústí- transtorakální a transezofageální echo – počet cípů, kalcifikace, stupeň hypertrofie/ dilatace, orientačně

gradienty mezi LK a aortou…- katetrizace – koronarografie, přesné měření transaortálního gradientu

Th – náhrada aortální chlopně – symptomatičtí pacienti s významnou stenózou- ustupuje hypertrofie komory, zlepšení systolické i diastolické fce- u některých je ale ireverzibilní a dochází k tzv afterload mismatch- mechanické a bioprotézy

- farm – léčba srdečního selhání, zábrana tachykardii- nedáváme arteriodilatancia (↓ systémový tlak a ↑ gradient na aortálním ústí → synkopy)

Aortální regurgitacePtg - aortální insuf. ↑ objemová zátěž levé komory (objem je navýšen o regurgitovaný objem) → excentrická

hypertrofie s postupnou dilatací levé komory

42

Page 43: lf1.czlf1.cz/wp-content/uploads/228-vypracovane_otazky_interna... · Web viewprognoza – 50 % dosáhne kompletního uzdravení do 6 měsíců, 20 % má prodloužený průběh –

Vypracované otázky na INTERNU made by Edita Homolková, Monika Ambroziová, Adam Král, Petr Vermach a Vítek Křehnáč

- zvýšení tepového objemu, snížená periferní rezistence → ▲systolicko – diastolická amplituda- objemové i tlakové přetížení: 1. fáze ▲ EF LK

2. fáze ▲ EF LK, ↑↑ EDV, ↓ ESV (větší objem krve, při nezměněném tlaku v komoře) … elongace vláken, zvýšení napětí ve stěně … vysoký endsystolický i enddiastol. Wall stress – excentrická hypertrofie

3. fáze ▲ EF LK, ↑↑ EDV, ↑ ESV 4. fáze ▼EF LK, ↑↑ EDV, ↑↑ ESV

Etio - dilatace kořene aorty – Marfanův syn., ankylozující spondylartritida, syfilitida- poškození chlopně - porevmaticky, méně často po endokarditidě (perforace cípů), po traumatu, SLE, RA,

kongenitální- postižení anulu s normální chlopní – hypertenze, prolaps cípů do defektu IVS

KO – dlouho asymptomaticky- projevy kongestivního i dopředného srdečního selhání

Dg – auskultace - diastolický šelest nad aortální chlopní – maximum v Erbově bodě (3.-4. mezižebří vlevo), začíná hned po 2. ozvě

- velký tepový objem (>sTK), porucha uzávěru chlopně (<dTK, až neměřitelný), rychlý s vysokou amplitudou = Corriganův puls = pulsus altus et celer

- příznaky z objemné pulzové vlny (dunění v hlavně při pulzové vlně, viditelná pulzace karotid- rychlá únavnost, palpitace, při hrozící dekompenzaci angina pectoris + známky levostranné srd. insuf.- Mussetův příznak – potřásání hlavou s každou systolou- ECHO – retrográdní diastolický tok….- katetrizace – kontrola koronární aterosklerózy

Th – náhrada chlopně (i kořene aorty), záchovné operace- léčba hypertenze (zabránění dilatace kořene aorty), snaha snížit diastolický tlak při počínající dilataci (ACEI,

CCB, diuretika)

21.A ONEMOCNĚNÍ HLUBOKÉHO ŽILNÍHO SYSTÉMU, TROMBÓZA HLUBOKÝCH ŽIL

Felbotrombóza = hluboká žilní trombóza- postihuje nějakou část hlubokého žilního sys (nejčastěji DK, pánevní pleteně)- riziko plicní embolie, chronické trombembolické plicní hypertenzeEtio – hyperkoagulační stav, stáza krve, porušení endotelu- predisponují vrozené/ získané trombofilní stavy, věk, obezita, imobilizace, kmenové varixy DK, cestování

letadlem, HAK…KO – bolest, otok, cyanóza (lividní ale teplá), pocit napětí, palpační bolestivost, bolest v lítku při dorzální flexiDg – D-dimery +

- USG (duplexní sono) – velikost žíly, průchodnost, intraluminální obsah (stlačitelnost žíly sondou)- ascendentní kontrastní flebografie – zobrazení před léčbou trombolitikem- CT flebografie, spirální CT – ileofemorální a tirohrudní trombózy

Th – zástava růstu tormbu, rozpuštění trombu, prevence embolizace- konzervativně - ne imobilizace!, bandáž končetiny, nízkomolekulární hepariny + warfarin (fondaparinux,

dabigatran)- lokální trombolýza – rtPA/ altepláza současně s heparinem

- mladí, bez KI, trombóza proximálně uložená- chir – při KI trombolýzy a antikoagulace (po porodu)- kavální filtry – do DDŽ, při nemožnosti antikoagulace/ emboliích i při antikoagulaci

21.B ONEMOCNĚNÍ KŮRY NADLEDVIN

Adrenokortikální insuficience – Addisonova choroba – viz 3.BCushingův syndrom – 77.APrimární hyperaldosteronismus – důsledek autonomní nadprodukce aldosteronuConnův syndrom – primární hyperaldosteronismus způsobený autonomní nadprodukcí aldosteronu adenomem

43

Page 44: lf1.czlf1.cz/wp-content/uploads/228-vypracovane_otazky_interna... · Web viewprognoza – 50 % dosáhne kompletního uzdravení do 6 měsíců, 20 % má prodloužený průběh –

Vypracované otázky na INTERNU made by Edita Homolková, Monika Ambroziová, Adam Král, Petr Vermach a Vítek Křehnáč

Kongenitální adrenální hyperplazie – skupina chorob AR, nedostatečná aktivita některého z enzymů steroidogeneze

- ↓ kortizolu → ↑ ACTH → stimulace kůry → ↑ steroidních metabolitů před enzymovým blokem (androgeny) + urč deficit gluko a mineralokortikoidů

Deficit 21-hydroxylázy – nejčastější, prevalence 1:14 000- ↓ kortizol a aldosteron + ↑ androgeny, manifestace hyponatremií,

hyperkalií a dehydratací u klasické formy, u neklasické spíše nadbytek androgenů (v různém věku)

- léčba glukokortikoidy

22.A RIZIKOVÉ FAKTORY ATEROSKLERÓZY, POSOUZENÍ CELKOVÉHO KARDIOVASKULÁRNÍHO RIZIKA

= chronické progresivní onemocnění cévní stěny charakterizované místní akumulací lipidů a dalších komponent krve a fibrózní tkáně v intimě arterií, provázené změnami v médii cévní stěny

- vyvíjí se jako chronický zánět s nadměrnou proliferativní odpovědi intimy a médie tepen na různé podněty, zejména na modifikované LDL

Fáze vývoje aterosklerózyČasné fáze – hromadění lipidů

1. izolované pěnové buňky odvozené z makrofágů2. tukové proužky – hromadění pěnových buněk, obsahujících intracelulárně akumulované lipidy3. intermediární léze – malá množství extracelulárně uložených lipidů, pocházejících z

odumřelých pěnových buněk4. aterom – vznik lipidového jádra, tvořeného extracelulárně akumulovanými lipidy

Pozdní fáze – intimální proliferace a nasedající trombóza1. fibroaterom – proliferace bb. hladkého svalstva v intimě a ↑ syntéza extracelulární matrix,

obsahující kolagenní a elastická vlákna, která vytváří vazivovou vrstvu nad lipidovým jádrem

2. komplikovaná léze – kalcifikace, ruptura nebo exulcerace, krvácení do ateromu, vznik trombu

- k rozvoji aterosklerotických změn přispívají různé komponenty krve a arteriální stěny: lipoproteiny , endotelové buňky, monocyty/makrofágy, T-lymfocyty, trombocyty, buňky hladkého svalstva

- nejvýznamnější rizikový faktor aterosklerózy jsou aterogenní lipoproteiny, zejména LDLAterosklerotický plát

- je tvořen jádrem s hojnými extracelulárně uloženými lipidy a fibrózního obalu (čepička, kryt), sestávajícího se z vazivové tkáně s převahou kolagenu a dále proteoglykanů, obklopující svalové buňky

- Stabilní plát - silnější a neporušenou fibrózní čepičkou- Nestabilní pláty - mají v jádře uloženo větší množství lipidů, pěnových bb. a T lymfocytů a fibrózní kryt je tenký

s ↓ obsahem kolagen- jsou náchylnější k ruptuře, která je odpovědná za většinu akutních koronárních příhod- k nestabilitě plátu přispívá zánětlivý proces, k němuž dochází v místech nahromadění

makrofágů a T lymfocytů (makrofágy jsou zdrojem metaloproteinázy, kolagenázy, stromelysinu, které mohou zeslabovat fibrózní kryt. Naopak T lymfocyty přítomné v plátu produkují interferon-γ potlačující syntézu kolagenu buňkami hladkého svalstva, a CD40 stimulující syntézu metaloproteinázy

- ruptura plátu je důvodem ke krvácení do plátu a vytvoření tromb- viz ot. 5.A

Rizkové faktory - neovlivnitelné - věk (muži nad 45, ženy postmenopauzální), mužské pohlaví, genetika- ovlivnitelné - dyslipoproteinémie (>LDL, <HDL, >TGL), AHT, DM, metabolický syndrom (obezita,

DM2, >TGC, AHT), kouření- nedostatek pohybu, stres, hyperhomocysteinémie, hyperfibrinogenémie,

protilátky proti fosfolipidům- hsCRP, ↑ fibrinogen

- SCORE – ženy/ muži, kuřák/ nekuřák, systolický tlak, celkový cholesterol, věk- - vysoké riziko >5 %

44

Page 45: lf1.czlf1.cz/wp-content/uploads/228-vypracovane_otazky_interna... · Web viewprognoza – 50 % dosáhne kompletního uzdravení do 6 měsíců, 20 % má prodloužený průběh –

Vypracované otázky na INTERNU made by Edita Homolková, Monika Ambroziová, Adam Král, Petr Vermach a Vítek Křehnáč

22. B NEHODGKINSKÉ MALIGNÍ LYMFOMY

- různorodá sk. chorob- nejčastější malignitou ve věkové sk. 60-65 let, incidence vzrůstá, s nárůstem AIDSEtio – imunosuprese – HIV, terapeutická (transplantace), EBV (Burkittův lymfom), HTLV-1 (T-leukemie/ lymfom

dospělých), HVC (lymfoplasmocytární lymfom s kryoglobulinémií II), HHV-8- Helicobacter pylori (extranodální MZL MALT), Chlamidia sp. (MALT lymfom oční spojivky), Campylobacter

jejuni (střevní MALT lymfom)- herbicidy, pesticidy, radioterapie, cytostatika- vznik neoplastickou transformací imunokompetentních bb. na různých etážích jejich postupného vyzrávání;

nádorová bb. → klonální expanze, zachovává si fční a migrační charakteristiky svých normálních protějšků- NHL i mimo lymfatickou tkáň = extranodální, zvl. formou jsou maltomy = vycházejí z lymf. tkáně sliznic

KO – postihuje hl. mízní uzliny → postižení jakéhokoli orgánu → příslušná symptomatologie- 2/3 pac. zvětšení mízních uzlin (Waldeyerův okruh, krční, axilární, inguinální)- abnormal. krev. obraz (anemie, trombocytopenie, lymfocytóza při leukemizaci), alterace jaterních testů, obraz

renální insuficience, kašel a dušnost při postižení hilových uzlin, výpotky, poruchy trávicího traktu, otoky- celkové příznaky: horečky, pocení, váhový úbytek, nechutenství- B-symptomatologie: horečky nad 38 °C, ↓ váhy > 10 % během 6 měsíců, noční pocení- často ložisko mimo lymf. uzlinu (GIT, ŠŽ, kůže, orbita)

Dg – imunohistologické vyšetření zmraženého řezu, trepanobiopsie, PET-CT, lumbální punkce (MR), biopsie jater…Klasifikace - komplikovaná (více než 40 definovaných typů NHL), pro dg. nutné „ druhé čtení“ histol. nálezu dalším

patologem + lab. + klinika- WHO - dělení na B (CD20) a T (CD3) lymfomy, sk. lymfomů prekurzorových a tzv. periferních

- NHL s nízkým st. malignity (indolentní průběh, odpověď na CHT i radioterapii)- NHL s vysokým stupněm malignity (bez léčby velmi rychlá progrese, agresivní CHT-kurativní

úč.)- stadium I – postižena 1 lymfatická/ extralymfatická oblast- stadium II – postiženy 2 a více skupin uzlin na 1 straně bránice/ 1 extralymfatické a lymfatické

postižení na 1 straně bránice- stadium III – postižení uzlin na obou stranách bránice s možným lokálním postižením

extralymfatického orgánu- stadium IV – difuzní/ diseminované postižení 1 a více extralymfatického org. (bez postižení uzlin)

- játra, kostní dřeň, plíce, kostiTh – chemoterapie – hlavně kombinovaná (alkylační látky – cyklofosfamid, mitotické jedy – vincristin,

antracyklinová atb – adriamycin, kortikoidy) = CHOP- HDT + ASCT = vyskokodávkovaná terapie + autologní transplantace krvetvorných bb – kryokonzervace

krvetvorných bb, následná terapie s limitující myelotoxicitou (maximum usmrcení nádorových bb.)

- u pacientů s agresivními lymfomy, kteří neodpověděli na první dva cykly záchranné léčby- alogenní transplantace – využívání GVL reakce (graft versus lymphoma) k likvidaci patologického klonu

- rizikové díky možnému rozvinují GVHD (graft versus host disease) - hl. u folikulárního lymfomu, periferní T-lymfom aj.

- radioterapie – lymfomy jsou radiosenzitivní, iniciálně málokdy- u lokalizovaného onemocnění, paliace v relapsu

- imunoterapie – anti CD20 – rituximab, ofatumumab- po navázání apoptóza/ aktivace komplementu/ cytotoxická reakce od ADCC

- anti CD52 – alentuzumab, CLL, perifení T-lymfoproliferace- s navázáným izotopem 90Y (β zářič)

- podpůrná léčba – G-CSF, EPO, atb…1. Prekurzorové lymfomy

- lymfoblastový lymfom (LL) – v 90 % pochází z T-bb. - považován s akutní lymfoblastovou leukemií (ALL) za jednu jednotku- časté extranodální postižení, agresivní průběh

45

Page 46: lf1.czlf1.cz/wp-content/uploads/228-vypracovane_otazky_interna... · Web viewprognoza – 50 % dosáhne kompletního uzdravení do 6 měsíců, 20 % má prodloužený průběh –

Vypracované otázky na INTERNU made by Edita Homolková, Monika Ambroziová, Adam Král, Petr Vermach a Vítek Křehnáč

2. B-buněčné lymfomy- Difúzní velkobuněčný B-lymfom (DLBCL) – u nás nejčastěji (45 %), věkový medián 60 let, heterogenní sk, CD20

positivní, relativně agresivní průběh- kombinovaná imunochemoterapie (R-CHOP)- Primárně mediastinální B-lymfom (MBCL) – 2 % lymfomů, věkový medián 35 let

- primární lokalizace v mediastinu, často postihuje pleuru, perikard- z medulárních lymfoidních bb. thymu (obstrukce DC či sy VCS)

- Primární velkobuněčný lymfom CNS – 1 %, neurologická symptomatologie- stereotaktická biopsie (chirurgický zákrok na mozku klasicky, gama nožem,

jehož cílem je odstranění/zničení nežádoucího ložiska nebo postižené oblasti, případně biopsie, stereotaxe je přesné zaměření ložiska v mozku a je základem pro chirurgický zákrok)

- Folikulární lymfom (FL) – 20 %, věkový medián 60 let, u 95% translokace t(14;18) – poškození regulace apoptózy (bcl-2), 1/3 pacientů má ↑ LDH

- pozvolný průběh, opakované relapsy- radioterapie a R-CHOP

- Lymfom z plášťových buněk (MCL) – 5 %, více muži, spíše nepříznivá prognóza- translokace t(11;14) → ↑ exprese cyklinu D1 (uvádí bb do buněčného cyklu)

- Lymfom z malých lymfocytů (SLL) – lymfom typu CCL, věkový medián 60 let, diagnóza často v pokročilé fázi (postiženy uzliny i dřeň)

- Lymfoplazmocytický lymfom (Waldenströmova makroglobulinemie) – medián věku 65 let- často paraprotein IgM, diseminované postižení uzlin, dřeně a sleziny- může být kryoglobulin (Raynaudův syn.), kloubní bolesti, periferní neuropatie

- Extranodální lymfom z marginální zóny MALT typu – nodální/ stenický/ extranodální (nejčastější)- žaludek, plíce, děloha, kůže, oční spojivka, slinné žlázy- významná stimulace antigenem H. pylori (eradikace může vést k regesi, amoxycylinn,

metronidazol, cyklofosfamid)- prognóza dobrá (dlouhodobé přežití 80 %)

- Splenický lymfom z marginální zóny – lymfomové bb v periferní krvi a dřeni, splenomegalie, vzácně postiženy uzliny, dobrá prognóza

- splenektomie, imunochemoterapie v případě progrese- Nodální lymfom z marginální zóny – vzácný, horší prognóza, léčby jako u folikulárního lymfomu- Burkittův lymfom - endemický výskyt v malarických oblastech Afriky (EBV, postižení dětí s typickou orofaciální

lokalizací, 100% vysoká prolferativní aktivia), je agresivní- agresivní CHT - potenciálně kurabilní

3. T-bunečné lymfomy- Anaplastický velkobuněčný lymfom (ALCL) – kožní (velmi dobrá prognóza)/ systémová varianta (postiženy hl

uzliny)- systémová varianta – CD30+, t(2;5) → ALK protein

- ALK+ - mladší, lepší prognóza, kurabilní- ALK- - starší, horší prognóza, extranodální postižení, ↑ LDH

- Perifení lymfomy blíže nezařaditelné (PTL NOS – not otherwise specified) – heterogenní sk, primárně nodální- nepříznivá prognóza, diagnostika v pokročilém (extranodálním) st.

- méně časté: angioimunoblastický T-lymfom – celkové příznaky, horečka, generalizovaná lymfadenopatie, hepatosplenomegalie, polyklonální gamapatie

Adult T-cell leukemia/ lymfoma (ATLL) – spojeno s HTLV-1- Kožní T-lymfomy - Mycosis fungoides - nejčastější, zpočátku indoletně

- svědivý enantém - premykotické, ekzematoidní stádium (olupující se ložiska podobná ekzému), infiltrační stádium - plošná, vyvýšená tuhá ložiska; tumorózní stádium - polokulovité nádory, rozpadající se ve vředy, generalizované st. - adenomegalie, viscerální postižení

- I.st - symptomaticky – lokálně kortikoidy, antipruriginóza- II - IIIst. - fotochemoterapie, lokální CHT (mustargen)

- Sézaryho syndrom - erytrodermie, generalizovaná adenopatie, neoplastické T-lymfocyty s cerebriformním jádrem = Sézaryho bb. v perif. krvi, kůži a v uzlinách

46

Page 47: lf1.czlf1.cz/wp-content/uploads/228-vypracovane_otazky_interna... · Web viewprognoza – 50 % dosáhne kompletního uzdravení do 6 měsíců, 20 % má prodloužený průběh –

Vypracované otázky na INTERNU made by Edita Homolková, Monika Ambroziová, Adam Král, Petr Vermach a Vítek Křehnáč

- nepříznivá prognóza- Primárně kožní CD30+ kožní lymfomy – 2. nejčastější sk. kožních T-lymfomů

- Primárně kožní velkovuněčný lymfom CD30+ (kožní ALCL) – lokalizované noduly/ tumory

- Lymfomatoidní papulka (LP) – papilózní až nodulózní výsev (trup, končetiny)- dobrá prognóza

- Podkožní T-lymfom podobný pankulitidě – mnohočetné podkožní noduly- Extranodální T-lymfomy jiné než kožní – NK/T-lymfom nazálního typu – hl. Asie a Střední a Jižní Amerika, vliv

EBV, agresivní- tumorózní infiltrace , kostní a cévní destrukce nasální

dutiny, paransal. sinů, nasofaryngu- T-lymfom spojení s enteropatií – častěji u celiakie, bolesti břicha,

perforace střeva, nepříznivá prognóza- Hepatosplenický T-lymfom – postihuje játra, slezinu a často i dřeň, bez

uzlin, celkové příznaky

23.A HEMATEMEZA, MELÉNA, ENTERORAGIE

Hematemeza- zvracení červené krve nebo zvracení s příměsí krve- příznak krvácení do horní části zažívací trubice, např. z peptického vředu (prudké arteriální krvácení) nebo

jícnových varixů (průměrně 500 ml) při cirhóze jater- vředy (a eroze (60 %), varixy (10%), Malloryho-Weissův sy (5%), ca žaludku (3%), zbytek neobjasněn- pokud je krev tmavé barvy má svůj původ obvykle v duodenu a je doprovázená melnou- při zvracení kávové sedliny při krvácení do žaludku, které spontánně ustalo- vyžaduje urychlené vyšetření gastroskopie a léčbu - masivní ztráta krve může vést k šoku

Meléna- dehtovitě černá a mazlavá stolice obsahující natrávenou krev s charakteristickým zápachem- příznak krvácení do horní části zažívací trubice, např. ze žaludečního vředu. Vyžaduje rychlé vyšetření, protože

silné krvácení může vést k šoku- >100 ml krve po 8 hodinách po krváceni- černé zbarvení stolice ovšem může být způsobeno i složením potravy borůvky či některými léky železo,

adsorpční uhlíEnteroragie

- krvácení ze střeva, zejm. tlustého (zdroj distálně od duodenojejunálního ohbí)- divertikulární choroba tlustého střeva (40 %), angiodysplazie (20 %), mezenteriální ischemie (10-15 %),

idiopatické střevní záněty, postendoskopicky (polypektomie), po NSA, postradiačně, infekce, při aortální stenóze (získaný deficit von Willbranbova faktoru)

- z tračníku (tmavě červené gelatinózní stopy krve, homogenně tmavě červené zabarvení stolice), krvavé průjmy (kolitida), z rekta (proužky světle červené krve na povrchu stolice, červená stolice po červené řepě

- na rozdíl od melény, při níž krev pochází z vyšších částí zažívacího ústrojí a je dehtovitě zbarvená částečným natrávením, krev při enteroragii má červené zbarvení

- může být způsobena střevními záněty proktokolitidou, úplavicí, hemoroidy, nádory aj.- vyžaduje vyšetření rektoskopií, koloskopií- déle trvající enteroragie může způsobit anemiiDg – endoskopie, koloskopie, rektoskopie

- zobrazovací metody – radiointervenční angiografie – krvácení do horní části GIT, při pokračujícím a recidivujícím krvácení (musí být >1 ml/min), po neodhalení krvácení endoskopií

Th – endoskopie - při krvácení do horních částí GIT je třeba před endoskopií vypláchnout žaludek, nemocný je při vyšetření monitorován (pulsní oxymetr, EKG, neinvazivní krevní tlak)

- provést ideálně do 2 hodin od přijetí (nejpozději do 12 hod od přijetí)- pacientovi je podána frakcionovaně analgo-sedace a kyslíková podpora.- Termokoagulační techniky - mikrovlnná koagulace, laserová fotokoagulace, argonová plasma

47

Page 48: lf1.czlf1.cz/wp-content/uploads/228-vypracovane_otazky_interna... · Web viewprognoza – 50 % dosáhne kompletního uzdravení do 6 měsíců, 20 % má prodloužený průběh –

Vypracované otázky na INTERNU made by Edita Homolková, Monika Ambroziová, Adam Král, Petr Vermach a Vítek Křehnáč

- Elektrotermokoagulační techniky - monopolární a multipolární elektrokoagulace- Injekční metody - injikovat adrenalin, sklerotizující látky nebo tkáňová (fibrinová) lepidla- Mechanické metody - nasazení speciálních kovových svorek („hemo-klipy“) a ligace (za použití

gumových kroužků)- farmakologie - i.v. terlipressin (Remestyp, syntetický analog vazopresinu, má výrazný vasokonstrikční a

protikrvácivý účinek) při krvácení komplikujícím portální hypertenzi (zdrojem: jícnové varixy, kongestivní gastropatie při portální hypertenzi) je v akutní fázi nejčastěji podáván

- i.v. nitráty, somatostatin a jeho analoga - snižující portální hypertenzi - úprava narušených hemokoagulačních poměrů (čerstvě zmražená plasma)- při non-variceálním krvácení je farmakologická léčba méně efektivní. Při kapilárním krvácení

(např. u postižení gastrointestinálního traktu při urémii) je kromě terlipressinu možno i.v. podat etamsylat (Dicynone, redukuje kapilární krvácení zlepšením funkce destiček (adheze), nemá vliv na hemokoagulační faktory)

- při krvácení při vředové chorobě je v akutní fázi i.v. aplikován H2-blokátor nebo inhibitor protonové pumpy (omeprazol, pantoprazol) (tyto léky doplňují, ale nenahrazují endoskopickou léčbu krvácení)

- k prevenci krvácení - u vředové choroby gastroduodena s pozitivním průkazem Helicobacter pylori - eradikace helicobacterové infekce

- u krvácení ze žaludku a z duodena - H2-blokátory nebo inhibitory protonové pumpy a dále cytoprotektivní léky (misoprostol, sukralfát)

- ke snížení portální hypertenze jsou podávány neselektivní β-blokátory

- Postup při krvácení z konečníku – anamnéza (jestli je to poprvé nebo opakovaně, jestli uz měl nějakou nemoc střeva, operaci nebo hemoroidy, jestli užívá antikoagulační léčbu, jestli má známky anemického syndromu)

- laboratoř - krevní obraz (jestli je pokles Hb - pak to je spíš protrahované krvácení, anebo pozor masivní!!, pak leukocyty a CRP - známky zánětu (?), mohlo by to být třeba spolu s průjmem, bolesti břicha, hledáme třeba infekční nebo jiný střevní zánět)

- vyšetřit p.r., jestli je to čerstvá krev nebo meléna, jestli není stenóza nebo hmatný tu, pokud je tam čerstvá krev → většinou koloskopie (může ukázat tumor - polyp - nebo ca, nebo někdy divertikulózu s divertikulitidou, nebo se najdou hemoroidy nebo píštěl, fissura, když ne, a pacient má obtíže → CT)

- zobrazovací vyšetření - začínáme sono břicha - zánět střevní stěny (?), tumor (?), malá pánev (?)23.B METODY NÁHRADY FUNKCE LEDVIN

- léčba terminální fáze renálního selhání: hemodialýza, peritoneální dialýza, transplantace ledvin1. Hemodialýza

- zahájení – cl urea >30 mmol/ l, cl kreatinin 600-800 μmol/ l, pokles clearance pod 0,25 ml/ s, u DM dříve- příprava - cévní přístup, očkování proti HBV, léčba erytropoetinem, sledování nefrologem (1/3 „pacientů z

ulice“)- princip - odstraňování látek difuzí, odstráňování vody a v ní rozpuštěných látek ultrafiltrací

- ↓ hladinu dusíkatých katabolitů, K, P, upravit acidobazickou rovnováhu (ABR), odstranit retinovanou tekutinu

- 3x týdně po 4 hod, variabilní - th - hypertenze, anémie, porucha kalciumfostátového metabolizmu

- styk krve s cizorodým materiálem - nebezp. TE kom, proto heparin (kontinuálně nebo intermitentně), možná i bezheparinová dialýza(?)

- cévní přístup - zajištění dostatečného přítoku krve (alespoň 200-300 ml/ min)- dočasný u akutního renálního selhání (ARS) – 2cestný katétr ve v. jugularis- trvalý – klasický arteriovenózní podkožní píštěl - radiocefalická fistule, žilní štěpy, protézy- komplikace - hematom, trombóza, aneuryzma, infekce, stenózy- hyperkinetická cirkulace - ↑ srdeční výdej (pozor u pacientů se ↓ systolickou fci)

- u srdečního selhání - permanentní žilní katetr ve v. jugularis/ subclavia

48

Page 49: lf1.czlf1.cz/wp-content/uploads/228-vypracovane_otazky_interna... · Web viewprognoza – 50 % dosáhne kompletního uzdravení do 6 měsíců, 20 % má prodloužený průběh –

Vypracované otázky na INTERNU made by Edita Homolková, Monika Ambroziová, Adam Král, Petr Vermach a Vítek Křehnáč

- dieta - dostatečný příjem bílkovin a energetický přísun, příjem tekutin dle diurézy- pacienti s oligoanurii - restrikce K a P, suplementace vitB, C

- akutní komplikace - hypotenze, křěče (příliš rychlá HD), arytmie (hypokalemie), krvácení (heparinizace)- chronické komplikace - projevy kom chronického selhání ledvin, i dialyzační th

- KV - ICHS, IM, srdeční selhání, CMP - častá morbidita, hospitalizace i mortalita (50 %)- obtížně ovlivnitelné - ani erytropoetin, statiny

- hypertenze - do 40 %pacientů, et. chronické převodnění (hodnotí se při suché hmotnosti), vhodné ACEI/ CCB/ β-blokátory

- hypertrofie LK - u 75 %, tlakové i objemové přetížení, častá systolická dysfunkce- uremická perikarditida - vzácně, známka nedostatečné HD, hemodialýza každý den- infekce - infekce cévních přístupů, uroinfekce (DM, polycystické ledviny), HBV, HCV- periferní polyneuropatie - sy neklidných nohou (vit. B)- uremický pruritus – 90 %nemocných, těžko ovlivnitelné (antihistaminika, UV, hemoperfuze)- amyloidóza - ukládání β2-mikroglobulinu do pojivovývh tkání (synovie,kosti, pouzdra šlach i nervů)

- KO – sy karpálního tunelu, bolesti kloubů, th - NSA, analgetika, fyzikální th- intoxikace Al3+ - dříve, když se používala neupravená voda

2. Peritoneální dialýza- princip - dialyz. membrána je peritoneum

- v břišní stěně peritoneální katétr (chirurg. zaveden) a dial. roztok se do břicha napouští a vypouští- kontinuální ambulantní peritoneální dialýza – roztok ponechán v břiše 6 hod, výměna 4x denně- automatizovaná peritoneální dialýza – stačí 1x denně i to v noci

- vhodná u dětí, oběhově nestabilních pacientů, pacientů s problematickým cévním přístupem- neomezuje tolik jako HD- KI - srůsty v břišní dutině, neřešitelné kýly, kolostomie, aktivní střevní onem., acsites, nespolupracující- komplikace – peritonitida, zejména stafylokoková (nesterilní manipulace)

- KO - teploty, bolesti, zvracení perit. dráždění nebo zelený dialyzát- th - celkové i lokální atb

3. Transplantace- dnes rutinní léčba CHSL- indikace - každý nemocný s CHSL bez KI- KI - ohrožují příjemce na životě při vlastní operaci nebo v pooper. období (SS, akutní infekce)

- nemoci omezující dožití (malignity, CHS jater, AIDS), nespolupracující - prognóza – 1 roční přežívání je víc než 90 %(závisí na HLA shodě, věku dárce příjemce, IS th, cytotoxické Ab)-dárce – živý - geneticky příbuzný – svobodně dobrovolně rozhodnutí, lepší přežívání štěpů

- zemřelý - smrt mozku potvrzená mozkovou panangiografií- bez známek irreverzibilního pošk. Ledvin, bez infekčních onemocnění- předpokládaný souhlas

-postup - štěp se ukládá do pravé jámy kyčelní, sleduje se rozvoj funkce štěpu (diuréza, sérový kreatinin)- imunosupresivní th - podmínka dlouhodobé fce štěpu – trvalá IS

- udržovací profylaktická IS – 3kombinace – KS, mykofenolát (azatioprim), takrolimus- antirejekční IS - zvrátit akut. rejekci (↑ dávky. i.v. KS, u rezistentních poly/ monoklon. Ab)

- poly a monoklonální Ab – taky po operaci u rizikových pacientů -NU - Cy-nefrotoxický, zhoršuje krevní tlak, takrolimus - nefrotox., azatioprim -

myelotoxicita, hepatotoxicita- KS chronicky - zhoršení hojení ran, osteoporóza, steroidní DM, zhoršení krev. tlak

- komplikace časné - akutní tubulární nekróza (ATN) (anurie, pomalý rozvoj fce štěpu)- rejekce – hyperakutní - vzácná, humorální forma rejekce, vznik do 24 hod. od operace

- AB0 inkompabilita, preformované Ab u příjemce proti HLA a +kříž. zk.

- akutní - vznik nejčastěji v prvních 3 měsících po transplantaci- KO - náhlé zhoršení fce štěpu (bolest, ↑ teplota, ↓ diurézy, hypertenze)

- příznaky mohou chybět- dif.dg. - ATN, Cy nefrotoxicita, obstrukce vývod. cest moč., trombóza cév- dg - histologie z perkutánní biopsie

49

Page 50: lf1.czlf1.cz/wp-content/uploads/228-vypracovane_otazky_interna... · Web viewprognoza – 50 % dosáhne kompletního uzdravení do 6 měsíců, 20 % má prodloužený průběh –

Vypracované otázky na INTERNU made by Edita Homolková, Monika Ambroziová, Adam Král, Petr Vermach a Vítek Křehnáč

- th - pulzy KS - většinou zabere, když ne, tak biolog. léčba (anti CD3)- moč. píštěl, trombóza cévy štěpu, infekce (moč.cest, ran )- IS - infekce CMV - leukopenie, teplota, orgánové postižení, th gancyklovir

- oportunní infekce - největší riziko je v prvních 6 měsících po T

- komplikace pozdní - KV – kvůli hyperlipidémii, obezitě, hypertenzi (částečně vlivem KS, Cy)- th - ACEI, antagonisty aldosteronu, blok. Ca kanálu

- DM - (KS)- chronické jaterní selhání – azatioprim, Cy, virové hepatitidy- maligní tumory - častější než v běžné populaci

- chronická nefropatie štěpu - postupně se zhoršující fce štepu- progreduje proteinurie, hypertenze hůř kontrolovatelná- prevence - ↓ incidenci a závažnost akut. rejekcí, th hypertenze, hyperlipidémie

24.A VIROVÉ HEPATITIDY

- inkubační doba: VHA (15-45 d), VHB + VHD (30-180 d), VHC (15-150 d), VHE (15-60 d)- u všech může infekce probíhat různou intenzitou- často proběhne inaparentně, abortivně (krátce, chřipkový či dyspeptický syndrom)- typická je forma ikterická, u všech je však častější forma anikterická- cholestatická forma vypadá jako obstrukční žloutenka - problémy v diagnostice- maligní či fulminantní forma - jaterní selhání, vysoká úmrtnost- v průběhu VH sledujeme prodromální stádium, stádium jaterního poškození a stádium rekonvalescence- stádium jaterního poškození trvá 2-8 týdnů, když se to nezlepší - protrahovaný průběh - až do chronicity (zánět

trvající déle než 6 měsíců)- nákaza může též vést k asymptomatickému nosičství- v období rekonvalescence jsou časté funkční poruchy GIT (nechutenství, nesnášení tučných jídel...), často též

posthepatické hepalgie (bolesti v krajině jater - podmíněno srůsty mezi jaterním pouzdrem, bránicí a peritoneem, vymizí)

- první biochemická změna u VH je zvýšení transamináz - již v prodromálním stádiu- u akutních stoupá více ALT než ASTHepatitida A

- virus značně odolný na vnější vlivy, vylučuje se stolicí, už 2 týdny před vznikem příznaků a ještě asi týden po skončení, nemocný je nejvíce nakažlivý před koncem inkubace

KO - závažnost nákazy stoupá s věkem (u malých dětí je 90% asymptomatických), jaterní selhání je vzácné, v 10% protrahovaná forma (ale nevede do chronicity)

Dg - vyšetření specifických protilátek IgM v séru, negativita testu u imunokompetentních osob nákazu vylučuje- IgM přetrvávají v séru 3-6 měsíců po nákaze

Hepatitida B- jedno z nejzávažnějších virových onemocnění člověka, nositelem viru 5% populace - přenos parenterálně, pohlavním stykem, perinatálně- každý pátý nosič umírá na cirhózu, každý devátý na hepatocelulární caPtg - jeho genom je částečná dvoušroubovice cirkulární DNA, genom je jedinečný, každá část DNA něco kóduje (i

víc věcí než jednu)- HBcAg - „core“ - nukleokapsidový protein, má před sebou ještě část DNA tzv. pre-core a když se to přeloží

celý (pre-coe + core) - vznikne HBeAg- HBsAg - „surface“ - dává ho na membránu hepatocytů, je na obalu viru a mnoho tohoto antigenu

produkují hepatocyty volně do oběhu- DNA polymeráza - reverzní transkriptáza, zabírá většinu genomu- HBX - protein z regionu X, je nezbytný pro virovou replikaci, předpokládá se, že hraje roli při vzniku

hepatocelulárního caKO - často inaparentně (hlavně děti a imunodef. pacienti)

- Akutní hep B - prodromální stádium - až několik týdnů, často artralgie či vyrážky- žloutenka trvá déle než u VHA- nejhorší komplikace - rozvoj jaterního selhání - hlavně u starších a vyčerpaných

50

Page 51: lf1.czlf1.cz/wp-content/uploads/228-vypracovane_otazky_interna... · Web viewprognoza – 50 % dosáhne kompletního uzdravení do 6 měsíců, 20 % má prodloužený průběh –

Vypracované otázky na INTERNU made by Edita Homolková, Monika Ambroziová, Adam Král, Petr Vermach a Vítek Křehnáč

- do chronicity - 90% novorozeneckých, 30-40% dětských, 5-10% dospělých- Chronická hep B - buď jako následek akutní nebo primárně - bez zjevné akutní fáze

- při replikaci vznikají různé mutanty viru - precore mutanta – (netvoří HBe antigen - častěji jaterní selhání, rychlejší cirhóza, málo citlivá na interferon)

- Asymptomatické nosičství VHB - bez klin., biochem. ani bioptických příznaků, jen přítomnost HBsAg v séru- může spontánně vymizet, ojediněle se z toho může vyvinout chronická VHB, proto se

takovýto objevivší se pacient dispenzarizuje, zjistíme HBeAg a anti HBe, abychom zjistili aktivní replikaci viru, zjišťujeme ALT

- nebezpečí v těhotenství - 90% riziko, že během porodu nakazí dítě (proto se provádí vyšetření na HBsAg u všech těhotných, když je pozitivita, novorozenec se pasivně imunizuje)

Dg - sérologie pomocí ELISA- jako první známka nákazy - HBsAg (i několik týdnů před ostatním), v průběhu úzdravy ze séra mizí- když se nedá prokázat HBs (u malého množství VHB), zjistím IgM proti HBc- vyšetření replikační či integrované fáze chronické VHB - je e antigen - HBe a IgM proti ní svědčí o replikační

fázi, anti-HBc zůstávají celý život a jsou markery prodělané infekce, anti-HBs vznikají při akutní infekci ale hlavně po vakcinaci

- nejcitlivější marker infekce - PCRHepatitida C

KO - většina proběhne subklinicky, průběh zpravidla anikterický, ikterus častěji u starších- jaterní selhání vzácně, z 80% přechází do chronicity, vývoj cirhózy je pomalý, urychluje ho VHB a alkohol- asymptomatické nosičství je vzácné

Dg - průkaz Ig anti-VHC- tvorba Ig může být opožděna, při podezření na akutní VHC, opakovat- malá souvislost mezi biochemií a histologií (i mírně zvýšené ALT, velké změny)

Hepatitida D- původce je defektní virus delta, k replikaci potřebuje VHBKO - superinfekce vede ke zhoršení jaterních funkcí - selhání nebo rychlejší cirhóza, při současné infekci je to

většinou lepšíDg - specifické Ig ELISou

Hepatitida E- virus se šíří fekálně-orální cestou (v rozvojových zemích s nedostatečnou úrovní zásobování nezávadnou vodou,

v ČR ojediněle, většinou import)- klinicky podobná VHA, žloutenka výraznější, selhání častější- do chronicity nejde, závažné u těhotných

Komplikace- cholestáza (5%, hepatitida s intrahepatální cholestázou) - výrazné ↑ bilirubinu a cholestatických enzymů- protrahovaná a recidivující hepatitida – konst./ intermit. > transaminázy > 3 měsíce, ale < 6 měsíců- fulminantní hepatitida - VHA, VHB i VHC mají riziko < 1 %, VHE < 2% (u těhotných až 20%), VHD > 2%

KO - těžký ikterus, ascites, hemoragická diatéza, rozvoj jaterního komatu- extrahepatální komplikace - imunokomplexový sy (při VHB až 10%, artralgie, event. exantém)- perzistence viru (nosičství) - HVB (dospělý 5-10%, novorozenec > 90%), VHC (80%), VHD (pň superinfekci nosiče

HBsAg > 90%)- chronická hepatitida - nezhojení hepatitidy do 6 měsíců- hepatocelulární ca - pozdní komplikace VHB a VHC, závisí na věku nákazy (horší u novorozenců), genetických

faktorech (horší u asiatů), zvýšené riziko při chron. hepatitidě, jaterní cirhóze, kouření, alkoholismu apod.

Diferenciální diagnóza hepatitid- anamnéza - RA - nemoci jater, žlučových cest, familiární hyperbilirubinemie a hemolytické stavy- OA - totéž, nemoci, které by mohli průběh VH ovlivnit - cukrovka, imunodeficity, operace, transfúze krve- Epidemiologická A - pátráme po styku s infekcí, jiné infekce v okolí, očkování, možnost nákazy sexuálně, pobyt

v přírodě (závadné zdroje vody...)- FA - antikoncepce! a hormonální preparáty, antidepresiva, analgetika- Abúzus - drogy- NO - příznaky, které předcházely

51

Page 52: lf1.czlf1.cz/wp-content/uploads/228-vypracovane_otazky_interna... · Web viewprognoza – 50 % dosáhne kompletního uzdravení do 6 měsíců, 20 % má prodloužený průběh –

Vypracované otázky na INTERNU made by Edita Homolková, Monika Ambroziová, Adam Král, Petr Vermach a Vítek Křehnáč

- může jít o poškození při jiných infekcích, toxické poškození alkoholem, hepatotoxiny, houby- vždy je nutné vyloučit žloutenku z obstrukce žluč. cest- dynamika ALT - největší hodnoty transferáz - virové hepatitidy (prodromální stádium - 2x, po dvou týdnech až

50x zvýšeny, okolo 8. týdne se normalizují)- rychlý vzestup (cca 30x) - toxické poškození jater- lékové a alkoholové intoxikace - mírnější vzestup

Terapie virových hepatitid- základ - klidový dietní režim- léčba akutních hepatitid - symptomatická a podpůrná, kromě VHC, kde časná léčba interferonem může

zabránit přechodu do chronicity- z infekčního odd. odchází postižení po zmizení žloutenky a poklesu transferáz pod trojnásobek normy,

následuje kontrola na hepatologii- interferon, z virostatik - lamivudin

Dif. Dg - virové infekce - herpesviry, EBV (výrazná lymfadenopatie), CMV, coxsackie, exotické viry jako arboviry (žlutá horečka, horečka Dengue), arenaviry, Marburg, Ebola

- bakteriální infekce - brucelóza, Q horečka, leptospiróza- parazitární infekce - malárie, schistozomóza, jaterní motolice, poléková a alkoholová hepatitida- exacerbace chronické hepatitdy, ostatní jaterní onemocnění (autoimunitní hepatitida, PBC, nádory)

Dg - A, SP, screening (HBsAg, HCV-RNA)- lab - ↑ transamináz (de Ritis kvocient = AST/ALT <1). pň cholestáze (↑ sérový bilirubin, ↑ bilirubin a

urobilinogen v moči, ↑ GMT a ALP), ↑ Fe, elektroforéza (↑ γ-globulinová frakce), event. ↑ zánětlivé markery (leukocytóza, ↑ FW, ↑ CRP)

- při těžkém/ fulminantním průběhu známky jaterní insuf. (↑ cholinesteráza, ↓ Quickův čas, ↓ albumin)

- sérologie - IgM (průkaz nedávné infekce), IgG (průkaz prodělané infekce, výjimkou je VHB, kdy při akutní fázi i exacerbaci je > anti-HBc IgM)

Prog - pravděpodobnost vyléčení VHA (100%), VHB (90%), VHC (s virostatiky > 50%), VHD (90% při koinfekci, špatná prognóza při infekci nosiče HBsAg), VHE (98%, u těhotných 20% fulminantní průběh)

Profylaxe - aktivní imunizace- VHA (jedinci anti-HAV neg. se zvýšeným rizikem infekce (cestovatelé, zdrav, pracovníci), 2 dávky IM v odstupu 6 měsíců), VHB + VHD (známka úspěšnosti = titr anti-HBsAg > 100 j./l)

- pasivní imunizace - VHA (I - kontakt s nemocným s VHA, podat do 10 dnů), VHB + VHD (I - novorozenec HbsAg + matky, poranění neočk. personálu krví, očkuje je vždy aktivně i pasivně, nejlépe do 6 hodin)

Sérologické markery VHA

 

Sérologické markery relabující VHA  Sérologické markery akutni VHB Sérologické markery chronické hepatitidy B

52

Page 53: lf1.czlf1.cz/wp-content/uploads/228-vypracovane_otazky_interna... · Web viewprognoza – 50 % dosáhne kompletního uzdravení do 6 měsíců, 20 % má prodloužený průběh –

Vypracované otázky na INTERNU made by Edita Homolková, Monika Ambroziová, Adam Král, Petr Vermach a Vítek Křehnáč

24.B ONEMOCNĚNÍ PŘÍŠTÍTNÝCH TĚLÍSEK

Primární hyperparatyreóza- autonomní nadprodukce PTH- častěji ženy, 400/ 100 000/ 1 rok- prognóza příznivá, chirurgické řešení kurativní v 95 %, horší u vzácných karcinomůEtio – 80 % adenom, 15 % primární hyperplazie, karcinomKO – osteoporóza (hl. úbytek kortikalis), osteitis fibrosa cystin (ložiska subperiostální resorpce, lebka, falangy),

↑ ALP- nefrolitiasa (u 15 %), nefrokalcinóza, možná progrese poruchy koncentrační schopnosti ledvin → polyurie,

polydipsie, renální insuficience- peptický vřed žaludku a duodena, akutní pankreatitida, dyspepsie, nauzea, zvracení, zácpa- častěji arteriální hypertenze, poruchy rytmu až zástava (v systole)- slabost, únavnost, poruchy osobnosti, deprese- svalová slabost, únavnost, změny EMG- epulis gigantocellularis, chondrokalcinóza, záchvaty pseudodny

Dg – hyperkalcémie (norma 2,2-2,7 mmol/l) i jen intermitentní, hypofosfatémie (norma 0,65-1,65 mmol/l), může být mírná metabolická hyperchloremická acidóza, hyperfosfaturie, hyperkalciurie, ↑ PTH

- denzitometrie kostí, skiagram lebky a rukou, renální fce, USG krku, MR a CT při podezření na retrosternální lokalizaci

Th – paratyreoidektomie- estrogeny u postmenopauzálních žen, bisfosfonáty, selektivní modulátory estrogenních receptorů,

kalcimimetika (aktivují kalcium sensing receptor → ↓ sekrece PTH)- léčba hyperkalcémie – hydratace, někdy forsírovaná diuréza, bisfosfonáty parenterálně (pamidronát),

kalcimimetikum cinacalcet, hemodialýzaSekundární a terciární hyperparatyreóza

Sekundární hyperparatyreóza - ↑ PTH v důsledku hypokalcémie (negativní kalciové bilance)- postupně může docházet k hypertrofii tělísek

Terciární hyperparatyreóza – autonomní sekrece PTH v hyperplastických tělískách vzniklých na základě sekundární hyperparatyreózy

- hyperkalcémie, častěji u lidí s pokročilou renální insuficiencíHypoparatyreóza

- syndrom z nedostatečné sekrece PTH vedoucí k hypokalcémii- vzácnéEtio – chirurgicky odstraněná příštítná tělíska – komplikace hlavně tyreoidektomie

- tranzitorně po odstranění adenomu tělísek (fyzio utlumení těch zdravých)

- idiopatická – autoimunitní postižení, i v rámci polyglandulárního syndromu- familiární – vzácné

- AR – mutace genu pro PTH nebo transkripčního faktoru- AD – mutace genu pro kalcium sensing receptor (konstituční aktivace)

- DiGeorgův syndrom, Wilsonova choroba (ukládání Cu), těžká hypomagnezémie, infiltrace metastázamiKO – rychlé ↓ Ca2+ - tetanické křeče, ↑ nervosvalová dráždivost, karpopedální spasmy, křeče, parestézie

v obličeji a končetin- pomalé ↓ Ca2+ - psychické příznaky, slabost, únavnost, apatie, poruchy osobnosti, úzkost, poruchy visu

v důsledku katarakty, poruchy intelektu- Chvostkův příznak (záškub svalů obličeje při poklepu na tvář před čelistním kloubem v

místě, kde probíhá obličejový nerv n. facialis), Trousseasův příznak (křečovité postavení ruky vyvolané omezením přítoku krve do horní končetiny. Je známkou latentní tetanie), hyperaktivita šlachových reflexů, suchá kůže s deskvamací, křehké nehty, vypadávání vlasů, katarakta, ↑ nitrolební tlak

Lab – hypokalcémie, hyperfosfatémie, ↓ PTH, u chronické formy kalcifikace bazálních ganglií, méně pak kalcifikace kůže, prodloužení QT intervalu

Th –chronická forma – vit D (8000-150000 UI/den D2), suplementace Ca (1-2 g/den)

53

Page 54: lf1.czlf1.cz/wp-content/uploads/228-vypracovane_otazky_interna... · Web viewprognoza – 50 % dosáhne kompletního uzdravení do 6 měsíců, 20 % má prodloužený průběh –

Vypracované otázky na INTERNU made by Edita Homolková, Monika Ambroziová, Adam Král, Petr Vermach a Vítek Křehnáč

- akutní forma – Ca i.v. (kalcium glukuronát), kalcitriol, Mg Pseudohypoparatyreóza

- dědičné onemocnění, rezistence tkání na PTH

25.A ARTERIÁLNÍ HYPERTENZE

- nejčastější KVO, prevalence 20-50 %- rizikový faktor pro ICHDK, ICHS, CMP- normální TK = 130/85 mm Hg (optimální 120/80)= opakovaně vysoký TK - 140/90 mm Hg, izolovaná systolická hypertenze >140/<90 mm Hg- hypertenze – 1. Stupeň – 140-159/ 90-99

- 2. Stupeň – 160-175/ 100-109- 3. Stupeň - >180/ >110

Pat - ↑ minutový srdeční objem, ↑periferní rezistence nebo obojí (TK = MSO * PR)Klasifikace - Primární (esenciální) hypertenze - nejčastější (>90 %), často součást metabolického syndromu

(obezita, DM2, dyslipidemie, hyperurikémie, EHT)- ptg neznámá, kombinace multifaktoriálni dědičnosti s rizikovými faktory

(obezita, nadbytek solení, stres), faktory genetické, nerovnováha endogenních mechanismů s převahou presorických faktorů

Dg - vyloučení příčin sekundární hypertenze (per exclusionem)- Sekundární hypertenze (<5%) – důsledek jiného přesně def. patologického stavu

- viz ot. 73.B- těhotenská – v těhotenství tlak značně kolísá (nejdřív klesá a v 3. trimestru roste)

- klasifikace – neexistující – v těhotenství pokračuje z dřívější doby- těhotenstvím indukovaná – objeví se po 20. týdnu a není doprovázená

proteinurií- kombinace obou přdchozích – v těhotenství vysoký TK ještě roste a

objeví se proteinurie- preeklampsie – nově vzniklá hypertenze s proteinurií po 20. Týdnu

- systola nad 160 a/ nebo diastola nad 110, bolesti hlavy, poruchy vizum krvácení do sítnice, edém papily…

- eklampsie – známky preeklampsie s těžkou hypertenzí a generalizovanými křečemi či komatem

- komplikace: abrupce placenty, CMP, plicní edém, DIC, intrauterinní růstová retardace, předčasný porod, umrtí plodu

Th – klid na lůžku, hospitalizace- metyldopa- atenolol, metoprolol, amlodipin

Dle vývojových stádií:Stadium I – prosté zvýšení TK bez orgánových změnStadium II – přítomny orgánové změny (hypertrofie LK - zvedavý úder hrotu, stěna >12 mm tlustá,

mikroalbuminurie pak proteinurie, aterosklerotické změny koronárních tepen)...bez změn funkce orgánů

Stadium III – orgánové změny s poruchou nebo selháváním jejich funkce (CMP, renální selhání, srdeční selhání) Stadium IV – maligní hypertenze – náhlý vzestup TK s fibrinoidní nekrózou arteriol a malých arterií

(neuroretinopatie, renální selhání na podkladě nefrosklerózy, hypertenzmí encefalopatie a rychle progredující levostranné selhání)

KO - dlouho asymptomatický, časné ranní bolesti hlavy (ustupují se zvýšením polohy hlavy), hučení v uších, nervozita, závratě, palpitace, námahová dušnost, epistaxe; dále viz dělení dle stádií

Dg – stanovit, zda jde o hypertenzi, urcit jeji tíži + orgánové změny, vyloučit sek. hypertenzi- A, SP, TK i pulz na obou HK (koarktace aorty, při níž je na pravé ruce vyšší), poslech břicha (?aneuryzma, ?

stenóza a. renalis), oční pozadí)- Lab - vyšetření moči (bílkovina, clearence za 24 hodin, katecholaminy/ 24 hod (feochromocytom)), iontogram

(hypokalémie (?Connův sy, glykémie, cholesterol, TAG, dexametazonový test (?Cushingův sy)

54

Page 55: lf1.czlf1.cz/wp-content/uploads/228-vypracovane_otazky_interna... · Web viewprognoza – 50 % dosáhne kompletního uzdravení do 6 měsíců, 20 % má prodloužený průběh –

Vypracované otázky na INTERNU made by Edita Homolková, Monika Ambroziová, Adam Král, Petr Vermach a Vítek Křehnáč

- EKG, RTG hrudníku, echo, doppler sono ledvin (?stenóza)Kompl - emergentní hypertenzní krize - při náhlé vzniklé těžké hypertenzi, vede k orgánovým poškozením

- hypertenzní encefalopatie - riziko CMP, hyperperfúze mozku s edémem mozku a papily- levostranné srdeční selhání - riziko edému plic

- předčasná arterioskleróza- ICHS + levostranné srdeční insuficience- poškození mozku- poškození ledvin - benigní nefroskleróza, časně mikroalbuminurie- aneuryzma břišní aorty - nejčastěji infrarenálně, indikace k operaci při průměru >5 cm- disekce aorty

Th – viz ot. 92.B

25.B GLOMERULONEFRITIDY (GN)

- imunologický proces se zánětlivým charakterem postihující glomeruly -v glom. Najdeme neutrofily, makrofágy, proliferaci mesangia (v širším smyslu sem zarazujeme i glomerulopatie,

ktr. nemají vyjádřenou zánětlivou složku, nemají aktivované imunit. mechanizmy, na nemocech glom. se totiž podílejí různé etiologie)

Dělení – klinické – primární - izolované postiž. ledvin- sekundární - při onem. postihujícím i jiné orgány

- časový vývoj – akutní - renální insuficience za několik dnů- RPGN - rychlá ztráta fce, chron. selhání ledvin za několik týdnů- chronické - vyvíjejí se léta, progrese do chron. ren. insuficience s náhradou fce ledvin

Etio - genetické faktory- imunitní f. - depozice imunokomplexů, cirkulující Ab

- v konečném důsledku dojde k poškození kapilár proteázami (aktivovaný komplement nebo makrofágy), fibrin se dostane do moč. prostoru a vyvíjí se srpky

- infekce - sekundární příčiny někt. GN- metabolické a hemodynamické f. - DM, amyloidóza, hypertenze

Dg - KO- biopsie - světelná mikroskopie, IF, elektron. mikroskopie

- význam - samotné stanovení dg, posoudíme aktivitu nemoci (agresivita th), chronicitu (prognóza)Primární Glomerulopatie (GP)

Neproliferativní GN- nefrotický sy (velká protinurie selektivní/ neselektivní), není zvýšená buněčnost glom., poškození podocytůNefrotický syndrom s minimálními změnami glom. (minimal change disease)

- nejčastější příčina NS u dětí (do 90%), u dospělých 20% (sekund. Příčina - tumor)- etio – nejasná, ↑ propustnost pro albumin (asi nějaký cirkulující permeabilní faktor)- KO - náhlý vznik otoků dolních konč. očních víček

- není hematurie, hypertenze, pokles RF- labornš selektivní proteinurie (albuminurie)- ↓ celková bílkovina i albumin

- hist. obraz - elektrón. mik. - splynutí pedicel podocytů, jinak nic (děti nebioptujeme, nejdřív se zaléčí, když nezareagují, tak bioptujeme)

- th – prednison 1mg/ kg 8-12-16 týdnů (když vyvinou remisi, postupně ↓) (kostikosenzitivní)- relapsy - obdobně (kortikodependentní - po snížení dávky relaps)- nereagují = kortikorezistentní CFA 2 mg/ kg/ den nebo CyA 5 mg/ kg/ den (není často

úspěšná)- symptomatická th NS (ACEI, hyperlipidémie, trombotické komplikace)

- komplikace th - poruchy růstu u dětí, obezita, porucha glukózivé tolerance, dyslipid., oční katarakta, osteoporoza

- prognóza - po úspěšné th. uplná normalizace RFFokálně segmentální glomeruloskleróza (FSGS)

- skleróza části glom. trsů některých glom. (část glom. a některých glom.)- etio - primární

55

Page 56: lf1.czlf1.cz/wp-content/uploads/228-vypracovane_otazky_interna... · Web viewprognoza – 50 % dosáhne kompletního uzdravení do 6 měsíců, 20 % má prodloužený průběh –

Vypracované otázky na INTERNU made by Edita Homolková, Monika Ambroziová, Adam Král, Petr Vermach a Vítek Křehnáč

- sekundární - genetické predispozice- infekce (HIV), morbidní obezita, ageneze 1 ledviny, vezikoureterální reflux v

dětství- pravděpodobně se obnažená glom. membrána nalepí na pariet. bb. Bowman. pouzdra

- KO - asymptomatická proteinurie (PU) nebo plně vyjádřený NS (+ mikroskop. hematurie, hypertenze, ↓ RF)

- dg - biopsie- prog - závisí na vstupní PU, RF, odpověď na th KS, dlouhodobá je závažná- th - malá asymp. PU - neléčime, IS th jen pro primární FSGS

- NS - vysoké dávky KS (aspoň 3měs.)- kortikorezistentní - Cy- 5 mg/ kg aspoň 5 měs. (časté relapsy, ale nižší riziko progrese do

chron. RI)Membranózní nefropatie (MN)

- stluštění bazální membrány (BM) a depoziceimunokomplexů mezi BM a podocyty- BMdělá výrustky (spikes), IK depozita se inkorporují- nejčastější příčina NS u dospělých (nad 40 let)- dg - nález na svět. mik., IF – IgG + komplement C3, elektron. mik. - depozita- idiopatické-sekundární – polékové - zlato, penicilamin, NSAI (nesteroidní antiflogistika)

- infekční - HBV, syfilis, malárie, EBV- tumory - solidní (colon, plice, žaludek...) i lymfomy- systémový lupus erytematodes - typ V

- KO – proteinurie, mikroskopická hematurie, hypertenze + ↓ RF se přidružují sekundárně- prog - třetina běhěm 10 let vyvine CHRI

-60 % neléčených - spontánní remise - th - dle etio., vyloučení tumorů (RTG, sono, gynekolog. vyš, vyš. prostaty, okultní krvácení),

vyloučení infekcí - pak dg idiopatické MN- dále dle tíže PU - do 4 g/ l nízká - renoprotektivní th (ACEI, sartany, aliskiren, diuretikum),

pozorování- 4 – 10 g/ l střední – kombinovaná KS + chlorambucil (v pulzech, 15 dní th,

15 dní přestávka)- nad 10 g/ l těžká - KS + CyA

- vysoké riziko tromboembolických kom. - antiagregační thProliferativní GP

- smíšený moč. nález – proteinurie + hematurie, zmnožení bb. uvnitř glom- vyšetření hematurie - fázový kontrast - čerstvá moč (na vyš. posíláme pacienta, ne moč, pacient se má 1 hod

před vyš. vymočit, vypít max 100 ml)- glom. Krvácení - nález akantocytů, sférocytů (víc než 5 %)- dif. dg. HU (neglom.) - infekce, litiasa, tumory intraureterálně, v pánvičce, záněty

intersticia, warfarin INR 10, trombocytopenie (ITP)- HU fyziologicky po infekci, tonsilektomii, vakcinaci, traumatu, velké tělesné námazeMesangioproliferativní GN

IgA nefropatie - depozice IgA v mesangiu glom. (biopsie, IF)- nejčastější GN ve světě i u nás- etio nejasná, nejspíš porucha terminální glykosylace IgA- sekundární - chron. jaterní choroby, neoplastické syndromy, idiopatická proktokolitis,

ankylozující spondylartritida, mykosis fungoides, sprue, dermatitis herpetiformis, HIV

- KO - variabilní průběh, někteří jen mikroskop. HU, ataky makroskop. HU při nějaké infekci

- kolem poloviny pomalá progrese do RI (za 20let)- špatná prog. - rychlejší progrese (podle tíže PU, hypertenze, RF)

- th - th hypertenze - ACEI, antagonisté aldosteronu- th PU-KS- progrese do RI-KS + CFA

56

Page 57: lf1.czlf1.cz/wp-content/uploads/228-vypracovane_otazky_interna... · Web viewprognoza – 50 % dosáhne kompletního uzdravení do 6 měsíců, 20 % má prodloužený průběh –

Vypracované otázky na INTERNU made by Edita Homolková, Monika Ambroziová, Adam Král, Petr Vermach a Vítek Křehnáč

Henoch-Schönleinova purpura - multiorgánová vaskulitida dětí- postižení ledvindo 1/2 pacientů- KO - artralgie, břišní koliky, purpura extenzorové části dolních i horních

konč.- po zaléčení bez následků

SLE typ II - větš. malý moč. nález, th podle extrarenální aktivity onem.Membranoproliferativní GN

Membranoproliferativní GN - difuzní postožení všech glom., zesílení BM (až 2kontura), IF svítí IgG, IgM- v jakémkoli věku- KO - těžká PU, NS, HU, ↓ hladina komplementu- 3 typy - dle hist. obrazu (umístění depozit)

- mechanizmu aktivace (aktivace komplementu)- prog - většinou nepříznivá

SLE typIII (fokální proliferativní GN),typ IV (difuzní proliferativní GN) – th - kombinovaná IS – KS + CFA (pulzy) nebo mykofenolát, CyA

Akutní postinfekční GNAkutní poststreptokoková GN - steriní následek streptokokové infekce

- vzácná (atb), častější subklinické formy (v rodinách, kolektivech), mladší věk. skupiny

- může i jiné agens (stafylokoky, syfilis, malárie, coxackie, EBV, HBV...)- po tonsilitidě, faryngitidě, pyoderma, endokarditidě či břišním abscesu- KO – 1-2 týdny po inf. v krku, 3-6 týdny po kožní inf.

- makroskop HU – moč barvy hnědého čaje, coca coly- periorbitální otoky - akutní nefritický sy- nespecif. Příznaky: slabost, nevolnost, nechutenství- v krvi ↑ ASLO, aktivovaná klasická cesta komplementu

- biopsie se obvykle nedělá, u sporných př.- proliferace mesangiálních i endotel. bb., IF- + C3 a IgG

- th – symptomatická - otoky, hypertenze- klidový režim + restrikce solí a tekutin- většinou spontánní úprava, když ne KS- nosiči - atb (pnc, klaritromycin, roxytromycin), omezit kontakt

- prog. - zhojí se bez následků

26.A TUBERKULÓZA PLIC

- podléhá povinnému hlášení- podezření na TBC - osoba u které se objevily příznaky nebo znaky vzbuzující podezření na TBC- případ TBC - nemocný s prokázanou TBC na základě bakteriologického průkazu - definitivní případ TBC - nemocný s prokázaným mycobacterium tuberculosis complex- granulomy s kaseifikační nekrózou- zdrojem je člověk vylučující mykobakteria přímo mikroskopicky (ve sputu), inkubace 4 týdny až 2 rokyEtio - původcem TBC je mycobacterium tuberculosis (Kochův bacil), aerobní nesporulující a k zevním vlivům

poměrně odolná tyčkaKlinické formy - Latentní forma – většina osob infikovaných TBC neonemocní

- Manifestní forma – manifestace závisí na mnoha faktorech (genetika, celkový stav jedince, virulence a počet inokulovaných mykobakterií, délka a intenzita expozice infekci), vyšší riziko mají starší, imunokompromitovaní

- Primární TBC – reakce hostitele naprvní kontakt s mykobakteriemi, vytváří primární complex (ložisko infekce + postižení uzliny) → mykobakteriémie, většinou spotnánní zhojení + přecitlivělost na tuberkulin

- Postprimární TBC – vzniká u osob již infikovaných a nejčastěji postihuje plíce (reinfekce/ reaktivace tbc)

KO - primární TBC - většinou u dětí, výjimečně se projeví teplotou malátností a nechutenstvím

57

Page 58: lf1.czlf1.cz/wp-content/uploads/228-vypracovane_otazky_interna... · Web viewprognoza – 50 % dosáhne kompletního uzdravení do 6 měsíců, 20 % má prodloužený průběh –

Vypracované otázky na INTERNU made by Edita Homolková, Monika Ambroziová, Adam Král, Petr Vermach a Vítek Křehnáč

- postprimární TBC - probíhá akutně nebo chronicky a projevuje se nápadnou únavou, nechutenstvím, hubnutím, ↓ výkonnosti, subfebriliemi, nočním pocením a kašlem

- kašel je zprvu suchý, později produktivní s produkcí mukopurulentního sputa nebo s hemoptýzou, dušnost závisí na závažnosti procesu

- může přejít ve fibrokavernózní postižení plic nebo proběhnout rychle jako tuberkulózní pneumonie

- miliární TBC - generalizované onemocnění s rychlým celkovým chátráním a subfebriliemi, vzácně probíhá pod obrazem septického stavu s hepatosplenomegalií (Landouzyho sepse)

- často je snížená imunita a nemocný bude tuberkulin negativní- TB pleuritida – vyskytuje se asi ve 3 % tuberkulóz – vzniká serózní exsudát s velkým množstvím fibrinu,

makrofágů a lymfocytů- TB nitrohrudních uzlin - dnes vzácné – kašel s možnou hemoptýzou při negativním skiagramu hrudníku - mimoplicní TBC - vyskytuje se cca u 15% nemocných, může být postižen jakýkoliv orgán

- Uzliny – krční, axilární nebo inguinální (nebolestivé zvětšení bez erytému, někdy centrální fluktuace)

- Skelet – nejčastěji páteř, osteolytické změny imitují metastázy, můžou vznikat artritidy - GIT – postižení ileocékální oblasti - Ledviny – unilaterální pyelonefróza s hematurií a dysurií (často pyurie bez nálezu bakterií),

bývá i zánět varlat i prostaty- Postižení kůže – je vzácné, projevuje se jako skrofuloderma (ulcerace podkožních uzlin),

lupus vulgaris (torpidní nafialovělá pomalu progredující léze na obličeji a končetinách), nodózní erytém, indurativní erytém

- Bazilární meningitida – prodromy: slabost, anorexie, bolest hlavy a zvracení- typické příznaky: krční ztuhlost, parézy hlavových nervů, delirium,

kómaDg - fyzikální nález – často bývá normální nebo velmi chudý – vlhké chrůpky, bubínkový poklep nad kavernou,

oslabené dýchání a zkrácený poklep u pleurálního výpotku - RTG – nález je charakteristický ale není specifický – TB ložiska a infiltráty v dorzálním a apikálním segmentu

horního laloku nebo v apikálním segmentu dolního laloku (stejnou lokalizaci mívá tuberkulom), podezření na TBC zvyšují rozpadové procesy

- miliární rozsev, uzlíky v horních partiích plic subpleurálně, zvětšené nitrohrudní uzliny, pleurální výpotek nebo fibróza

- HRCT – přesnější detekce drobných lézí - Mikrobiologické vyšetření – vyšetření sputa odebraného nalačno, cílené vyšetření aspirátu bronchiálního

výplachu nebo tekutiny z BAL- Histologie – vzorky plic, pleury nebo uzlin (biopsie), nález kaseifikujícího epiteloidního granulomu- Tuberkulinový test – průkaz opožděné hyperreaktivity proti antigenům mykobakterií (nemá pro stanovení

diagnózy zásadní význam)Th -kombinovaná, dlouhodobá, kontrolovaná léčba antituberkulotiky

- minimální doba léčby je 6. měsíců – 2 měsíce ústavní léčba 4kombinací antituberkulotik a následně ambulantní léčba 2- až 3kombinací antituberkulotik

- Antituberkulotika: Rifampicin – 10 mg/ kg per os max. 600 mgIzoniazid – 5 mg/ kg per os max. 300 mgPyrazinamid - 25 mg/ kg per os max. 2 g Etambutol - 25 mg/ kg per os v denním režimu pak 15 mg/ kg denně + intermitentně 2 x

týdně 45 mg/ kg (3x týdně 30 mg/ kg)Streptomycin - 1g i.m

- rezistence – multirezistence (rezistence na rifampicin a izoniazid), polyrezistence (rezistence na izoniazid, rifampicin, flurochinolony a jeden aminoglykosid) extrémní polyrezistence (rezistence na základní antituberkulotika, flurochinolony a aminoglykosidy – vyléčitelnost nižší než 50 %)

26.B SYSTÉMOVÁ SKLERODERMIE, POLYMOSZITIDA, DERMATOMYOZITIDA

Systémová sklerodermie- závažné, život ohrožující onemocnění

58

Page 59: lf1.czlf1.cz/wp-content/uploads/228-vypracovane_otazky_interna... · Web viewprognoza – 50 % dosáhne kompletního uzdravení do 6 měsíců, 20 % má prodloužený průběh –

Vypracované otázky na INTERNU made by Edita Homolková, Monika Ambroziová, Adam Král, Petr Vermach a Vítek Křehnáč

- systémové, chronické postižení kůže, pohybového aparátu i orgánů způsobené fibrotickou sklerotizací periferních i viscerálních cév (plíce, srdce, ledviny)

- etio?- patogeneze: protilátky stimulují fibroblasty a endotelie k nadprodukci kolagenu- dle průběhu a tedy i závažnosti se dělí na tři formy - difuzní – kůže celého těla

- limitující – od lokte a bérce distálněji, dekolt, obličej- sine sclerodermia – změny vnitřních orgánů

KO - na začátku Raynaudův fenomén, myalgie, artralgie a krepitus ve šlachách, kožní změny, postupně se vyvíjí- edematózní změny (opocená, prosáklá, pohyblivá proti spodině)- skleróza - napjatá, vyhlazená kůže, nepohyblivá proti spodině- místy hyperpigmentace místy vitiligo, ztráta adnex, člověk se cítí jako v malých rukavicích- špičatý nos, circumorální rýhování- postižení cév → plicní hypertenze!- ukládání kalcifikací do podkoží – bolí ležet - Sine sclerodermia - onemocnění začne námahovou dušností (fibrotizace plic)- orgánové změny - achalázie jícnu…dysfagie

-artritidy-mono, oligo, poly, retrakce svalových skupin- dušnost a suchý kašel, plicní fibróza (inspirační fenomény) plicní hypertenze- nevýkonnost myokardu, palpitace, nevýznamná perikarditida, převodní poruchy, AV

blok, arytmie- hypertenze, sklerodermická renální krize, vaskulitida postihuje glomeruly

Lab - ANA – nespecifické- Anticentromerové protilátky u limitované sklerodermie- Antii SCL 70u difuzní

Dg - kožní biopsie, kapilaroskopie nehtového valu prokáže ischemizaciTh – kortikoidy, cyklofosfamid, ACE inhibitory u renální krize, blokátory Ca kanálu, Pentoxyfylin u RaynaudaJiné formy onemocnění - Morfea, lineární sklerodermie – lokalizované formy bez orgánových změn

- CREST, Calcinoza, Raynaud, Esofagus, Sklerodermie, Teleangiektázie je variantou limitované sklerodermie

Polymyozitida- nehnisavé zánětlivé onemocnění více příčně pruhovaných svalů- vyskytuje se častěji u žen, i idiopatické formy dětské- etiologie primární není známa- sekundární formy se vyskytují např. jako součást jiných systém. onemocnění/ jako paraneoplastický syndrom- patogeneticky se uplatňují imunopatologické děje, postiženy jsou svalové cévy a ve svalech bývá lymfocytární

infiltraceKO - únavností, popř. bolestí svalů; svaly mohou být palpačně bolestivé a později dochází k svalové atrofii

- postiženy jsou svaly pletenců končetin, v těžkých případech i svaly dýchací- 4hlavé svaly stehenní → chůze o široké bázi, po schodech s oporou o zábradlí, z dřepu vstává „šplháním“

rukama po těle, nedokáže vzpažit, u těžších případů je pacient upoután na lůžko s neschopností zvednout hlavu, aktivně se posadit a s rizikem selhání funkcí dýchacích svalů

- dermatomyozitida - je-li současně přítomno poškození kůže- červenohnědý periorbitální („heliotropní“) exantém, příznak horního torza („límcový“

exantém na rozhraní krku a trupu), Gottronův příznak (papulózní exantém na hřbetě prstů a ruky) aj.

- dojde-li k atace s výraznějším poškozením svalů, mohou být v krvi detegovány i známky rozpadu svalových vláken kreatinkináza, AST, myoglobin, LD

Dg - EMG, biopsie svalů, imunologické vyšetření autoprotilátky ENA, Jo-1 (anti-histidyl tRNA syntetáza)Th - kortikoidy, imunosuprese

- v případě paraneoplastického původu může dojít k ústupu po odstranění nádoru

27.A CHOLELITIÁZA A CHRONICKÁ CHOLECYSTITIDA

Cholelitiáza- prevalence 115 % u mužů a 24 % u žen

59

Page 60: lf1.czlf1.cz/wp-content/uploads/228-vypracovane_otazky_interna... · Web viewprognoza – 50 % dosáhne kompletního uzdravení do 6 měsíců, 20 % má prodloužený průběh –

Vypracované otázky na INTERNU made by Edita Homolková, Monika Ambroziová, Adam Král, Petr Vermach a Vítek Křehnáč

- konkrementy (cholesterolové/ smíšené 80%, bilirubinové 20%)- cholecystolitiáza/ choledocholitiáza- 5 F : forty (věk), female, fatty, fertility, fair (etnicita)Etio - věk, obezita, častěji ženy, dieta (nadbytek tuků, málo vlákniny), těhotenství

- cholesterolová CL – sekrece cholesterolem hypersaturované žluče, poruch acidifikace žlučníkové žluče…- pigmentová CL – hemolytické stavy, cirhóza, dlouhodobá parenterální výživa, infikovaná žluč…- smíšená CL - nejčastější

Dg – lab - ↑ bilirubin, ALP, GMT, elevace transamináz (AST, ALT) při sepsi- USG (hodnocení kontrakční schopnosti žlučníku, zvětšení, ztluštělá stěna, dilatace žlučovodů)- MRCP, ERCP

Th - asymptomatická forma (nevyžaduje léčbu, pouze při nálezu "porcelánového žlučníku" léčbu, protože je zvýšené riziko ca žlučníku)

- chir. – laparoskopická cholecystektomie – konkrementy v nefunkčním žlučníku/ potenciální terén pro ca (porcelánový žlučník, žlučníkové polypy, akutní cholecystitida, hydrops a empyém žlučníku, perforace žlučníku, biliární píštěl)

- postcholecystektomický syn. – 20-40 % má abdominální dyskonfort- ERCP – papilosfinkterotomie, intrakorporální litotrypse u velkých konkrementů/ extrakorporální rázovou

vlnouKO - asymptomatický (75%), symptomatický (25%)

- biliární kolika – po 3-5 hod po jídle, při pasáži konkrementu žlučovými cestami kolikovitá bolest v pravém hypochondriu (ev. epigastriu) s propagací do pravého ramene nebo do zad (vyloučit pankreatitidu, IM, plicní embolii), bolest trvá 15 min-5 hod, někdy je nauzea, zvracení (nepřináší úlevu)

- biliární dyspepsie - nespecifické příznaky, tlaky/ bolesti, meteorismus, nesnášenlivost některých jídel (mastné, pečené, káva)

- akutní cholecystitida a pericholecystitida – jako komplikace blokády d. cholecysticus (retrográdní infekce a mechanické a chemické vlivy), zánětlivá náhlá břišní příhoda

- septický stav, lokální dráždění peritonea, teploty, silná a stálá bolest pod pravými žebry- Murphyho příznak - při hluboké palpaci žlučníku v pravém hypochondriu při současném

hlubokém nádechu pacient náhle pro bolest nádech zastaví- hydrops a empyém žlučníku – zaklínění konkrementu v krčku/ d. cysticus, obsah se několikanásobně zvětší a

je odbarvený s větším obsahem hlenu- po biliární klice přervává stálá tlaková bolest v pravém podžebří, citlivá a tuhá rezistence

pod pravým okrajem jater- při infekci se zaplňuje hnisem

- biliární pankreatitida – zaklínění v oblasti Vaterské papily, retrográdní tok žluče do pankreatického vývodu- bolest v epigastriu trvalá a intenzivní s vyzařováním k páteři a do zad, zhoršuje se

v poloze na zádech, nauzea a zvracení- leukocytóza, ↑ sérových amyláz a jater. enzymů (hl. GMT a ALP) a obstrukční ikterus

- obstrukční ikterus- akutní cholangitida - Charcotova trias = bolest v pravém hypochondriu + ikterus + horečka- chronická cholecystitida – chronický zánět vznikající při cholecystolitiáze, ztluštělá stěna (i kalcifikace),

žlučník může být zjizvený a svraštělý, omezení evakuační schopnostiKO – dyspepsie (tlak v epigastriu, nadýmání, nauzea) s vazbou na jídlo

27.B VROZENÉ NEMOCI LEDVIN

Ageneze 1 ledviny- ledvina je hypertrofická, může být ektopie, malrotace- podkovovitá ledvina (splynutí)

Vezikoureterální reflux (VUR)- abnormalita VU přechodu s následním refluxem při močení, genet. faktoryKO - u dětí noční pomočování (enuréza), časté moč. infekce

- vyšší stupeň VUR - refluxová nefropatie (FS GS)Dg - cystografie, sono ledvin (jizvení), izotopové vyš.

60

Page 61: lf1.czlf1.cz/wp-content/uploads/228-vypracovane_otazky_interna... · Web viewprognoza – 50 % dosáhne kompletního uzdravení do 6 měsíců, 20 % má prodloužený průběh –

Vypracované otázky na INTERNU made by Edita Homolková, Monika Ambroziová, Adam Král, Petr Vermach a Vítek Křehnáč

Th - u nižšího stupně - atb, u vyššího - antirefluxní plastikaCystická onem. ledvin

-heterogénní skupina dědičných onem. s přítomností cystAD polycystická choroba ledvin

- nejčstější dědičné onem. ledvin, 1:500-100- mutace genu - PKD1 pro polycystin1 (16 chrom) - výskyt 85 %, rychlejší progrese do RI (54 let)

-PKD2 pro polycystin2 (4. chrom) - pomalší progrese do RI (74 let)- cysty mohou být i v jiných orgánech (játra)

KO - bolesti v bedrech, hypertenze, recidivující moč. infekce, HU, nefrolitiázaDg - anamnéza (+rodinná)

- sono (mnohonásobně zvětšená ledvina, cysty), CT, MRTh –symptomaticky hypertenze, moč. inf.

- transplantaceKomplikace - akutní pyelonefritida (febrilie, bolest, třesavka)

- infekce v cystách (kultivace moče), ↑ prokalcitonin, leukocyturie- nefrolitiáza - makroskop. HU (ruptura cévy v stěně cysty)- častější výskyt aneuryzmatu na mozk. cévách

AR polycystická choroba ledvin- vzácná- mutace genu PKDH1 pro fibrocystinKO - nejzávažnejší forma je neslučitelná se životem (oligohydramnion + plicní hypoplázie)

- lehší formy - arteriální hypertenze, portální hypertenze, RI až selhání v období adolescenceKomplikace - krvácení z jícnových varixů, trombocytopenie, hypersplenizmus (anémie), cholangoitida

Nefronoftíza- AR, vzácná- mutace NPH 1-4 (nefrocystin)KO - manifestace během jiné nemoci – polydipsie, polyurie, velké ztráty solí, anémie, postupně selhání L

- může být kombinace s retinitis pigmentosa (až slepota)- cysty jen u části pacientů

Th - symptomatická, transplantaceDřeňová cystická ch.L

- dospělá forma (chronické selhání v 4.-5. dekádě) a juvenilní formaKO - zhoršená koncentrační schopnost L

- hypertenze, hyperurikémie (urea se nevylučuje), malá PU- cysty prokážeme u pacientů s RI

Th - symp., dostatečný příjem tekutin a solíTuberózní skleróza

- vzácné onem. s AD dědičnostíKO - hamartomy všude možně (kůže, mozek, oči, srdce...) a dělají tam problémy

- makrosk. hematurie, bolest, hypertenze, vývoj RI- angiomyolipomy (benigní), můžou se vyskytovat i maligní Tu

Von Hippel-Lindauova choroba- multisystémové nádorové onem., AD dědičnost, tumor supresorový gen VHL- karcinom L + nádory očí, mozku, míchy, nadledvin, pankeatu (vaskularizované)- každý rok kontrola výš uvedených systémůTh - chirurgická

Vrozená onem. glomerulůSy tenkých membrán (benigní familiární HU)

- mutace α3 nebo α4 řetězce kolagenu IV (na X chromosomu)- izolovaná glom. HU - nález se dlouhodobě nemění, dobrá prognóza

Alportův sy-mutace α5 řetězce kolagenu IV- časně - ztenčení BM, progrese do postupného stluštění a rozvláknění BM- extrarenální projevy - percepční nedoslýchavost až hluchota, přední lentikokonus

61

Page 62: lf1.czlf1.cz/wp-content/uploads/228-vypracovane_otazky_interna... · Web viewprognoza – 50 % dosáhne kompletního uzdravení do 6 měsíců, 20 % má prodloužený průběh –

Vypracované otázky na INTERNU made by Edita Homolková, Monika Ambroziová, Adam Král, Petr Vermach a Vítek Křehnáč

KO - trvalá mikroHU asi od 5 let, postupně makroHU při infekci, renalní selhání okolo 25 let- poruchy sluchu, hypertenze, PU až NS

Kongentiální nefrotický syndrom- mutace genů pro podocytární proteiny- projevy hned při narození (finský typ), nebo později

Vrozené tubulopatieBartterův sy

- mutace genů pro kanály v Henleovy kličce- hypokalemie, hypochloremická metabolická alkalóza (HMA), ↑ ztráty Na, K, Cl močí, normotenze- únava, tetanie, křeče, netolerují zátěž, pozor na maligní arytmie

Giltemanův sy- mutace genu pro Na Cl kotransportér v dist. tubuluKO - vyvíjí se až v dospělosti, podobný jako u Barttera

- když najdu pacienta s hypokalemií, tak si snad spomenu na Barttera a GiltemanaFabryho nemoc

- dědičný deficit enzymu α-galaktozidáza- renální postiž. - depozita v tubulech (PU, mikroHU, hypertenze, progrese do RI)- extrarenálně - angiokeratomy, akroparestézie, sníž. pocení, postiž. myokarduTh - rekombinantní enzym

28.A VŘEDOVÁ CHOROBA ŽALUDKU A DUODENA

- vřed proniká min do muscularis mucosae,všude, kde je HCl (jícen, žaludek, duodenum, ektopicky v Meckelově divertiklu)

- vznikají často na slizničních rozhraních (cirkulace?)- zánětová – gastritida (Helicobacter)- sekundární – lékové (nejčastější), stresové (Curlingův při popáleninách), endokrinní (Zollinger-Ellisonův,

hyperparatyreóza), hepatogenní, při respirační insuficienci, stařeckéEtio – agresivní faktory –HCl

- Helicobacter pylori – vyvolává antrální gastritidu, 90 % vředů v bulbu- produkce amoniaku (ureáza), vakuolizační cytotoxin (u 65 % kmenů)

- NSA, kys. acetylosalicylová, žluč, kouření, porucha mikrocirkulace (stres)- projektivní faktory – normální skladba a množství hlenu, mikrocirkulace, hydrogenkarbonáty v žaludeční

šťávě, regenerace epitelu, endogenní prostaglandinyDg – endoskopie – jediná prokáže bulbitidu, řešení krvácení

- biopsie pro průkaz H. pylori a rozpoznání karcinomu (podobný obraz)- lab – gastrin v séru při Zollinger-Ellisonově syn. (+ dif dg hyperacidity)

- ureázový test v histologických řezech- dechová zkouška po podání urey se 14C- specifický antigen H.p. ve stolici

Prognóza – u většiny léčitelné, u 10 % je choroby sužujícíKomplikace – hemateméze, meléna – kdykoli, často po NSA

- penetrace – do pankreatu, do dutiny břišní ani ne- úporné bolesti, tvorba srůstů a reaktivní zánět

- stenóza pyloru – vzácně u pylorického a duodenálního vředu- díky edému nebo fibrotizací- pocit plnosti, zvracení

Th – režimová opatření – dostatek klidu a spánku- každá strava má neutralizační účinek, ale i stimuluje produkci šťáv (ne káva, čaj, cola, alkohol)- nekouřit, ne NSA

- farmakoterapie – eradikace H.p. – inhibitory H+ pumpy + claritromycin, amoxicilin, azitromycin (úspěch v 90 %)

- když po 1 měsíci od léčby není prokázán- sekundární vředy – zakázání NSA, inhibitory H+ pumpy (na 3-6 týdnů), antagonisti H2 rec,

antacida pro tišení obtíží

62

Page 63: lf1.czlf1.cz/wp-content/uploads/228-vypracovane_otazky_interna... · Web viewprognoza – 50 % dosáhne kompletního uzdravení do 6 měsíců, 20 % má prodloužený průběh –

Vypracované otázky na INTERNU made by Edita Homolková, Monika Ambroziová, Adam Král, Petr Vermach a Vítek Křehnáč

- chir – jen při komplikacích (nezvládnutelné/ opakované krvácení, penetrace, perforace)Duodenální vřed

- nejčastěji muži 20-40 letKO – epigastrická bolest ve stření čáře/ vpravo od ní, nalačno (2 hod po jídle), často v noci

-na bolest působí jídlo a antacida- pokud je na zadní straně, může se bolest šířit do zad- často pyróza a kyselá regurgitace, říhání, zácpa, vyzvracení většinou uleví- ↑ hmotnosti díky úlevě po jídle (jí pořád)- únavnost, spavost, často v rodinné anamnéze

Žaludeční vřed- výskyt stejný u obou pohlaví, není typický věkKO – bolest je méně časově vázána na jídlo – závisí na lokalizaci (čím výše, tím dříve se bolest objeví)

- angulární a midogastrický vřed – bolest za 1 hod, u subkardiálního brzy po jídle- palpační bolestivost v epigastriu- vzplanutí 1-2x do roka (jaro a podzim)- po eradikaci recidiva H.p. v 10 %

28.B SJOGRENŮV SYNDROM

- systémové, chronické autoimunitní onemocnění postihující exogenní žlázy- etiologie je neznámá- patogeneze je taky neznámá, uvažuji o CMV, EBV, HLA DR 3 (primární SS, DR 4 sekundární SS), prostředí,

hormonální vlivy, psychologické vlivy, asociace s lymfomy- postihuje typicky ženy mezi 40. až 60 rokem (včetně)KO - projevuje se zvětšením gl. parotis (příušnice to nebudou, viz postižená skupina)

- může být nesymetrická neerozivní artritida, jedná-li se o primární Sjogrenův sy- může se vyskytovat sekundárně při: RA (to je ale symetrická, erozivní artritis), SLE, sklerodermii- glandulární projevy - suchá keratokonjuktivitis, popisují písek v oku, fotofobie

- Schirmerův test (papírek do oka) – objektivizace suchosti oka- Bengálská červeň pěkně obarví nekrozy na rohovce

- xerostomie (vyšší kazivost zubů), zvětšení slinných žláz, dysfagie, gastritis s achlorhydrií- suchost v nose, epistaxe, rhiniopharyngitis, chron. bronchitis (!!! pozor může vést

k intersticiální plicní fibróze a plcní hypertenzi a na tu se umírá!!!), dráždivý kašel- Extraglandulární projevy - neerozivní artritis

- Raynaudův fenomén (spíše u sklerodermie)- hepatomegalie- lymfadenopatie- vaskulitida projevuje se petechiemi od lýtka dolů- intersticiální pneumonitida- hubnutí, únava

Lab - ↑ FW, CRP v normě, mírná anémie, hypergamaglobulinemie, protilátky proti všemu (pozitivita RF, ANA, ENA, SSA(Ro), SSB(La), AMA

Th – symptomatická - oční kapky Lacrisyn, časté pití, žvýkačky, lubrikační gely, zvlhčující mléka- extraglandulární – Metotrexát, Prednison + Cyklofosfamid

29.B SEKUNDÁRNÍ GP

Diabetická nefropatie – ot.67.A a 97.AHenoch-Schönleinova purpuraAmyloidóza ledvin

- ukládání proteinových hmot do tkání v orgánechKO – nefrotický syndrom (10-20 g/ l) u ledvinného postižení, jinak pestrý obraz, závisí na lokalizaci- části amyloidu - amyloidové fibrily (specifické), amyl. protein P (váže se na něj Kongo červeň), apolipoproteinyTypy - AL am. - ukládání lehkých řetězců κ a λ

- paraproteinémie - MGUS, Waldenström, B-CLL, lymfomy

63

Page 64: lf1.czlf1.cz/wp-content/uploads/228-vypracovane_otazky_interna... · Web viewprognoza – 50 % dosáhne kompletního uzdravení do 6 měsíců, 20 % má prodloužený průběh –

Vypracované otázky na INTERNU made by Edita Homolková, Monika Ambroziová, Adam Král, Petr Vermach a Vítek Křehnáč

- AA am. - ukládá se SAA - protein akutní fáze- chron. záněty v organizmu (revmat. artritis, Crohn, ankylozující spond., bronchiektázie, tbc)

- hereditární formy, u dialyzovaných,...SLE - lupusová nefritida

- postihuje do 75% nemocných se SLE- ukládání imunokomplexůKO - může být nefritický sy (erytrocyturie, PU), leukocyturie bez infekce, může být NS (40 %)

- může být i obraz RPGN - zvlášť s antifosfolipidovým syhist. dělení - 6 typů, ne stadia (pacient si svůj typ nese celý život)

- typI - normální nález- typII - mesangiální GN- typik - fokálně proliferativní GN

Kryoglobulinémie- nefrotický syndrom, membranoproliferativní GNTh – plasmaferéza, kombinovaná imunosuprese- stav charakterizovaný přítomností kryoglobulinů v krvi s následnou hyperviskozitou plasmy a aglutinací erytrocytů

ucpávání, resp. trombóza drobných cév, poruchou funkce trombocytů a funkce endotelu krvácení- nápadné jsou petechie na kůži a na slinicích, mohou být drobné nekrózy i infarkty vnitřních orgánů, bývá cyanóza akrálních

částí těla a sekundární Raynaudův fenoménSklerodermie

- manifestace hl. v zimních měsících u mužů s difuzním postižením kůžeKO – proteinurie, malignií hypertenze, rychlý rozvoj renální insuficienceTh – ACEI, dialýza

Myelomová ledvina- precipitace lehkých řetězců Ig v tubulech- rychlý rozvoj akutního renálního selháníTh – hydratace, alkalizace, normalizace K+, cytostatika, plazmafereza

Hemolytickouremický sy (HUS)- trombotická mikroangiopatie s intravaskulární trombózou, trombocytopenií a mikroangiopatickou

hemolytickou anémií- dominuje postižení ledvinEtio – poškození endotelu verotoxinem (shiga toxin, E. coli) KO – trombocytopenie <100*109/ l, purpura, mikroangiopatická hemolytická anémi

- selhání ledvin s oligoanurií, hypretenze- destičkové tromby, ischemie glomerulů

Dg – viz KO, pokud není jasné – biopsieTh – pokud verotoxin – NEPODÁVAT ATB a léky zpomalující motilitu střeva

- stav se většinou upraví sám

30.A AKUTNÍ A CHRONICKÁ PANKREATITIDA

Akutní pankreatitidaEtio – 50-60 % způsobeno nemocemi žlučových cest = akutní biliární pankreatitid

- abúzus alkoholu v 20-40 % (ovlivňuje přímo metabolické procesy pankreat. bb. a aktivuje intraacinárně enzymy)

- polékové postižení (furosemid, hydrochlorothiazid, merkaptopurin, cyklosporin, azathioprin, valproát, karbamazepin, erytromycin, rifampicin, tetracyklin, ACEI…)

-metabolické příčiny (hyperkalcémie, těžká hypertriglyceredémie)- infekce virem karotidy, coxackie, AIDS, virové hepatity, protozoa- tupá poranění, iatrogenně (ERCP, perioperačně), idiopaticky

Dělení – těžká – 15-20 %, často fatální, - u akutní nekrotizující s fokální nekrózou smrt v 15 %, pokud totální nekróza až 50 %- nejčastěji úmrtí na septické komplikace- multiorgánové selhání + lokální komplikace (absces, nekróza, akutní pseudocysta)

- lehká – orgánové dysfce- v naprosté většině plná reparace

64

Page 65: lf1.czlf1.cz/wp-content/uploads/228-vypracovane_otazky_interna... · Web viewprognoza – 50 % dosáhne kompletního uzdravení do 6 měsíců, 20 % má prodloužený průběh –

Vypracované otázky na INTERNU made by Edita Homolková, Monika Ambroziová, Adam Král, Petr Vermach a Vítek Křehnáč

- akutní edematózní forma s výpotkem v dutině břišní/ retroperitoneu – u lehké formy, tekutina ve žláze nebo těsném okolí

- pankreatická nekróza – lokální/ difuzní, typicky s preripankreatickou tukovou nekrózou, u těžší formy- akutní pankreatická pseudocysta- absces pankreatu

KO – bolest ve střením epigastriu a mezogastriu, propagace do páteře, je stálá, intenzivní a neproměnlivá- bolest se zhoršuje v pohoze na zádech, v předklonu úleva- často zvracení- u těžké formy tachykardie, hypotenze, teplota, hypovolemický šok (selhání ledvin, oligurie, anurie), rychle

nastupující paralytický ileus, respirační insuficience, dušnost, cyanóza, ARDS- mramorování v okolí pupku a tříslech (redistribuce krve, modrá a žlutavá zabarvení na kůži v důsledku

retroperitoneálního hematomu)- psychická alterace

Dg – náhlá prudká bolest v epigastriu s propagací do stran břicha a do páteře, zvracení, porucha střevní pasáže, teplota, šokový stav

- Lab - ↑ amyláz v séru a moči (CAVE! I u mimoděložního těhotenství, perforované žaludeční vřed, ischemie splanchniku) alespoň 3x nad normu

- ↑ lipázy (je specifičtější, ale nastupuje pomaleji)- lipázy a amylázy nerostou hned

- leukocytóza, hypokalcémie je nepříznivá pro prognózu (usazuje se do nekróz), ↑ ery a hematokritu (reakce na hypovolemii)

- CRP >150 (norma 0-10) svědčí pro těžkou formu- sledovat K+ pro posouzení ledvinného selhání (jako důsledek hypovolemického šoku)- sledovat: glc, Ca2+,K+, Na+, Cl-, bilirubin, aminotransferáz, ABR, celk. bílkovina, albumin- USG – může být těžko proveditelné, ale hl. pro vyloučení biliární etio

- monitorace vzniku komplikací (cystoid, absces, trombóza lienální žíly)- CT – průkaz nekrózy (za 72 hod nejdříve) + doplnit punkci pro kultivaci- endosono – průkaz biliárního kamene v distální části ductus choledochus (pokud se před tím nepodařilo

ERCP)- ERCP – hl. při biliární etio, ne u alkoholické- MR cholangiopankreatikografie

Th – 1. a 2. týden ba JIP (možné komplikace), 3. a 4. týden bakteriální komplikace– inhibice žaludeční sekrece zavedením sondy a odsávání žaludečního obsahu- hypovolemický šok – vznik atonického ileu, selhání ledvin, tekutiny do retro- a peritoneálního prostoru

(několik litrů v 1. fázi)- koloidní a krystaloidní roztoky (6000–8000 ml/24 h)

- enterální výživa do jejuna- parenterální výživa u velmi těžkých případů- inzulin při hyperglykémii- opioidy - fentanyl i.v. (morfin ↑ tonus Oddiho svěrače), tramadol, epidurální analgezie (dýchání omezeno

bolestmi a bazálními atelektázami)- blokátory H+ pumpy- Atb cíleně (50 % nekróz je již od začátku infikovaných), cefalosporiny 3. generace, metronidazol, chinoliny,

(ciprofloxacin, ofloxycin), karbapenemy (imipenem, meropenem)- podpora dýchání – komplikace jako výpotky, atelektázy, pneumonie, pleurální sufuze- při biliární etio endoskopická papilosfinkterotomie- drenáž abscesů pod CT kontrolou- chir – spíš léčba komplikací, odstranění rozsáhlých nekróz a abscesů, krvácení, pseudocysty, cévní

obstrukce…Chronická pankreatitida

- incidence lehce roste, 7/100 000= vleklý zánět pankreatu vedoucí k ireverzibilním změnám a náhradě fční tkáně vazivem → exo- a endokrinní

nedostatečnostEtio – abúzus alkoholu – 80 %

65

Page 66: lf1.czlf1.cz/wp-content/uploads/228-vypracovane_otazky_interna... · Web viewprognoza – 50 % dosáhne kompletního uzdravení do 6 měsíců, 20 % má prodloužený průběh –

Vypracované otázky na INTERNU made by Edita Homolková, Monika Ambroziová, Adam Král, Petr Vermach a Vítek Křehnáč

- tuková degenerace acinů alkoholem a jejich poškození kyslíkovými radikály, destrukce acinů duktální hypertenze z obstrukce pankreatických vývodů proteinovými zátkami, fibrózy z epizod akutní pankreatitidy

- cystická fibróza – mutace CFTR prokazovány u idiopatické chron. pank. bez kliniky cystické fibrózy- hereditární – velmi vzácné, mutace genu pro trypsinogen (autoaktivace)

- ↑ riziko ca pankreatuDělení – klasifikace TIGAR-O

- toxicko metabolická forma – alkohol, nikotin, hyperlipidémie, urémie, léky, toxiny- idiopatická – s časným a pozdním začátkem nemoci, tropická forma- genetická – hereditární chron. pankreatitida s katioologickým trypsinogenem

- AR forma (CFTR mutace u odob s mukoviscidózou)- deficit α-1-antitripsinu

- autoimunní- rekurentní akutní pankreatitida spojená s morfologickýmí změnami chronické pankreatitidy (ischemická,

postiradiační, postnekrotická)- obstrukční – obstrukce vývodu, stnóza Vaterské papily, cystoid utlačující vývod, pankreas divisum, jizva

po traumatickém postižení žlázyKO – stálá bolest po několik hodin/ dnů v okolí pupku šířící se pod oba žeberní oblouky, dostavuje se po jídle

(radši nejedí), možno i do zad, mezi lopatky- hubnutí- obstrukční ikterus (porucha odtoku stenouzou intrapankreatické části žlučovodu – pozor na bezbolestný

ikterus u ca pankreatu)- stolice častá, mastná s nestrývenými zbytky- u pokročilých forem diabetes- pankreatický ascites u 3 % (je v něm ↑ amylázy a bílkovin), možný i současný hydrothorax

Dg - ↑ amylázy a lipázy, chymotrypsin ve stolici (citlivost jen 50 %)- rtg ukáže intrapankreatické kalcifikace- na USG velikost a nehomogenita, rozšíření pankreatického vývodu nad 3 cm!- na CT prostorová orientace, kalcifikace, zobrazení vývodů a cystomů- MR – cholangiopankreatikografie- ERCP hl. pro terapeutické výkony-endoSONO

Lab – stanovení CFTR, protilátky (antilaktoferrin, antikarboanhydráza II, revmatoidní faktor, antinukleární antigen (ANA), protilátky proti hladkým svalům (ASMA), sérové globuliny…)

Th – dieta – absolutní zákaz alkoholu, ne živočišné tuky, rostlinných ne více než 60 g/den, sacharidy dle diabetu- substituce pankreatických enzymů (kapsle)- analgetika (paracetamol, tramadol), někdy stačí jen spasmolytika- prokinetika (často bývá spojeno s dysmotilitou střeva)- kortikoidy u autoimunitních - endoskopické protětí papily u obstruktivní algické formy (uvolnění toku pankreatického sekretu), zavedení

stentu při stenóze, léčbě cystomů…- chir – drenáž nebo resekce (nejčastěji hlavy)

30.B HODGKINOVA CHOROBA

= Hodgkinův lymfom = systémové lymfoproliferativní onemocnění, bb Reed-Sternberg nenesou lymfocytární znaky- ve značném % vyléčitelné; častěji muži, výskyt má 2 vrcholy → 15-30 roků a po 50. roceEtio - genetická predispozice (HLA fenotypy) a vztah k porušené imunitě

- zákl. dg. Elementem = buňky Reedové-Sternberga, jejichž méně vyzrálé prekurzory jsou tzv. Hodgkinovy bb. (centrofolikulární původ)

- v bb kontitutivní aktivita transkripčního faktoru NFκB- v nádorové transformaci a proliferaci se uplatňuje infekce EBV (vyšší frekvence inf. mononukleózy

v anamnéze) a HIV- produkují cytokiny, chemokiny → celkové příznaky (IL-1 horečka, TGFα hyperplazie uzlin, fibróza...)

66

Page 67: lf1.czlf1.cz/wp-content/uploads/228-vypracovane_otazky_interna... · Web viewprognoza – 50 % dosáhne kompletního uzdravení do 6 měsíců, 20 % má prodloužený průběh –

Vypracované otázky na INTERNU made by Edita Homolková, Monika Ambroziová, Adam Král, Petr Vermach a Vítek Křehnáč

Klasifikace – klasický HL – modulárně sklerotický- smíšeně buněčný- na lymfocyty bohatý- s deplecí lymfocytů

- nodulární HLKO - nemoc začíná v lymfatické uzlině, šíří se do sousedních uzlin (axiální distribuce) → Hodgkinovy bb. lymfatickou

cestou → mízní uzliny, systémová cirkulace → od začátku jako systémové onem.- rozsah uzlinové postižení = 4 klinická stádia (význam pro terapii)- projeví se nebolestivým zvětšením mízní uzliny (krk 80%, L strana), axily, ingviny, RTG hrudníku - zvětšení

mediastina - zvětšením lymf. uzlin, lépe CT- může být i hepatomegalie a splenomegalie- celkové příznaky - horečky i cyklické (Pelova-Ebsteinova typu), svědění kůže, malátnost, únava, hubnutí,

bolest v uzlinách po požití alkoholu (vzácné), adenomegalie, splenomegalie, při zvětšení mediastina dráždivý kašel, syndrom horní duté žíly, výpotky, infiltrace parenchym. orgánů či skeletu

Lab – CT (PET), scinti kostí, trepanobiopsie- ↑ sedimentace- u pokročilých forem leukocytóza , lymfopenie, anémie, eosinofilie

Th –lokalizovaná stádia - bez rizikových faktorů – 2 cykly ABVD (adriamycin + neomycin + vincristin + dacarbazin) a radioterapie na původní lokalizaci tumoru

- s rizikovými faktory …Prognóza - správně a včas léčená choroba vyléčitelná až v 95 %, v pokročilých stádiích až v80 %

31.A KOLOREKTÁLNÍ KARCINOM

- 95 % všech nádorových onem tlustého střeva, ze žlázového epitelu- 2. nejčastější malignita po ca prsu/ plic- tubulární/ tubulovilózní/ virózní, s nízkou/ vysokou dysplazií- intramukózní = neinvazivní (nepřesahuje lamina propria mucosae) se moc metastaticky nešíří (sliznice není

lymfaticky drénovaná), ale může být i invazivní- 90 % je sporadických, zbytek hereditární (mutace APC a mismatch repair genu)- nejčastěji v 70-80 letech, v levém tračníku (30 % rektum, 26 % sigmiodeum, 8 % descendens), v transversu 13 %,

v ascendentu 9 % a v céku 14 %- incidence 95 mužů a 63 žen na 100 000 ob, 5ti leté přežití 40%- rizikové: kouření, červené maso, vysoké množství tuků, obezita, hyperinzulinémie- projektivní: vláknina, polynenasycené MK, kys. listová, HAKKO – až u pokročilé formy (stenóza, anémie)

- dle lokalizace a typu expanze (intra-/ extraluminální- vpravo – 36 %, sideropenická a mikrocytová anémie- vlevo – 64 %, manifestní krvácení a poruchy pasáže, střídání průjmu a zácpy- v rektu – 30 %, tenesmy- dyskonfort, nadýmání, váhový úbytek, nechutenství, slabost- při penetraci symptomy dle orgánu (pneumaturie, odchod stolice močí…)

Dg – fyzikální nález většinou nenápadný (projevy anémie…), hmatná rezistence, zvětšená tvrdá játra (metastázy), ascites

- středně zvýšená sedimentace, mikrocytární hypochromní anémie, elevace jaterních enzymů (metastazy)- koloskopie s biopsií, USG/ CT pro staging nádoru, CT malé pánve + endorektální sono při postižení rekta

(hloubka invaze, postižení uzlin)- rtg plic

Th – dle Dukesovy a TMN klasifikace, kurativní/ paliativní- chirurgie – závisí na lokalizaci, velikosti a stagingu, kombinace s radio- a chemoterapií (neadjuvantně,

adjuvantně)- endoskopická resekce dostačující u nádorů neprorůstající lamina muscularis mucosae- pravý tračník → pravostranná hemikolektomie a ileo-transverso-anastomóza- levý tračník → levostranná hemikolektomie a transverso-rekto-anastomóza

67

Page 68: lf1.czlf1.cz/wp-content/uploads/228-vypracovane_otazky_interna... · Web viewprognoza – 50 % dosáhne kompletního uzdravení do 6 měsíců, 20 % má prodloužený průběh –

Vypracované otázky na INTERNU made by Edita Homolková, Monika Ambroziová, Adam Král, Petr Vermach a Vítek Křehnáč

- radioterapie - výhradně u ca rekta- ozařování pokročilých nádorů před a po resekci/ amputaci konečníku- u inoperabilních paliativně

- chemoterapie – 5-fluorouracil + leukovorin (derivát kys. listové) / oxaliplatina- biologická léčba u metastáz (bevacizumab – protilátka proti VEGF, cetuximab – protilátka

proti EGF)Screening – detekce asymptomatických prekanceróz (pokročilé adenomy)

- test okultního krvácení (pseudoperoxidázová aktivita hemu, imunologický test lidského hemoglobinu) ve stolici pro lidi nad 50 let 1x rok + koloskopie při pozitivitě testu

- osoby nad 55 let buď koloskopie a na 10 mají pokoj/ TOKS 1 za 2 rokyDispenzarizace – koloskopie v pravidelných intervalechPrognóza – jen 30-40 % dg v lokalizovaném stádiu s nadějí na vyléčení v 90 %

- 20-30 % dg v diseminovaném stádiu s 5ti letým přežitím 4-12 %

31.B ANÉMIE HEMOLYTICKÉ, ZÍSKANÉ

1. IMUNITNÍ HEMOLYTICKÉ ANEMIEAutoimunitní hemolytická anemie

- vazba protilátky na ery s následnou hemolýzou- porucha kooperace mezi Tc, Ts a B lymfocyty → dysregulace vede k nedostatečné supresi tvorby protilátek

proti vlastním antigenům nebo paradoxně ke stimulaci jejich tvorby- SlE, RA, sklerodermie, ulcerózní kolitis, antifosfolipidový syndrom, maligní lymfomy, karcinomy, thymom,

Kaposiho sarkom, EBV, HIV, HCV, hypogamaglobulinemie- idiopatická - etio?, často po akutní virové inf.- chladové protilátky - po mykoplazmové infekci, také u SLE, lymfomů, karcinomů

- IgM, vazba při teplotním optimu 4°C (chladnější akrální partie organismu) → snazší aktivace komplementu (je to pentametr) → intravaskulární hemolýza

- v teplejších oblastech se protilátky uvolní, ale zůstane navázána C3b → vychytání makrofágy ve slezině

- tepelné protilátky – IgG, vazba na ery při teplotním optimu 37°C → aktivuje komplement → vychytány ve slezině makrofágy = extravaskulární hemolýza (intravaskulární hemolýza jen při vysokém titru protilátek)

- bitermický hemolyzin – IgG, vede k paroxysmální chladové hemoglobinurii- výskyt u lues a spalniček

KO – akutně - rychle se rozvíjejíci se anemizace (ikterus, bolesti břicha, zvracení)- chronicky – hl. u starších, postupná anemizace, žloutenka, hemoglobinurie, hepatosplenomegalie

Lab – středně těžká i těžká anemie (<50 g/l Hb), makrocytární, ↑ retikulocyty (10-30 %), mírná leukocytóza, trombocyty v normě kromě Evansova sy (= autoim. hem. an. + trompcytopenie)

- sférocyty, penízkovatění ery, v dřeni hyperplazie normoblastické krvetvorby- ↑ přímého a nepřímého bilirubinu, v moči urobilinogen- Coombsův test přímý = přímý antiglobulinový test - aglutinace senzibilizovaných ery nemocného

heterologními protilátkami anti-IgG či anti-C3b- nepřímý Coombsův test - průkaz volných autoprotilátek, ty reagují s typovými ery a poté jsou tyto ery

aglutinovány stejně jako v přímém testuTh – glukokortikoidy (↓ tvorby protilátek a jejich afinity, receptorů pro Fc fragment a C3b), v komb.

s cyclofosfamidem- těžká hemolýza – imunosuprese + imunoglobuliny (kompetice o receptory) + plazmaferéza, splenektomie- u chlad. protilátek - transfúze s deleukotizovanými ery a ohřát na 37°C

Poléková hemolytická anemie- vazbaí na bílk. nosič - působí jako hapteny a vyvolávají tvorbu protilátek po 7-10 dnech ponasazení léku

- penicilin, ampicilin, cefalosporiny se - tvorba imunních komplexů adsorbujících se spolu s komplementem na povrch ery - phenacetin, rifampicin,

chinidin, hydrochlorthiazid…- stimulace tovrby protilátek proti některým antigenům ery - methyldopa

Hemolytická choroba novorozenců

68

Page 69: lf1.czlf1.cz/wp-content/uploads/228-vypracovane_otazky_interna... · Web viewprognoza – 50 % dosáhne kompletního uzdravení do 6 měsíců, 20 % má prodloužený průběh –

Vypracované otázky na INTERNU made by Edita Homolková, Monika Ambroziová, Adam Král, Petr Vermach a Vítek Křehnáč

- Rh - matka a Rh+ plod; při druhé graviditě (u matky protilátky proti Rh, senzibilizují ery plodu)- novorozenecká žloutenka až hydrops fetalis- vzestup bilirubinu 3. den po porodu, u matky nález anti-Rh(D)- prevence rozvoje, matce se podá anti-Rh protilátka vyvazující zbylé senzibilizující ery plodu po ukončení prvé

gravidityTh – fototerapie, výměnná transfúze

2. NEIMUNITNÍ HEMOLITICKÉ ANEMIE= mechanické poškození ery s jeho fragmentací, rozpadem a odstraněním makrofágyMikroangiopatická hemolytická anémie

- rozrušení integrity membrány ery mechanickou překážkou měnící průsvit cévy či depozity fibrinu- některé vyvolávající příčiny vedou k aktivaci koag. systému → DICKO - HUS (hemolyticko-uremický sy) a TTP (trombotická trombocytopenická pupura) - akutní stav

s přítomností mikrotrombů v cirkulaci - CNS, ledviny, hemolytickou anémií a trombocytopenií- HELLP syndrom – komplikace gravidity, hemolýza, ↑ jaterní enzymy, ↓trombocyty, křeče, poruchy zraku,

srd. selhávání, DIC- Mikroangiopatická hemolýza CNS - podklad preeklampsie v těhotenství- u vaskulitidy, akutní GN, rejekce štěpu, A-V malformace, po rekonstrukci chlopní, poškození cévní stěny

bakteriálními toxiny, léky (ticlodipin, tacrolimus, mitomycin..), nádoryLab – anémie s retikulocytózou, sférocyty, akantocyty, schistocyty (krvinky tvaru skořápky či helmy), jejich

fragmenty- TTP – typické ELFO von Willebrandova faktoru- ↑LDH, ↑ přímý i nepřímý bilirubin, ↑ kreatinin a urea, proteinurie, hemoglobinurie

Th - TTP/ HUS - plazmaferéza + čerstvě zmrazená plazma- antikoagulancia, kortikoidy

Pochodová hemoglobinurie - mechanické příčinyClostridium perfringens, hadí jedy - způsobí hemolýzu ery

32.A SYNKOPA A KOLAPS = 1.A

32.B LEDVINY A HYPERTENZE

Ledviny jako oběť hypertenze- kontrola arteriální hypertenze má vliv na zpomalení progrese chronikého renálního selhání a na redukci

proteinurie u diabetické nefropatie- nejlépe 120-130/75 mm Hg při proteinurii, jinak stačí 130/85 mm HgTh – ACEI/ ATI – zpomalují progersi chron. ren. Infuf. O 30-50 % a u lidí s DM snižují riziko vzniku

mikroalbuminurie- ↓ glomerulárního tlaku, protizánětlivé, snižují oxidační stres, inhibují fibrogenezi v renálním

intersticiuLedviny jako původce hypertenze

- 10 % sekundárních hypertenzíRenoparenchymatózní hypertenze

- s progresí renálního selhání rost výskyt hypertenze nezávisle na etiologii ren. selhání, při zahájení dialýzy má hypertenzy 90 %

U glomerulopatií – vzácně u nefrotického syndromu s minimálními změnami, u 20-40 % nemocných s membranózní nefropatií, a ještě více u proliferativních GN

- často u lupusové nefritidy (navození remise obvykle normalizuje TK- u 50-70 % nemocných s diabetickou nefropatií

U polycystických chorob – hypertenze u 50-70 % ještě v době normálních renálních fcí

Renovaskulární hypertenze – hypertenze související s hemodynamicky významnou stenózou renálních tepen/ tepny (min 70-80 %)- fibromuskulární dysplazie u mladých žen, ateroskleróza → ↓ perfuze ledviny → zánik glomerulů a fibróza

intersticia = ischemická nefropatie

69

Page 70: lf1.czlf1.cz/wp-content/uploads/228-vypracovane_otazky_interna... · Web viewprognoza – 50 % dosáhne kompletního uzdravení do 6 měsíců, 20 % má prodloužený průběh –

Vypracované otázky na INTERNU made by Edita Homolková, Monika Ambroziová, Adam Král, Petr Vermach a Vítek Křehnáč

Th – ACEI/ ATI – ale u bilaterální stenózy může dojít ke snížení GF a ↑ sérového kreatininu a hyperkalémii (nutno zkontrolovat před th a pak ještě 3 týdny po zahájení th)

- revaskularizace – u mladých žen s fibromuskulární dysplazií

33.A RESPIRAČNÍ INSUFICIENCE = 2.A

33.B TUBULOINTERSICIÁLNÍ NEFRITIDY

= zánětlivá onemocnění postihující primárně tubuly a renální intersticium- akutní/ chronické, bakteriální/ abakteriálníAkutní pyelonefritida = akutní bakteriální intersticiální nefritida

- často dochání k šíření z dolní močových cest do horních- rozlišovat aktuní nekomplikovanou pyelonefritidu u maldých zdravých žen a aktuní komplikované a chronickéNekomplikovaná

KO – bolest v boku, teplota často >38°C, nauzea, zvracení, bolestivost v kostovertebrálním úhluDg – sediment – leukocyturie, mikroskopická hematurie

- moč na kultivaci (E. coli, klebsiella), vyloučit gravidituTh – nekomplikovaná – cotrimoxazol, trimetoprim, ciprofloxacin

- těžký stav – iniciální hospitalizace s atb i.v. (ceftriaxon, fluorochinolony)- rehyddratace

Komplikovaná- vznikají abscesy v ledvině/ perinefricky/ nekrózou papily- ↑ riziko u pacientů s obstrukcí cest, diabetem a rezistentními bakteriemiKO – jako u nekomplikované + únava, slabost, nauzea, bolesti břicha

- příznak renální koliky s mikro-/ makroskopickou HUEtio – E. coli, citrobacter, enterobycter, pseudomonas aeruginosa, st. AureusTh – hospitalizace, atb i.v.

Akutní alergická intersticiální nefritida- po několika dnech/ týdnech u lidí, kteří užívali β-laktamová atb/ NSAKO – fabrilie, kožní změny, artralgie, eozinofilie, eozinofilurie, hematurie, malinká proteinurie, ↓ renálních fcíTh – vysazení léků

- glukokortikoidyVezikoureterálmí reflux

- projevuje se často recidivujícími infekcemi močových cestEtio – vrozený defekt (zkrácení submukózní části ureteru, při mikci jde moč retrográdně)- těžkých refluxů je proteinurie v důsledku hyperfiltrace ve zbývajících nefronech- u 40 % dojde k spontánnímu vymizení do 2 let u dětíDg – sono, CT, mikční cystografie, statická scinti ledvin (posouzení poškození ledvin)Th – chirurgie

Analgetická nefropatie- chronická intersticiální nefritida vedoucí k fibróze a atrofii ledvin → chronická insuficience, nekróza papil- po několikaletém užívání hl. kombinovaných analgetik (fenacetin + analgetikum + kofein/ ASA + paracetamol +

kofein)- ohroženo 3-4 % populace (hlavně ženy)- analgetický syndrom: analgetická nefropatie, hypertenze, akcelerovaná ateroskleróza, stenózy renálních tepen,

peptické vředy, chronická pankreatitida, anémie, porucha kostního mtb, zvýšená pigmentace kůže

34.A KARDIOMYOPATIE = 3.A

34.B METABOLICKÝ SYNDROM

= dyslipidémie (HDL <1,03 mmol/l u mužů a <1,28 mmol/l u žen, TAG >1,7 mmol/l) + ↑TK (>130/ 85 mm Hg) + obezita (obvod pasu >102 cm u mužů a 88 cm u žen) + porucha glukózové tolerance (gly nalačno >5,6 mmol/l nebo DM II)

70

Page 71: lf1.czlf1.cz/wp-content/uploads/228-vypracovane_otazky_interna... · Web viewprognoza – 50 % dosáhne kompletního uzdravení do 6 měsíců, 20 % má prodloužený průběh –

Vypracované otázky na INTERNU made by Edita Homolková, Monika Ambroziová, Adam Král, Petr Vermach a Vítek Křehnáč

- nejvýznamnější rizikový faktor pro DM II a KVO (1,5-4 krát)- 20-30 % dospělých EvropanůKO – abdominální obezita – nežádoucí je hlavně více metabolicky aktivní viscerální tuk

- inzulinová rezistence – hyperinzulinémie, hyperglykémie- arteriální hypertenze – klidnější je 24 hod sledování, KVO posuzovat i podle tabulky SCORE- dislipidémie - ↑ TAG, ↑alipoproteinu B, ↑ malých denzních LDL (aterogenní, snadněji pronikají arteriální

stěnou), ↓ HDL- možná asociace s nealkoholovou steatózou jater, cholecystolitiázou, SAS, protrombotickým a prozánětlivým

stavem, anxiózně-depresivní poruchou, syndromem polycystických ovárií, obezitou indukovaným hypogonadizmem mužů a ↓ testosteronu a SHBP

Th – doporučení realistického snížení hmotnosti a její udržení (ideálně 5 – 15 % za 6 měsíců)- dieta - ↓ energet příjmu o 15-30 % → redukce 0,5-1 kg/týden, ↓ nasycených tuků a cholesterolu,

jednoduchých cukrů, alkoholu- fyzická aktivita – zlepšení metabolických parametrů (lipidový profil, inzulinová senzitivita, ↑ BM)- psychoterapie- farmakoterapie – antiobezitikum orůstat (váže střevní lipázu)- chirurgická léčba obezity

35.A PORTÁLNÍ HYPERTENZE, KRVÁCENÍ Z JÍCNOVÝCH VARIXŮ = 4.A

35.B SYNDROM DISEMINOVANÉ INTRAVASKULÁRNÍ KOAGULACE

= chorobný stav, u kterého dochází z různých příčin k narušení hemostatické rovnováhy a k rozvoji generalizované intravaskulární koagulace

- DIC = samostatná chorobná jednotka, představuje však syndrom vznikající při jiném zákl. onem. → mnohočetné tromby v mikrocirkulaci (ischemie) + spotřeba části hemostatických činitelů (krvácivý stav)

Etio - generalizované infekce – hlavně G- s endotoxémií, viry, rickettsie- komplikace těhotenství a porodu - septický potrat, embolizace plodové vody, zadržený mrtvý plod,

odloučení lůžka, eklampsie- malignity - generalizace ca, plic, prostaty, GIT, akutní leukemie (hl. promyelocytární AML)- chirurg. stavy - rozsáhlá poranění, popáleniny, velké operač. výkony, použití mimotělního oběhu- imunopatol. stavy - anafylaktické reakce, hemolytické potransfúzní reakce, vaskulitidy- šokové stavy

- uvolnění tkáňového faktoru z poškozených tkání či jeho buněčné expresi v generalizovaných zánětlivých procesech - infekční činitelé aktivují trombocyty nebo poškozují cévní stěnu a činí ji trombogenní → aktivuje se fibrinolytický systém → mikrotromby → ischemie + konsumpce destiček a koagulačních faktorů → krvácivý stav

- ischemická ložiska mohou prokrvácet- druhotná fibrinolýza → odstranění destičkových a fibrinových depozit (štěpení fibrinogenu a f. V a VIII) →

prohloubení krvácivého stavuKO - pestrý, měnlivý, latentní fáze (jen laboratorní nález)/ manifestní fáze (klinika)

- petechie, krvácení v místě vpichů u lehké formy- těžší formy - rozsáhlé sugilace (plošná krevní podlitina) na kůži, neztišitelná krvácení do sliznic/ operač. ran ,

metroragie- dramatický obraz - navíc projev org. postižení ischémií, akutní renální a jaterní selhání, insuficience nadledvin

(Waterhouseův-Fridrichsenův syndrom) či postižení CNS, rozvoj šokové plíceLab - pozitivita D-dimerů, průkaz solubilních komplexů v krvi = parakoagulační test (ethanolový gelifikační a

protaminový precipitační test)- kompenzovaná fáze: zkrácení koagulačního času aPTT- dekompenzovaná fáze: prodloužení koagulačních časů, ↓ destičky a fibrinogen, možná i anemie, schistocyty

Th - zákl. chorobný proces! - vede ke spont. úpravě hemostatických procesů- rozvoj DIC = heparin 10 000 - 15 000 IU kontinuálně za 24 hod, dostatečná konc. antitrombinu (AT)- manifestace + rozvoj krvácivých projevů = koncentráty trombocytů + zmražená krevní plazma +AT III (i

koncentráty fibrinogenu)- podání proteinu C u sepse

36.A ISCHEMICKÁ CHOROBA SRDEČNÍ – PATOGENEZE, KLINICKÉ FORMY= 5A

71

Page 72: lf1.czlf1.cz/wp-content/uploads/228-vypracovane_otazky_interna... · Web viewprognoza – 50 % dosáhne kompletního uzdravení do 6 měsíců, 20 % má prodloužený průběh –

Vypracované otázky na INTERNU made by Edita Homolková, Monika Ambroziová, Adam Král, Petr Vermach a Vítek Křehnáč

36.B MYELOPROLIFERATIVNÍ CHOROBY (MIMO CML)

= stavy, jejichž společným rysem je primární proliferace hematopoetických kmen. bb. myeloidní řady jak v kostní dření, tak extramedulárně

- je-li ve všech zárodečných řadách → panhypercelularita k. dřeně, navíc extramedulární hematopoéza ve slezině, játrech, v periferii erytrocytóza, granulocytóza, trombocytóza

- skupina Ph negativních myeloproliferacíPrimární myelofibróza

- proliferace hlavně megakaryocytární a granulocytární řady s fibrotizací dřeně a extramedulární hemopoezou- etio?, benzen, ionizační záření- fibrotizace dřeně je sekundární, vzniká proliferací nenádorových fibroblastů (jako reakce na myeloproliferativní

změny)KO – prefibrotická fáze – hypercelulární dřeň s minimální fibrózou, v krvi může být třeba jen tromnocytóza

- fibrotická fáze – fibróza dřeně až osteoskleróza, hepatosplenomegalie, leukoerytroblastóza v perifení krvi- ↓ váhy, slabost, nechutenství, noční poty, pocit plnosti, dušnost, poruchy pasáže- splenomegalie u 90 %, hepatomegalie u 50 % → portální přetlak, ascites- u 20 % přechod do akutní leukemie

Dg – prefibrotická fáze – mírná leukocytóza a trombocytopenie, Hb neklesá pod 100 g/l, mírná splenomegalie- fibrorická fáze – pancytopenie, leukoerytroblastická reakce, ↑ CD34+ v periferní krvi,

Th - transplance hematopoet. bb - jediná kurabilní th., u mladších při výrazné progresi- cytoredukční léčba, ery masa, trombokoncetráty, splenektomie/ ozáření sleziny, EPO

- prognóza 3-5 let, přičina smrti (interkurentní inf., krvácení, portální hypertenze, městnavá srd. slabost, 5% přechod do AL)

Primární polycytémie = Polycythaemia vera, m. Vaquez- mutace JAK2 (fyzi aktivována přes EPO-R) → ↑ stimulace erytropoesy, leukopoesy a trombopoesy- může přejít v akutní leukemiiKO – pre-polycytemidká fáze – mírná erytrocytóza

- polycytemická fáze - ↑ krevní masy- post-polycytemická fáze (spent) – myelofibróza, pancytopenii, mladší- bolesti hlavy, závratě, dušnost, poruchy zraku, tinitus, pruritus po koupeli, trombotické komplikace,

splenomegalie, hepatomegalieDg – Hb >185 g/l u mužů a >165 g/l u žen, leukocytóza se ↑ neutrofilů, různě vysoké trombocyty, ↑ celk. ery

masa RCM = red cell mass (↑ viskozita)- koncentrace EPO je ↓ principem zpětné vazby - průkaz mutace genu JAK2- hypercelulární dřeň s proliferací všech 3 řad (možná fibróza u spent fáze)- saturace arteriální krve kyslíkem je u polycytémia vera je normální !, ↓FW

Th – udržení HTK <0,45 a trombocytů <400*109/l- venepunkce, erytrocytoferézy, kys. acetylosalicilová- cytoredukce – IFN-α u mladších, u starších hydroxyurea

- prognóza - chronické, benigní, medián přežití 8-11 let, ohrožení CMP, srdeční selháníPrimární trombocytémie

- chronické myeloproliferativní onemocnění postihující megakaryocytovou řadu- KO – i bez příznaků, trombotické i krvácivé příhody (fčně defektní), parestezie, gangrény, vertigo, bolesti hlavy,

poruchy vizu, splenomegalie možná - vzácně přechod do myelofibrózy

- Lab – trombocytóza >450*109/l, leukocytóza (do 15*109/l), dřeň je hypercelulární- nejsou erytroblasty a mladší formy granulocytární řady v periferní krvi

- Th – asymptomatičtí pacienti kys. acetylsalicylová + INF-α/ anagrelidem- hydroxyurea, trombocytaferéza, anagrelid - antiagregační úč. + tlumí maturaci megakaryocytů nebo INF-α,

ASA

37.A AKUTNÍ INFARKT MYOKARDU ST ELEVACEMI = 6.A

72

Page 73: lf1.czlf1.cz/wp-content/uploads/228-vypracovane_otazky_interna... · Web viewprognoza – 50 % dosáhne kompletního uzdravení do 6 měsíců, 20 % má prodloužený průběh –

Vypracované otázky na INTERNU made by Edita Homolková, Monika Ambroziová, Adam Král, Petr Vermach a Vítek Křehnáč

37. B PORUCHY FOSFOKALCIOVÉHO METABOLISMU

- Ca a P jsou základní komponenty kostní tkáně- intracelulární Ca++ - jako “druhý posel” spouští řadu intracelulárních dějů- fosfáty - hlavní intracelulární aniont, součást membránových fosfolipidů, ATP, kreatinfosfátu, NADP, glukózo-1-

fosfátu- na úrovni střeva, kosti a ledvin zasahují:

- parathormon (PTH) - při hypokalcémii stoupne sekrece PTH v příštítných tělíscích a uvolněním Ca++

z kostí se kalcémie rychle normalizuje- PTH také ↑ resorpci Ca++ a exkreci P v ledvinách

- kalcitriol (1,25-dihydroxyvitamin D) vzniká syntézou v kůži, játrech a ledvinách; ovlivňuje hlavně dlouhodobou bilanci Ca++ - ↑ vstřebávání Ca++ z potravy, ↑vstřebávání P

- kalcitonin - secernován parafolikulárními buňkami štítné žlázy při ↑ Ca2+ a působí jako funkční antagonista PTH v kostech i v ledvinách

- celkově je tedy okamžitá koncentrace Ca++ hlavně pod kontrolou PTH a dlouhodobá bilance Ca++ hlavně pod kontrolou kalcitriolu

- Ca++ cirkuluje v plasmě jednak ionizované (asi 50 %), jednak vázané na bílkoviny (hlavně albumin) a jednak (malá část) jako rozpustné komplexy

- volná a vázaná frakce jsou v dynamické rovnováze a jen ionizované Ca++ má fyziologickou aktivitu- vazba je závislá na pH, při acidóze se snižuje, při alkalóze zvyšuje podíl vázaného Ca++

- fosfáty se v plasmě vyskytují v několika chemických podobách : HPO42-, NaHPO4

-, H2PO4- a další, konkrétní poměry

závisí m.j. na pH- většina P je ionizována, jen asi 12 % je vázáno na proteinyHyperkalcémie

- >2,6 mmol/l, hlavně vzestup ionizované frakce (závisí i na albuminémii)- závislá na PTH – primární/ sekundární hyperparatyreóza, familiární hyperparatyreóza (MEN I, MEN IIa),

familiární hypokalciurická hyperkalcémie (mutace kalciového receptoru)- nezávislá na PTH – malignity, intoxikace vit. D, chronická granulomatózní onemocnění, hypertyreóza,

feochromocytom, imobilizace, lithium, thiazydy…Primární hyperparatyreóza – zpravidla mírná hyperkalcémie do 2,75 mmol/ l, obvykle asymptomatické, kalciová

nefrolitiázaNádory - ↑ kostní resorpce u metastáz do skeletu

- myelom → RANKL → aktivace osteoklastů- mohou produktovat PTH related peptide, lymfomy mohnou produkovat kalcitriol

KO – mírná – asymptomatická, únava, deprese, zácpa- těžká – polyurii, polydipsie, dehydratace, anorexie, nauzea, svalová slabost, zkrácení QT, letargie,

zmatenost, komaTh – nemusí se spšchat, ale náhlý vzestup a těžká hyperkalcémie (3,5 mmol/l) vyžaduje léčbu

- rehydratace - 200 -300 ml/hod- kalcitriol i.v. – zvyšuje kalciurii a snižuje kostní resorpci- bisfosfonáty- glukokortikoidy inhibují tvorbu kalcitriolu a tím i střevní absorpci- hemodialýza

Hypokalcémie- pokles pod 2,1 mmol/l- hypoalbuminémie (ionizované zústává v normě), acidóza, rezistence na účinek PTH při hypomagnezémii- hypoparatyreóza – chir., aktinoterapie, utoimunitní polyglandulání sy., mutace kalciového rec- sekunární hypoparatyreóza – málo vit. D, rezistence na vit. D, chronické selhání ledvin, ztráty

(hyperfosfatémie, akutní pankreatitida, rozpad tumorů…- léky – bisfosfonáty, cinacalcet (aktivátor kalc. Rec.), foscarnetKO – tetanie (hl pod 1,9 mmol/l, horší při hypomagnezémii a alkalóze)

- Trousseauovo znamení – addukce palce, flexe zápěstí a metakarpofalangeálních kloubů při nafouknutí manžety nad úroveň systolického tlaku

- Chvostkovo znamení – kontrakce ipsilaterální svalů tváře vyvolaná poklepem na lícní nerv před ušním boltcem

73

Page 74: lf1.czlf1.cz/wp-content/uploads/228-vypracovane_otazky_interna... · Web viewprognoza – 50 % dosáhne kompletního uzdravení do 6 měsíců, 20 % má prodloužený průběh –

Vypracované otázky na INTERNU made by Edita Homolková, Monika Ambroziová, Adam Král, Petr Vermach a Vítek Křehnáč

- prodloužení QT na EKG- edém oční papily, psychické změny

Th – i.v. kalciumglukuronát (1-2 g v 10 % rozkoku za 10-20 min)- neaktuní formy p.o. kalcium a vit. D

Hyperfosfatémie- jen u lidí se ↓ fcí ledvin, při extrémním přívodu fosfátů dietou, uvolňování z tkání a ↑ zpětné resorpci

v proximálním tubulu- pseudohyperfosfatémie – u hyperglobulinémie, hyperlipidéme, hemolýzou a hyperbilirubinémií

- interference analytických metod- u syndromu z rozpadu nádorových bb. masivní uvolnění fosfátů, při rabdomyolýze, masivní hemolýza,

transfúze dlouho skladované krveTh – hemodialýza při současné hypokalcémii a ↓GF

- izotonický roztok NaClHypofosfatémie

- <0,8 mmol/l- často u alkoholiků, těžké sepse, traumat- redistribuce z ECT do ICT - ↑ inzulin, akutní respirační alkalóza, syndrom hladové kosti- ↓ střevní absorpce – málo v dietě, průjmy, steatorea, deficit/ rezistence na vit.D- ↑ renální exkrece – primární/ sekundární hypoparatyreóza, deficit/ rezistence na vit.D, osmotická diuréza,

Franconiho syn.KO – symptomatická <0,3 mmol/l

- dlouhodobá hypofosfatémie, snižuje tubulární resorpci Ca a Mg a zvyšuje renální produkci kalcitriolu a stimuluje kostní resorpci

- encefalopatie, srdeční selhání, slabost dýchacích svalů, dysfagie, ileus, hemolýza, ↓ chemotaxe a fagocytóza

Th – suplementace vit D, fosfáty jen u symptomatických a s tubulární poruchou

38.A CHRONICKÉ SRDEČNÍ SELHÁNÍ = 7.A

38.B PRINCIPY ÚČELNÉ HEMOTERAPIE, TRANSFÚZNÍ PŘÍPRAVKY A KREVNÍ DERIVÁTY. PŘEDPOKLADY DAROVÁNÍ KRVE, VYŠETŘENÍ DÁRCE

Účelná hemoterapie-spočívá v cíleném podání těch krevních komponent, jejichž obsah je v cirkulaci nemocného snížený, a který

může být příčinou život ohrožujících komplikací- v současné době se dávají transfúze plné krve vzácně, nahrazena transf. přípravky- cílem je dosažení max. léčebného efektu při min. ohrožení příjemce nežádoucími účinky- lékař podává transfúzi jen v odůvodněných případech, vždy určité riziko!

Transfúzní přípravky- připravují se na transfúzních odd. centrifugací, protivirově neošetřené (zatím)- vývoj PIT (patogen inaktivačních technik) přímo v transfúzních přípravcích na transfúzních odd. (psoraleny,

riboflavin)- skladování +4 +6 °C, 30-40 dní, kontinuální sledování hodnot1. plná krev – při dárcovském odběru 400-500 ml, expirace 21-35 dní

- obvykle dále zpracována na koncentrát erytrocytů, plazmu a koncentrát trombocytů (i granulocytů)

- jen ve vyjímečných situacích, krvácející pacienti se ztrátou >25 % celkového objemu krve2. erytrocyty – z plné krve nebo hemaferézou, malé příměsi, expirace 21-42 dní

- indikace u Hb <70 g/l, transfúze 1 jednotky → ↑ hematokritu o 3 % u dospělých a o 6 % u dětí- ery resuspendované – erymasa, plazma nahrazena roztoky s ochraným účinkem- ery resuspendované bez „buffy coatu“ - ↓ obsah leukocytů, pro opakovaně transfundované

pacienty

74

Page 75: lf1.czlf1.cz/wp-content/uploads/228-vypracovane_otazky_interna... · Web viewprognoza – 50 % dosáhne kompletního uzdravení do 6 měsíců, 20 % má prodloužený průběh –

Vypracované otázky na INTERNU made by Edita Homolková, Monika Ambroziová, Adam Král, Petr Vermach a Vítek Křehnáč

- erytrocyty deleukotizované – zbavení 99 %leukocytů, prevence alloimunizace a febrilních reakcí u pacientů senzibilizovaných HLA antigeny

- promyté ery – při alergických reakcích na plazmatické proteiny- zmražené ery – uchovávání auto a alogenních ery pro pacienty s prolátkami proti ery- ozářené ery – prevence GVHD u imunosuprimovaných příjemců transfúze- komplikace – časné: objehové přetížení, hypokalcémie díky citrátu, hemolýza z fyzikálních příčin,

horečka a šok z bakteriální kontaminace, hemolýza při inkompatibilitě (horčka, dušnost, tachykardie, bolest v bedrech, hypotenze, šok, zvracení, neklid, renální selhání)

- pozdní: přetížení Fe, infekce, pozdní hemolytická reakce se zrychlenou destrukcí ery, alloimunizace

3. Trombocyty – expirace 5 dní, za teploty 20-24 °C- minimální terapeut. dávka trombocytů je 2.1011 (trombocyty od 4 různých dárců)- profylakticky u pacientů s <5-10*109/l nebo u pacientů s vyšší hodnotou, ale symptomatických

a před invazivním výkonem- z plné krve – cca 0,5*1011 trombocytů, resuspendované v 50-70 ml plasmy- z aferézy – cca 2*1011 trombocytů- deleukorizované trombocyty z aferézy – prevence HLA alloimunizace a přesnosu CMV

4. Plazma - zmražená; 24 měsíců, T -25°C- drží se v karanténě 6 měsíců, pak se dárce opakovaně vyšetří na infekce- terapie krvácení, výměnná plazmaferéza u HUS a TTP

Krevní deriváty- složky plazmy, které se získávají ve frakcionačních zařízeních, protivirově ošetřené- frakcionací plazmy → f. VIII, IX, albumin, antitrombin III, i.v. imunoglobuliny

Předpoklady darování krve + vyš. dárce- způsobilost k odběrům (dobrý zdr. stav, vyhovující parametry krevního obrazu a moči, biochemie,

anamnestický dotazník, poučení, věk 18-60 let, >50 kg)- dárci s rizikovými faktory nebo rizikovým chováním nemohou krev darovat- každý dárcovský odběr - stanovení ALT, markry HIV 1 a 2; HBV, HCV (ELISA, NAT), syphilis, krevní sk. ABO, Rh,

nepravidelné protilátky proti erytrocytům- „jen“ negativní výsledky infekč. Markrů → transfúzní přípravky ke klin. podání

39.A FIBRILACE SÍNÍ, JEJÍ KOMPLIKACE A LÉČBA = 8.A

39.B STRUMA, ZÁNĚTY ŠTÍTNÉ ŽLÁZY, NÁDORY ŠTÍTNÉ ŽLÁZY

Záněty štítné žlázyAkutní tyreoiditida – hnisavý infekční zánět, vzácně

KO – bolestivé zduření, kůže nad teplejší a zarudlá, celk. příznaky- sedimentace, CRP, fce většinou bez alterace- možná aspirace tenkou jehlou (FNAB) pro cytologii a kultivaci

Th – ATB, při fluktuaci drenáž, chir. resekceSubaktuní tyroiditida = De Quervainova, granulomatózní, obrovskobuněčná

– relativně častá, spíš ženy, etio viry (adenoviry, Coxsackie, příušnice)KO – po virovém onemocnění HCD i bez něj

- bolestivé zduření (vystřeluje do čelisti a ucha), únava, zvýšená teplota, možná tranzitorní destrukční hypertyreóza

Dg - ↑ sedimentace, leuko a CRP spíš normální, ↓ TSH u trazirotní destrukce, USG, FNABTh – nesteroidní antirevmatika, glukokortikoidy, β-blokátory u tranzitorní destrukce

Chronická tyroiditida = Hashimotova, chron. lymfocytární/ autoimunní- senzitizace lymfocytů proti tyreoidálním antigenům (TgAb, TPOAb, TBAb)

75

Page 76: lf1.czlf1.cz/wp-content/uploads/228-vypracovane_otazky_interna... · Web viewprognoza – 50 % dosáhne kompletního uzdravení do 6 měsíců, 20 % má prodloužený průběh –

Vypracované otázky na INTERNU made by Edita Homolková, Monika Ambroziová, Adam Král, Petr Vermach a Vítek Křehnáč

- probíhá dlouho asymptomatickyKO – hypotyreosa

- struma – někdy v počáteční fázi onemocnění, mírné difúzní zvětšení, většinou nebolestivé, pak atrofizace nebo zvětšení při uzlovité přestavbě

Dg – protilátky, SONO, hormonyTh – substituce hypotyreózy

Riedelova tyreoiditida = invazivní fibrózní struma- vzácné, ženy středního a vyššího věku, etio nejasnáKO - asymetrické zvětšení, kamenně tuhá, fixyce a komprese okolí → dysfagie, dušnost,

chrapot – n. laryngeus recurens- většinou hypotyreóza (i hypoparatyreoza)

Struma = zvětšení štítné žlázy- difúzní/ nodózní (uni/ poly), eufunkční/ hyperfunkční/ hypofunkční, endemická- celosvětová prevalence 5 % (oblasti s málo jódem 75 %)Toxická struma – Grawes-Basedov, adenom (uninodulární, polynodózní), adenom produkující TSHEufunkční struma – při poruše syntézy T4 a T3, jódový deficit (fokální hyperplazie, nekrózy, hemoragie)

- nodózní – fokální tyreoitida, benigní adenom, karcinom (studený uzel), cysty- strumigeny – interference s vychytáváním a organifikací jódu, se syntézou a metabolismem

hormonů- soja, brokolice, zelí, kapusta, květák

Hypofunkční struma – chronická lymfocytární tyreoiditidaKO – zvětšení, konzistence

- retrosternální šíření – CT- Pembertonovo znamení – komprese vv. Jugulares → zarudnutí obličeje, dilatace krčních žil při vzpažení- syndrom horní duté žíly - silné zduření žil, cyanóza a edém na krku (Stokesův límec), postupně i na hlavě a

horních končetináchEndemická struma – deficit I, často provázeno hypotyreózou

- v těchto oblastech i snížený intelektu a kretenismus- stanovení joduje- ideální 150 – 300 μg I/den, těžký deficit při 25 μg/ denTh – suplementace, chir jen při útlaku okolních struktur

Karcinomy štítné žlázy= maligní epiteliální ca, většinou z folikulárních bb., vzácněji z medulární ca z parafolikulárních bb.- relativně částé (10/ 100 000 obv),častěji ženy, většinou asymptomatické, tuhé nebolestivéKO – tuhý uzel, někdy fixovaný, 3 % manifestace metastázami (regional. uzliny, plíce, kosti)Dg – cytologický průkaz po chirurg odstranění a histologický potom

- regionální uzliny fyzikálně, USG, citologicky a histologicky, CT hrudníku, scintigrafie (akumulace I)Th – diferencovaný ca (papilární a folikulární) – odpovídají na TSH a vychytávají I

- tyreoidektomie, odstranění postižených uzlin, radioablace případných zbytků a metastáz radiojódem (udělat vysoké hladiny TSH)

- nediferencovaný (anaplastický) ca – léčba dle stádia v TNM klasifikaci- supresní terapie hormony štítné žlázy (kompenzace hypotyreózy a uvedení do hraniční

hypertyreózy kvůli supremaci sérového TSH)- medulární ca – chirurgicky tumori i metastázy

40.A MITRÁLNÍ STENÓZA A MITRÁLNÍ REGURGITACE = 9.A

40.B DIABETES MELLITUS I. TYPU

- cca 7 % diabetiků v ČR- B-bb. mají velkou kapacitu (stačí 10-20 % funkčních bb.)Etio – autoimunní onemocnění s destrukcí B-bb. (inzulitida), je normální citlivost tkání na inzulin

- protilátky anti GAD, anti-IA2, IAA…- idiopatický – bez průkazu protilátek

76

Page 77: lf1.czlf1.cz/wp-content/uploads/228-vypracovane_otazky_interna... · Web viewprognoza – 50 % dosáhne kompletního uzdravení do 6 měsíců, 20 % má prodloužený průběh –

Vypracované otázky na INTERNU made by Edita Homolková, Monika Ambroziová, Adam Král, Petr Vermach a Vítek Křehnáč

- LADA (latent autoimunne diabetes in the adults) – v jakémkoli věku- virové infekce, kravské mléko, toxické látky, stres… ?- možná součást „sdružených autoimunit“ (štítná žláza, celiakie…)

KO – hlavně u mladých, dosud zcela zdravých- žízeň, polydipsie, polyurie, nykturie, ↓ hmotnosti, únava, poruchy visu, diabetické ketoacidotické koma- klasický typ – v dětství až adolescenci, rychlá progrese- LADA – ze začátku se podobá DM II

Dg – hyperglykemie (15-30 mmol/l, stanovení jako u DM II), ketoacitóza u mladého, dosud zdravého a štíhlého člověka

- často dehydratace (hypernatrémie, hyperchloremie, hyperkalemie), někdy i ↑ urey a kreatininu, ↑ lipidy (po kompenzaci diabetu vymizí), může být i ↑ amyláza (ale bez pankreatitidy)

- glykosurie, ketonurie- C-peptid, protilátky proti B-bb.

Th – inzulinoterapie – intenzifikovaný inzulinový režim (bazál-bolus, inzulin s prodlouženým účinkem 1 nebo 2 krát denně a krátkodobý inzulin v bolusu před jídlem)

- zahájení nižšími dávkami (přetrvává ještě vlastní produkce – fáze honeymoon, cave hypoglykémie)

- při infekčních onemocnění ↑ inzulin o 10-30 %- individuální dietní režim – regulovaná strava- životní styl – fyzická aktivita (nutno snížit dávku krátkodobého inzulinu, 1-2 h pojídle), kouření…- edukace pacienta i členů rodiny- psychosociální péče- pravidelné sledování – 1x/3 měsíce

41.A JATERNÍ CIRHÓZA = 10.A

41.B ONEMOCNĚNÍ GONÁD; POLYGLANDULÁRNÍ SYNDROMY (SYNDROMY MNOHOČETNÉ ENDOKRINNÍ NEOPLAZIE, AUTOIMUNITNÍ POLYGLANDULÁRNÍ DYNDROMY)

ONEMOCNĚNÍ GONÁD

POLYGLANDULÁRNÍ SYNDROMYAutoimunní polyglandulární syndromy

- autoimunní postižšní několika žláz APS-I – AR, vzácné (10/1 000 000), manifestace obvykle v dětství

- chronická mukokutánní kandidóza + autoimunitně podmíněná hypoparatyteóza + autoimunitně podmíněná adrenokortikální insuficience (diagnostická je přítomnost 2 znaků)

- mutace AIRE genu (nukleární protein a transkripční faktor, důležitý v navození imunitní tolerance)- hypogonadismus u 60 % žen a 45 % mužů, hypoparatyreóza, dystrofie nehtů, hypoplazie sklovinyTh – léčba jednotlivých součástí syndromu

APS-II – adrenokortikální insuficience (100 %)+ autoimunitní tyreopatie (70 %) + DM I (50 %)- manifestace v dospělosti (20.-40. rok), převaha žen, polygenní dědičnostTh – léčba jednotlivých součástí syndromu

SYNDROMY MNOHOČETNÉ ENDOKRINNÍ NEOPLAZIE- tumory žláz s vnitřní sekrecí často provázené hypersekrecíSyndrom MEN-1 – Wermerův syndrom

- AD, 2-20/ 100 000- delece/ mutace genu pro menin (tumor supresor)- primární hyperparatyreóza + tumory endokrinního pankreatu (nejčastěji Zollinger-Ellisonův syn, inzulonomy)

+ tumory hypofýzy- adenomy nadledvin, karcinoid, angiofibromy obličejeTh – léčba jednotlivých součástí syndromu

Syndrom MEN-2A – Sippleův syndrom- AD, vzácné, mutace RET protoonkogenu

77

Page 78: lf1.czlf1.cz/wp-content/uploads/228-vypracovane_otazky_interna... · Web viewprognoza – 50 % dosáhne kompletního uzdravení do 6 měsíců, 20 % má prodloužený průběh –

Vypracované otázky na INTERNU made by Edita Homolková, Monika Ambroziová, Adam Král, Petr Vermach a Vítek Křehnáč

- medulární ca štítné žlázy + feochromodytom + primární hyperparatyreózaSyndrom MEN-2B

- AD, vzácné- medulární ca štítné žlázy + feochromocytom + slizniční neuromy- marfanoidní habitus, střevní ganglioneuromy

42.A CHRONICKÁ ISCHEMICKÁ CHOROBA SRDEČNÍ = 11.A

42.B HYPOTALAMO-HYPOFYZÁRNÍ SYSTÉM A JEHO ONEMOCNĚNÍ; DIABETES INSIPIDUS CENTRALIS

Onemocnění hypotalamu- velmi pestrá symptomatologieEtio – nádory (tu hypotalamu a hypofýzy, kraniofaryngeom), meningitida, meningoencefalitida, granulomatózní

procesy, trauma, hemoragie, ischemie, vývojové vady (Kallmanův syndrom = anosmie a deficit gonadotropinů s nedostatečným vývojem pohlavních žláz, septooptický dysplazie)

KO – endokrinologické příznaky – GnRH, CRH, TRH,SST,GHRH, dopamin, ADH, oxytocin- neendokrinologické příznaky – poruchy příjmu potravy (hyperfagie, anorexie, kachexie)/ tekutin

(polydipsie, adipsie, esenciální hypernatrémie)/ termoregulace/ psychiky/ spánku/ vědomí/ vegetativního systému (srdeční rytmus, sfinktery, pocení)

Onemocnění adenohypofýzy – viz 47.BHypopituitarismusTumory hypofýzyProlaktinomy a hyperprolaktinémieAkromegalie a gigantismusCushingův syndrom a Cushingova choroba – viz 77.ATyreotopin produkující adenomyGonadotropiny produkující adenomy

Onemocnění neurohypofýzyDiabetes insipidus centralis –vzácný

- důsledek nedostatečné sekrece ADHEtio – porucha hypotalamických neuronů nebo zadního laloku hypofýzy

- primární u 1/3 (bez morfologické léze, autoimunita/ genetika?)- sekundárně traumatem, chirurgickým výkonem, zánětem, cévní lézí, infiltrativním procesem

KO – polyurie (až 10l/den), polydipsie (při zachovalých osmoreceptorech v organum vasculosum laminae terminalis)

Dg – vyšetřit každého s polyurií >2,5 l/den, mineralogram, osmolalita séra- koncentrační test – odnění tekutin až na 36 hod resp. do ztráty 3 a více % hmotnosti/

bezvědomí/ hypotenze…Th – substituce desmopresinem

Syndrom nepřiměřené sekrece antidiuretického hormonu (SIADH) – Schwartz-Bartterův syndromEtio – tumory (bronchogenní ca, mezoteliom, thymom, ca duodena, ca pankreatu, ca endometria, ca prostaty,

leukémie…), postižení CNS (tumory, abscesy, subdurální hematomy, záněty, degenerativní onemocnění, trauma, delirium tremens, hematomy, hydrocefalus), polékově (abusus nikotinu, fenothiazinů, bicyklických antidepresiv, desmopresin)

KO - ↑ ADH → euvolemická hypoosmolalita s hyponatrémií- neurologické příznaky (hyponatremická encefalopatie: bolest hlavy, nauzea, zvracení, zmatenost, křeče,

kóma, respirační selhání), mozkový edémDg – osmolalita plasmy <275 mOsm/kgH2O a moči >100 mOsm/ KgH2O

- euvolémie, ↑ vylučování Na+ při normální příjmu vody a sodíkuTh – akutní – korekce osmolarity a natrémie (postupně!!! → povinní a extrapontinní myelinolýza)

- roztok NaCl tak, aby koncentrace rostla 0,5-2 mmol/l/hod až do vymizení neurologických příznaků/ natrémie 125 mmol/l

- chronická – trvající více jak 48 hod, bez neurologických příznaků- restikce tekutin

78

Page 79: lf1.czlf1.cz/wp-content/uploads/228-vypracovane_otazky_interna... · Web viewprognoza – 50 % dosáhne kompletního uzdravení do 6 měsíců, 20 % má prodloužený průběh –

Vypracované otázky na INTERNU made by Edita Homolková, Monika Ambroziová, Adam Král, Petr Vermach a Vítek Křehnáč

Syndrom prázdného sedla- díky rozšíření subarachnoidálních cysteren do sedla (insuficience diafragma sellae, hemoragická infarzace

adenomů, neurochirurgická/ radiační terapie) a útlaku hypofýzy

43.A BRONCHOGENNÍ KARCINOM = 12.A

43.B UROLITIÁZA

- potihuje 4%, muže 2-3x čatějiEtio – hypersaturace moči kamenotvornými látkami + krystalizační centra + ↓ inhibitorů litogeneze (citrát, Mg 2+,

pyrofosfát…)- ↓ diuréza, ↑ exkrece Ca2+/ urátů/ oxalátů, abnormální pH (↓ u urátových/ cystibnovývh a ↑ u

kalciumfosfátových/ struvitových- hyperkalciurie – většinou s normokalcémií- hypocitráturie – u chron. průjmů, thiazidových diuretik, distální renální tubulární acidóza- hyperoxalurie – primární (porucha mtb glycinu), sekundární u poruchy resorpce tuků (biliární obstrukce,

chron. pankreatitida, Crohnova ch., resekce střeva…)- struvitové kameny – fosforečnan hořečnatoamonný, vzniká činností některých bakterií (proteus,

pseudomonas, klebsiella)- odlitkové konkrementy

- urátové kameny – u ↓ diurézy, při normální urémii- cystinové kameny – cystinurie, vrozená porucha renálního transportu cystinu, ornitinu argininu

KO – asymptomatické- mikro-/ makroskopická HU, - renální kolika – intenzivní (někdy přerušovaná) bolest v boku vyzařující do třísla- obstrukce cest s infekcí až urosepsí

Th – kolika – analgetika, spazmolytika, hydratace- hospitalizace při nekontrolovatelné bolesti, zvracení, infekcí, obstrukcí

- spontánně odchází kameny obvykle do velikosti 4 mm- extrakorporální litotrypse, endoskopická extrakce, extrakce perkutánní nefrotomií- dieta – diuréza alespoň 2 l/ den, normaliza přijmu Ca

- při oxalurii nejíst špenát, oříšky a omezit kávu a čaj- při hyperurikosurii omezit maso

- farm – thiazidy – redukují kalciurii o 50 % a ↓ exkreci oxalátů- při současné hyperkalciurii podávat kaliumcitrát k zabránění hypocitrátuii

- alopurinol – u hyperurikosurie- penicilamin – u cystinurie

44.A PLICNÍ EMBOLIE = 13.A

44. B AKUTNÍ LEUKÉMIE

-heterogenní sk., klonální proliferace nevyzrávajících leukemických blastů, útlak normální hematopoézy- hyatus leucaemicus – anemie, trombocytopenie, neutropenie, leukemické blasty- náhlá manifestace (ne akutní vznik), neléčení do několika týdnů umírajíAkutní myeloidní leukémie (AML)

- incidence roste s věkem, medián 65 let- častější výskyt u předchozích hematolog. onem. (MDS, myeloproliferace), genetické onem. (Downův syn.),

chemikálie, ionizující záření, chemoterapie (za 5-10 let, alkylační látky)- de novo AML – primární, není spojeno s jinou chorobou- AML s myelodysplastickými změnami – po léčbě malignity, vzniklá transformací MDSKO - nespecifické, pestré, chřipkovité příznaky (únava, slabost, subfebrilie), bolesti skeletu, anemie (dušnost,

bledost), hemoragická diatéza z trombocytopenie (petechie, hematomy, epistaxe, krvácení z dásní)- bledost, krvácivé projevy, v ústní dutině zánětlivé změny až nekrózy na dásních- oportunní infekce

79

Page 80: lf1.czlf1.cz/wp-content/uploads/228-vypracovane_otazky_interna... · Web viewprognoza – 50 % dosáhne kompletního uzdravení do 6 měsíců, 20 % má prodloužený průběh –

Vypracované otázky na INTERNU made by Edita Homolková, Monika Ambroziová, Adam Král, Petr Vermach a Vítek Křehnáč

- leukostáza (leu >50 000*109/l) → dušnost, bolesti hlavy, poruchy vizu- ne lymfadenopatie a splenomegalie (jen při sekundární etio, progrese CML/ myelofibrózy)- při neurologických symptomech provést lumbální punkci (infiltrace mening)

Lab - krevní obraz – leukocytóza s vyplavováním blastů, méně často cytopenie- diferenciál - hiatus leucaemicus (vedle zralých granulocytů různé % atypických blastů , střední vývojová

stádia granulocytů chybí = nejsou tam promyelocyty, myelocyty, metamyelocyty)- punkce k. dřeně – dostatečné k dg., ale typ imunologicky, cytogeneticky, cytochemicky-+ reakce na

myeloperoxidázu vylučuje lymfoidní původ leukémie- hypercelulární se zmnoženou granulopoezou (alespoň 20 %)

Klasifikace: FAB - morfologie a cytologie sedm forem AML (M1-M7)M0 - nediferencovanéM1 - bez vyzráváníM2 - s vyzrávánímM3 - promyelocytárníM4 - myelomonocytárníM5 - monoblastická a monocytárníM6 - erytroleukémieM7 – megakaryoblastická

WHO - molekulárněbiologické rozdíly, přesnější- existují nediferencované leukémie = hybridní (bifenotypové) leukémie se znaky myeloidní i

lymfoidní linie (léčí se jako leukémie myeloidní)Th – rozhodnout, jestli paliativní nebo kurativní

- chemoterapie - 1. Předváze - provádí se při leu >100.109/l (hydroxyurea + glukokortikoidy + leukocytaferéza)- 2. indukční terapie - při poklesu leu <50.109/l (protokol 3 + 7 = daunorubicin po 3 dny +

cytarabin v kontin. infúzi po 7 dnů)- 3. záchranná léčba - nebyla-li indukční th. Úspěšná (různé kombinace) - salvage- 4. postremisní th - konsolidační léčba kvůli přetrvávání reziduální leukémie

- transplantace – autologní/ alogenní (GvL)Prognóza - průběh neléčené AML je fatální (smrt během několika týdnů), příčinou jsou komplikující infekce

- celkové přežití mladých pacientů 30 %, u starších 15 %Akutní promyelocytární leukemie (APL)

- jedna z forem AML (dle FAB M3), ale jiná klinika a prognóza, 5 % případů AML- urgentní!, ale prognóza dobrá, medián 40 letKO – pancitopenie, laboratorní známky DIC, atypické promyelocyty s hypergranulární cytoplasmou v krvi i dřeni,

Auerovy tyče- krvácení ze sliznic, do kůže, retiny, plic, CNS (letální)

Th – indukční – ATRA (all-trans reinová kys.) + idarubicin- zvládnutí DIC a prevence krvácení

Akutní lymfoblastová leukémie (ALL)- nejčastější nádorové onemocnění dětí (kolem 4 let věku), u dospělých vzácnější (20 %), heter. sk. onem. - etio?, vzácné formy T-ALL (HTLV-1, EBV), ionizující záření, benzen- leukemický zvrat v thymu nebo dřeni, 75 % vzniká z B-bb a zbytek z T-bb.- u dětí přežití v 80 %, u dospělých ani ne 50 %- u dospělých je častá translokace t(9;22) = Ph chromosom, tyto leukémie mají nejhorší prognózu (přítomnost

lymfoidní i myelooidní řady = leukémie hybridní)- defekt regulace není spojen s maligním postižením pluripotenetní kmenové b (možno vyléčit bez BSCT)KO - obecné symptomy (únava, slabost, krvácivé projevy, anemické příznaky)

- anémie (slabost, dušnost, únava), trombocytopenie, infekce - hepatosplenomegalie, adenomegalie (mediastinální uzliny), infiltrace CNS (nitrolební hypertenze, paréza

hlavových nervů)- adenomegalie, mírná splenomegalie, hepatomegalie, mediasti. uzliny ( u T-formy), meningy

(↑nitrolebečního tlaku), poruchy vizu při infiltraci n. opticus, infiltrace varlatLab – ve dřeni a krvi různorodá populace blastů (myeloperoxidáza neaktivní), anemie, trombocytopenie,

neutropenieoblasty, anémie, trombocytopénie Klasifikace - EGIL ( European Group for Imunological charakterization of Leukemias) - dle imunofenotypu

80

Page 81: lf1.czlf1.cz/wp-content/uploads/228-vypracovane_otazky_interna... · Web viewprognoza – 50 % dosáhne kompletního uzdravení do 6 měsíců, 20 % má prodloužený průběh –

Vypracované otázky na INTERNU made by Edita Homolková, Monika Ambroziová, Adam Král, Petr Vermach a Vítek Křehnáč

Th – chemoterapie – vinkristin + prednisolon + daunorubicin + asparagináza + cyklofosfamid - léčba energická!- profylave leukemického postižení CNS - intrathékálně cytostatika (metotrexát)- konsolidační léčba - do 2-4 týdnů po skončení indukční léčby = velmi intenzivní chemoterapie, navodí

dlouhotrvající bezpříznakové období u mladších osobPrognóza - liší se dle typu, jiné u dětí a dospělých, dospělí s Ph = špatné, velmi špatná u relaps

45.A ZÁNĚTLIVÁ PLICNÍ ONEMOCNĚNÍ- PNEUMONIE, PLICNÍ ABSCES = 14.A

45.B UMĚLÁ VÝŽIVA

Enterální výživa- podávání farmaceuticky připravených výživných roztoků do GIT- cca 50-60 % energie v sacharidech, 30-40 % v tucích a zbytek bílkoviny/ peptidy, vitamíny, minerály, stopové

prvky v DDD- stimulace motility, udržení hepatobiliárního oběhu…Způsoby podávání – sipping – p.o., jako doplněk

- sonda – nazogastrická, nazojejunální- při riziku aspirace, obstrukce horní části GIT, poruchy vědomí/ polykání…- PEG (perkutánní endoskopická gastrostomie) – nad 6-8 týdnů- J-PEG

- bolusově/ kontinuální (do jejuna)KI – NPB, akutní krvácení do GIT, mechanický ileus, šokový stav, těžká hypoxie/ acidózaPolymerní – živiny ve své původní formě, měly by obsahovat vlákninu (KI u zúžení střevního lumen a těžkých

nespecifických střevních zánětů)- k podáváni do žaludku

Oligomerní – při průjmech, zácpě, pankreatické insuficienci/ akutní pankreatitidě (netřeba trávicí enzymy), syndrom krátkého střeva, těžká malabsorpce…

- do tenkého střevaKomplikace – intolerance – reflex, nauzea, zvracení, bolesti břicha…

- metabolické – hypo i hyper /hydratace/ kalémie/ fosfatemie/ natrémie…- dekubity, poranění při zavádění, zavedení do dýchacích cest (bezvědomí)

Parenterální výživa- podávání výživy do cévního systému- šok, nestabilní pacient, NPB, akutní krvácení do GIT + (akutní pankreatitida, těžký průjem, enterokutální píštěle,

zvracení, žaludeční atonie)- Systém all-in-one – průmyslově vyráběné/ magistraliter- podává se cyklicky, respekt nočního metabolického kliduPeriferní – méně komplikací, osmolarita 900 mosmol/l

- nižší obsah živin, částečná výživa, na 7-10 dníCentrální – větší množství, 1200 mosmol/l, riziko při kanilaci v. subclavia/ jugularis interna do vena cava superior

- venózní port (komůrka na konci katetru krytá membránou)/ tunelizované katetry (veden dlouhým podkožním tunelem)

46.A ISCHEMICKÁ CHOROBA DOLNÍCH KONČETIN = 15.A

47.A NESTABILNÍ ANGINA PECTORIS, AKTUNÍ INFARKT MYOKARDU BEZ ST ELEVACÍ = 16.A

47.B ONEMOCNĚNÍ ADENOHYPOFÝZY

Hypopituitarismus- nedostatečnost sekrece 1 nebo více hormonů pření hypofýzy- panhypopituitalrismus = ↓ sekrece všech hormonůEtio – útlak expanzivními procesy (adenomy, kraniofaryngeom, Ratkeho cysty, meningiomy, chordou,

aneurysma, granulomy), genetika, idiopatické, trauma, ozáření, autoimunita, ischemie, hemochromatóza

81

Page 82: lf1.czlf1.cz/wp-content/uploads/228-vypracovane_otazky_interna... · Web viewprognoza – 50 % dosáhne kompletního uzdravení do 6 měsíců, 20 % má prodloužený průběh –

Vypracované otázky na INTERNU made by Edita Homolková, Monika Ambroziová, Adam Král, Petr Vermach a Vítek Křehnáč

KO - ↓ GH - v dětství → zpomalení a zástava růstu, v dospělosti snížení aktivní hmoty a snížení fyzické výkonnosti a kognitivních fcí, snížení citlivosti na inzulin…

- ↓LH a FSH – hypogonadotropní hypogonadismus- ↓ TSH – centrální hypotyreóza- ↓ ACTH – centrální hypokorikalismus- ↓ prolaktinu - nevýznamné

Tumory hypofýzy- mikroadenomy (<1 cm)/ makroadenomy- afunkční/ s nadprodukcí hormonů- dle výchozích bbKO – endokrinologické

- z expanze – optický trakt - atemporální hemianopsie, defekty zorného pole, skotomy, slepota, porucha vnímání červené barvy

- hypotalamus – viz tam- sinus cavernosus – diplopie, oftalmoplegie, ptóza, poruchy citlivosti obličeje- frontální lalok – anosmie, poruchy osobnosti- bolest hlavy, hydrocefalus, demence, psychózy

Prolaktinomy a hyperprolaktinémie- prolaktinom je nejčastější nádor hypofýzy a původce hyperprolaktinémie↑ prolaktin – fyziologicky při námaze/ těhotenství/ kojení/ stres/ spánek

- farmakologicky navozená antidepresivy/ antipsychotiky/ neuroleptiky/ estrogeny/ gestageny/ blokátory H2-rec, verapamil, methyldopa, opiáty, nikotin

- patologicky u prolaktinomu/ akromegalie/ periferní hypotyreóza/ jaterní cirhózy/poškození stopky hypofýzy/ SLE/ renální insuficience

KO – poruchy menstruačního cyklu (anovulace, amenorea, oligomenorea, infertilita, galaktorea)- u mužů snížení libida, erektilní dysfce, někdy gynekomastie

Th – antagonisté dopaminu (bromocryptin, cabergolin) → normalizace až u 80 % pacientů a zmenšení nádoru i vymizení (CAVE apoplexie – intenzivní bolest hlavy, poruchy zraku)

- chirurgie, radioterapieAkromegalie a gigantismus

- z dlouhodobé expozice vysokým koncentracím GH (hypofyzární adenom)Gigantismus – před ukončením růstu (excesivní růst)Akromegalie – po ukončení růstu

- zvětšení akrálních částí těla (uši, nos, mandibula, rty), organomegalie (makroglosie…)- otoky (důsledek zmnožení vaziva a retence tekutin), hypertrofie potních žláz- akromegalická artropatie- postižení KVO (arteriální hypertenze, akcelerovaná ateroskleróza, akromegalická

kardiomyopatie)- inzulinová rezistence, zvýšené riziko kolorektálního ca, cefalgie

Dg – ↑ sérové koncentrace GH a IGF-I- orální glukózový toleranční test – u zdravého dojde k supresi GH- MR, oftalmologické vyšetření…

Th – neurochirurgie, agonisté dopaminu, superaktivní analoga somatostatinu, antagonisté GHCushingův syndrom a Cushingova choroba – viz 77.ATyreotropin produkující adenomy

- vzácné- hypertyreóza bez strumy, ↑ T4 a T3 se ↑ nebo normálním TSHTh – normalizace fce štítné žlázy + neurochirurgické odstranění

Gonadotropiny produkující adenomy- gonadotropinomy produkují FSH, LH a jejich podjednotky, často afunkční

48.A NESPECIFICKÉ STŘEVNÍ ZÁNĚTY = 17.A

82

Page 83: lf1.czlf1.cz/wp-content/uploads/228-vypracovane_otazky_interna... · Web viewprognoza – 50 % dosáhne kompletního uzdravení do 6 měsíců, 20 % má prodloužený průběh –

Vypracované otázky na INTERNU made by Edita Homolková, Monika Ambroziová, Adam Král, Petr Vermach a Vítek Křehnáč

49.A DYSLIPIDEMIE

= porucha normálního složení krevních tuků- skupina metabolických onemocnění hromadného výskytu (10kty % dospělé populace)- hyperlipoproteinémie a dyslipidémie je významný faktor pro KVO- díky ↑ syntéze a/ nebo ↓ odbourávání lipoproteinů- primární/ sekundární (hypotyreóza, nefrotický syn, DM)/ geneticky podmíněné- cílové hodnoty: CH 5 mmol/l, LDL-C 3 mmol/l, HDL-C 1 mmol/l, TG 1,7 mmol/lHypercholesterolémie - LDL

- akutní intermitentní porfyrie, mentální anorexieSmíšená nebo kombinovaná HLP – LDL + VLDL

- hypotyreóza, nefrotický syn, akromegalie, léčba diuretiky/ kortikoidyHypertriglyceridéme - VLDL

- DM, uremie, obezita, alkoholismus, léčba některými β-blokátory, dysgamaglobulinémie, lipodystrofie, glykogenóza typu I, SLE

Reziduální riziko – riziko KVO zůstávající po léčba statiny a dosažení cílové hodnoty LDL-C- roli hrají další rizika - hypertenze, DM, kouření, hladiny HLP a DLP, obezita centrální…

Geneticky podmíněné DLPFamiliární hypercholesterolémie – AD, ↓ množství LDL-rec (nejsou tvořeny/ nejsou transportovány do

membrány/ porucha vazby lipoproteinové částice/ rec nejsou internalizovány)- homozygoti 1:1000 000, heterozygoti 1:500Lab – izolované ↑ CH při normální nebo mírně ↑ TGTh – statiny, LDL-aferéza

Polyfenní hypercholesterolémie – mírná hypercholesterolémie do 8 mmol/lFamiliární kombinovaná hyperlipidémie – výskyt 1:50-100

- ↑ CH a/ nebo TG, ↑ LDL a VLDL a apolipoproteinu BKO – často asymptomatické a projeví se až KVO

- arcus semilis cornae, xantelesma palpebrarum, xantomy (kůže, šlach)- u těžkých smíšených jako komplikace akutní pankreatitida- ateroskleróza (UZ karotid a velkých tepen DK, vyšetření kotníkových tlaků)

Th – nefarmakologická – dieta – celková E, aby se udržela hmotnost, tuky celkem 25-35 % E (nasycené živočišné <7 %), sacharidy 50-60 % E, bílkoviny, vláknina, vitamíny a další mikronutrienty

- cholesterol <200 mg/den, u hypertenze NaCl <7-8 g/den- pro ↑ HDL – redukce nadváhy, rybí tuk (3 rybí jídla/ týden), sója

- fyzická aktivita – aerobní cvičení 20-30 min 4-5x týdně/ 45-60 min 2-3x týdně, 60-75 % TFmax

- ↓ váhy – o 5 %- nekouřit

- farmakologická – LDL-C je primární cíl (↓ o 1 % vede ke snížení rizika KVO o 1-2 %)- statiny – inhibitory 3 HMGCoA reduktázy, blokace syntézy cholesterolu v játrech

- ↓ o 30-40 % při běžné dávce 5 mg/ den- účinek závislý na dávce (pravidlo 6 %. 2x → 6 %, 3x → 12 %...)- NÚ: myopatie- artrovastatin, simvastatin, rosuvastatin, fluvastatin

- fibráty - ↓ TG a ↑ HDL- antagonisté PARPα rec → ovlivnění transkripce genů- účinek závisí na závažnosti dyslipidémie- fenofibrát

- niacin - ↑ HDL nejúčinněji (o 20-30 %) a ↓ TG- NÚ: flush (zrudnutí obličeje a i horní části těla) – výskyt snižuje laropiprant

- ezetimib – inhibitor vstřebávání cholesterolu ve střevě- dochází ke ↑ cholesterolu v játrech → kombinace se STATINY (↓ CH o více než

50 %)- pryskyřice – blokace vstřebávání žlučových kyselin

- hlavně u izolované hypercholesterolémie, netoxické, ženy ve středním věku

83

Page 84: lf1.czlf1.cz/wp-content/uploads/228-vypracovane_otazky_interna... · Web viewprognoza – 50 % dosáhne kompletního uzdravení do 6 měsíců, 20 % má prodloužený průběh –

Vypracované otázky na INTERNU made by Edita Homolková, Monika Ambroziová, Adam Král, Petr Vermach a Vítek Křehnáč

- ! vit A, D, E, K, zácpa

49.B ONEMOCNĚNÍ PLEURY

a) Hydrotoraxb) Exsudativní pleuritidac) Hemotorax d) Empyéme) Chylotoraxf) Pneumotorax – ot. 68.Bg) Fibrotorax h) Nádory pleury

Pleuritida- zánět společného prostoru parietální a viscerální pleury infekčního/ neinfekčního původu- fyziologická resorpční kapacita 500-700 ml/ den- infekční - má exudativní, fibrinopurulentní a organizující fázi

- spolu s pneumonií, po traumatu a operacích- neinfekční – kolagenózy (SLE, RA, Sjögrenův syn.,) a vaskulitidyKO – bolest na hrudi (iritace parietální pleury – ostré píchání/ křeče při kašli), tachypnoe, kašel, dušnost (menší

dechové exkurze pro bolest a komprese plicního parenchymu), dráždivý a neproduktivní kašel- při rozvoji empyémů bývá opětovný nárůst teploty

Dg – fyzikálně – oslabené sklípkové dýchání, přitlumený až temný poklep, fremitus pectoralis (palpuje se chvění hrudní stěny přenášené z fonace pacienta. Vyšetření se provádí dlaněmi/ ulnární hranou ruky, které se symetricky přikládají na obě poloviny hrudníku a intenzita chvění se srovnává. F. p. zesiluje nad kondenzovanou tkání, zeslabuje nad výpotkem, srůsty, emfyzémem apod.)

- sedimentace, CRP, prokalcitonin- USg, zadopření a bočný rtg hrudníku, CT- punkce – biochem, cytologie, mikrobiologie

Th – drobné výpotky – systémová atb- kompikované parapneumonické – drenáž, opakované průplachy fyziologickým/ desinfekčním roztokem- empyém – drenáž + výplachy + streptokináza/ urokináza

Exsudativní pleuritida - hlavní příčinou exsudace je nespecifický zánět plic (para- a metapneumonický výpotek), TBC, plicní i mimoplicní maligní tumory (ca plic, prsu), PE, procesy v břiše (pankreatitida, absces), systémová onemocnění

Th - léčba základního onemocnění, odstranění co největšího množství tekutiny (zamezení vzniku fibrotoraxu)

Empyém - nahromadění hnisu v pleurální dutině při hnisavých procesech plic, mediastina a břišní dutiny- klinicky jde většinou o septický stav s bolestí na hrudi (nemocný je schvácený, febrilní, RTG –

homogenní zastření na hrudní stěně, punktát je smetanově hustý, páchnoucí s vysokým obsahem neutrofilů)

Th - ATB dle citlivosti, často je nutná drenáž s proplachem FR, instilací ATB nebo streptokinázyFibrotorax - lokální nebo difuzní ztluštění pleury jako následek zánětu či traumatu

- srůsty vedou k omezení plicních funkcí – restriktivní ventilační porucha- příčinou byla dříve TBC, nyní jsou to hlavně záněty plic a pleury nebo azbestózaTh - dekortikace – odstranění ztluštělé pleury chirurgicky

Pleurální výpotek = Hydrotorax- akumulace tekutiny v pleurální dutině (>10 ml u 70 kg člověka)- transudáty hl. u systémových onem. a exudáty u pleurálních procesůtransudát – chudý na bílkoviny, při ↑ hydrostatického intravaskulárního tlaku/ ↓ intravaskulárního onkotického

tlaku (levostranné srdeční selhání, plicní embolie, hypalbuminemie při onemocnění jater či ledvin, konstriktivní perikarditidy nebo mitrální stenózy)

- specifická hmotnost < 1,015, celkovou bílkovinou <30 g/l, negativní Rivaltova zkouška, Lightova kritéria

- Ravaltova zkouška – provede se pomocí kys. octové (pozitivní u exsudátu)84

Page 85: lf1.czlf1.cz/wp-content/uploads/228-vypracovane_otazky_interna... · Web viewprognoza – 50 % dosáhne kompletního uzdravení do 6 měsíců, 20 % má prodloužený průběh –

Vypracované otázky na INTERNU made by Edita Homolková, Monika Ambroziová, Adam Král, Petr Vermach a Vítek Křehnáč

- Lightova kritéria: Je-li poměr bílkovina v punktátu/ bílkovina v séru <0.5 nebo poměr LDH v punktátu/ LDH v séru <0.6 nebo koncentrace LDH v punktátu <2/3 normální koncentrace LDH v séru

Th - léčíme základní onemocnění, opakované punkce nejsou indikoványKO – asymptomatické u menšího rozsahu

- dušnost, pleurální bolest, suchý kašel, komprese mediastina při extrémě velkých výpotcích (↓ plnění komor, útlak žil…)

Dg – fyzikálně výpotky >300-400 ml, oslabené až neslišitelné sklípkové dýchání, přitlumený až temný poklep- rtg hrudníku, SONO, punkce, CT, MRI

Th – řešení příčiny, punkce/ tunelizované pleurální katery (ven z těla/ do peritoneální dutiny)

Hemotorax - přítomnost krve v pleurální dutině nejčastěji jako důsledek úrazu (poranění hrudní stěny a zlomenina žeber)

dalšími příčinami jsou maligní nádory, hemofilie či antikoagulační léčbaTh - traumatický hemotorax se léčí hrudní drenáží či chirurgickou intervencí, zbytek podle vyvolávající příčiny

Chylotorax- přítomnost chylu v pleurální dutině, punkcí získáme opaleskující tekutinu mléčného vzhledu bohatou na lipidy Etio - poškození ductus thoracicus nádorem či traumatem, u žen lymfangioleiomyomatóza- chylotorax navozuje malnutrici a imunodeficit- při traumatu indikujeme ligaci ductus thoracicus, u nádoru radioterapie + parenterální výživa

Nádory pleury - nejčastější jsou sekundární nádory (metastatické postižení pleury), primární nádor – mezoteliom je vzácnýPrimární nádory benigní – fibrom, lipom a lokalizovaný mezoteliom

- okrouhlý/oválný, někdy pendulující tumor - většinou je diagnostikován jako náhodný nález na skiagramu hrudníku- můžeme pak provést diagnostickou punkci nebo pacienta rovnou indikovat k resekci

Primární nádory maligní – maligní difuzní mezoteliom - vzácný maligní nádor vyvolaný nejčastěji azbestem- latence mezi expozicí a vznikem nádoru je cca 30 letKO - dušnost, kašel, nechutenství, hubnutíDg - RTG – homogenní zastření (nádorová masa + tekutina), histologické

vyšetření (odběr biopsie při torakoskopii)Th - chemoterapie a aktinoterapie (většinou neúspěšná), ojediněle

chirurgická resekceMetastatické nádory – jsou poměrně časté a jsou doprovázeny tekutinou v pleurální dutině (někdy

hemoragickou)- metastázy na pleuře tvoří: ca plic, mammy, žaludku, ledvin, pankreatu a ovariaKO - pleurální bolest, kašel, progredující dušnost, nechutenství

- fyzikální nález – poklepové zkrácení, oslabené dýchání a bronchofonieDg - nález maligních elementů v punktátu, histologické vyšetření biopsie pleuryTh - hrudní punkce, drenáž (ovlivnění dušnosti), cytostatika (intrapleurálně bleomycin),

při rychlém doplnění tekutiny lze provést pleurodézu

50.A DIABETES MELLITUS II. TYPU

= chronické ↑ glykémie nad normální hodnoty při inzulinové rezistenci a relativ/ absolutně ↓ sekreci inzulinu- 92 % diabetiků (celkem 8 % populace)- často sdružen s viscerální obezitou, arteriální hypertenzí, dyslipidémií, hyperurikémií a lehkou elevací

aminotransferáz v důsledku jaterní steatózy- hl. kombinace genetiky, obezity a nedostatku pohybové aktivityRizikové skupiny – DM v rodinné anamnéze, gestační diabetes/ porod dítěte >4 kg, obezita, hypertenze,

hyperlipoproteinémie, porušená glc tolerance, věk >40KO – klasické příznaky většinou nejsou vyjádřeny, spíše únava, mírně zvýšený pocit žízně a močení

- náchylnost k infekcím (urogenitální, kožní)

85

Page 86: lf1.czlf1.cz/wp-content/uploads/228-vypracovane_otazky_interna... · Web viewprognoza – 50 % dosáhne kompletního uzdravení do 6 měsíců, 20 % má prodloužený průběh –

Vypracované otázky na INTERNU made by Edita Homolková, Monika Ambroziová, Adam Král, Petr Vermach a Vítek Křehnáč

- hyperosmolární hyperglykemické kóma při gly >30-40 mmol/l- mikrovaskulární a makrovaskulární komplikace

Dg – náhodná gly - >11 mmol/l (opakovaně) + klinika- gly nalačno - poslední jídlo před 8 hod, >7 mmol/l

- hypo <3,8, normo 3,8-5,6, IGT (impared glucose tolerance) 5,6-6,9, diabetes >7- orální glukózový toleranční test (oGTT) – 75 g glc v 200 ml vody

- gly za 2 hod >11 mmol/l- glykovaný Hb - >4,75 %, ukazatel za poslední 2 měsíce

Sledování komplikací – oftalmologie – diabetická retinopatie, katarakty- nefrologie – ionty, urea, kreatinin, mikroalbuminurie

- při závažnější nefropatii (kreatinin >200 μmol/l) ke specialistovi-vyšetření DK – neurolog dle potřeby- TK – při ambulantní kontrole/ holter- lipidy – pravidelně 1/ rok- EKG

Th – viz 97.B

50.B MYOKARDITIDY

- zánět myokardu, intersticia a cév vyvolané (generalizovanou) infekcí virovou/ bakteriálními toxinyEtio - viry (50%) - coxackie B1-5 (velice nebezpečné), coxackie A a B, herpesvlry, influenza virus, HIV

- bakterie – difterická (poškození toxinem), Borrelie burgdorferi, β-hemolytické streptokoky, při bakteriální endokarditidě (stafylokoky)

- mykózy- protozoa - Trypanozoma cruzi (Chagasova nemoc)- neinfekční - revmatická horečka, kolagenózy, vaskulitídy, postradiační, idiopatická Fiedlerova myokarditida

KO - většinou asymtomatický nebo mírný průběh, vzácně fulminantní, při chronické formě je možný přechod v dilatační kardiomyopatii

- únava, slabost, palpitace (extrasystoly, komorové tachyarytmie, AV blokáda), tachykardie, až srd. insuf. s oběhovým selháním a šokem

- myalgie, artralgie, subfebrilieDg - A, SP

- EKG - sinusová tachykardie, často extrasystoly, AV blokády, prodloužení PQ u revmatické horečky, známky poškozeni vnitřní vrstvy myokardu (deprese ST, oploštělé, až negativní T)

- ECHO - někdy při perimyokarditidě períkardiální výpotek, poruchy kinetiky, dilatace- RTG hrudníku - známky srd. insuf. - dilatace stínu a známky plicní venostázy- scintigrafie se značenými antimyozinovými protilátkami (pozitivní i při IM a někdy i dilatační kardiomyopatii),

katetrizace s endomyokardíální biopsií- Lab - nespecifické známky zánětu, někdy ↑ kardiomarkery (CK, CK-MB, troponin T a I), virologické +

bakteriologické vyšetření- MR - nejlepší

Komplikace - srd. insuf., arytmie, dilatační kardiomyopatie (pozdní komplikace virové myokarditidy)Th - kauzální - léčba revmatické horečky, boreliózy nebo difterie, při Chagasově nemoci nifurtimox, antivirová

terapie proti některým virům- symptomatická – klid!!!, profylaxe tromboembolie, léčba komplikací, dočasná mechanická náhrada srdce- širokospektré ATB (amoxycilin, proti sekundární bakteriální infekci) + nesteroidní antirevm. (kys.acetylsalyc.

0,5-1g/ 3denně) + kortikoidy (prednison 40-60 mg denně)

51.A TROMBOFILNÍ STAVY

= porucha hemostázy se zvýšenou tendencí ke vzniku trombózy- defekty inhibitorů koagulace, patologie koagul. faktorů a nedostatečnost fibrinolytického systému1. Hereditární trombofílie

- Leidenská mutace = rezistence k aktivovanému proteinu C (bodový molekul. defekt f. V) v místě proteolytického působení aktivovaného proteinu C (50% všech vrozených trombofílií)

86

Page 87: lf1.czlf1.cz/wp-content/uploads/228-vypracovane_otazky_interna... · Web viewprognoza – 50 % dosáhne kompletního uzdravení do 6 měsíců, 20 % má prodloužený průběh –

Vypracované otázky na INTERNU made by Edita Homolková, Monika Ambroziová, Adam Král, Petr Vermach a Vítek Křehnáč

- 3 % populace- Defekty proteinu C, S – AD, homozygotní formy → neslučitelné se životem, heterozygoti - dospělí

- deficit plazminogenu či zvýšená aktivita PAI-1- defekt f. XII.- léčba heparinem

- Defekty antitrombinu – riziko při poklesu pod 50 %- snížení inhibice f Xa a IIa, protože snižují jeho vazbu s heparinem/ aktivaci heparinem/

proteázovou aktivitu- Disfibrinogenémie – mutace → snadnější přechod v polymerní fibrin- Porucha fibrinolýzy – nedostatečné uvolnění tPA/ rezistence plazminogenu vůči aktivaci tPA/ ↑ PAI v plasmě- Hyperhomocysteinemie – >18,5 mol/l, při blokaci metabolismu metioninu i homocysteinu

- zřídka žilní trombóza, riziko u preeklampsie a v šestinedělí2. Získané trombofilní stavy

- poškození endotelu, zpomalení průtoku, turbulentní proudění, trombocytémie, hyperviskózní syndrom u polyglobulie, paraproteinémie, leukémie, těhotenství

- zánětlivá reakce – infekt, trauma, pooperačně (nejvíc 10.-14. den)- IL-1, TNF-α, IFN-γ, IL-6 → aktivace endotelu, ↑ proteinů akutní fáze (CRP, fibrinogen, FVII,

FVIII, PAI)- maligní proces – produkce TNF-α, hl. diseminované adenokarcinomy- antifosfolipidový syn. – negativně nabité fosfolipidy na většině bb., SLE, systémové choroby pojiva, maligní

lymfomy- 60 % žilní trombózy, 30 % CMP, 10 % jiné, opakované aborty

- DIC, AFP, p.o. kontraceptiva, ...Th - prevence vzniku manifestní trombózy

- nízkomolekulární heparin, warfarin (INR 2,0-3,0), bandáže DK

51.B ONEMOCNĚNÍ PERIKARDU, SRDEČNÍ TAMPONÁDA

Akutní perikarditidaEtio - infekční – coxsackie, influenza, CMV, HIV, kandida, stafylokoky, streptokoky, tbc

- imunologická - revmatická horečka, SLE, Dresslerův sy (do 1-6 týdnů po IM nebo chirurgickém zákroku, může být horečka, zvýšené zánětlivé markery), sklerodermie

- při urémii, po úraze, nádorová, postradiační, hypotyreózaKO – bakteriální - sepse

- suchá pericarditida (sicca, nejč. při IM/ urémii) - píchavá bolest za sternem zesilující se vleže, při hlubokém nádechu a při kašli, třecí šelest (nejlépe slyšitelný při dolním sternu při výdechu a v předklonu, charakter jako když třeme vlasy o sebe)

- exsudativní pericarditida (exsudativa, nejč. pň vir./ rev. h./ tbc) - při přechodu ze suché do exsudativní formy mohou být oslabené srdeční ozvy a vymizet třecí šelest

Kompl. - srdeční tamponáda (při rychlém vytvoření výpotku)Dg - A, SP,

- EKG - při postižení vnější vrstvy myokardu elevace ST úseku (je ale narozdíl od IM prox. konkávní a není patologický kmit Q)

- ECHO - velmi dobrá metoda,v případě bez exsudátu však nic neprokáže, ev. punkce perikardu s biopsiíTh - kauzální - léčba infekce (dle kultivace), revmatické horečky, při alergické perikarditídě kortikoidy, při

urémii dialýza- symptomatická – analgetická NSA (ibuprofen 3x 400 mg) pokud do 48 h nenastane účinek → kortikoidy,

při idiopatické perikarditídě ter. pokus s kolchicinem, odlehčující punkceChronická perikarditida

- pokud trvají příznaky více jak 3 měsíce- jizevnaté změny po proběhlé akutní perikarditídě, částečně srostlý a kalcifikovaný svraštělý perikard omezuje

diastolické plnění komor a způsobuje tak městnáni (konstriktivní perikarditida)Etio – často u autoimunních chorob (SLE)KO - pravostranné městnání - ↑ CVP, Kussmaulův příznak (paradoxní vzestup tlaku v krčních žilách pň hlubokém

nádechu), hepatomegalie (někdy s ascitem), edémy

87

Page 88: lf1.czlf1.cz/wp-content/uploads/228-vypracovane_otazky_interna... · Web viewprognoza – 50 % dosáhne kompletního uzdravení do 6 měsíců, 20 % má prodloužený průběh –

Vypracované otázky na INTERNU made by Edita Homolková, Monika Ambroziová, Adam Král, Petr Vermach a Vítek Křehnáč

- nízký srdeční výdej - slabost, námahová dusnost, někdy pulsus paradoxus (pokles TK při nádechu o více než 10 mm Hg)

Dg -A, SP (poslech- časné diastolické klapnutí když rychle rozvíjející se komory narazí na tuhý perikard), EKG (nízká voltáž, negativní T, někdy fibrilace síní), ECHO, rtg hrudníku (skořápkovité ohraničení srdce kalcifikovaným perikardem)

Th - dekortikace srdce, perikardektomie, operace se musí provést včas, jinak dojde k atrofii srdce a po operaci k akutní dilataci

Perikardiální výpotekEtio - transsudace (městnavé srdeční selhání, urémie)

- exsudace (serózní u virové, purulentní u bakteriální)- krvácení - hypotyreózy

KO - obrazem perikarditídy nebo tamponády srdeční, při rozsáhlejším výpotku rozšířené poklepové ztemnění a srdeční ozvy se stávají tichými

Dg - ECHO – suverénní a nejužitečnější metoda, dle velikosti anechogení zóny (výpotek) určíme i přibližné množství výpotku, také pomáhá při perikardiocentéze

- RTG – registruje výpotek od cca 400-500 ml, rozšíření srdečního stínu bez známek plicní venostázy ukazuje na výpotek

- EKG – difuzní snížení voltáže kmitůSrdeční tamponáda

- nahromadění tekutiny v perikardu → ↑ perikardiální tlakKO - sinusová tachykardie (kompenzace sníženého tepového objemu)

- ↑ náplň jugulárních žil s pulzací zvýrazněnou v inspiriu (Kussmaulovo znamení)- kolísání systémového tlaku v závislosti na respiraci (pulsus paradoxus)- hypotenze

Dg - ECHO – diastolický kolaps obou síní, diastolický kolaps pravé komory, zvětšení pravé komory v inspiriu a současné zmenšení komory levé → pulsus paradoxus

Th – evakuace- perikardiocentéza pod echo kontrolou- chirurgicky (chirurgická fenestrace perikardu, perikardektomie)

52.A METABOLICKÉ OSTEOPATIE. PAGETOVA CHOROBA

Osteoporóza- úbytek kostní hmoty a změna její kvality vedoucí ke ↑ lomivosti- 6-7 % populace ČR, postmenopauzální ženy, staří muži- většina zlomenin žen po 55. roce a mužů po 65. letech- kompresní zlomeniny obratlů, fr distálního předloktí, fr proximálního femuruEtio – převaha osteoresorpce nad novotvorbou (max kostní hmoty v 25 letch, úbytek 0,3-0,5 %/ rok do 70 let)

- primární, involuční – vystupňované změny typické pro fyziologické stárnutí- senilní – po 70. roce, ženy:muži 2:1, nedostatek vit D a Ca2+, ztráta trabekulární i kortikální kosti- postmenopauzální – nedostatek estrogenů, ztráta hl. trabekulární kosti- idiopatická – juvenilní osteoporóza, idiopatická osteoporóza mladých dospělých

- sekundární – Cushingův syn., hypogonadismus, tyreotoxikóza, primární hyperparatyreóza, DM I, malabsorpční syn, chronické jaterní choroby, nespecifické střevní záněty, chron. renální insuficience, mnohočetný myelom, malignity, dlouhodobá imobilizace, chron. zánětlivá onemocnění (hl. revmatologická), glukokortikoidy, supresiní th hormony štítné žlázy, antikoagulační léčba

KO – asymptomatické, náhodný nález při rtg/ vyšetření rizikových osob, zlomeniny při nepřiměřeném násilíDg - denzitometrie –metoda DAX (dual-energy X-ray absorptiometry), posouzení kostního minerálu, g/cm2

- T-skóre – vztaženo k průměru mladých osob- hodnotí se proximální femur a bederní páteř- norma BMD >-1 T-skóre, -1 - -2,5 osteopenie, >-2,5 osteoporóza

- trg páteře – kompresivní fraktury, změny patrné až při úbytku 30 % kostní hmoty- anamnéza – zlomeniny, bolesti zad

88

Page 89: lf1.czlf1.cz/wp-content/uploads/228-vypracovane_otazky_interna... · Web viewprognoza – 50 % dosáhne kompletního uzdravení do 6 měsíců, 20 % má prodloužený průběh –

Vypracované otázky na INTERNU made by Edita Homolková, Monika Ambroziová, Adam Král, Petr Vermach a Vítek Křehnáč

- fyzikální vyšetření – hrudní kyfóza, tělesná výška, svalová síla, koordinace a stabilita stoje- lab – úroveň remodelace

- Ca a fosfáty v normě- novotvorba kosti: kostní alkalická fosfatáza, N-propeptid kolagenu I (PINP)- kostní resorpce: kostní kyselá fosfatáza, C-terminálni telopeptid kolagenu I (CTX-I)- urea, keratinin, ALP (pokud je 1,2x vyšší než horní hranice normy, není to primární osteoporóza), GMT,

TSH, glykemie, bílkoviny (paraprotein), sedimentaceTh – suplementace vit D a Ca2+ - 1 g Ca/den, 800 UI vit. D/den

- antiresorpční léky u lidí s vysokým kostním obratem nedosahující ještě kritické hranice pro vysoké riziko vzniku zlomenin (s resorpcí se zastavuje i novotvorba), protože mohou ↓ riziko zlomenin

- osteoanabolické léky při primárně snížené novotvorbě (např. steroidní osteoporóza) a u osteoporózy s ↑ nemodelační aktivitou

- selektivní modulátory estrogenních receptorů (SERM) – antagonisté v reprodukčním sys., v kosti a játrech jako agonista rec

- raloxifen - tlumí osteoresorpci, ↓ riziko ca prsu, ↑ riziko trombembolie- hl. postmenopauzální ženy, léčba na 4-5 let

- bisfosfonáty – syntetická analoga pyrofosfátu tlumící resorci (↑ afinita ke kosti)- u ↑ resorpce (postmenopauzální, senilní)- nutný dostatečný příjem vit. D a Ca (jinak osteomalacie)- NÚ: horní dyspepsie, KI: hypokalcemmie, hypovitaminóza D, osteomalacie

- stroncium ranelát – organická sůl Sr, stimuluje novotvorbu a tlumí resorpci, při postmenopauzální osteoporóze

- kalcitonin –inhibice osteoklastů, přímé analgetické účinky (lososí kalcitonin jako nosní sprej)- postmenopauzální ost. s fr obratlů a bolestmi, při KI bisfosfonáty a raloxifenu

- hormonální substituční léčba (HRT) – tlumí resorpci, ↑ riziko ca prsu a trombembolie- androgeny – u mužů s hypogonadizmem před 50. rokem, ↑ riziko ca prostaty- parathormon – osteoanabolický při intermitentním podávání, léčba na 18 -24 měsícůPostmenopauzální osteoporóza – u ¼ žen po 50. roce

- ↓ estrogenů → ↑ resorpce, toto období trvá 8-10 let, ztráta hlavně trabekulární kosti- asymptomatická (až pak fraktura)

Senilní osteoporóza – rozvoj po 70. roce, více postiženy ženy- nedostatečný příjem vit. D a Ca, vede k sekundární hyperparatyreóze, ztráta trabekulární i

kortikální kostiSteroidní osteoporóza – nejčastější sekundární, prevalence fraktury u dlouhodobé léčby je 30-50 %, výrazné

riziko osteoporózy od 5 mg/ den- přímá inhibice osteoblastů, úbytek hlavně trabekulární kosti

Osteomalacie- ↓ mineralizace novotvořeného osteoidu, kosti měknou, jsou náchylné k deformitám a frakturám- časté ve stáří, u některých onemocnění GIT a renální insuficience Rachitis – v dětsví, poruchy zrání růstových ploténekEtio - ↓ vit. D – málo v dietě, málo slunečního záření, malabsorpce, porucha sytézy kalcitriolu v ledvinách

(renální insuficience, vit. D-dependentní rachitis typu I), porucha syntézy 25-hydroxykalciferolu v játrech, mutace genu pro rec vit. D

- hypofosfatémie - ↑ renální ztráty (genetická porucha tubulů ledvin, orogenní osteomalacie), ↓ vztřebávání fosfátů

- acidóza – renální insuficience, renální tubulární acidóza- léky – bisfosfonáty, fluoridy

KO – deformace a bolesti skeletu (difúzní, neurčité, citlivost kostí na poklep), svalová slabost (díky hypokalcemii)- kolébavá kachní chůze

Dg - hypokalcémie, hypofosfatémie (obvykle mírná), ↑ alkalická fosfatáza a parathormon, hypokalciurie (<1 mmol/24 h)

- rtg – Looserovy zóna přestavby (pseudofraktury, projasnění kortikalis kolmo k dlouhé ose kosti), deformity dlouhých kostí

- BMD většinou pod -4 T-skóreTh – substituce vit. D spolu s Ca – v případě poruchy hydroxylace vitamínu, podáváme aktivní formu kalcitriol

89

Page 90: lf1.czlf1.cz/wp-content/uploads/228-vypracovane_otazky_interna... · Web viewprognoza – 50 % dosáhne kompletního uzdravení do 6 měsíců, 20 % má prodloužený průběh –

Vypracované otázky na INTERNU made by Edita Homolková, Monika Ambroziová, Adam Král, Petr Vermach a Vítek Křehnáč

- fosfáty – při hypofosfatemii (magistraliter roztok 13,6 % natrii dihydrogenphosphorici)Pagerova choroba

- lokalizovaná multifokální remodelace vedoucí ke zduření a deformacím kostí- nejčastější metabolická osteopatie, prevalence 1-8 %Etio – nejasné, genetika + perzistující virová infekce

- vystupňovaná osteokalstická resorpce s druhotným zvýšením kostní novotvorby (neplnohodnotná – místo lamelární je „plsťovitá“)

KO – moostotická/ polyostotická forma, často páteř, femur, lebky, pánev, strnum- klinické problémy jen 10 %- bolesti, zduření (může vést k útlaku nervu), lokální ↑ teploty- u 1-2 % vziká osteosarkom

Dg – rtg – fokusy osteoresorpce, kostní zduření s chaotickou strukturou kosti- lab - ↑ ALP (často velké) při normě GMT, PTH, fosfáty a Ca obvykle v normě

Th – bisfosfonáty Osteitis fibrosa cystica – při primární hyperparatyreóze (10 %)

- ložiska subperiostální resorpce až kostní cystyOsteogenesis imperfecta – skupina dědičných chorob, poruchy kolagenu I

- deformity skeletu, fraktury, porucha dentinu, ochablost vazů, hluchotaOsteopetróza – vrozená ch., ↓ osteoresorpce díky poruše osteoklastů, zvýšená denzita a křehkost, útlak kostní

dřeně a nervůFibrózní dysplazie – ložiskové postižení skeletu, benigní expandující kostní léze

- bolestivé zduření, deformity, fraktury

52.B BRADYARYTMIE A RAMÉNKOVÉ BLOKÁDY

- <60/ min- na podkladě poruchy automacie SA nebo blokády vedení vzruchu z SA nebo AV uzlu na komory- pokud je sinusový rytmus a na EKG nenajdeme vlny P, pak je porucha v SA uzlu, pokud vlny P jsou, jedná se o AV

blokáduSinusová bradykardie

- nejčastější, fyziologicky se může vyskytovat ve spánku (vagová aktivita), atleti často pod 40/ min- při léčbě β-blokátory, verapamilem, digitalisem , amiodaronem (dávku snižujeme podkun klesá pod 50/ min

nebo při manifestaci syn. ↓ minutového srdečního výdeje)Etio – sick sinus syndrom, hypotyreóza, hypotermie, nitrolební hypertenze, akutní IM spodní stěny

- při 35/ min zvážit SA blokádu 2. Stupně 2:1Sick sinus syndrom (SSS)

- syndrom na podkladě dysfce SA uzlu, výskyt roste s věkem (idiopatická degenerace SA), u ICHS a kardiomopatiíKO – sinusová bradykardie (jinak nevysvětlitelná), sinusové pauzy, sinoatriální blokády

- tachykardicko-bradykardická forma – sdružen se supraventrikulární tachykardií (nejčastěji fibrilace síní)- dlouhé pauzy po spontánní/ léčebné kardioverzi fibrilace síní, selhává automacie v

SA i ostatních centrechSinusová zástava (sinus arrest) – díky výpadku automacie SA (výpadek P vln), při dlouhé pauze se pak může

vyskytnout junkční/ ivdioventrikulání stah- sinoatriální blokáda 2. stupně (intermitentní blokáda převodu z SA na síň) má na EKG pauzu a

výpadek P vlny a pokud je pauza delší než 2násobek bazálních PP intervalů, klasifikuje se jako sinusová zástava

- SA blokádu 3. stupně (kompletní) nelze odlišit- léčba pouze při symptomatické formě (bývá u spodních infarktů, často s hypotenzí), atropin

Th - při bradykardii/asystolii z důvodu SSS implantace kardiostimulátoru (nejčastější indikace)Respirační sinusová arytmie

- periodické urychlení SA frekvence při nádechu a zpomalení při výdechu- fyziologické (reflexní změny vagu), hlavně v dětském věku

Atrioventrikulární blokády- porucha převodu ze síní na komory- blokáda nejčastěji v AV uzlu (suprahisálně), ale i intrahisálně a infrahisálně (čím dál, tím větší riziko)

90

Page 91: lf1.czlf1.cz/wp-content/uploads/228-vypracovane_otazky_interna... · Web viewprognoza – 50 % dosáhne kompletního uzdravení do 6 měsíců, 20 % má prodloužený průběh –

Vypracované otázky na INTERNU made by Edita Homolková, Monika Ambroziová, Adam Král, Petr Vermach a Vítek Křehnáč

Kongenitální AV blok 3. stupně – náhradní junkční rytmus nad 40/ min, bez potíží, není indikace pro kardiostimulátor

- po IM (hlavně spodní stěny, uzávěr pravé koronárky), virové myokarditidy, borelióza, Chagasova choroba, idiopatická degenerace převodního systému (46 %), ICHS, kardiomyopatie, kardiochirurgická operace

1. stupeň – prodloužené vedení na komory, PR interval > 200 ms- neléčí se

2. stupeň – intermitentní výpadky vedení, na EKG pauza s nepřevedenou vlnou P (nenásleduje QRS)- 1. typ, Wenckebach – výpadku předchází postupné prodlužování PR intervalu

- možné fyziologicky u mladých a atletů, asymptomatické není indikováno pro kardiostimulátor

- někdy jako NÚ bradykardizující/ antiarytmické léčby- 2. typ, Mobitz – nedochází k prodlužování PR a je pauza 2násobná než bazální RR interval

- vziká při poruše infrahisální (často s blokádou nějakého Tawarova raménka, či porucha nitrokomorového vedení s rozšířením QRS)/ intrahisální

-3. stupeň – kompletní přerušení vedení, náhradní rytmus nejčastěji z distální části AV uzlu/ komor (junkční, idioventrikulární)

- na EKG nezávislá aktivita síní a komor, junkční (úzké QRS) a idioventrikulární rytmus (široké QRS) je pravidelný o frekvenci <50/ min

Th – odstranění potenciálních příčin (vysadit antiarytmika a dioxin)- atropin pouze u symptomatických (0,5-1 mg i.v.) k potlačení vagové aktivity- při současné léčbě antiarytmiky nebo digoxinem se tyto léky vysadí, při AIM s AV blokádou (dušnost,

hypotenze, psychická alterace) atropin nebo dočasná kardiostimulace, při chronické AV blokádě s příznaky srdečního selháváni implantace kardiostimulátoru (dovolí podávat betablokátory i digoxin)

53.A KRVÁCIVÉ STAVY Z PLAZMATICKÝCH PŘÍČIN

(APTT) Vnitřní systém Zevní systém ( Quick, TT)kontakt

XII. XIIa

XI. XIa tkáňový faktor ( tromboplastin)

IX. IXa VIIa VII.

X. Xa X.

protrombin trombin

fibrinogen fibrin monomer + fibrinopeptid A,B

fibrinový polymerXIII. XIIIa

stabilní fibrinová sraženinaQuick, TT test - zevní koagulační systém (VII., Ca 2+, aktivovaný tkáňový tromboplastin) i společný ( VII., V., X., I.)

- norma 12- 15 s; INR = international normalized ratio TT sled. plazma/TT kontrolní vzorek 0,9-1,28- prodloužen u avit. K., fyziologicky u novorozenců (málo f. VII)...

APTT = aktivovaný parciální tromboplastinový čas - vnitřní koag. systém (XII., XI., IX., VIII.) a poté i společný (X., V., II., I.)

- norma 28-40 s; prodloužen u hemofílie A (↓ f. VIII) a B (↓ f. IX.)...

1. Vrozené - vzácné, závažné- drobná kožní poranění se staví normálně, neúměrné hematomy, spontánní krvácení do kloubů a měkkých tkání- Hemofílie A, B, C, a deficit ostatních f., von Willebrandova chorobaHemofílie A - XR dědičnost, výskyt 1:15-20 000, ↓ plazmat. konc. f.VIII (těžká forma 1-5 j/dl, lehká 5- 30j/ dl)

KO - na nevelké podněty → hematomy (podkožní, svalové, šíří se), opakovaná spontánní krvácení do velkých kloubů → pocit napětí, bolest (kloub je oteklý, citlivý, napjatá ale neprokrvácená kůže) → hemofilické artropatie (deformity kloubů až ankylóza), krvácení do retroperitonea → iritace m. psoas → semiflekční postavení dané DK - imituje NPB!

91

Page 92: lf1.czlf1.cz/wp-content/uploads/228-vypracovane_otazky_interna... · Web viewprognoza – 50 % dosáhne kompletního uzdravení do 6 měsíců, 20 % má prodloužený průběh –

Vypracované otázky na INTERNU made by Edita Homolková, Monika Ambroziová, Adam Král, Petr Vermach a Vítek Křehnáč

Dg - prodloužení APTT, stanovení konc. f. VIII, RTG kloubů; prenatální dg. analýzou DNA (choriové klky 10.-12. týden či pupečníková krev 18. týden)

Kompl - krvácení do šlach. pouzder a periostu → pseudotumory, útlak okolí; u léčených koncentráty f.VIII → infekce VHC, VHB, HIV (dříve)

Th - substituce f. VIII (koncentráty v lyofilizované formě Octanate, Immunate, Fanhdi, rekombinantní Refacto)

- lehké formy - desmopressin (analog vasopresinu, zvyšuje konc. endogenního f. VIII a vWF vyplavením ze zásobních granul) - infúze či intranasálně; analgetika (paracetamol, tramadol, tilidin, pentazocin)

- klid + studené obklady + šetrná rehabilitaceHemofílie B - XR; ↓ f. IX., 5-10 x méně častá

- Th - f. IX ve formě koncentrátů (Octanine F, Immunine)Hemofílie C - (↓f. XI); nedostatek fibrinogenu a koag. f. II., V., VII., X, XIII - velmi vzácnévon Willebrandova choroba - AD; nejčastější vrozená koagulopatie 1:10000, deficit vWF (adheze trombocytů

k subendoteliu, nosič pro f. VIII)- typ 1 – parciální deficit vWF, typ 2 - funkčně insuficientní vWF, typ 3 – kompletní deficitKO - krvácivé projevy kolísají od banálních až po středně těžký stupeň, hl. do sliznic,

menoragie, epistaxe, po extrakci zubu, do kloubů velice vzácněDg - prodloužená doba krvácivosti a APTT; snížená agregace trombocytů po podání

ristocetinu (antibiotikum využívané k laboratornímu vyšetření hemostázy, trombocyty v přítomnosti vWF po přidání r. agregují)

Th – desmopresin – i.v./ s.c./ intranasálně, účinek 6 h, po opakování klesá, individuální účinek- neefektivní u typu 2

- substituce – koncentráty z plazmy, rekombinantní f VIIa- u typu 3

- antifibrinolytika – alternativní, u menších krvácení- ne u krvácení do močového traktu (ucpání močovodů koagulem)

2. Získané- jako součást komplexní poruchy hemostázyJaterní poruchy - defekty f. II., V., VII., IX., X. (kromě f. V se tvoří v játrech)

- jaterní onem. (vázne tvorba f.) → Th: K-plazma, koncentráty protrombinového komplexu, čerstvě zmražená plazma, antifibrinolytika + glukokortikoidy

Nedostatek vit. K - obstrukční ikterus, malabsorpční sy pankreatického či střevního původu, kumariny - Warfarin → Th Kanavit

Cirkulující antikoagulancia - činitele s antikoagul. účinkem, které se objevují za patologických okolností v krvi- protilátky namířené proti koag. faktorům nebo proti fosfolipoproteinům- autiprotilátky jako izolovaný projev autoimunity a nádorových onemocnění

(idiopatické)/ u hemofilie A/ v poporodním období/ bez primárního postižení- nejčastěji inhibitory FVIIII, velmi vzácně FIX (možné i ostatní)

Inhibitor FVIII/ IX u hemofilie A – imunitní reakce na terapii FVIII/ IX v krevních derivátech (aloprotilátka, IgG)- u 5-10 % pacientůKO – zkrácení poločasu účinnosti terapie, prodloužení aPTTTh – malé krvácení – komprese, ledování

- velké krvácení – substituce lidským rekombinantním případně prasečím FVIII/ IX, DDAVP, cyklofosfamid

- navození imunitní toleranceZískané inhibitory FVIII – u pacientů bez hemofilie A, je to autoprotilátka, může dojít ke spontánnímu vymizení

KO – kožní/ slizniční/ intramuskulární krvácení (GIT, urogenital, CNS), ne do kloubů- pordloužení aPTT nekorigovatelné normální plasmou, ↓ FVIII

Th – jako u předchozího, kortikoidy, cyklofosfamid… plasmaferezaDIC – ot. 35 BSyndrom primární hyperfibrinolýzy – u ca tlustého střeva, ca prostaty, kdy může dojít k vyplavování činitelů

přímo aktivující fibrinolytický sys. → plazmin odbourá fibrinogen a FV a FVIII- negativita testů průkazu solubilních komplexů v plazmě, ↓ fibrinogen, D-dimery v norměTh – PAMBA, substituce fibrinogenu (mražená plasma)

92

Page 93: lf1.czlf1.cz/wp-content/uploads/228-vypracovane_otazky_interna... · Web viewprognoza – 50 % dosáhne kompletního uzdravení do 6 měsíců, 20 % má prodloužený průběh –

Vypracované otázky na INTERNU made by Edita Homolková, Monika Ambroziová, Adam Král, Petr Vermach a Vítek Křehnáč

53.B TUMORY JATER A PODJATERNÍ KRAJINY

(adenom, hemangiom, hepatocelulární ca - cirhóza, HBV, HCV, napodobuje cirh. uzly,roste rychle invazivně,↑ α fetoprotein) karcinom žlučníku-adenokarc, rychle do jater šíří + meta, chir, palia: drenáž) tu. žlučových cest (adenokarcinom, ca horní třetiny, střední a dolní třetiny, stř. a dolní resekabilní)Nesidiom z B buněk, gastrinom; karcinom pankreatu (100% mortalita, etio: kouření, tuk, alkohol; adenokarcinom; v hlavě p. 60%; CA19.9, CA242; ter: chir (duodenopankreatektomie) paliativní (cholecystoduodenoanastomóza)Nádory jater

Benigní nádory jater- hemangiom – nejčastější, častěji u žen, vzácně krvácení do dutiny břišní při povrchové lokalizaci

KO - většinou asympt., náhodný nález na UZ (hyperechogenní)- hepatocelulární adenom - u žen 30-40 let užívajících estrogenní kontraceptiva

- asymptomatické, u 1/3 ruptura a hemoperitoneum, event. maligní zvrat v hepatocelulární ca

- fokální nodulární hyperplazie (FNH) – u žen mezi 30-50 rokem, hypertrofovaná část jater. parenchymu- adenom žlučovodů - velice vzácný, většinou asympt., náhodný nález na UZ, u hepatocelulárního

adenomu časté komplikaceDg - dopplerSONO, kontrastní spirální CT, MRI s kontrastní látkou spec. pro játraTh - vysazení a KI estrogenů a anabolických steroidů u hepatocelulárního adenomu

a FNHMaligní nádory jater

- hepatocelulární ca – 2 % všech tumorů, více u mužů (v tropech nejčastější maligní nádor u mužů), téměř vždy spojen s jaterní cirhózou (hl. v důsledku hemochromatózy a hepatitidy B, HCV, aflatoxin B)

KO – až pozdější stádia palpační bolestivost v pravém hypochondriu, hubnutí, event. šelest nad játry, ascites, paraneoplastické příznaky (horečka, polyglobulie)

Dg – α-fetoprotein (AFP), dopplerSONO, CT, MRI, intraoperační UZ- při potenciálně kurativním nálezu nesmí být provedena punkce tenkou jehlou

(2% riziko implantačních metastáz)Th - u solitárního nádoru resekce jater/ transplantace (lze jen u 10-30 %)

- paliativně lokální terapie (etanol. injekce pod UZ kontrolou, laser, kryoterapiecholangiokarcinom – vzácný

- cholangiokarcinom – častěji doprovází primární sklerozující cholangitidu, špatná prognóza (nemožná resekce

KO – ikterus- sekundární, metastatické - velice časté, u více než ½ nemocných s tumory GITu

- kolorektální ca, tu plic, ca močového měchýře,, melanomy, Hodgkinovy i non-Hodgkinovy lymfomy

Dg - UZ (mnohočetná ložiska s hypoechogenním okrajem s centrální nekrózou a kompresí/ dislokací cév)

Karcinom pankreatu – viz ot. 77.BNádory žlučníku

- v 80 % vzniká při současné cholecystolitiáze, při kalcifikaci stěn žlučníku, kongenitální biliární systy, polypy žlučníku, nosičství Salmonelly

KO – nespecifické, překryté symptomy cholesytolitiázy a cholecystitidy- interus (špatná prognóza, infiltrace jaterního hilu), hepatomegalie, ascites, hmatná rezistence v místě

žlučníkuDg – lab cholestázy: ↑ bilirubin, ALP, GMT, nádorové markery CA 19-9 a CEA, USG, MR, CT, ERCP, PTCTh – chirurgie (60-80 % umírá do roka po operaci)

Nádory žlučových cest- adenokarcinomy šířící se per kontinuitatem do okolí (jaterní hilus, pankreas, žlučník) a lymfogenně do uzlinKlatskinův nádor – rakovina oblasti junkce hepatiku, šíří se submukóznmě a může napodobovat sklerózující

cholangitiduEtio – hl. primární sklerózující cholangoitida…KO – anorexie, nauzea, hubnutí, nebolestivý ikterus, recidivující cholangitidy

93

Page 94: lf1.czlf1.cz/wp-content/uploads/228-vypracovane_otazky_interna... · Web viewprognoza – 50 % dosáhne kompletního uzdravení do 6 měsíců, 20 % má prodloužený průběh –

Vypracované otázky na INTERNU made by Edita Homolková, Monika Ambroziová, Adam Král, Petr Vermach a Vítek Křehnáč

Dg – lab známky cholestázy, SONO, ERCP, MRCPTh – chirurgické odstranění tumoru a drenáž žluče, jinak vše ostatní paliativní

Nádory Vaterovy papily- benigní a maligní adenomyEtio – nejčastěji geneticky podmíněna (familiární adenomatóza)KO – symptomatologie choledocholitiázy (kolika, ikterus)Dg – lab parametry cholestázy, USG, CT, ERCP…

- koloskopie k vyloučení současných adenomů/ ca tlustého střevaTh – chir. odstranění s bilioenterální anastomózou

- u inoperabilních dren perkutánní transhepatální cestou/ endoskopicky transpapilárně

54.A PORUCHY METABOLISMU SODÍKU A VODY

HYPONATREMIE- norma 135-145 mmol/l- klinické projevy při 125 mmol/l u akutně vzniklé, u chronické mohou projevy začít až při 110 mmol/l- riziko u diuretik, alkoholiků, chronické malnutriceKO – edém mozku + ↓ pO2

- bolest hlavy, apatie, zmatenost, křeče, koma Lab - osmolalita séra, K+, MG2+, osmolalita moči, ztráty Na+ močí, TSH, ranní kortizol- V +somolalita: Sosm = 2* Na+ + Surea + Sglc (mmol/l)- tonicita = efektivní osmolalita – urea, volně přestupuje mezi kompartmenty + přesuny vody, když dlouhodobě,

přeorganizování organ. látek v bb. – NE rychlá dialýza!Isotonická hyponatrémie - pseudohyponatrémie, laboratorní artefakt, koncentrace Na+ v plasmě zůstává

normální- 280 – 295 mosm/ kg- SIADH (syndrom nepřiměřené sekrece ADH, Schwartzův-Bartterův syndrom) - ↑ ADH →

euvolemická hypoosmolalita a hyponatremieKO – neurologické příznaky (bolest hlavy, nauzea, zmatenost, zvracení, křeče, koma,

respirační selhání)- mozkový edém

Etio – bronchogenní ca, mezoteliom, ca duodena/ pankreatu/ prostaty/ endometria/ ureteru/ nazofaryngu, leukemie

- tu mozku, abscesy mozku, subdurální hematomy, demyelinizační onem., kraniocerebrální traumata, delirium tremens

Th – inhibitory receptorů vasopresinu (tolvaptan)- hypotyreóza

Hypertonická hyponatrémie – retence vody > renece Na+

- >295 mosm/ kg- UNa - >20 mmol/l → nefrotický syn. a sehlání ledvin

- < 20 mmol/l → chronické srdeční selhání, ascitická jaterní cirhóza, otoky- ↑ nefektivní osmolality jiným solutem (glukóza, infúze manitolu) než Na+, hypertonicita ECT vede

k přesunu vody z buněk, což naředí Na+ v plasmě za vzniku hyponatrémieHypotonická hyponatrémie – nejčastější, ↓ Na+ → ↓ osmolality

- <280 mosm/ kg- renální ztráty – diuretika, osmotická diuréza, cheon. Intersticiální nefritida, Addisonova ch.- extrarenální ztráty – průjem, pocení, 3. prostor (popáleniny, střevní obstrukce, pankreatitida)- příčina - excesívní příjem volné vody při primární polydipsii nebo porušená schopnost ledvin

reagovat na hypoosmolalitu plasmy adekvátním zředěním moči (absolutním nebo relativním nadbytkem ADH)

- hypoosmolalita ↓ sekreci ADH, avšak tato inhibice může být zrušena např. snížením efektivního cirkulujícího volumu

- hypersekrece ADH však může být nezávislá (SIADH) - léky, nádory, poruchy CNS, stres.Th – mírná hyponatrémie – monitorace, vysazení diuretik a hypotonických infůzí, korece hapokalémie a

hypomagnezémie

94

Page 95: lf1.czlf1.cz/wp-content/uploads/228-vypracovane_otazky_interna... · Web viewprognoza – 50 % dosáhne kompletního uzdravení do 6 měsíců, 20 % má prodloužený průběh –

Vypracované otázky na INTERNU made by Edita Homolková, Monika Ambroziová, Adam Král, Petr Vermach a Vítek Křehnáč

- asymptomaická hyprevolemická a euvolemická – omezit příjem tekutin na <1 l/den- asymptomaická hypovolemická – doplnit volum izotonickým roztokem NaCl- těžká hyponatrémie s encefalopatií - pomalá korekce (min. 48 hod) hyponatrémie roztokem NaCl (0,9 %/ 3

%), rychlost přírůstku 0,5-2 mmol/l/h- kalkulace změny sérové koncentrace po podání 1000 ml koncentrovaného rozkoku NaCl:

Změna Na = (koncentrace Na v roztoku – koncentrace Na v séru)/ celková tělesná voda (60% váhy u mužů a 50% u žen) + 1

HYPERNATRÉMIE - sérová konc. >146 mmol/l- vede k buněčné dehydrataci (mozek), ↑ smykové tření v cévách (trombóza, krvácení)- osmolalita moče < osmolalita séra = centrální/ nefrogenní diabetes insipidus- hyperosmolární moč - s ↑ Na (>100 mg?/l)

- s ↓ Na (< 25mg/l) – pocení, snížený příjem vodyKO – žízeň, apatie, slabost, zmatenost, křeče, bezvědomíLab – Na v moči a séru, glc, urea, kreatininTh – 50 % mortalita

- hypotonické roztoky (5% glc)-akutní hypernatrémie – do 24 hod, možná rychlá korekce- kalkulace změny natrémie po podání 5% roztoku glc:

Změna Na = (konc. Na v podávaném roztoku(0 ) – koncentrace Na v séru)/ celková tělesná voda +1

Dehydratace: Izotonická hypovolemie - Souběžný deficit vody a Na způsobuje zmenšený objem ECT s normální osmolalitou

- při zvracení, průjmech, ztrátách izotonické tekutiny – píštěle, popáleniny, punkce ascitu- Projevuje se tachykardií, hypotenzí, synkopou, křečemi, žízní, polyuriíTh - substituce izotonické tekutiny (0.8% roztok NaCl), při zvracení podáme bilanční roztoky

(Ringerův roztok, Hartmanův roztok)Hypotonická hypovolemie - Ztráta vody i Na (sodík převládá) – pokles osmolality ECT způsobí hypovolemii a

expanzi ICT- často hrazením ztrát tekutin pitím čisté vody nebo infuzí glukózy bez hrazení ztrát Na- projevuje se únavou, hypotenzí, tachykardií, poklesem CŽT , křečemi a horečkou + vzestup

celkové bílkoviny a hematokritu. Th - substituce izotonickými až hypertonickými roztoky NaCl

Hypertonická hypovolemie - Deficit čisté vody bez ztrát Na – voda difunduje z ICT do ECT, je snížen objem ECT + hyperosmolalita

- nedostatečným přívodem čisté vody, nadměrnými ztrátami pocením, při diabetes insipidus, osmotické diuréze, náhradou tekutin pitím hypertonické tekutiny

- projevuje se žízní, suchostí v ústech, poklesem CŽT, hypotenzí, tachykardií, zvýšenou tepotou, bezvědomím až kómate

Th - pití čisté vody nebo infuze bezsolutové vody ve formě 5% 10% nebo 20%glukózy, poloviční nebo třetinové roztoky Darowa. Úprava musí být pozvolná, při rychlé změně osmolality hrozí edém mozku

Hyperhydratace: Izotonická hypervolemie -Nadbytek vody i sodíku - Zvětšený objem extracelulární tekutiny při normální

osmolalitě- Příčinou bývá nadměrné podávání izotonických infuzí nebo kardiální a renální insuficience- Projevem jsou otoky, výpotky, dušnost, selhání oběhu, vzestup CŽTTh - léčba primárního onemocnění, omezení příjmu tekutin a solí, diuretika, hemofiltrace.

Hypotonická hypervolemie -Nadbytek čisté vody -> zvýšení objemu ICT i ECT + snížení osmolality - Nadměrné podání hypotonických roztoků, zvýšená sekrece ADH, selhání jater apod..- Projevuje se slabostí nauzeu, otoky, poruchami vědomí, dušností a vzestupem CŽTTh - léčba primární poruchy, restrikce bezsolutové vody, osmotická diuretika, hemofiltrace

Hypertonická hypervolemie -Nadměrný přívod vody i sodíku s převahou sodíku – zvýšení osmolality, zvětšený objem ECT.

95

Page 96: lf1.czlf1.cz/wp-content/uploads/228-vypracovane_otazky_interna... · Web viewprognoza – 50 % dosáhne kompletního uzdravení do 6 měsíců, 20 % má prodloužený průběh –

Vypracované otázky na INTERNU made by Edita Homolková, Monika Ambroziová, Adam Král, Petr Vermach a Vítek Křehnáč

- Vzniká po podání hypertonických roztoků, hypersekrecí hormonů kůry nadledvin- Projevy: srdeční selhání, otok, průjem, zvracení, poruchy vědomíTh - omezení soli a tekutin, osmotická diuretika, hemofiltrace

54.B NEMOCI JÍCNU

Hiátová hernie- častá anatomická odchylka- přesun gastroezofageální junkce a/ nebo části žaludku do mediastinaKO - často bez kliniky, zvracení, krvácení s erozemi a záněty

- při velké hernii bolest, dysfagieDivertikly jícnu

- pravé – tvořeny celou stěnou- pseudodivertikly – výchlipky mukózy procházející otvorem ve svalovině- pulzní – při intraluminálním přetlaku

- Zenkerův v cervikální oblasti- trakční – tahem z okolí (tbc lymfadenitida)Dg – Rtg s kontrastní látkou

Vrozené vadyAtrézie jícnu – relativně časté (1:4500 živě narozených)Duplikatura jícnu – větčinou klinicky němá, dysfagie/ respirační porucha

Porucha motility jícnu- primární – achalázie, difúzní spasmus jícnu, louskáčkovitý jícen, hypertenzní dolní jícnový svěrač- sekundární – sklerodermie, dermatomyositida, DM, alkoholismus, myastenie, sclerosis multiplex,

parkinsonismus…KO – dysfagie (horní/ dolní, stálá/ intermitentní/ paroxysmální)Dg – ezofagogastoduodenoskopie, kontrastní Rtg (polykací akt)

- manometrieAchalázie jícnu - porucha hybnosti jícnu neznámé etio, zánětlivý infiltrát a ireverzibilní degenerativní změny

v plexus myentericus (více postiženy inhibiční neurony), možná tvorba autoprotilátek- nedostatečné uvolnění svalstva na přechodu jícnu a žaludku, chybí peristaltika, ↑ klidový tlak,

městnání jídla a slin vedoucí k dilataci (až 2 l)- pseudoachalázie – podobný klinický obraz díky jiné chorobě (nejčastěji ca jícnu)- vigorózní achalazie – manometricky vyšší amplitudy kontrací s bolestmi na hrudiKO – dysfagie, odynofagie, regurgitace

- z počátku paradoxní dysfagie, hubnutí, stálé bolesti na hrudi (enormní dilatace jícnu)Dg – rtg polykacího aktu, endoskopie v počátku normálníTh – uvolnění fční stenózy nitráty/ blokátory Ca kanálu (nifedipin)

- aplikace botulotoxinu- balónková dilatace, chirurgie

Difúzní jícnový spasmus – nejasná etio- neprogredující nepravidelná dysfagie, bolest- zhoršení po studeném/ teplém a ve stresuTh – blokátory Ca kanálu, nitráty, chirurgie

Louskáčkovitý jícen – nadměrná peristaltika s vysokými a dlouhými peristaltickými vlanami- bolest i bez vazby na jídlo

Dysmotilita při systémových chorobách pojiva – nejčastěji u sklerodermie- může být i prvním projevem (před postižením kůže)- může být zhoršováno nedostatečností exokrinních žlaz (potřeba sousta zapíjet), regurgitace,

rozvoj refluxní ezofagitidy- často kanadidové infekce, krvácení, dráždění vedoucí k tvorbě striktur

Refluxní choroba jícnu- reflux je do urč. míry fyziologický- refluxní ezofagitida při makroskopických známkách zánětu v distálním jícnu, NERD (non erosive reflux disease)

pouze při mikroskopickém průkazu a klinice

96

Page 97: lf1.czlf1.cz/wp-content/uploads/228-vypracovane_otazky_interna... · Web viewprognoza – 50 % dosáhne kompletního uzdravení do 6 měsíců, 20 % má prodloužený průběh –

Vypracované otázky na INTERNU made by Edita Homolková, Monika Ambroziová, Adam Král, Petr Vermach a Vítek Křehnáč

Etio – díky pH refluxátu a době jeho působení- porucha motility, ↓ tonus dolního jícnového svěrače (iatrogenně blokátory Ca kanálů, nitráty a

aminofyliny), kouření, kofein, zvedání břemen, těhotenství…KO – pyróza (80%), regurgitace, dysfagie, bolest na hrudi, odynofagie, říhání, zvracení, bolest v epigastriu

- možný podíl na chronickém kašli, laryngitida, globus, asthma, ↑ kazivost zubůDg – endoskopie s biopsií (NERD), 24hod pH-metrieKomplikace – vřed, krvácení, stenózy, aspirace ve spánku, Barettův jícen (inkompletní intestinální metaplazie,

prekanceróza adenom, u 1-20 %)Th – stravovací návyky – více malých porcí, jíst nejpozději 3 h před spaním, zvednout horní část postele

- farmaka – inhibitory H+ pumpy – omeprazol, pantoprazol, ráno nalačno- antagonisté H2 rec – ranitidin, famotidin, menší účinek, - prokinetika – itoprid

Ezofagitidy Korozivní – kyseliny, louhy,tetracykliny, clindamycin, bisfosfonáty…

- mediastinitida, stenózy- propláchnutí a odsátí sondou, vysoké kortikoidy, atb, analgetika, ataraktika (uklidňovadla)

NádoryBenigní – leiomyom, fibrom, lipom

KO – většina asymptomatické, příčina krvácení do GIT, dysfagie- endoskopie s biopsií, endosonografie

Maligní – rizikové: horká strava, koření (spinocelulární ca), alkohol, nikotin, nitrosaminy, obezita, Barettův jícenKO – dysfagie, úbytek váhy, kachexie, regurgitace, chrapot

55.A CHRONICKÁ LYMFATICKÁ LEUKÉMIE ( CLL) A PŘÍBUZNÉ CHOROBNÉ STAVY

- periferní lymfoproliferace z B-bb., nejčastější typ leukemie, častěji postihuje muže, medián 70 let- vzniká kumulací zralých afunkčních lymfocytů i bb. v G0 a G1 fázi - díky chronické antigenní stimulaci BCR, uplatňuje se i deregulace miRNA (nekódující RNA, váže se na

komplementární RNA a tak inhibuje její translaci – Bcl2 → omezení apoptózy)- rozlišení CLL s mutovaným a nemutovaným genem pro IgVH (variabilní oblast těžkého řetězce)- autoimunní komplikaceKO - bývá asymptomatická - náhodný nález, heterogenní onem. - nestejný průběh choroby

- zvětšené periferní uzliny, játra, slezina- aktivní CLL – horečky, noční poty, hubnutí, únava- pokročilá CLL - anémie, trombocytopenie, autoimunní hemolytická anémie a trombocytopenie

- projev tkáňové infiltrace lymfocyty (zvětšené lymf. uzlin ve všech lokalizacích, drobné až velikost mandarinky; jsou elastické, nebolestivé, volně pohyblivé)

Dg – absolutní lymfocytóza >5*109/l po dobu 3 měsíců- typický klonální imunofenotyp CD5+, CD19+, CD23+ (Monoklonální B-lymfocytóza = nalezena pozitivita

imunofenotypu CLL, ale krevní obraz je normální)- zralé bb s kondenzovaným chromatinem, Gumprechtovy stíny (poškozená jádra fragilních leukemických

lymfocytů)- není nutná biopsie (leukemická infiltrace >35 %)

Prognóza – horší u nemutovaného IgVH, exprese CD38 - celková doba přežití 1-20 let

Klasifikace - dle Raie: stadium 0 - jen lymfocytózastadium 1 - lymfocytóza + adenomegaliestadium 2 – lymfocytóza + adenomegalie + hepato/ splenomegaliestadium 3 - lymfocytóza + anémie s konc. Hb <110 g/lstadium 4 - lymfocytóza s trombocytopenií <100*109/l

Th – pacienty bez selhání kostní dřeně nebo obtíží plynoucích z adenomegalie, celkových příznaků a známek rychle progredujícího onemocnění jen sledujeme, neléčíme

- imunochemoterapie FRC – rituximab, fludarabidit, cylkofosfamid v 6 cyklech- pokud selže, alemtuzumab (antiCD52), ofatumumab (antiCD20), kombinovaná

chemoterapie, vysokodávkové kortikoidy, allogenní transplantace

97

Page 98: lf1.czlf1.cz/wp-content/uploads/228-vypracovane_otazky_interna... · Web viewprognoza – 50 % dosáhne kompletního uzdravení do 6 měsíců, 20 % má prodloužený průběh –

Vypracované otázky na INTERNU made by Edita Homolková, Monika Ambroziová, Adam Král, Petr Vermach a Vítek Křehnáč

- radioterapie paliativněKomplikace - Richterův syn. – transformace v průběhu choroby do agresivního lymfomu (hl difuzní velkobuněčný

DLBCL)- recidivující infekce, porucha imunity, granulocytopenie → herpes zoster, atypické pneumonie, CMV,

přemrštěná odpověď na jakoukoli vakcinaci- tendence ke vzniku autoimunitní hemolyt. anémie či ITP, Evansův sy

Prolymfocytární leukémie - může být z B nebo T bb. (B-PLL, T-PLL)- 1 % z lymfatických leukemií- symptomatický průběh (tzv. B-symptomy), rychle ↑ počet leukocytů v periferní krvi- B-PLL – >55 % větších bb s nukleoly, CD20 a CD22

- masivní splenomegalie bez adenomegalie- imunochemoterapie s rituximabem

- T-PLL – středně velké bb, nukleolus, CD2,3,7, častěji CD4+- hepatosplenomegalie s adenomegalií, u 1/5 výpotky a kožní infiltráty- imunochemoterapie s alemtuzumabem

Trichocelulární leukémie = Hairy cell leukemia (HCL) - 2 % z lymfatických leukemií, postihuje více muže- KO – celkové příznaky, hubnutí, únava, horečky, břišní dyskonfort, infekce- Lab - infiltrace kostní dřeně a masivní splenomegalie, možná infiltrace i jater a uzlin vzácně

- pancytopenie s malým množstvím kolujících nádorových bb (velké, kondenzovaný chromatin, vláskové výběžky ne povrchu)

-v bb tartarát rezistentní kyselá fosfatáza- Th – 2-chloro-deoxyadenosin (kúra 3 týdny), možno opakovaně- Prognóza - dobrá

55. B ONEMOCNĚNÍ MEDIASTINA

Akutní mediastinitida- akutní zánět mediastina vzniká přestupem infekce z okolí (jícen, plíce – zánět, absces, empyém) – jde o závažný

stav s vysokou mortalitouKO - horečka, retrosternální bolest, při poškození jícnu je dysfagie a podkožní emfyzém krkuDg - RTG – rozšířené mediastinum, zastření kostofrenického úhlu, mediastinální emfyzémTh - drenáž mediastina + parenterálně ATB

Chronická mediastinitida - charakteristická je tvorba vaziva někdy označována jako idiopatická fibróza mediastina (Ormondova choroba),

občas bývá sdružena i s fibrózou retroperitonea při které dochází k útlaku močovodů a vzniku hydronefrózy- Etiologie nejasnáDg - RTG – nepravidelně rozšířený stín mediastina, někdy přítomnost kalcifikačních stínůTh - glukokortikoidy

Nádory mediastina - nádory vyrůstají ze struktur mediastina- nemocní bývají asymptomatičtí, nádor bývá odhalen náhodně při RTG hrudníku z jiné indikace- přehled tumorů a pseudotumorů:

a) Nádory předního mediastina – thymom, lymfom, retrosternální struma, teratomb) Nádory středního mediastina – lymfom, perikardiální/ bronchogenní cystac) Nádory zadního mediastina – neurinom, ca jícnu, aneurysma aorty, hiátová hernie

KO - zřídka se objevuje chrapot, kašel, dušnost a bolest na hrudi, syndrom horní duté žílyDg - dle RTG – lokalizace, denzita, vaskularizace, kalcifikace aj..Th - většinou je indikována chirurgická resekce

56.A AKUTNÍ RENÁLNÍ SELHÁNÍ

= náhle vzniklá neschopnost ledvin odstraňovat z organismu splodiny metabolizmu- retence dusíkatých katabolitů vede k narušení homeostázy někdy provázené syndromem urémie (anorexie,

nauzea, zvracení, průjmy, známky hemoragické diatézy, polyneuritida psychické změny až kóma)- u 30 % nemocných na JIP

98

Page 99: lf1.czlf1.cz/wp-content/uploads/228-vypracovane_otazky_interna... · Web viewprognoza – 50 % dosáhne kompletního uzdravení do 6 měsíců, 20 % má prodloužený průběh –

Vypracované otázky na INTERNU made by Edita Homolková, Monika Ambroziová, Adam Král, Petr Vermach a Vítek Křehnáč

Ptg – 20 % průtoku je nutritivní, 80 % je funkční, při částečně sníženém průtoku se naruší fce, ale až při velkém snížení dojede k požkození parenchymu

Etiologie - prerenální – nedostatečná perfůze ledvin (55% případů)- renální – morfologické poškození parenchymu ledvin (40% případů)- postrenální – obstrukce močových cest- příčiny se často kombinují → ARS je často výsledkem souhry více faktorů

Prerenální příčiny Hypoperfuze ledvin - nejdříve postižen pouze funkční průtok, nutritivní až později

- po obnovení adekvátního průtoku je možná rychlá úprava fce ledvin- další zhoršení renální perfúze ale nakonec může vést

k ischemickému poškození ledvin a ke vzniku akutní tubulární nekrózy (ATN)

- příčiny preren. selh.: hypovolémie, nízký ♥ výdej, systémová vazodilatace (efektivní hypovolémie), selhání intrarenální autoregulace, hyperviskózní syndrom, bilaterální obstrukce renálních cév

- hypovolémie → aktivace karotických baroreceptorů a ♥ volumoreceptorů → aktivace sympatiku + systému renin-angiotenzin-aldosteron + vazopresinu (ADH) + aktivace syntézy endotelinu → vazokonstrikce cév ve splanchnické oblasti, retence Na+ a H2O a udržení perfúze životně důležitých orgánů → ↓ renální perfuze → stimulace syntézy vazodilatač. látek v ledvinách (prostacyklin, prostaglandin E2, NO) → dilatace aferentních arteriol + aktivace angiotenzinu II udržuje kontrakcí eferentních arteriol intraglomerulární tlak a tím i glomerulární filtraci

Renální příčinyPřímé poškození parenchymu ledvin - nejčastěji ischemické nebo toxické a nekrózou tubulárních bb., dále pak

renální vaskulitida, rychle progredující GN, akut.interstic.GNIschemické ARS – v důsledku protrahované těžké ischémie se ↓ nutritivního průtoku a morfolog.

poškoz. ledvin, neustupuje po odstranění příčiny- ↓ průtoku krve (hlavně kůrou), ↓ permeability glomerulární kapilární stěny, reflex

filtrátu poškozenou stěnou tubulů do intersticia ledvin, tubulární obstrukce- déletrvající ischémie endotelu renálních cév ↓ produkci vazodilatačních působků

v endotel. bb. a stimuluje uvolnění vazokonstrikčních látek → prohloubení ischémie- nejcitlivější jsou části nefronu uložené v zevní části dřeně ledvin a

v kortikomedulárním přechodu (pars recta proximálního tubulu a tlustá část vzestupného raménka Henl. kličky) → nekróza tubulárních buněk

- během reperfúze a reoxygenace hrozí poškození bb uvolněnými volnými radikály- nekrotické tubulární bb. umožní zpětný reflex glom. filtrátu, odloupané epitelie

mohou ucpat lumen tubulů → ↑ intratubulární tlak → zástava GF → nadměrný přísun solutů do oblasti macula densa v dist. tubulu → aktivace tubuloglom. zpětné vazby s konstrikcí aferentních arteriol a ↓ GF

Nefrotoxické ARS – exogenní/ endogenní toxické látky se mohou koncentrovat v tubulárních epitelových buňkách/ v intersticiu dřeně ledviny (protiproudový koncentrační mechanismus)

- ATB - aminoglykosidy, amfotericin B, cefalotin, vankomycin, klindamycin, β-laktamová ATB (cefalotin, cefaloridin), sulfonamidy (obstrukce tubulů krystaly)

99

Page 100: lf1.czlf1.cz/wp-content/uploads/228-vypracovane_otazky_interna... · Web viewprognoza – 50 % dosáhne kompletního uzdravení do 6 měsíců, 20 % má prodloužený průběh –

Vypracované otázky na INTERNU made by Edita Homolková, Monika Ambroziová, Adam Král, Petr Vermach a Vítek Křehnáč

- cytostatika - cis-platina, metotrexát, ifosfamid, cyklosporin- radiokontrastní látky - ischémie i přímá tubulotoxicita, intratubulární precipitace

proteinů nebo krystalůRabdomyolýza a hemolýza – zejména při hypovolémii a acidóze, intratubulární precipitace Hb,

myoglobinu a urátů- traumata velkých svalových skupin, svalová ischémie, komprese svalů, nadměrné

cvičení, křeče, prochlazení, přehřátí, infekce (legionela, chřipka), MTB poruchy (hypokalémie, hypofosfatemie), myopatie, léky (kokain, fibráty), toxiny (hadí jedy, CO), inkompatibilní transfuze, mechanické poškození erytrocytů při mimotělním oběhu, akutní hemolytická krize při hemolytické anémii

Intratubulární obstrukce (hyperoxalurie)Akutní urátová nefropatieHyperkalcémieTěhotenství (hypovolémie, sepse, tox.vlivy)Choroby postihující ledvinné cévy, glomeruly a intersticium – HUS, trombotická trombocytopenická

purpura, těhotenská toxémie, DIC, sklerodermie, malig. hypertenze, glomerulonefritidy a vaskulitidy, akutní alergická intersticiální nefritida

- fáze renálního ARS: I. Iniciální – ledviny vystaveny vyvolávajícímu inzultu, vzniká ARS, které lze zvrátit léčbou a udržet nemocného v neoligoanurickém ARS

II. fáze rozvinutého ARS s oligoanurií (7-14 dnů) – ohrožení hyperhydratací, iontovou dysbalancí, acidózou, retencí dusíkatých katabolitů, od konce 1. týdne ARS se může projevit urémie

III. reparace – postupný návrat ren. fce, nejdříve se vrací diuréza, která při nedostatečné koncentrační schopnosti ledvin přechází do polyurie a pak ↑ GF a ↓ azotémie, porucha koncentrační schopnosti přetrvává i řadu měsíců

Postrenální příčiny– podmínkou vzniku ARS je oboustranná obstrukce močových cest nebo jednostranná při jedné funkční ledvině- v prvních několika hodinách průtok krve postiženou ledvinou ↑ vlivem dilatačního účinku prostaglandinů na

aferentní arterioly- ↑ tlak v dutém systému nad překážkou a ↑ intratubulární tlak vedou k zástavě filtrace v glomerulech a ↑

cévní intrarenální rezistence vyvolá ↓ průtoku krve ledvinou- po uvolnění několikahodinové obstrukce se průtok krve ledvinou i filtrace normalizují, po delší obstrukci to trvá

déle, dlouhodobá obstrukce vede k tlakové atrofii renálního parenchymu- příčiny: obstrukce ureteru, obstrukce močového měchýře a jeho hrdla, obstrukce močové trubice

100

Page 101: lf1.czlf1.cz/wp-content/uploads/228-vypracovane_otazky_interna... · Web viewprognoza – 50 % dosáhne kompletního uzdravení do 6 měsíců, 20 % má prodloužený průběh –

Vypracované otázky na INTERNU made by Edita Homolková, Monika Ambroziová, Adam Král, Petr Vermach a Vítek Křehnáč

KO - Prerenální ARS – žízeň, ortostatická hypotenze, tachykardie, snížená náplň krčních žil, suchost sliznice, snížený kožní turgor, absence potu v axilách (důsledek hypovolemie např. po velkých krevních ztrátách, dehydrataci apod…)

- potvrzením klinické diagnózy je nízký CŽT nebo tlak v plicnici v zaklínění (plnící tlak LK)- Renální ARS– probíhá ve 3 fázích: iniciální fáze – ledviny jsou vystaveny inzultu vyvolávající ARS, které lze v

této fázi zvrátit (neoligoanurické ARS má lepší prognózu a rychleji dojde k úpravě funkce ledvin)

fáze rozvinutého ARS – oligoanurie trvající 7 – 14 dní, nemocnému hrozí hyperhydratace s iontovou dysbalancí, acidózou a retencí katabolitů, po 1. týdnu se mohou objevit příznaky urémie

fáze reparace – postupný návrat renálních funkcí (zprvu polyurie pro sníženou koncentrační schopnost ledvin) – nemusí být restituce ad integrum

- Postrenální ARS – pomalu progredující ARS je asymptomatické, při náhlém vzniku distenzní bolest nad symfýzou někdy kolikovitého charakteru vystřelující do třísel a na vnitřní stranu stehna, u nemocných s neuropatií, neurologickými chorobami či léčených anticholinergiky uvažujeme o tzv. neurogenním měchýři

Dg - sediment – prerenální – chudý- renální – hnědavé válce s tubulárními buňkami, mikrohematurie, lehká proteinurie

- u intersticiální nefritidy bělavé granulární válce, u akut. glomerulárního poškození ery válce nebo dysmorfické erytrocyty, u obstrukce pyurie a hematurie

- exkreční frakce sodíku – rozlišení prerenálního ARS od ischemického a nefrotoxického ARS→ prerenální ARS (retence Na) - ↓ EF Na, moč koncentrovaná s ↑ osmolalitou→ renální ARS (tubulární buňky neumí koncentrovat moč) – EF Na ↑, osmolalita moči

přibližně jako osmolalita plazmy- USG- ascendentní pyelografie – při podezření na obstrukci

- komplikace ARS vznikají v důsledku ztráty renální fce a vystupňovaného katabolismu (hyperhydratace, hyponatremie, hyperkalemie, hypokalcemie, hyperfosfatemie, mtb acidóza)

prevence - zachování intravaskulárního volumu, udržení adekvátní srdeční fce, minimalizace expozice nefrotox. l., při cytostatické terapii alopurinol (prevence urátové nefropatie) s forsírovanou diurézou a alkalizací moči

- po požití etylenglykolu podáme alkohol + dialýzaTh - korekce hypovolémie – FR, 20 % manitol (osmoticky aktivní plazmaexpandér, osmotické diuretikum,

vazodilatans, ↓ edém, stabilizuje fci mitochondrií) 100-250 ml za 10-30 min- kličková diuretika – furosemid - ↓ aktivní transport Na v tlusté části Henl. kličky (↓ spotřeba O2 a ATP)

- hradit ztráty tekutin – ½ FR (0,45% NaCl) + 5% glu 1:1- monitorace hydratace – měření CŽ tlaku, bilance tekutin, labor. vyš.

- nízké dávky dopaminu- terapie hyperkalémie (! K >6,5 mmol/ při hyperkatabolismu vede k ♥ zástavě) → urgentní dialýza

- pro ↓ membr. účinku → Ca i.v., Na i.v., β mimetika i.v. 1-2 mg- odpora utilizace K v buňkách → glu + inzulin i.v., NaHCO3 i.v.- otenciace fekální eliminace – iontoměniče p.o., klyzma

- terapie hypokalemie (hrozí v polyurickém stadiu ATN, při úpravě MTB acidózy nebo při anabolismu) → 20 ml 7,5% KCl v 500 ml FR

- dietotepapie – zabránit vzniku katabolismu a současně dosáhnout co nejmenší produkce zplodit mtb dusíku- v akut. stadiu parenter. výživa, enter. výživa NG sondou, restrikční ↓proteinová dieta

- dialýza – indikace: K >6 mmol/l, hyperhydratace se ♥ selháním, těžká mtb acidóza, klin. vyjádřená urémie, intoxikace dialyzovatelným jedem, hyperazotemie (urea > 35 mmol/l, kreatinin > 500-600 µmol/l), hyperkalcemie > 4,5 mmol/l, hyperurikemie >1000 µmol/l, oligoanurie > 3 dny

101

Page 102: lf1.czlf1.cz/wp-content/uploads/228-vypracovane_otazky_interna... · Web viewprognoza – 50 % dosáhne kompletního uzdravení do 6 měsíců, 20 % má prodloužený průběh –

Vypracované otázky na INTERNU made by Edita Homolková, Monika Ambroziová, Adam Král, Petr Vermach a Vítek Křehnáč

56.B ŠOK

= akutní hemodynamická proucha s generalizovaným ↓ perfuze tkání pod úroveň nezbytnou pro zachování jejich fcí a integrity

- tachykardie (>100/min)/ bradykardie, periferní vazokonstrikce (studená bledá opocená kůže) oligurie (<20 ml/ h), acidóza, poruchy vědomí

Kardiogenní šok= selhání srdce jako pumpy- ↓ srdečního výdeje pod 1,8-2,2 l/ min/ m2 (norma >2,5)- sTK < 80-90 mm Hg, end-diastolický tlak v levé komoře >20 mm Hg (norma 5-12, = tlak v plicních kapilárách)Etio - ↓ kontraktility (IM, kardiomyopatie), objemové přetížení se ↑ preloadu (insuf. chlopní, zkraty), tlakové

přetíženi se ↑ afterloadu (stenózy chlopní, plicní embolie), obtížné plnění srdce (tamponáda, konstrikt. perikard.), arytmie

Dg - vlhké chrůpky, dušnost, RTG (známky plicního městnání), ECHO, plicnicový katétrTh - kauzální - reperfúzní terapie (při IM), operace (ruptura srdeční stěny, ruptura papilárního svalu), odlehčující

punkce (tamponáda), fibrinolýza/embolektomie (plicní embolie), antiarytmika (arytmie)- symptomatická - zvýšená poloha homí ½ těla, oxygenoterapie, sedace/ analgetika, dopamin

Obstruktivní šok - při významné překážce krevní cirkulace- nereaguje na doplnění cirkulujícího objemu a ionotropikaEtio – masivní plicní embolizace, srdeční tamponáda, tenzní pneumotorax, obstrukce mitrální/ trikuspidální

chopně trombem, disekující aneurysma aortyDg – CT, D-dimery, echo…

Hypovolemický šok Etio – ztráta cirkulujícího volumu, zrtáty tekutin (průjem, zvracení, pocení), přesun tekutiny do intersticia

(pankreatitida, rozsáhlé trauma), popáleniny, Addisonská krize- hypotenze při ztrátě 20 % krve- klesá rozdíl systolického a diastolického tlakuDg – dehydratace (↑ urey a kreatininu a hematokritu), nízký centrální žilní tlak

Distribuční šok- generalizovaná vazodilatace- septický šok - při G- sepsi, DIC, hypotenze s teplými růžovými končetinami, příznaky sepse (horečka, zimnice,

třesavka)- anafylaktický šok – může být navozena ještě bronchokonstrikce s expirační dušností/ edémem horních cest

dýchacích a inspirační dušností- neurogenní šok – ztráta periferního cévního tonu díky úrazu míchy

3 fáze šoku:1. stadium kompenzované hypotenze

- pokles výdeje, hypotenze, ↓snížení kontraktility- kompenzace: sympatoadrenální sys → tachykardie, redistribuce oběhu s konstrikcí v splanchniku,

ledvinách, svalech a kůži; selektivní dilatace v koronárním a mozkovém řečišti; pokud je konstrikce dostatečná tak dočasně tlak udrží normální (za cenu perif.ischemie a acidózy); je plně reverzibilní

- aktivace renin-angiotenzin-aldosteronu, vazopresinu2. stadium dekompenzovane hypotenze

- prohlubuje se hypotenze a klesá i perfuze životně důležitých orgánů (srdce, mozek)- hypoperfuze ledvin → anurie, CNS → somnolence až koma, srdce → ↓ výdej a prohlubuje tak

hypotenze- stále je ještě reverzibilní

3. ireverzibilní fáze šoku – vede neodvratně k smrtiOrgánové změny při šoku

- makro a mikrocirkulace – centralizace, vazokonstrikce pre i postkapilární → působením kys. metabolitů dojde k dilataci arteriol → zvýší se tlak v kapiláře → transsudace, únik tekutiny z řečiště → další prohloubení acidózy a hypoperfuze

- krev – ztráta tekutin → hemokoncentrace - obstrukce řečiště a zpomalení proudu

102

Page 103: lf1.czlf1.cz/wp-content/uploads/228-vypracovane_otazky_interna... · Web viewprognoza – 50 % dosáhne kompletního uzdravení do 6 měsíců, 20 % má prodloužený průběh –

Vypracované otázky na INTERNU made by Edita Homolková, Monika Ambroziová, Adam Král, Petr Vermach a Vítek Křehnáč

- plíce – hypoxie, porucha mikrocirkulace, ↓ perfuze, arteriovenózní zkraty, tyto funkční a morfologické změny vedou k selhání plic = šoková plíce

- PaCO2 klesá a PaO2 se nemění- ledviny - ↓ perfuze,↓ glom. filtrace → oligo až anurie dočasně udržují intravaskulární objem- střevo – edém, krvácení, průnik bakterií → pseudomembrány, průnik endotoxinů do oběhu- játra – poškození až po dlouhé ischemii- hemokoagulace – může vést k DIC- acidobazická rovnováha - anaerobní metabolismus → metabolická acidóza

Th – udržení středního arteriálního tlaku na 75-80 mm Hg (minimálně 65 mm Hg)- poloha – horizontální s mírnou elevací DK- zabránění podchlazení - analgosedace – i.v., ze začátku krátkodobý benzodiazepin

- ne ↓ dýchání a tk!!! (morfin)- oxygenace/ ventilace – snížení dechové práce- expanze volumu – před podáním ionotropika

- krystaloidy – izotonické – fýzák, v cirkulaci zůstane jen ¼ volumu (zbytek se přesune do intersticia, 500-1000 ml při zahájení léčby

- hyperosmolární – přesun tekutin z intersticia do oběhu, při přednemocniční léčbě

- koloidy – albumin, polysacharidy, hydroxyetylenškroby…- volumexpandéry

- krevní transfuze, mražená plasma – krvácení, krevní zráty- ionotropika - ↑ srdeční výdej a ↑ periferní cévní rezistenci

- noradrenalin - dopamin – při relativní bradykardii- dobutamin – při kariogenním šoku- milrinom (inhibitor fosfodiesterázy)- kontinuální podávání infuzí/ pumpou!

- léčba acidózy – infúze 8,4 % Na HCO3

- BE (base exces) * 0,3 * hmotnost- kortikoidy – anafylaktický šok- antihistaminika – anafylaxe- preventivní heparinizace – proti trombembolismu a DIC, kontinuálně 10 000j/ 24 hod- glykemie – 7-8 mmol/l- energie – parenterálně

57.A CHRONICKÉ KOMPLIKACE DIABETU

- úmrtnost 2x vyšší než u nediabetiků- neezymová glykace → AGEs → rec na endotelu a makrofázích → zánět- oxidační stresMIKROVASKULÁRNÍ KOMPLIKACE

- prekapiláry + kapiláry + postkapiláry → ztluštění BM degenerace pericytů, hypertrofie endotelových bb. (z hypoxie)

- u DM I v prvních 5-10 letech relativně vzácný, pak strmě roste- u DM II často přítomny už v době dgDiabetická nefropatie – iniciální glomerulární hyperfiltrace (často bez kliniky a laboratorních nálezů),

mikroalbuminurie (30-300 mg/ 24 h) až proteinurie, pak ↓ glomerulární filtrace až renální selhání - návštěva nefrologa při sérovém kreatininu 200 μmol/l, při atypickém/ rychlém zhoršování fcí- roste výskyt sekundární arteriální hypertenzeTh – dodržení KI metforminu a dalších, přestup na inzulin

- ACEI, sartany (blokátory AT1), cílový tlak 130/85 mm Hg- statiny- nekuřáctví, redukce hmotnosti

Diabetická retinopatie – nejčastější příčina slepoty ve vyspělých zemích, až u 70 % DM I a 25 % DM II

103

Page 104: lf1.czlf1.cz/wp-content/uploads/228-vypracovane_otazky_interna... · Web viewprognoza – 50 % dosáhne kompletního uzdravení do 6 měsíců, 20 % má prodloužený průběh –

Vypracované otázky na INTERNU made by Edita Homolková, Monika Ambroziová, Adam Král, Petr Vermach a Vítek Křehnáč

- ↑ permeabilita kapilár → depozita lipidů, neovaskularizace sítnice, retrovitreání a intravitreální krvácení, trakční odchlípení sítnice

- neproliferativní DR - dilatace kapilár, mikroaneurysmata, uzávěry, krvácení- proliferativní DR – nekvalitní novotvorba kapilár, krvácení, fibrotizací neovaskularizované tkáně

dochází k trakčnímu odchlípení- diabetická makulopatie – fokální/ difúzní/ ischemický/ smíšený edémTh – laserová fotokoagulace sítnice

Diabetická neuropatie – metabolické a mikroangiopatické vlivy (ischemie díky postižení vasa nervorum)- periferní – symetrická senzitivně motorická polyneuropatie – nejčastější, bolesti/ abnormální pocity

v končetinách nebo ↓ citlivosti na bolest, teplo, chlad a vibrace- bolesti distální části DK hlavně v klidu a teple

- asymetrická – postižení jen 1 nebo několika nervů- úžinové syndromy, diabetická amyotrofie (postihuje pánevní pletenec, 4hlavý sval stehenní a psoaty, akutní nástup, bolest, nápadná atrofie stehenního svalstva a neschopnost chůze ze schodů, etio nejasná, často spontánně mizí)

- autonomní – u 20-40 %- nevolnost, zpomalené vyprazdňování žaludku, pocit plnosti, ↓ motilita střeva se

zácpou, nebo naopak průjmy, poruchy pocení, ↑ TF, poruchy erekce, poruchy adaptace TK

- EMG, vyšetření monofilamentem (kožní citlivost), dotek tupým/ ostrým předmětemTh – odstranění bolesti a parestezií 3cylkickými antidepresivy

- gabapentin, dulotexin, kys. α-lipoová (thiokrová)Diabetická noha – povrchové i hluboké defekty, ischemická gangréna nebo flegmóna, deformity skeletu,

osteomyelitida, Charcotova osteoartropatie- díky ischemii, neuropatii, nasedající infekci, ↓ kapilární perfúzi a hypoxii- porucha inervace interoseálních svalů → zborcení klembyTh – péče o nohy (odstranění hyperkeratóz, ošetření drobných poranění, vhodná obuv…)

- sledovat kotníkové tlaky a index ABI, angiografieMAKROANGIOPATIE

- KV riziko 4x vyšší než ostatní populace- v podstatě akcelerovaná ateroskleróza

57. B URGENTNÍ STAVY V PNEUMOLOGII

Hemoptýza – ot 62.BPneumothorax – ot 68.B

- nahromadění Aspirace cizího tělesa

- častá u dětí a starých lidí -normálně jsou DCD chráněny kašlacím a laryngeálním reflexem- k aspiraci dochází často v bezvědomí, při poruchách polykacího aktu, u neurologických potížích, v opilosti či při

poruchách jícnu- kromě cizích těles lze aspirovat žaludeční obsah (Mendelsohnův syndrom), krev, hnis či tekutinyKO - závisí na velikosti aspirovaného tělesa a jeho lokalizaci – je-li nad hlasivkovou štěrbinou vzniká dramatický

stav s rizikem udušení, při zachycení tělesa v dolním lobárním bronchu mohou být symptomy nevýrazné (záchvatovitý kašel, může být stridor, afonie, dušnost, laryngospazmus, hemoptýza)

- aspirace bývá zaměněna za astmatický záchvat či pneumonii (20 – 30% aspirací není včas diagnostikováno)Dg - anamnéza x klinický obraz x RTG hrudníku (RTG kontrastní předměty, ložisková hyperinflace, atelektáza,

posuny mediastina) a bronchoskopie (dg + odstranění tělesa)- po delší době dochází ke vzniku granulační tkáně (může pak těleso zakrývat), za stenózou vyvolanou cizím

tělesem se tvoří opakované záněty, které mohou vyústit v karnifikaci plic (indikace k plicní resekci)Laryngospazmus

- fyziologický ochranný reflex bránící aspiraci

104

Page 105: lf1.czlf1.cz/wp-content/uploads/228-vypracovane_otazky_interna... · Web viewprognoza – 50 % dosáhne kompletního uzdravení do 6 měsíců, 20 % má prodloužený průběh –

Vypracované otázky na INTERNU made by Edita Homolková, Monika Ambroziová, Adam Král, Petr Vermach a Vítek Křehnáč

-při podráždění dochází ke kontrakci konstriktorů laryngu, někdy dochází k patologickému vystupňovanému laryngospazmu

KO - nemožnost inspiria, zástava dechu a panický strach (postižený vnímá záchvat jako nekonečně dlouhý), spontánně odeznívá během desítek sekund

- laryngospazmus lze zrušit jiným reflexem (úder na hrudník, povytažení kořene jazyka, polknutí studené tekutiny apod.)

Th - u neurotiků indikujeme benzodiazepiny, při zánětlivé etiologii podáváme ATBStatus asthmaticus

- protrahovaná dušnost trvající několik hodin nebo nereagující na léčbu- je to život ohrožující stav, který vyžaduje okamžité podání glukokortikoidů i.v. (možno i subcutánně

s adrenalinem) + β2 mimetika v nebulizaci + oxygenoterapie a dostatečná hydrataceNekardiální plicní edém

- ARDS - ot. 2.A

58.B VARIXY DOLNÍCH KONČETIN, CHRONICKÁ ŽILNÍ INSUFICIENCE

Varix = povrchově uložená dilatovaná a elongovaná žíla- primární – spontánně vziklý díky vzrozené méněcennosti žilní stěny

- deficit kolagenu, hydrostatický tlak (stání, obezita…), hormonální vlivy (těhotenství, HAK), věk (degenerace)

- sekundární – při narušení toku v hlubokém žilním systému (flebotrombózy, insuficience žilních chlopní)- intradermální (metličky, do 1 mm)/ retikulární (intradermální, do 4 mm)/ kmenové (subkutánní, postižení v.

safena magna nebo parva)Chronická žilní insuficience

= klinický stav plynoucí z poruchy žilního návratu z končetin, je pro něj charakteristická žilní hypertenzeKO – kosmetické problémy

- bolest, pocit tíhy/ tenze, pocit únavy, noční křeče, pálení/ svědění kůžeKomplikace – konžní změny - purpura, edém, hyperpigmentace, ekzém, až bércový vřed

- varikoflebitida, krvácení z varixůDg – vyšetření ve stoje – teplota, barva, turgor, ochlupení, otoky…

-duplexní sono Th – chirurgicky odstranit/ skleroterapie

- venotonika – koňský kaštan- komprese, pohyb

59.B FIBROTIZUJÍCÍ PLICNÍ PROCESY

- Idiopatické intersticiální pneumoniea) Idiopatická plicní fibróza/kryptogenní fibrotizující alveolitida b) Deskvamativní intersticiální pneumoniec) Akutní intersticiální pneumonied) Lymfocytární intersticiální pneumonie e) Nespecifická intersticiální pneumonief) Kryptogenní organizující pneumonie (obliterující bronchiolitida)g) Familiární plicní fibróza

IPF- specifická forma chronického fibrotizujícího intersticiálního procesu nejasné etio s histologickým obrazem

intersticiální pneumonie-přežití 2-3 rokyEtio - není známá, jedná se pravděpodobně o uniformní patologickou odpověď organismu na infekční a neinfekční

agens, která způsobují poškození výstelky alveolů vedoucí k progredujícímu jizvení (asi porucha epitelizace)KO - klidová posléze námahová dušnost, únava a kašel, při těžší hypoxemii i cyanóza

- u většiny pacientů se objevují paličkovité prsty s nehty tvaru hodinového sklíčka - poslechově nalézáme krepitus nad bazemi plic

105

Page 106: lf1.czlf1.cz/wp-content/uploads/228-vypracovane_otazky_interna... · Web viewprognoza – 50 % dosáhne kompletního uzdravení do 6 měsíců, 20 % má prodloužený průběh –

Vypracované otázky na INTERNU made by Edita Homolková, Monika Ambroziová, Adam Král, Petr Vermach a Vítek Křehnáč

- typický je pomalý průběh, ale mohou se vyskytnout epizody akutní exacerbace – klinické zhoršení, pokles plicních funkcí a radiologický obraz „mléčného skla“ - alveolitida

Dg - RTG – zmnožená plicní kresba (až retikulace)- HRCT – fibróza + obraz voštinovité plíce (hlavně baze plic)- fční vyšetření plic – restriktivní porucha ventilace + porucha plicní poddajnosti, typický nález je hypoxemie

(námahová a později klidová) a snížený transfer faktor pro CO- BAL – metoda pomocná (diferenciální diagnostika), typické je zmnožení granulocytů- chirurgická biopsie plic- HRCT + biopsie plic – základní diagnostika IPF

Th – málo účinné cokoli (kortikosteroidy, imunosupresiva, cytostatika), N-acetylcistein, inhibitory TGFβ a PDGF- transplantace plic

Deskvamativní intersticiální pneumonitida- vyskytuje se u mužů ve třetí a čtvrté dekádě, 2x vice ukuřákůKO - dušnost, suchý kašel, paličkovité prsty, variabilně krepitus, restrikční porucha ventilaceDg - BAL větší příměs lymfocytů, - lepší prognóza než IPF- přežití 70 % 10 let

Respiratorybronchiolitis- associatedinterstitiallungdisease- RB-ILD- vyskytuje se u mužů ve třetí a čtvrté dekádě, 2x vice u kuřáků,KO - dušnost, kašel, obstrukční nebo restrikční porucha ventilace- AM s kuřáckými inkluzemi v BAL- přežití variabilní

Akutní intersticiální pneumonie- začíná náhlesystémovými příznaky, restrikční poruchou ventilace, poruchou výměny krevních plynů s

následkem časné hypoxemieRespirační selhání typu ARDS

- BAL – průkaz granulocytů, občas lymfocytů- přežití dny navzdory UPV, pokud po kortikosteroidech regreduje, nezanechává fibrózu

Lymfocytární intersticiální pneumonie- infiltrace plicní tkáně lymfocyty (možná souvislost s lymfomy)- sdružuje se s některými autoimunitními nemocemi - typicky Sjogrenův syndromu, neinfekční manifestace HIV

infekceDg - klinický obraz, HRCT obraz mlhovitých opacit a centrilobulárních nodulů se ztluštěním bronchovaskulárních

svazků a interlobulárních sept a s obrazem drobných cyst, chirurgická plicní biopsie Th - systémově kortikoidy

Nespecifická intersticiální pneumonieKlasifikace - Buněčná NSIP

- Fibrotická NSIP - Smíšená NSIP

- fibrotická NSIP má nejhorší prognózu, podtyp buněčný a smíšený mají prognózu lepší – lze ovlivnit protizánětlivou léčbou

- NSIP může být idiopatická, většinou je sdružená se systémovými nemocemi (revmatoidní artritida, systémová sklerodermie, Sjogrenův syndrom), polékovým postižením, imunopatologickými stavy, nebo s exogenní alergickou alveolitidou

Exogenní alergická alveolitida-difúzní granulomatózní proces rozvíjející se po opakované expozici antigenům i disponovaného jedince- ptáci, houbym termofilní bakterie, nízkomolekulární sloučeninyKO – akutní – teploty, třesavka, slanost, dušnost, tlak na hrudi, suchý kašel, na rtg obraz mléčného skla

- vymizí do 48 h- subakutní a chronická – dlouhodobá nizká koncentrace antigenu, progrese námahou dušnosti, slabost,

nechutenství, hubnutí, na rtg retikulonodulace až voština→ plicní fibróza → pravostranné srdeční selhání- restrikční ventilační porucha,

- bilaterální krepitus při bazích, tachypnoe, paličkovité prsty, IgG proti antigenu, plicní biopsieTh - korikosteroidy

106

Page 107: lf1.czlf1.cz/wp-content/uploads/228-vypracovane_otazky_interna... · Web viewprognoza – 50 % dosáhne kompletního uzdravení do 6 měsíců, 20 % má prodloužený průběh –

Vypracované otázky na INTERNU made by Edita Homolková, Monika Ambroziová, Adam Král, Petr Vermach a Vítek Křehnáč

60.A PLAZMOCYTOM

= mnohočetný myelom, Kahlerova nemoc- 4/ 100 000 ob., starší osoby (50 - 60 let), častěji muži- etio?, snad vliv chron. infekce, ionizující záření, chemikálie, genetické aberace (translokace v místě kódu pro těžké

řetězce, deregulace cyklinu D), mikroprostředí (IL6, TNFα, VEGF, adhezivní molekuly)- maligní proliferace z B-lymfocytů na úrovni jeho diferenciace v plazmatickou b.- maligně transformovaný plazmocyt = základ buněčného klonu, produkuje molekuly Ig identických fyzikálních,

chem., biolog. vlastností (monoklonální gamapatie)- homing = usídlení myelomových bb. v k. dřeni (adhezní molekuly), určují klin. Obraz

- produkce řady cytokinů → ↑aktivita osteoklastů = osteolyt. ložiska skeletu (RANK/ RANKL/ OPG/ MIP-1)- ↓ konc. funkčních Ig → závažné infekceKO - CRAB = hyperkalcémie, renální poškození, anémie, kostní poškození

- indoletní myelomy - v počátku bez obtíží, u 70% dominuje bolest v zádech a žebrech, napodobují „ revmatická“ onem., lumbago, neuralgie, na rozdíl od bolestí při kostních meta (bolí častěji v noci) je bolest vázaná na tělesnou aktivitu, spontánní fraktury kostí

- lehké řetězce se vylučují ledvinami → poškození (přímá tubulární toxicita, tvorba válců, vychytávání paraproteinu a tvorba depozit) → atrofie a degenerace tubulů → myelomová ledvina

- hyperviskózní syndrom - bolest hlavy, poruchy vizu- selhání ledvin při myelomové ledvině, (urychluje dehydratace, hyperkalcémie - deprese, letargie, slabost,

hyperurikémie)- malátnost, hubnutí, dušnost při prův. anémii- klasická trias (dřeňová plazmocytóza + osteolyt. Ložiska + paraprotein sérum/ moč)- anémie – normocytární, normochromní, z útlaku- opakované infekce- koagulační poruchy

Dg - ↑↑ FW, imunoelfo moči a séra - paraprotein, v moči Bence-Jonesova bílkovina (lehké řetězce), ALP, albumin, celková bílkovina

- skiagrafie skeletu - difúzní osteolytická ložiska (možná neurologická symptomatologie z útlaku páteře)- sternální punkce - >10% atyp. plazmocytů- stanovení Ca2+ v séru (hyperkalcémie), kys. močové (hyperurikémie), β2-mikroglobulin↑- u nesekretorického MM doporučeno stanovení volných řetězců v séru (citlivější)- poměr γ:κ

Th – chemoterapie: cyklofosfamid, dexamethason, melphalan, lenalidomid (derivát thalidomidu, inhibice TNFα), - autologní transplantace k. dřeně- radioterapie - u bolestivých kostních ložisek- bisfosfonáty (pamidronát), denosumab (protilátka proti RANK ligandu)- hydratace, plasmaferéza, trans. erymasy, EPO- nízkomolekulární heparin- parciální remise = ↓ paraproteinu o 50 %

Prognóza - přežití 6 -7 let, relaxujeKlasifikace – stadium I – max. 1 osteolytické ložisko, norma Ca2+,

- stadium II – mezi I a II- stadium III – bla bla- doutnající myelom – paraprotein větší než u MGUS, bez symptomatologie- nesekretorický myelom – jako MM, bez paraproteinu- plazmocelulární leukemie – plazmatické bb vyplaveny do krve- solitární kostní plazmocytom a extracelulární plazmocytom – bez CRAB

60.B KOMPLIKACE INFARKTU MYOKARDU

Mechanické- většinou u STEMI- jizva – hypokineze až akineze - stunning = omráčený myokard – porucha kinetiky několik dní i po otevření tepny

107

Page 108: lf1.czlf1.cz/wp-content/uploads/228-vypracovane_otazky_interna... · Web viewprognoza – 50 % dosáhne kompletního uzdravení do 6 měsíců, 20 % má prodloužený průběh –

Vypracované otázky na INTERNU made by Edita Homolková, Monika Ambroziová, Adam Král, Petr Vermach a Vítek Křehnáč

- aneuryzma – u transmurálních lézí, vyklenování v důsledku tlaku krve v LK- ruptura stěny – mitrální insuficience při ruptuře papilárního svalu a rozvoj plicního edému

- interventrikulárního septa – vede k selhání pravé komory, holosystolický šelest v prekordiu- volné stěny – tamponáda, hemoperikard

Perikardiální výpotek a perikarditida- časná poinfarktová perikarditída, pozdní poinfarktová perikarditída- častěji u rozsáhlých infarktů přední stěnyTh - kys. acetylsalicylová 500 mg á 8 h

Arytmie- poruchy převodního sys. - AV blokády II. a III. stupně hl. u infarktu spodní stěny

- atropin + dočasná kardiostimulace- fibrilace síní – někdy hemodynamicky významná (hypotenze), vyšší riziko náhlé smrti z maligní arytmie

(fibrilace komor a komorová tachykardie - amiodaron)- kardiogenní šok - vysoký plnící tlak levé komory až 20 mm Hg, nízký srdeční index, hypotenze, porucha perfúze

orgánů, T: dopamin 5-20 ug/ kg/ min nebo noradrenalin 2-10 ug/ kg/ min

61.A OSTEOPORÓZA – viz 52.A

61.B AKUTNÍ ZÁNĚTY ŽLUČNÍKU A ŽLUČOVÝCH CEST

Akutní cholecystitida– jako komplikace blokokády d. cholecysticus (retrográdní infekce a mechanické a chemické vlivy), zánětlivá

náhlá břišní příhoda - septický stav, lokální dráždění peritonea, teploty, silná a stálá bolest pod pravými žebry, bolest v ětšinou trvá

déle než hodinu- Murphyho příznak - při hluboké palpaci žlučníku v pravém hypochondriu při současném hlubokém nádechu

pacient náhle pro bolest nádech zastavíDg - RTG břicha (konkrementy), UZ (ztluštění stěny, konkrementy), krevní obraz (leukocytóza)Th - hospitalizace + konzultace s chirurgem, klid na lůžku, analgetika (při mírné bolesti diclofenac/pethidin, při

silné morfin + atropin), antibiotika (cefuroxim, při těžkém průběhu + metronidazol), doplnění tekutin- operace (do 5 dní od příznaků) - při progresi onemocnění bez reakce na léčbu, empyém, perforace- chirurgická - cholecystektomie s antibiotickým krytím před i po výkonu, PTCholecystectomia

- u těžkých stavů možná PTcholecystostomie- cholecystektomii můžeme provést buď okamžitě (do 72h) nebo odloženě (za 2-3 měsíce)

- konzervativní - klid, hladovka, tekutiny parenterálně, při závažném průběhu parenterální výživa, ledovat podžebří, proti bolesti buscopan i.v. (při neúspěchu pethidin (Dolsin))

- na chronickou žlučníkovou dietu přecházíme velmi pomalu- ATB indikace (amoxicilin, gentamicin, při podezření na anaeroby kombinovat s

metronidazolem)- lehké případy bez nutnosti chir. léčby, u těžkých forem se septickými komplikacemi, jako

předoperační přípravaAkutní cholangitida

- úpná/ částečná obstrukce (lithos, zánět, nádorová masa)- infekce postihuje i intrahepatální žlučovody (a i může přjít na parenchym)- nejčastěji G- (E. coli, klebsiela, haemophilus influ.)KO – Charcotova trias (horečka, interus, biliární bolest), známky sepse (splenomegalie), hepatomegalieTh – urentní ERCP, atb

62.A NÁDORY LEDVIN

KO – bolest v mezogastriu, hmatný tumor, makroskopická hematurieDg - USG, CT (propagace do renálních žil?), MRBenigní - Angiomyolipom - benigní, nemetastazuje, často oboustranný

- přítomnost zralých tukových bb, svaloviny a cév- může dojít k jeho ruptuře s krvácením do retroperitonea

108

Page 109: lf1.czlf1.cz/wp-content/uploads/228-vypracovane_otazky_interna... · Web viewprognoza – 50 % dosáhne kompletního uzdravení do 6 měsíců, 20 % má prodloužený průběh –

Vypracované otázky na INTERNU made by Edita Homolková, Monika Ambroziová, Adam Král, Petr Vermach a Vítek Křehnáč

- Tumor z juxtaglomerulárních bb – produkuje reninKO – signifikantní hypertenze a hypokalémie (sekundární hyperaldosteronismus)

- fibrom, hemangiom, leiomyomMaligní – Grawitzův tu, Adenokarcinom - vychází z prox. tubulu, častěji u AD polycystózy, podkovovité ledviny,

Hippel Lindaua, abúzus nikotinuKO – triáda: bolest (lumbalgie) + makrohematurie + hmatný tumor, teploty, anémie,

hypertenze, metastázy do plic a kostíDg – sono, CT, kostní scintigrafieTh - primárně chirurgická (radikální nefrektomie = odstranění ledviny, nadledviny,

tukového pouzdra, Gerotovy fascie), při meta do skeletu paliativní radioterapie - Wilsonůt tu (nefroblastom) - jeden z nejčastějších tumorů dětského věku, hlavně u dětí do 4 let

KO - hmatný tumor s vyklenutím břicha, bolest, hypertenze, úbytek hmotnosti, nechutenství, metastazuje do plic a jater

Dg – palpace - šetrná a jen jedenkrátTh – chirurgická transperitoneální nefrektomie s retroperitoneální lymfadenektomií + chemo

- Uroteliální – na přechodu epitelu moč. cest/ měchýře/ pánvičky/ ureteru- kůřáci, průmyslové toxiny, abúsus analgetik- hematurie- ca pánvičky a ureteru, ca močového měchýře

- Metastázy – vzácné, a spíš klinicky němé- ca plic (50 %), ca žaludku, ca prsu, lymfomy…

62. B HEMOPTÝZA

= vykašlávání krve – větší nebo menší příměs krve ve sputu, jejím zdrojem je nutritivní oběh plic (aa. bronchiales a její větve)- vykašlávaná krev je zpěněná a jasně červená (při masivním krvácení hrozí udušení)Etio - zánětlivé i nádorové plicní procesy, hemoragické diatézy, mitrální stenóza, vaskulitidy apod. KO - úzkost, neklid, může se objevit dušnost

- objektivně můžeme slyšet chropy středních nebo velkých bublinDg - RTG hrudníku

- vyšetření hemokoagulace (trombocyty, aPTT, INR) - bronchoskopie- při podezření na PE provedeme perfuzní sken plic- není –li uvedenými metodami vysvětlena příčina lze indikovat bronchiální angiografie

Th - závisí na vyvolávající příčině- doporučuje se klid na lůžku v polosedě + led na hrudník + podání antitusik, anxiolytik, mražené plazmy, ATB

a hemostyptik (kanavit, ascorutin, PAMBA)- při bronchoskopii lze vyjmout cizí těleso, ošetřit krvácející místo či provést tamponádu bronchu/ přechodný

uzávěr pomocí balonkového katetru- bronchoskopii indikujeme při středně těžké hemoptýze, u masivní hemoptýzy volíme konzervativní terapii- někdy je indikována plicní resekce nebo arteficiální embolizace (nutné vyloučit existenci spojek bronchiálních

a spinálních arterií)- masivní hemoptýza - zajištění ventilace O2 - odsání krve z DC, selektivní endotracheální intubace k ochránění

zdravé plíce- zajištění cirkulace - zajištění i.v. vstupu, infúze fyziologického roztoku, monitorace

tlaku, pulsu, saturace- konzílium hrudního chirurga

63.A INFEKCE MOČOVÝCH CEST A AKUTNÍ PYELONEFRITIDA

Akutní pyelonefritida – ot. 33.B

Infekce močových cest Nekomplikované – u mladých negravidních žen

109

Page 110: lf1.czlf1.cz/wp-content/uploads/228-vypracovane_otazky_interna... · Web viewprognoza – 50 % dosáhne kompletního uzdravení do 6 měsíců, 20 % má prodloužený průběh –

Vypracované otázky na INTERNU made by Edita Homolková, Monika Ambroziová, Adam Král, Petr Vermach a Vítek Křehnáč

- mikroorg. Endogenního původu (fekální flóra), ascendentně/ hematogenně- 50-60 % žen alespoň 1 v životě- E. coli, St. Saprophyticus, Klebsiella, proteus

Komplikované – u oslabených a imunodeficitních, močový katért, u těhotných, DMSignifikantní bakteriurie – přítomnost >105 bakterií/ ml → 80 % pravděpodobnost přítomnosti bakteríí

v měchýřiLeukocyturie – při bakteriurii (pokud ne → agens nebylo zachyceno kultivací)

- sterilní leukocyturie – při intersticiálních nefritidách, analgetická nefropatie, někdy i u aktivních GNAsymptomatická bakteriurie – u 5 % žen a 0,1 % mužů

- th není nutná, spontánní vymizeníInfekce dolních cest močových – akutní cystitida/ uretritida

- dysurie, pálení, nucení na močení, bolest nad symfýzou, pokud je zvšená teplota i infekce horních cest

Uretrální syndrom - opakované dysurie, častější nucení na moč, nesignifikantní bakteriologický nález v močiBakteriální prostatitida - akutní – bolest v podbřišku, febrilie, obtížná mikce, per rektum bolestívá a zvětšená

- chronická – podobné, recidivy infekce moč. cest, krev ve spermatuTh - nekomplikovaná forma (kotrimoxazol, fluorochinolony 3-5 dní), ↑příjem tekutin k udržení diurézy nad

2000ml- komplikovaná rekurentní (reinfekce - jako nekomplikovaná, relaps - profylaktická několikaměsíční ter.

chemoterapeutiky na noc, kombinovaná s udržením vysoké diurézy přes den)

63.B METABOLICKÉ NEMOCI JATER (NEALKOHOLICKÉ STEATOHEPATITIDA, PORFYRIE, WILSONOVA CHOROBA, HEMOCHROMATÓZA)

Nealkoholická steatóza jater, nealkoholická steatohepatitida (NSAH)- akumulace tuků v hepatocytecj přesahující 5-10 % hmotnosti jater, NSAH je pokročilou fází steatózy- NASH může progredovat do cirhózyEtio – obezita, metabolický syndrom. Inzulonová rezistenceKO – většinou bez obtížíDg - ↑ AST a ALT, dobře na SONO, k přesnému určení biopsieTh – změna životosprávy, snížení hmotnosti

Wilsonova choroba- AR onemocnění vedoucí ke kumulaci Cu ve tkáních (mozek, játra), prevalence 1:30 000Etio - dědičné onemocnění způsobené deficitem měď transportující ATPázy (ATP 7B) vedoucí ke kumulaci Cu –

- kumulace Cu v orgánech vede k nadbytku volných radikálů a k poklesu redukovaného glutathionu → peroxidace membránových lipidů, poškození proteinů bohatých na SH skupiny a poškození DNA

KO – většinou se projeví do 15 let, asymptomatičtí mají jen ↑ aminotransferázy- postupně se vyvijí fibróza a cirhóza- neurologické postižení se projevuje nejčastěji poruchami motoriky- Kayserův-Fliescherův prstenec – hnědozelená pigmentace periferie rohovky- hemolýza v 15 % (hl. u fulminantní formy), portální hypertenze, poruchy hormonální/ růstu, renální

acidóza…- jaterní onemocnění se může manifestovat v pěti klinických formách: akutní hepatitida/ jaterní steatóza/

fulminantní jaterní selhání/ chronická aktivní hepatitida/ jaterní cirhóza (nejčastější forma manifestace)Dg - základním screeningovým vyšetřením je stanovení hladiny ceruloplasminu v séru – je ↓ a ↑ volné Cu v séru

- měď v moči – konstantní nález u symptomatické formy WD, vylučování se zvýší po podání penicilaminu- měď v játrech – je nejpřesnějším diagnostickým parametrem a stanoví diagnózu WD, Cu v sušině- Kayser – Fleischerův prstenec je typickým příznakem Wilsonovy choroby

Th - omezení příjmu Cu (mořské ryby, kakao, ořechy)- penicilamin zvýší vylučování do moči, pyridoxin, zinek (snižuje resorpci ve střevě)- transplantace jater je jedinou možnou formou léčení fulminantní formy Wilsonovy choroby, kde má

vynikající výsledky. Bez transplantace jater končí tato forma Wilsonovy choroby smrtí nemocnéhoHemochromatóza

- AR dědičná choroba s nadměrným vstřebáváním Fe- prevalence 2-5/ 1000, výrazněji postiženi muži

110

Page 111: lf1.czlf1.cz/wp-content/uploads/228-vypracovane_otazky_interna... · Web viewprognoza – 50 % dosáhne kompletního uzdravení do 6 měsíců, 20 % má prodloužený průběh –

Vypracované otázky na INTERNU made by Edita Homolková, Monika Ambroziová, Adam Král, Petr Vermach a Vítek Křehnáč

Etio – mechanismus nejasný, ↑ (2x) vstřebávání Fe ve střevě → poškození hepatocytů, cholangiocytu, acinárních a B bb. pankreatu, kardiomyocytů, gonadotropních bb hypofýzy a epitelií kloubů

KO – většinou projevy po 40. Roce- hepatomegalie, jaterní cirhóza, abnormální pigmentace kůže, bolest kloubů, kardiomegaliie, inzulinová

rezistence a DM, impotence, amenorea, svalová slabost, často hepatocelulární caDg - ↑ aminotransferáz a protrombinový čas, ↑ saturace transferinu a ↑ ferritinu, hodně Fe v jaterní sušiněTh – venesekce (vyplavení Fe a normalizace hladiny ferritinu)

- cheláty- kompenzace DM, abstinence

- prognóza neléčené hemochromatózy špatná (2 roky po objevení komplikací), jinak okPorfyrie

- jsou vrozené nebo získané choroby způsobené poruchou v syntéze hemu s hromaděním porfyrinů nebo jejich prekurzorů v tkáních

- erytropoetický pool hemu pro výrobu hemoglobinu a hepatální pool pro tvorbu enzymůErytropoetické - kongenitální erytropoetická porfyrie – AR, velmi vzácná, defekt uroporfirinogen III kosyntetázy

→ ulkádání a vylučování uroporfyrinu IKO – exkrece tmavě červené moči, těžká fotosenzitivita kůže (puchýřky různé

velikosti, které praskají a infikují se → krusty → jizvy, mutilace), hepatosplenomegalie, dohněda zbarvené zuby, hemolytická anémie, splenomegalie

Dg - ↑ uroporfirinu I v moči, stolice fluoreskuje v UV světle kvůli koproporfirinu I, (fluoreskují i zuby, sérum a erytrocyty)

Th – absolutní ochrana před sluncem, transplantace kostní dřeně- erytropoetická protoporfyrie – AD, ↓ aktivita ferrochelatázy, úprotoporfyrin se hromadí v ery,

játrech a kůžiKO – kožní fotosenzitivita (při expozici UV → pálení, svědění, urtikárie, bez

mutilace, jaterní příznaky (↑ konjug. bilirubinu, ↑ aminotransferáz, ↑ AF), žlučové kontrementy

Dg - průkaz protoporfyrinu v erytrocytechTh – kožní příznaky léčíme β-karotenem, u jaterních cholestyramin a

antioxidans vit. EHepatální - akutní intermitentní porfyrie AIP – AD, nadprodukce porfobilinogenu (PBG) a ALA

Etio – latentní (nejčastěji do 3. decenia), pak dochází k ↓ jaterních zásob hemu (indukce cytochromů léky, stres, operace…)

KO – typické jsou záchvaty, v období mezi nimi bez obtíží- abdominální symptomy (kolikovitá břišní bolest, zvracení, zácpa)- neurologická symptomy (bolest hlavy, parézy nn., extrapyramidová symptomatologie, mozečkové symptomy, parestezie, svalová slabost, atrofie svalů)- psychiatrická symptomy (změny osobnosti, deprese, dezorientovanost, hysterie)- nespecifické kardiovaskulární projevy (hypertenze, tachykardie…)

Dg. – stanovení ALA a PBG v moči, moč je červenáTh – při záchvatu : strava parenterálně, úprava vodního a iontového metabolismu,

preventivně širokospektrá ATB, chlórpromazin i.v., infúzní podání hemu → rychlý pokles PBG i ALA; mezi záchvaty se doporučuje dieta s ↑ sacharidy jako prevence záchvatů, fenothyaziny, beta-blokátory u hypertenze

- hereditární koproporfyrie – AD, velmi vzácné, defekt koproporfyrinogenoxidázy v mitochondriíchKO – podobně jako ostatní jaterní porfyrie, často asymptomatická

- porfyria variegata (PV) – AD, akutní hepatální porfyrie, nedostatek protoporfyrinogenoxidázyDg. - ↑exkrece protoporfyrinu a koproporfyrinu stolicí, při akutní atace jsou

↑↑ vylučovány močí ALA, PBGKO – kožní postižení jako u PCT se kombinuje s hepatálním, neurologickým a

psychiatrickým o.Th – jako u AIP

- porfyria cutanea tarda (PCT) - AD, nejčastější porfyrie, manifestace po 40. roce, převaha mužů

111

Page 112: lf1.czlf1.cz/wp-content/uploads/228-vypracovane_otazky_interna... · Web viewprognoza – 50 % dosáhne kompletního uzdravení do 6 měsíců, 20 % má prodloužený průběh –

Vypracované otázky na INTERNU made by Edita Homolková, Monika Ambroziová, Adam Král, Petr Vermach a Vítek Křehnáč

- defekt uroporfyrinogen-dekarboxylázy v jaterní tkáni → nadprodukce porfyrinů (skladovány v játrech a uvolňují se do plazmy a jsou vylučovány močí - tmavá), častý hepatocelulární ca

Etio - kombinace vrozeného deficitu se zevními faktory (alkohol, drogy, hormonální antikoncepce, hepatitida C)

KO – fragilita kůže a fotodermatóza s rychlou tvorbou puchýřů (s čirým obsahem) → měkké ulcerace a sekundární infekce, hypertrichózou

- možná mnohočetná ložiska v játrech na SONU, často patolog. jaterní testyDg – jaterní tkáň červeně fluoreskuje, ↑porfyrinu v moči, ↑ALT, ↑AST, ↑GMT,

biopsie jaterTh – venepunkce (1krát za 14 dní 500ml) a léčba malými dávkami antimalarika

chlorochinuSekundární, získané porfyrie - následkem intoxikace organismu hepatotoxickými látkami (olovem, organickými

rozpouštědly, apod.) nebo důsledkem zánětu, který postihuje játra a žlučové cesty. Průběh onemocnění může být akutní nebo chronický.

64.B INFEKČNÍ ENDOKARDITIDA

= mikrobiální zánět endokardu- často postihuje srdeční kalcifikované a vadou postižené cholpně a chlopenní náhrady- vegetace – hmota z trombocytů, fibrinu, mikrobů a leukocytů

- pohybují se s krevním proudem, monou se drolit a embolizovat- může dojít k perforaci cípu chlopně, vzniku paravalvulárním abscesu- dělení na IE na nativních chlopních/ u intravenózních narkomanů/ u chlopenních náhradEtio - α-hemolytické streptokoky (60%), stafylokoky (20%), enterokoky, G-, mykózy, vzácně Coxiella burneti,

chlamydie, mykoplazma, HACEK (H. influenzae, Actinobacillus, Cardiobacterium, Eikenella, Kiingella)KO - horečka, sepse, tachykardie, obecné známky zánětu (slabost, nechutenství, hubnuti, artralgie, pocení)

- srdeční příznaky - nově vzniklý nebo zhoršující se šelest, až známky srd. insuf. při těžším postižení chlopně- kožní příznaky - petechie, Oslerovy uzlíky (imunokomplexová vaskulitida, hlavně na bříškách prstů), paličkové

prsty- septická embolizace (do končetin, mozku, ledvin), postižení ledvin (téměř vždy glomerulonefritida),

splenomegalieDg – A (toxikomani, operace srdce, katetrizace), SP, hemokultura (3 páry pro aerobní + anaerobní), ECHO (zobrazí

vegetace)Th - 4-6 týdnů (2 týdny po skončení horečky), vždy musí být hospitalizován

- ATB + symptomatická (antipyretika, diuretika, antiarytmika)- výhradně baktericidní ATB, vysoké dávky ATB parenterálně, preferujeme kombinace ATB- cílená terapie: streptokoky - Penicilin G + gentamycin

stafylokoky - oxacilin + rifampicin + gentamycinenterokoky - ampicilin + gentamycinkandidy - amfotericin B + flucytosin

Profylaxe - u pacientů s chlopenními vadami se před invazivními zákroky (zubař) podáváme amoxicilin (3 g 1h před a pak 0,5 g 6h po)/klindamycin, při vysokém riziku + gentamycin

65.A CHRONICKÁ MYELOIDNÍ LEUKÉMIE

- nejčastější myeloproliferativní onem., 25% všech leukémií u dospělých - 5. dekáda, častěji muži, vzácná u dětí- etio?, zevní faktory (ioniz. záření + kancerogenní chemikálie)- přítomnost philadelphského chromozomu (v 95 %, Ph chromosom), vzniká reciprokou translokací 9q+, 22q- →

přesun protoonkogenu c-ABL (receptorová tyrosinkináza) z oblasti chromosomu 9 do oblasti chromozomu 22 označované jako BCR (breakpoint cluster region) → fúzní gen BCR-ABL kóduje chimérický protein p210 BCR/ABL

s aberantní tyrosinkinázovou aktivitou (↑ aktivita, autofosforylace, v cytoplasmě) → klonální choroba postihující hematopoet. kmen. b. → intenzivní proliferace leukemického klonu

112

Page 113: lf1.czlf1.cz/wp-content/uploads/228-vypracovane_otazky_interna... · Web viewprognoza – 50 % dosáhne kompletního uzdravení do 6 měsíců, 20 % má prodloužený průběh –

Vypracované otázky na INTERNU made by Edita Homolková, Monika Ambroziová, Adam Král, Petr Vermach a Vítek Křehnáč

- k maligní transformaci dochází u CML na úrovní hemat. kmen. b. → Ph chromozom i v granulocytech, monocytech, prekurzorech ery a trombocytů, i B-lymf

- proliferace patol. klonu vytlačí normální krvetvorbu- přítomnost Ph chrom. → genetická labilita myeloidních bb. → další mutace a vznik nových malig. klonů s větší

proliferační aktivitou → přerůstají původní leukemický klon tzv. chronické fáze CML → přechod to tzv. blastické transformace

KO – 3 fáze - chronická fáze - mírná leukocytóza, min. subj. stesky, později dominuje splenomegalie a postupný přechod do akceler. f.

- akcelerovaná fáze- nárůst odchylky v karyotypu leukemického klonu, vzestup indexu alk. fosfatáz v neutrofilech a bazofílie, přechod dále..

- maligní fáze = blastický zvrat, metamorfóza - klin. i lab. známky AL- blastický zvrat myeloidního typu – 2/3- blastický zvrat lymfoidního typu (méně)- i rozvoj blastického zvratu ve formě extramedulárního postižení (CNS, kosti, kůže),

leukemické tumorózní útvary = chloromy či myelosarkomy- zpočátku nenápadný (jako náhodný nález)- únava, bolesti kloubů a svalů, subfebrilie, hubnutí, tlak v levém podžebří či pocity plnosti v nadbřišku při

výrazné splenomegalii (bolest napětím pouzdra zvětšující se sleziny), při perisplenitis - bolest krutá, propagace do levého ramene

- priapismus, Sweetův syndrom= horečky s bolestivými nodulárními kožními lézemi- splenomegalie - obrovská, dušnost - při anémii, krvácení při trombocytopenii, infekční komplikace

Lab - perif. krev - změny v bílé složce, leukocytóza 50 – 250*109/l, posun doleva, Gumprechtovy stíny (zbytky rozpadlých bílých krvinek resp. jader těchto buněk v krevním nátěru; časté u pacientů s leukemiemi u CLL, ale také u akutních leukemií)

- všechny elementy vývojové řady granulocytů, vyšší % bazofilů- k. dřeň hyperplastická, převaha granulocytární řady, poměr myeloidní řady k červené řadě 10-50:1 (norma je

2-4:1), blastů > 5 %- chron. fáze CML - silný ↓ až chybění alkalické fosfatázy v neutrofilech (index snížen pod 20, norma 20-80)- v akcelerované fázi CML - opětovný vzestup alk. fosfatázy, blastů ve dřeni > 5-20 %- blastický zvrat – ve dřeni blastů > 20 %- cytogenetika → Ph chromozom (FISH)

Th - allogenní transplantace hematopoet. kmen. bb. (jediná kurativní metoda, až po selhání léků)- v počátečním obd. léčby je lékem volby imatinib (Glivec) = kompetitivní inhibitor vazebného místa pro ATP →

blok autofosforylace, 400 mg/den až do ztráty citlivosti, poté nastupuje nový inhibitor dasatinib- v akcelerované fázi -imatinib ve vyšší dávce + cytarabin či hydroxyurea- blastický zvrat se léčí jako AL, ale s horším výsledkem

Prognóza - chron. fáze: 3-4 roky, akcelerovaná f. = 1-1,5 roku, blastický zvrat = 3-6 měsíců, prognóza Ph negativních forem je horší

65.B ZÁCPA A PRŮJEM

Zácpa- subjektivní symptom, nelze definovat (když potřebuje projímadlo)- může být symptomem doprovázející onemocnění GIT, nebo může být fční = habituální a pak je to vlastní

chorobná jednotkaExtraintestinální příčiny – psychosociální a psychické důvody (hl. u malých dětí)

- chronická medikace, psychiatrická onemocnění, endokrinní a systémové ch. (hypotyreóza, hyperparatyreóza), porfyrie, DM

- neurologické (starší, spojeno s dysfagií, zvracením)GIT příčiny – zmenšený intraluminální obsah (málo vlákniny a nody, hladovění, dehydratace)

- porucha motility- intraluminální překážka (stenóza tlustého střeva při ca)- porucha mechaniky defekace (porucha relaxace pánevního dna, ochabnutí stěny rekta, vymizení

rektoanálního reflexu)

113

Page 114: lf1.czlf1.cz/wp-content/uploads/228-vypracovane_otazky_interna... · Web viewprognoza – 50 % dosáhne kompletního uzdravení do 6 měsíců, 20 % má prodloužený průběh –

Vypracované otázky na INTERNU made by Edita Homolková, Monika Ambroziová, Adam Král, Petr Vermach a Vítek Křehnáč

Dg – anamnéza, laboratorní vyloučení metabolických a endokrinologických poruch, koloskopie u lidí nad 50 let

- defekografie, EMG svalů pánve a análních svěračů, anorektální manometrie, irigografieTh – úprava diety (↑ vlákniny a tekutin, péče o stolici po ránu)

- salinická projímadla při pokusu obnovení defekačního reflexu (i parasympatomimetika)Průjem

- řídká (> 75% voda) stolice 3x denně ve zvýšeném množství (> 250 g/den)Etio - infekce - bakterie (E. coli, salmonely, shigely, Campylobacter jejuni, Vibrio cholerae), viry (Norwalk,

rotaviry), protozoa (Entamoeba histolytica, Giardia lamblia)- intoxikace - bakteriální toxiny (S. aureus, B. cereus, Cl. perfringens), jedovaté houby, rtuť, arzén- léky - ATB, laxativa, kolchicin, cytostatika- potravinové alergie- onemocnění spojené s maldigescí - gastrektomie, celiakálnl sprue, deficit laktázy, malabsorbce žlučových

kyselin, insuf. exokrinního pankreatu- nespecifické střevní záněty - m. Crohn, ulcerózní kolitida- GIT nádory- endokrinní příčiny - hypertyreóza, medulární ca štítné žlázy, karcinoid, gastrinom, VlPom- diabetická neuropatie, funkční porucha (dráždivý tračník)

Ptg - osmotický průjem - při hladovění se stav upravuje (deficit laktázy, celiakální sprue)- sekreční průjem - bakteriální toxiny, hormony → ↑ sekrece elektrolytů a vody ze ↑ aktivace cAMP

v enterocytech- exsudativní průjem (zánětlivý) - infekce, NSZ, kolorektální ca, radioterapie, cytostatika- porucha motility - dráždivý tračník, resekce žaludku, vagotomie, diabetická neuropatie

KO - postižení tenkého střeva - vodnaté objemné průjmy, bez příměsi hlenu/ krve, stolice obsahuje nestrávenou potravu

- postižení tlustého střeva - menši objem stolice, často s příměsi hlenu/ krve, časté nucení na stoliciDg - délka trváni, frekvence, konzistence, objem, barva, příměsi, bolesti břicha, vztah k příjmu potravy, léky?,

reakce na hladovění?, pobyt v zahraničí?, operace?, radioterapie?- Lab - vyšetření stolice (krev, leukocyty, tuk, kultivace), základní laboratorní vyšetření (sedimentace, KO,

elektrolyty, kreatinin, sérologie)- kolonoskopie s biopsií

Th - ATB, infekce s těžkým průběhem, příměsi krve, horečkou → perakutně kotrimoxazol/ chinolony, při podezření na pseudomembranózní kolitidu (Cl. dificile) vysadit vyvolávající ATB a podat metronidazol

- rehydratace, úprava vnitřního prostředí, event. spazmolytika (při křečovitých bolestech břicha)- rehydratační roztok – Na 3,5g + NaHCO3 2,5g + KCl 1,5g+Glc 20g + aqua ad 1000ml

66.A REVMATOIDNÍ ARTRITIDA A JUVENILNÍ IDIOPATICKÁ ARTRITIDA

Revmatoidní artridita- primárně zánětlivé onemocnění, zkracuje život o 5-10 let- ženy 3:1 muži prevalence 1% incidence 50/100000 (v Praze ročně onemocní 500 lidí)Definice - klinická: symetrická polyartritida, postihující predilekčně DRK, RC s prodlouženou ranní ztuhlosti,

postihující převážně ženy s manifestcí mezi 30, a 40. rokem věku- patologicky anatomická: perzistující synovialitida s místní destrukcí chrupavky, kostními erozemi a

různým stupněm mimokloubních změnEtio - iintolerance neznámého antigenu s dysfunkčním imunitním systémem

- může za to trimolekulární reakce, to je souhra HLA bb na APC, antigenu a TCR, přes několik mnoho cytokinu (pro zkoušku postačí, zapamatujete-li si TNF alfa, IL 1, IL 6) dochází k zánětu buněk pojivové tkáně synovialis = pannus (synovialocyty, a stimulací osteoklastů a tvorbou metaloproteináz dochází k postupné resorpci kostní hmoty)

KO – palpační bolestivost nad kloubní štěrbinou, deformity kloubů, prsty labutí šíje (RA i SLE)- destrukce kloubů (eroze až ankylóza)- revmatické uzle (snadno hmatatelné na laterální straně předloktí) diagnostice pomůže histologie, mohou

být všude i v plicích

114

Page 115: lf1.czlf1.cz/wp-content/uploads/228-vypracovane_otazky_interna... · Web viewprognoza – 50 % dosáhne kompletního uzdravení do 6 měsíců, 20 % má prodloužený průběh –

Vypracované otázky na INTERNU made by Edita Homolková, Monika Ambroziová, Adam Král, Petr Vermach a Vítek Křehnáč

- dochází-li k tomuto procesu v atlanto-axiálním kloubu, dochází k atlanto axiální disociaci (rozvolnění lig. transversum) při prudkém nárazu hrozí posunutí s následnou náhlou smrtí nebo v lepším případě kvadruplegií

- je-li na RTG AOD >3 mm je indikována chirurgická fixace kalvy ke krční páteři- RA je provázena i vaskulitidou

Dg kritéria - ranní ztuhlost >1 hod- artritida 3 a více oblastí- artritida ručních kloubů- symetrická artritida- revmatické uzly- sérové revmatické faktory: RF – zvláštní protilátka proti Fc fragmentu IgG, je nespecifická (objevuje se

i při SLE, SS)anti CCP (=ACPA) maximálně specifický (cyklický citrulinový peptid, je v

něm arginin vyměněn citrulinem, je známkou probíhající apoptozy, necitrulinovaný peptid je součástí ECM kosti i chrupavky)

- antiCCP je tak specifický, že jeho přítomnost je dg kritériem i bez přítomnosti klinických znaků RA

- výše jeho titru určuje závažnost a průběh onemocnění- rentgenové změny: I: žádné destruktivní změny

II: mírné destruktivní změnyIII: destrukce + deformityIV: ankylóza

- extraartikulární příznaky: na základě vaskulitidy vznikají různé kožní defekty, vředy až gangrény, intersticiální plicní fibróza

Hodnocení aktivity nemoci + Zhodnocení efektu léčby DAS 28 (dinase aktivity score, 28 nejdůl. kloubů)- sedimentace mm/hod- počet bolestivých kloubů z 28 určených (HK + kolena)- autohodnocení na 10cm úsečce- složitý vzorec s odmocninami i logaritmy vyplivne číslo: >5.1 vysoká aktivita

>3.2 střední aktivita>2.6 nízká aktivita<2,6 remise

Th - 4 linie léčby- NSAID zlepšuje život, ale neprodlužuje přežití- DMARD - chorobu modifikující léky (sulfasalazin, Metotrexát, Leflunomid) ovlivní zánět, ale netlumí RTG

progresi, začíná účinkovat za 8-12 týdnů- kortikoidy - mohutný protizánětlivý účinek v průběhu několika dní v prvních dvou letech mohou i

zpomalit RTG progresi- Biologická léčba - púsobí na časné patogenetické mechanismy

- INFLIXIMAB anti TNF alfa, TOCILIZUMAB anti IL6, Anti IL1, RITUXIMAB antiCD20Juvenilní idiopatická artritida

- sk. onemocnění projevujícíhc se chronickou synovitidou 1 nebo více kloubů u dětí <16 let- prevalence1 / 1000 dětíKO – záleží na věku v době začátku, 7 klinických forem (systémová/ / a / / nediferencovaná)

- oligoartikulární typ – nejčastější, dívky předškolního věku, často kolenní kloub- ranní ztuhlost, bolest, otok- při pozitivitě ANA (antinukleární protilátky) riziko akutní přední uveitidy

- polyartikulární séropozitivní – nejméně častý, podobný dospělé RA, častěji dívky nad 10 let- polyartikulární séronegativní – školní věk, i velké klouby- systémová = Stillova choroba – 10-20 % dětí jakéhokoli věku

- celkové příznak (horečka), prchavá vyrážka na trupu a končetinách, myalgie, artralgie, bolest v krku a břicha, uzlinový synrom, hepatosplenomegalie

- ↑ reaktanty akutní fáze, ↑ ferritin, leukocytóza, anémie, trombocytóza, negativní autoprotilátky

115

Page 116: lf1.czlf1.cz/wp-content/uploads/228-vypracovane_otazky_interna... · Web viewprognoza – 50 % dosáhne kompletního uzdravení do 6 měsíců, 20 % má prodloužený průběh –

Vypracované otázky na INTERNU made by Edita Homolková, Monika Ambroziová, Adam Král, Petr Vermach a Vítek Křehnáč

- artritida za měsíc po prvních projevech- juvenilní psoriatická artritida – postižení velkých i malých kloubů, psoriáza v rodinné anamnéze- asociovaná s entezitidou – často chlapci, klouby DK (achilovka, kolem pately…)

- bolest a otokTh – jako u RA dospělých

66.B NEMOCI KONEČNÍKU A ANU

Hemoroidy- nejčastější onemocnění GIT, až ¼ populaceEtio – vrozená dispozice, hypertrofie vnitřního análního sfinkteru, porucha vyprazdňování (zácpa/ průjem), ↑

nitrobřišního tlaku (těhotenství, usilovné tlačení)Onemocnění vnitřních hemoroidálních pletení – nebolestivé krvácení, na konci stolice a toaletku, anémie

vzácněji- bolest jako projev komplikace v podobě trombózy a tromboflebitidy, intersfinkterický absces,

anální trhlina- často permanentní bolestivý tlak v konečníku vystupňovaný při defekaci- svědění a pálení kolem končníku jako důsledek intermitentního prolapsu objemných vnitřních

hemoroidů do análního kanáluOnemocnění zevních hemoroidálních pletení – téměř nikdy nekrvácí, klinicky se projeví při komplikacích jako je

trombóza zevních hemoroidů, perianální hematom díky ruptuře stěny hemoroidální pleteně (velmi bolestivé, po námaze a prochladnutí)

Dg – aspekce, vyšetření per rektum, anoskopie, rektoskopieTh – režimová opatření – vláknina, pravidelná měkká stolice, ne usilovné tlačení, fyzická aktivita

- lokálně – mastě, krémy, čípky s adstringentními, vazoaktivními a antiseptickým působením, anestikum - na intermitentní krvácení venofarmaka (tribenosid), hemostyptika (Ca sůl karboxycelulózy), NSA u

zánětu a kortikosteroidy- chirurgie – ligace, termokoagulace, skleroterapie, staplerová hemoroidektomie

Anální fisura- vředovitý defekt začínající v zevním okrakji, může dosahovat až linea debata- etio?, ischemie- akutní/ chronickáKO – řezavá bolest při defekaci, čerstvá krev

Prolaps rekta- kopletní protruze do análního kanálu, častěji u žen- jako důsledek tlačení, po poroduKO – zácpa, obtížné vyprazdňování, i inkontinence a krváceníDg – fyzikálně (aby zatlačila jako na stolici)Th – chirurgická staplerová resekce

Anální karcinom- vzácné, v 80 % spinocelulární ca, 16 % adenokarcinomy- rizko při HPV infekci, HIV, imunosuprese, Crohnova ch., chronická perianální píštělKO – krvácení z konečníku, citlivost v anální oblasti

67.A ONEMOCNĚNÍ ŠTÍTNÉ ŽLÁZY

Hypotyreóza, myxedém- nedostatečná sekrece/ působení hormonů štítné žlázy - časté, trvale postihuje 1 % populace (v 60 až 5 %), převážně ženyPrimární, periferní – nejčastěji

- chronická lymfocytární (autoimunní, Hashimotova) tyreoiditida, totální tyreoidektomie, terapie radiojódem, vrozené defekty syntézy hormonů, jódový deficit, novorozenecká (transplacentární přenos protilátek blokujících rec pro TSH), poléková (Li, amiodaron, interferon α)

- ↓ T4-free (norma 9-22 pmol/l), ↑ TSH

116

Page 117: lf1.czlf1.cz/wp-content/uploads/228-vypracovane_otazky_interna... · Web viewprognoza – 50 % dosáhne kompletního uzdravení do 6 měsíců, 20 % má prodloužený průběh –

Vypracované otázky na INTERNU made by Edita Homolková, Monika Ambroziová, Adam Král, Petr Vermach a Vítek Křehnáč

- protilátky proti tyroidální peroxidáze a proti tyreoglobulinu, protilátky blokující TSH recSekundární, hypofyzární – při hypopituitarismu, v klinice zahrnuje i terciární (hypotalamickou)

- ↓ T4-free, ↓/ norma TSHKO – u lehké formy nespecifické potíže

- v dětství – zpomalení růstu a psychomotorického vývoje, - v dospělosti – zpomalení mtb, reversibilní- únavnost, zimomřivost, ↑ hmotnosti, hypercholesterolémie, suchost kůže, myxedém (depozice

glykosaminoglykanů do podkoží, kůže a svalů), suchost a vypadávání vlasů, zácpa (i ileus), metroragie, hypermetrorea, ↓ kontraktility myokardu a TF (i perikardiální výpotek, ↓ voltáž komorových komplexů), mělké a pomalé dýchání, anémie (většinou normocytární, při těžké hypotyreóze), svalová slabost/ křeče, parestézie, porucha koncentrace/ paměti, apatie, deprese, zhrubělý až chraplavý hlas

Dg – USG (velikost, struktura, prokrvení), CT/ MR u retrosternálních strumTh – substituce T3/ T4/ kombinace

- 1x denně, ráno, nalačno- po tyreoidektomii plná očekávaná dávka- u hypotyreózy začínáme nízkou dávkou

Subklinická – primární hypotyreóza bez kliniky, ale již ↑ TSHOligosymptomatická – časté u geriatrických pacientůMyxedémové koma – vzácné, mortalita 50 %, vystupňované příznaky hypotyreózy

- slabost, letargie, hypoventilace, hypoglykémie, hypotermie, koma- hyponatrémie, respirační insuficience s acidózou, anémie, hypercholesterolémie

Hypertyreóza – viz 8.BZáněty štítné žlázy – 39.BUzly – viz 39.BKarcinom – viz 39.B

68.A OBEZITA

= nadměrné množství tuku v organismu- genetika a vliv zevního prostředí (50:50)- norma % tuku: ženy do 30%, muži do 20 %- ↑ zátěž skeletu a kloubů, ↑ zátěž KV aparátu, DM II, metabolický syndrom, hormonální odchylky, nádory (GIT,

gynekologické, Grawitz), dušnost, SAS, syndrom polycystyckých ovárií (u 5-10 % žen)- mortalita u obezity I. a II. stupně 2x vyššíBMI – výška cm/ kg2

- <18,5 podváha, norma 18,5 -24,9, nadváha 25 -29,9, obezita I 30-34,9, ob II 35-39,9, ob III >40Obvod pasu – metabolické riziko

- ženy: mírné nad 80 cm, výrazné nad 88 cm- muži: mírné nad 94, výrazné nad 102 cm

Tuk – aromatáza + ↓ SHBG → ↑ riziko karcinomů prsu, cervixu- mechanická ochrana, izolace, energetická zásobárna- vychytávání TAG – hlavně postprandiálně, ochrana před steatózou - tvorba hormonů – adiponektin - plazmatická hladina negativně koreluje s množstvím tukové tkáně (hl.

viscerální), TNF-α ihibuje promotor adiponektinu - ↑ citlivost buněk k inzulinu- v koster. svalu a v játrech ↑ expresi molekul zapojených do oxidace MK

a ↓ obsah TAG - antiaterogenní efekt

- leptin - produkt OB genu (7q31), produkován bílou tukovou tkání (hl. podkožní, méně viscerální), žaludkem, placentou, mléč. žlázou

- plazmat. hladina exponenciálně roste s přibývající tuk. tkání - snižuje příjem potravy, vliv na dlouhodobou regulaci energetické bilance- iniciace puberty, imunitní a zánětová odpověď, hematopoeza, angiogenenze,

růst kostí, hojení ran- ↑: přejídání, glukokortikoidy, insulin, agonisté β3 adrenergních receptorů

117

Page 118: lf1.czlf1.cz/wp-content/uploads/228-vypracovane_otazky_interna... · Web viewprognoza – 50 % dosáhne kompletního uzdravení do 6 měsíců, 20 % má prodloužený průběh –

Vypracované otázky na INTERNU made by Edita Homolková, Monika Ambroziová, Adam Král, Petr Vermach a Vítek Křehnáč

- ↓: dlouhé hladovění, TNF-α, IL-1/6, vysokotučná jídla, cigarety- rezistin - plasmatická hladina roste s přibývající tukovou tkání (hl. viscerální)

- blokuje působení inzulinu- zdrojem systémového zánětu (TNFα, IL-1, IL-6…)

Th – „metabolická paměť“ - ↓ glykémie na začátku vede za 20 let ke snížení mortality- dieta - ↓ energetického příjmu, vychází z dosavadních zvyklostí pacienta (14ti denní jídelní deníček)

- musí být doprovázena fyzickou aktivitou!- omezení E o 10-15 %/ den (u většiny cca 1000-2000 kJ/den), při větším omezení se váha snižuje

hlavně úbytkem vody- fyzická aktivita – brání adaptaci organismu na snížený energetický příjem

- energetický výdej = bazální metabolismus (60-70 %) + pohyb (20-25 %) + posprandiální termogeneze (5-12 %)

- udržení aktivní tělesné hmoty (nesníží se BM)- psychoterapie – nutná- farmakoterapie – při nedostatečnosti diety a pohybovvé aktivity a BMI >30, užívání obvykle 3-12 měsíců

- orlistat – blokátor střevních lipáz- bariatrická chirurgie – indikace u BMI >40/ BMI >35 + komorbidity (hypertenze, DM II, dyslipidémie)/ i

pacienti s BMI <35, kteří již někdy měli BMI vyšší- KI: závažné poruchy příjmu potravy (bulimie…), alkoholismus, drogová závislost,

dekompenzované psychiatrické onem- restriktivní – nutná spolupráce, dietní režim

- laparoskopická adjustabilní bandáž žaludku, tubulizace žaludku (sleeve gastrectomy)

- malabsorpční – vyřazují část tenkého střeva, provádějí se málo- kombinované – biliopankreatická diverze, Roux-en-Y-gastrický bypass- pokles o 10kty kg v prvních 2 letech, kompenzace DM II

68.B PNEUMOTHORAX

- stav, kdy se do interpleurální štěrbiny dostává vzduch a kdy dochází ke kolapsu plíce- nastává omezení dýchací plochy s poruchou ventilace, perfúze a difúze, tlakové změny v pohrudničkní dutině

spolu s koabovanou plící zatěžuje krevní oběh- fční rezervy plic jsou schopny vyrovnat se i s vyřatením 1 ½ plic, pokud je plicní parenchym intaktníEtio - Spontánní pneumothorax - primární – u jinak zdravých lidí (u mladých vysokých asteniků kuřáků)

- sekundární – komplikace plicních chorob, prasklá emfyzematická bula, plllicní systa, tbc kaverna

- Traumatický pneumothorax - penetrující – v rámci polytraumat- nepenetrující – poranění jícnu, rupture bronchů

- Iatrogenní pneumothorax – při invazivních vyšetření, zavádění centrálního žilního katetru do v. subclaviaDělení PNO - zavřený - při jednorázovém vniknutí vzduchu mezi pleurál. listy, intrath. tlak může zůstat negitví

- otevřený - pleurál. dutina komunikuje s okolním prostředím v expiriu i v inspiriu- mediastinum se v exp. i insp. vyklenuje vždy opačným směrem (paradoxní pohyb

mediastina)- může dojít k stlačení velkých cév a rozvoji těžké respir. insuficience

- ventilový - nejzávažnější forma, s každým inspiriem se zvětšuje objem vzduchu v pleurál. Dutině- v expiriu nemůže vzduch unikat ven, s každým inspiriem dochází k ↑ nitrohrud. TK →

přetlačování mediastina na zdravou stranu → útlak velkých cév, alterace srdeč. činnosti a vznik šokového stavu

KO - typicky se objevuje náhlá dušnost a bolest na hrudi- fyzikálně - neslyšitelné dýchání a bubínkový poklep

Dg - RTG - obraz kolabované plíce – perihilózní kulovité zastínění, nízko položená brániceKomplikace – Hydropneumothorax – serózní tekutina

- Pyopneumothorax – hnisavá tekutina - Hemothorax – krev – časté recidivy

Th – klid + vysokoprůkotokový O2 (10-15 l/ nim)

118

Page 119: lf1.czlf1.cz/wp-content/uploads/228-vypracovane_otazky_interna... · Web viewprognoza – 50 % dosáhne kompletního uzdravení do 6 měsíců, 20 % má prodloužený průběh –

Vypracované otázky na INTERNU made by Edita Homolková, Monika Ambroziová, Adam Král, Petr Vermach a Vítek Křehnáč

- jednorázová jehlová aspirace- hrudní drenáž do 2./3. mezižebří v medioklavikulární čáře pomocí lahví s podtlakem, kyslík, antitusika,

analgetika, sedativa- případně sutura plíce či bulektomie

- u ventilového je třeba okamžitě převést na otevřený (vpich tupou jehlou nad horní okraj žebra v medioklavikulární čáře

69.A LEDVINY A DIABETES

= Diabetická nefropatie- hlavní příčina chronického renálního selhání ve vyspělých zemích- 30-50 % dialyzovaných- mají zvýšení KV rizikoEtio – genenetické faktory, tvorba AGEs, chronická hyperglykemie, metabolizace glc na sorbitol, hypergly aktivuje

sys. Renin-angiotenzin-aldosteron přes sukcinátový rec- hyperglykémie → neenzymová glykace → ztluštění bazální membrány a mezangiální expanze- hyperglykémie → ↑průtok glomeruly, hyperfiltrace, mikroalbuminurie → následná modulární interkapilární

glomerulosklerózaKO - 1. St. latentní - klinicky asymptomatické, ↑průtok, ↑filtrace, při dekompenzaci tranzitorní

mikroalbuminurie) - 2. St. incipientní - mikroalbuminurie20-200 μg/min = 30-300 mg/24 hod, často doprovázeno ↑ tk,změny

bazálních membrán, expanze mezangia- 3. St. manifestní - albuminurie > 300 mg/24 hod a proteinurie > 500 mg/den, plně rozvinutý nefrotický syn.

- větčina má hypertenzi a dochází k ↓ GF- 4. St. chronické renální insuficience - dialýza by měla být zahájena při vzestupu kreatininu nad 400 μmol/l)

Komplikace – aterosklerotická stenóza renální arterie- nekróza papily – recidivující močové infekce- autonomní neuropatie močového měchýře-asymptomatické bakteriurie – u žen

Th – léčba hyperglykémie, hypertenze, dyslipidémie, antiagregační terapie- metformin KI u sérového kreatininu >200 μmol/l- proteiny max 0,8g/kg/24h

69.B JATERNÍ SELHÁNÍ

Akutní selhání jater- syndrom vzníkající při zániku hepatocytů- INR >1,5 + jakýkoli stupeň encefalopatie bez přítomnosti cirhózyEtio – vir hep B/ C (40-70 %), léky a toxiny (paracetamol, aflatoxin, halotan, isoniazid), metabolické poruchy

(Wilsonova ch., Reyův syn.), kardiovaskulární onemocnění (jakákoli forma šoku, Bucc-Chiary sy., levostranné srdeční selhání)

KO – u paracetamolu může být několik hodin bez obtíží (LD 13-25 g), ale léčbu je nutné zahájit okamžitě- u muchomůrky zelené se bojdví bolesti břicha a (krvavý) průjem do 6- 24 po požití, AJS za 3-4 dny- mladé dívky s amenoreou, hemolýzou a potižením jater Wilsonova choroby- ikterus, koagulopatie (hemoragická diatéza z nedostatku srážecích faktorů, DIC), porucha vědomí (jaterní

encefalopatie), ostatní (foetor hepaticus, flapping tremor, hyperventilace (amoniak), hypotenze (encefalopatie)

Kompl - edém mozku, krvácení z varixů, hypoglykémie (snížená glukoneogeneze), selhání ledvin, sepse, respirační komplikace

Dg - anamnéza, SP, spec. vyšetření (měření intrakraniálního tlaku, EEG)- Lab - ↑ jaterní testy, ↑ bilirubin, ↑ amoniak, Quickův čas ↓ 20%, ↓ srážecí faktory, trombocytopenie,

hypokalémie, hypoglykémieTh - léčba příčiny (eliminační léčba u otrav, ukončení těhotenství)

- otrava paracetamolem – při hospitalizaci do 6 hod po požití provést výplach žaludku- antidotum N-acytylcystein p.o./ i.v., (aktivní uhlí), hemodyalýza, hemoperfuze

119

Page 120: lf1.czlf1.cz/wp-content/uploads/228-vypracovane_otazky_interna... · Web viewprognoza – 50 % dosáhne kompletního uzdravení do 6 měsíců, 20 % má prodloužený průběh –

Vypracované otázky na INTERNU made by Edita Homolková, Monika Ambroziová, Adam Král, Petr Vermach a Vítek Křehnáč

- otrava amanitinem – výplach žaludku do 4 po požití, aktivní uhlí, forsírovaná diuréza (toxin se dobře filtruje)- kys. thioktová, silymarin, N-acetylcystein/ G-penicilin, hemoperfuze

- symptomatická – JIP, substituce elektrolytů, glukózy, srážlivých faktorů, AT III, parenterální výživa, profylaxe jaterního komatu (karence bílkovin, vysoká klyzmata, laktulóza, neomycin PO (redukce střevní flóry), profylaxe stresu, vředu, event. hemodialýza, transplantace jater

- edém mozku - manitol IV, zvýšená poloha horní poloviny těla, udržovat střední arteriální tlak o 50% vyšší než nitrolební tlak

70.A CHRONICKÁ RENÁLNÍ INSUFICIENCE

chronická renální insuficience = stadium chronického renálního onemocnění, kdy fce ledvin klesne na takovou úroveň (pod 20-25%) že dochází k výrazným změnám ve složení extracelulární tekutiny

- současně se projeví změny v mtb-endokrinní fci ledvin, které jsou vystupňovány při zátěži organismuchronické renální selhání = stav, kdy fce ledvin je snížená tak, že ledviny nejsou schopny udržet normální složení

vnitřního prostředí ani za bazálních podmínek, speciálních dietních a medikamentózních opatření - k prodloužení života nemocného je nutno použít metod nahrazujících fci ledviny (dialýzy,

transplantace)- bez použití dyalýzy se objevuje soubor příznaků – uremický syndrom (nauzea, zvracení, krvavé průjmy,

polyneuropatie, koma, perikarditida)- podle Dvořáčka: příznaky uremie – psychické (dezorientace, apatie až koma), gastrointestinální

(nauzea, zvracení, průjmy), respirační (prohloubené Kussmaulovo dýchání), kožní (hematomy), biochemické (hyperkalemie, hyperazotemie, zvýšená kreatininemie, acidóza

- ↓ GF do 75 % fyziol. hodnoty nevede ke změně ve složení vnitřního prostředí nebo jen ke změnám krátkodobým při vysokých nárocích na organismus

- k vzestupu sérového kreatininu dochází až při ↓ GF na 25 %→ stadium bez ↑ konc. sérového kreatininu = snížená fce ledvinposouzení fce ledvin - hodnota sérového kreatininu – chronickou dialyzační léčbu zahajujeme při kreatininu 500-

600 µmol/l = při poklesu clearance kreatininu pod 0,2 ml/s, v komplikovanějších případech už při kreatininu 400-500 µmol/l

patofyziologie tubulárních změn- chron. onem ledvin může vést k postupnému zániku fčních nefronů a k rozvoji CHSL- zánik nefronů je podmíněný jednak zákl. patol. procesem a je spojený s adaptivními změnami v reziduálních

nefronech, které umožňují přechodnou stabilizaci vnitřního prostředí organismu, z dlouhodobějšího hlediska ale adaptivní změny vedou k dalšímu organickému poškození glomerulů a tubulů až k jejich konečné skleróze

- postupný zánik fčních nefronů se projeví ↓ celkové GF a ↑ kreatininu v séru - ↓ clearance kreatininu a ↑ konc.kreatininu v séru umožňuje monitorování renální fce v čase

- fční adaptace reziduálních nefronů - změna fce reziduálních nefronů v průběhu chron. ren. onem. umožňuje zachování homeostázy vnitřního prostředí až do velkého úbytku aktivního renálního parenchymu - ↓ GF se ↓ množstvím profiltrované látky je v reziduálních nefronech kompenzován změnou intenzity tubulárních procesů - ↓ tubulární resorpce a ↑ tubulární sekrece → ↑ frakční exkrece sodíku a kalia (FE = clearance/ GF)

- reziduální diuréza - u CHRI je diuréza většinou normální nebo může být polyurie, u CHSL bývá oligoanurie- polyurie provázená polydypsií se sklonem k dehydrataci, polyurická izostenurie, celkové ztráty Na+ mohou být

vysoké → při přetrvávající ↑ FE Na+ dochází k dehydrataci, ↓ GF, ↑ sérová urea, kreatinin, někdy ↓ sérového Na+ (depleční hyponatremie), ↓ objemu extracelulární tekutiny → ↓ TK, tachykardie, slabost, svalové křeče – pro posouzení bilance Na+ stanovujeme odpady Na+ močí za 24 hod. (norm. 100-200 mmol)

- sérová koncentrace K+ může zůstat dlouho normální až do pokročilého stadia CHRI v důsledku ↑ vylučování K+ v reziduálních nefronech a ↑ extrarenální eliminace K střevem

- v terminálních stadiích (GF ↓↓↓) a při podávání kaliumretenčních farmak vzniká hyperkalemie (! nebezpečí bradykardie, arytmií až ♥ zástavy), k hyperkalemii může dojít i při uvolnění K+ z erytrocytů (hematomy, krvácení do GIT) nebo z tkáňových bb. při katabolických procesech

- může dojít i k hypokalemii (při extrarenálních ztrátách, při ↑ vylučování K+ do moči po diuretikách), při závažné hypokalemii svalová slabost, paréza končetin, nebezpečné je postižení dýchacích svalů a hypotonie až atonie hladkých svalů GIT (paralytický ileus), extrasystoly

120

Page 121: lf1.czlf1.cz/wp-content/uploads/228-vypracovane_otazky_interna... · Web viewprognoza – 50 % dosáhne kompletního uzdravení do 6 měsíců, 20 % má prodloužený průběh –

Vypracované otázky na INTERNU made by Edita Homolková, Monika Ambroziová, Adam Král, Petr Vermach a Vítek Křehnáč

- v pokročilejších stadiích ledvinových chorob se vyvíjí MTB acidóza – při ↓ GF se ↓ vylučování fosfátů a sulfátů, tubuly reziduálních nefronů ↓ syntetizují a vylučují amoniak a někdy je i porucha syntézy hydrogenkarbonátu v tubulárních bb. – zvětšující se aniontová mezera

- u některých nemocných se na MTB acidóze podílí renální tubulární acidóza z poruchy tubulární fce - ↑ pH moči a normální aniontová mezera

- MTB acidóza nemá progresivní charakter díky nárazníkové kapacitě kostní tkáně (vyplavuje se Ca 2+ z kostí), ale vyvíjí se renální osteopatie

- ↑ intenzita katabolických procesůMTB poruchy při CHSL

MTB bílkovin a AMK - ↓ přívod bílkovin v dietě → lepší využití dusíku- výpočet množství metabolizovaného proteinu na podkladě množství urey vyloučené do

moči v mmol/ 24 hod : (Uurea – V) : 4MTB sacharidů -1/3 nemocných má ↑glykemii nalačno, 2/3 mají abnormální glu toleranční test

- nutno vyloučit DM (zvlášť v polyurické fázi CHRI)- hladina inzulinu u CHRI zvýšená – tím se zvyšuje syntéza triacylglycerolů a VLDL v játrech,

inzulin reguluje aktivitu lipoproteinové lipázy- porucha glycidové tolerance způsobena postreceptorovou rezistencí periferních tkání vůči

inzulínu s možnou účastí malnutrice, acidózy, narušeného MTB kalia, hořčíku a kalciaMTB lipidů - ↑ celkové triacylglyceroly, ↑ VLDL, ↑ LDL, ↓ HDL, celkový cholesterol v mezích normálu nebo

lehce ↑, stejně tak i LDL cholesterol, celkový apoprotein A snížený- hyperlipoproteinemie již při ↓ GF pod 1 ml/s

progrese CHRI- postupný zánik reziduálních nefronů vede k terminálnímu selhání fce ledvin (rychlost zániku závislá na zákl.

patol. procesu)- jakmile dojde k zániku urč. kritického množství fungujících nefronů, dochází v reziduálních nefronech k fčním a

morfologickým změnám (↑ průtoku krve a ↑ GF v reziduálních nefronech, hypertrofie rezid. glomerulů), které po urč. dobu umožňují maximální výkon reziduálních nefronů, ale nakonec vedou ke skleróze glomerulů a dalšímu zániku nefronů

- 2 fáze: 1. zánik nefronů podmíněn zákl. onem.2. zánik nefronů mechanismem nezávislým na zákl. onem., hypertrofie, skleróza, zánik, indukce

tubulointersticiální sklerózyKO - až do pokročilých stadií nemusí mít žádnou symptomatologii, často zjištěno náhodně

- při poklesu GF pod 0,25 ml/s vzniká uremický syndrom (poškození řady orgánů, terminální fáze CHSL), dnes se skoro nevyskytuje, jen u nedialyzovaných

- anémie normocytární normochromní v důsledku ↓ produkce EPO a inhibice erytropoezy uremickými toxiny a parathormonem, tox. účinek aluminia, hemolýza, ztráty Fe při dialýze a krevních odběrech, nemocní na ni mohou být dobře adaptováni → TH: rekombinantní lidský erytropoetin, EPO + suplementace Fe s monitorací sérového ferritinu

- hemoragické diatézy – příčinou je trombocytopatie (porucha adheze a agregace destiček) a abnormality koagulačních faktorů → objevují se podkožní hematomy, epistaxe, krvácení do GIT, riziko ↑ krvácení při chirurg. výkonech

- renální osteopatie – na podkladě retence fosfátu, porucha hydroxylace 25-hydrocholekalciferolu na 1,25-cholekalciferol v ledvinách

- hypokalcemie a ↓ sérová koncentrace kalcitriolu vede k sekundární hyperparatyreóze (osteopatie se ↑ kostním obratem)

- deficit kalcitriolu, akumulace floru a aluminia mohou vést k osteomalacii a obě kostní choroby se mohou kombinovat

diagnostika renální kostní choroby:1/ vyšetření sérových koncentrací Ca, fosfátů, albuminu, ABR, ALP kostního izoenzymu, ACP, parathormonu2/ RTG – subperiostální resorpce falang, granulární atrofie kalvy s rozmazáním lamin3/ scintigrafie skeletu, kostní biopsie4/ CT nebo USG příštítných tělísek- renální osteopatie se vyvíjí řadu let a vyskytuje se až u pacientů v chron. dialyzačním programu

terapie CHRS: úprava příjmu proteinů a energie (0,3-0,8 g/kg/den)♦♦ úprava příjmu tekutin a Na

121

Page 122: lf1.czlf1.cz/wp-content/uploads/228-vypracovane_otazky_interna... · Web viewprognoza – 50 % dosáhne kompletního uzdravení do 6 měsíců, 20 % má prodloužený průběh –

Vypracované otázky na INTERNU made by Edita Homolková, Monika Ambroziová, Adam Král, Petr Vermach a Vítek Křehnáč

♦ úprava příjmu K♦ úprava ABR♦ úprava poruchy kalciofosfátového MTB♦ úprava KO♦ léčba arteriální hypertenze + zpomalení progrese renální insuficience♦ dialyzační léčba

Komplikace:1/ kardiovaskulární – koronární ateroskleróza, IM, hypertrofická a dilatační kardiomyopatie, perikarditida2/ poškození nervového systému – uremická encefalopatie (pseudoneurastenický syndrom, porucha vědomí,

křeče, cerebrovaskulární komplikace), poškození periferního nervového systému (sy periferní obrny – parestezie, křeče, sy neklidných nohou)

70.B POLYPY TRÁVICÍHO TRAKTU, HEREDITÁRNÍ POLYPÓZNÍ SYNDROMY

Familiární adenomatóza tlustého střeva (FAP)- mnohočetné adenomové polypy v celém tlustém střevě a konečníku v počtu vyšším než 100- u 1/3 adenomy duodena (tendence k malignizaci)- často v žaludku polypy = cysticky dilatované žlázky- ca tlustého střeva kolem 35 rokuDg –genetický screening v10-12 roce (mutace APC genu), koloskopieTh – totální kolektomie a ileo-pouch-anaání anastomóza (kolem 20. Roku)

- u některých vyšší dávky COX-2 inhibitorů (celekoxib) ke snížení rychlosti růstu polypůPrognóza – pokud dg včas, pak ok

71.A SYSTÉMOVÝ LUPUS ERYTHEMATODES

- systémové autoimunní onemocnění pojiva- prevalence 20-30 /100 000 ob, hl. ženy (5-10:1), začíná nejčastěji ve 2.-3. Dekádě (estrogeny?)Etio – genetické vlivy a zevní prostředí → polyklonální aktivace B-bb. a nespecifické ↑ tvorby protilátek (t-bb. jsou

naopak ↓) → ukládání imunitních komplexů → poškození různých orgánůKlasifikační kriteria: erytém v obličeji

diskoidní erytémfotosensitivitavřídky v ústechartritidyserozitidyrenální postižení (proteinurie, válce)neurologické postižení (křeče, psychóza)hematologické poruchy (anémie, leukopenie, trombocytopénie)imunologické poruchyantinukleární protilátky

KO – nespecifické příznaky…, horečka, váhový úbytek, katabolismus, kachexie, poškození pojivové tkáně- kůže – vaskulitida, fotosenzitivita, teleangiektázie, Raynaudův fenomén, trombembolie, motýlovitý erytém

(akutní), papulodeskvamózní ex (subakutní), diskoidní lupus (hojení jizvou, měsíce), svědění, alopecie- sliznice – nebolestivé vřídky v ústní dutině- klouby – mono/oligo/polyarteritidy, Rtg – neerozivní (bez destrukcí)

- periartikulární poróza = začátek artritidy- srdce – perikarditidy, myokarditida, Libman Sacksova verukózní endokarditida (hypertrofie lymfatické tkáně),

poruchy vedení, vrozená AV blokáda (díky antiRo protilátkám), akcelerace aterosklerózy- plíce – pleuritida, lupusová pneumonie, difúzní intersticiální pneumonitida, krvácení do plic, plicní hypertenze- CNS – psychiatrický syn: úzkostné stavy, akutní zmatenost, kognitivní/ afektivní poruchy

- neurologické: epilepsie, bolest hlavy, aseptická meningitida- krev – anémie (hemolytická), leukopenie, trombocytopenie, koagulopatie (antifosfolipidové protilátky →

trombembolie, potraty)- ledviny – lupusová nefritida asi u ½, proteinurie (>0,5 g/24 hod), hematurie, válce v moči

122

Page 123: lf1.czlf1.cz/wp-content/uploads/228-vypracovane_otazky_interna... · Web viewprognoza – 50 % dosáhne kompletního uzdravení do 6 měsíců, 20 % má prodloužený průběh –

Vypracované otázky na INTERNU made by Edita Homolková, Monika Ambroziová, Adam Král, Petr Vermach a Vítek Křehnáč

- antinukleární protilátky – anti-dsDNA, proti histonům, anti-ENALab – středně ↑ FW, CRP spíše v normě,

- pozitivita Anti-dsDNA, Anti-ssDNA, Anti-ssRNA, SSA(Ro), SSB(La) AntiHistoni, AntiNukleozomyTh – vyhnout se slunci, infiekcím a větší fyzické námaze

- lehčí formy – nízké dávky glukokortikoidů, Plaquenil (antimalarikum)- těžší formy – vyšší dávky glukokortikoidů, azathioprin (to je předvěc merkaptopurinu), Cyklofosfamid,

Cyklosporin A, Metotrexát- při antifosfolioidovém syn. antikoagulační léčba (INR 3-4)- biologická léčba - Rituximab protilátka proti CD20, způsobuje depleci B lymfomů a to potřebujeme, když

máme takový nadbytek autoprotilátek

71.B CHRONICKÁ BRONCHITIDA, BRONCHIEKTÁZIE

Chronická bronchitida - chronický produktivní kašel, který trvá alespoň 3 měsíce ve dvou po sobě jdoucích letech a u kterého jsou

vyloučeny jiné příčiny kašle + obstrukce- chronická bronchitida + emfyzém = CHOPNKlinické dělení - Jednoduchá CB s expektorací hlenu (simplex)

- Hlenohnisavá (mukopurulenta) - S obstrukcí (obstructiva)

Etio - Exogenní příčiny - opakované respirační infekce (hemofilus a pneumokok), vlivy pracovního i životního prostředí, kouření

- Endogenní příčiny - věk (incidence stoupá s věkem), pohlaví (spíše muži), individuální predispozice (reaktivita bronchů, atopie), hypo-/dysgamaglobulinemie, sinusitida

Ptg - hyperplazie a hypertrofie hlenových žlázek ( index dle Reidové)- zánětlivá infiltrace bronchiální stěny - dochází k obstrukci dýchacích cest a poruše distribuce vzduchu → hypoxie, vazokonstrikce a plicní

hypertenzeDg - anamnéza (kuřák, pracovní prostředí, recidivující infekce DCD)

- fyzikální vyšetření (může být bpn / známky obstrukce)- RTG – vyloučení ca plic - spirometrie - FEV1- bronchoskopie

Th - inhalační kortikosteroidy a bronchodilatancia (beta 2 – agonisté a anticholinergika)Bronchiektázie

- ireverzibilní dilatace průdušek středního a malého průsvitu provázené chronickou zánětlivou infiltrací bronchiální stěny a peribronchia, retence hlenu a recidivující infekce přispívají k progresi choroby

- mohou být vrozené, získané, asociované s jinou chorobou (cystická fibróza, záněty, intersticiální fibrotizující procesy…) nebo idiopatické

Etio - uplatňuje se slabost bronchiální stěny (zánětlivé a destrukční změny) ↑ se intrabronchiální tlak, dále působí vnější tah a ciliární dysfunkce

- retinovaný hlen obsahuje hodně proteolytické elastázy, bakterie v něm obsažené mnění své vlastnosti a tvoří biofilm

- dělí se na cylindrické, vakovité, ampuliformní, cystické a moniliformní (dnes převládají cylindrické bronchiektázie s výrazným postižením dolní části plic, rozdělují se podle etiopatogeneze)

KO - závisí na lokalizaci a rozsahu postižení, zpočátku bývá onemocnění asymptomatické (projevují se spíše vrozené bronchiektázie a bronchiektázie u imunodeficitech)

- projevy se stupňují v průběhu života, objevují se recidivující infekce, produktivní kašel (někdy s hemoptýzou) s expektorací velkého množství sputa (hnisavé, páchnoucí, objevuje se hlavně ráno)

- dalšími příznaky jsou dušnost, tělesná slabost, nechutenství, váhový úbytek, anémieDg - anamnéza – časté infekce, expektorace

- objektivně – vlhké chropy a pískoty- RTG – výrazné peribronchiální kresby, cystická projasnění v oblasti fibrózních změn- HRCT – dilatace bronchů s rozšířenou stěnou, pruhovitá větvení a noduly dávají obraz „pučícího stromu“ - Funkční vyšetření - prokáže obstrukční poruchu

123

Page 124: lf1.czlf1.cz/wp-content/uploads/228-vypracovane_otazky_interna... · Web viewprognoza – 50 % dosáhne kompletního uzdravení do 6 měsíců, 20 % má prodloužený průběh –

Vypracované otázky na INTERNU made by Edita Homolková, Monika Ambroziová, Adam Král, Petr Vermach a Vítek Křehnáč

Th – cílené atb, mukolytika (N-acytylcystein, bromhexin), expektorancia (guaifenesin), oxygenoterapie- polohové drenáže, bronchoskopické odsávání hlenů a zátek- chirurgická resekce u lokalizovaných bronchiektazií

72.A NEFROTICKÝ SYNDROM

- soubor příznaků a laboratorních známek zvnikající jako důsledek výrazné proteinurie obv. 3,5 g/24 h a více- nejčastěji u glomerulopatie s minimálními změnami (minimal changes disease), v dospělosti fokálně segmentální

glomerulosklerózy, diabetické nefropatii, lupusové nefritidy a amyloidóze ledvin- ↑ permeabilita glom. membrány pro albumin a při větším poškození i pro velké molekuly IgGKO - výrazné ztráty bílkovin vedou k hypalbuminemii/ hypoproteinémii → kompenzace ↑ tvorbou bílkovin a

lipoproteinů (hypercholesterolemie) v játrech - značné otoky, ev. transudáty, se vysvětlují retencí Na+ a tekutin- albuminémie <20 g/l, hypercholesterolemie >8 mmol/l- hyperkoagulační stav s rizikem trombóz

Th – otoky – restrikce Na+ v dietě, klid na lůžku- diuretika – kličková (furosemid) v kombinaci s manitolem/ hydrochlorthiazidem/ amilorid (šetřící K+)- albumin NE! (expanze volumu, riziko plicního edému)- ACEI/ ATI – u refrakterních otoků- pokud ohrožují na životě – bilaterální nefrektomie (embolizace)

- infekce- častá příčina smrti (hl. u dětí)- trombembolie – podíl kortikosteroidů a diuretik, prevalence 5-10 %

- mobilizace, heparin/ warfarin, ale ne profylakticky- u lidí s albuminem pod 20, ASA

-statiny – cíl je max. 2,6 mmol/l LDL

72.B DIVERTIKLY TRÁVICÍHO TRAKTU, DIVERTIKULÁRNÍ CHOROBA REKTOSIGMATU

Divertikly jícnu- viz 54.B

Divertikly tenkého střevaDivertikly duodena – v 90 % v sestupném raménku, jsou pulsní

- juxtapapilární a peripapilární (Vaterská papila na dně divertiklu, pankreatitidy, infekce cest a vznik lithiasy)

Divertikly jejuna a ilea – Meckelův divertikl – zbytek omfalo-enterické dučeje, u 2 % populace, na antimezenteriální straně ilea, 70-90 cm od ileocekální chlopně

- může mít zachovalé lumen, možnost ektopické žaludeční/ pankreatické tkáně

KO – manifestace jen v případě komplikací (krvácení, perforace, ileus)- Mnohočetné divertikly, pseudodivertikulóza – někdy u rozsáhlého postižení při

Crohnově chorobě (pokročilé strukturální změny, striktury střídané dilatacemi)

Divertikulání choroba- vícečetné divertikly v tračníku, jsou získané a nepravé (tvořeny sliznicí a submukózou), vytlačeny hlavněkolem

penetrujících cév (vasa recta)- nejčastěji v levém tračníku (hl. esovitá klička a sestupný tračník, 60-90 %)- počet se zvětšuje s věkem (v 70 letech u 60 %)KO – bez komplikací asymptomatické, nadýmání, plynatost, nepravidelná stolice, bolest v levém dolním

kvadrantu- akutní divertikulitida – u 20 % lidí s divertikly a u ½ proběhne jednorázově s úplnou úzdravou

- možná komplikace abscesem (krytá perforace střeva), píštělí (fibroprodukcee v okolí zánětu, komunikace s dělohou/ močovým měchýřem/ tenkým střevem), perforací a peritonitidou

- náhlé bolesti v levém hypo- a mezoeastriu, nauzea, porucha odchodu větrů, vzedmuté břicho, zvýšená teplota

- masivní krváceni – u 10 % nemocných, nejčastější příčina masivního krvácení z dolní části traávicí trubice

124

Page 125: lf1.czlf1.cz/wp-content/uploads/228-vypracovane_otazky_interna... · Web viewprognoza – 50 % dosáhne kompletního uzdravení do 6 měsíců, 20 % má prodloužený průběh –

Vypracované otázky na INTERNU made by Edita Homolková, Monika Ambroziová, Adam Král, Petr Vermach a Vítek Křehnáč

- enteroragie bez větších prodromů a bolestíDg – palpační bolestivost v levém (i pravém) dolním kvadrantu, bolestivá rezistence, teplota, známky

peritonitidy při komplikacích- irigografické vyšetření, CT-kolografie (posouzení transmurálního a extramurálního postižení), koloskopie je

relativně málo senzitivní (příčiny krvácení)- akutně USG a CT (MR), nativní rtg břicha- irigografie a koloskopie kontraindikovány v akutním stádiu- leukocytóza, ↑ FW a CRP

Th – vláknina 20-30 g/ den a tekutiny- u těžšího průběhu atb (metronidazol + ampicilin-klavulonát, cefriaxon, ciprofloxycin)- aminosalicyláty na zklidnění akutní fáze nebo probatika- chirurgie u závažných komplikací a recidiv (resekce, až po zklidnění stavu

73.A MALNUTRICE

= stav výživy, kdy deficit energie/ nutrietů má měřitelné účinky na tkáně/ tělo/ klinický stav= v praxi podvýživa (↓ energie/ makronutrientů/ mikronutrientů)- karence = selektivní nedostatek nutrientů- dobrý nutriční stav determinuje úspěšnost léčby, realimentace je značně obtížná- u 30-60 % hospitalizovaných a 3-4 % jsou přímo ohrožena na životě, u 40 % prodlužuje hospitalizaciEtio - ↓ p.o. příjem - nechutenství, nesoběstačnost, porucha vědomí/ polykání…

- porucha resorpce – syn. krátkého střeva, střevní píštěle, nespec. střevní záněty- porucha digesce – gastrektomie, pankreatobiliární nedostatečnost- mtb poruchy – hepatální/ renální/ kardiální/ respirační insuficience- ↑ E nároky a ztráty – operace, sepse, polytraumata, popáleniny, nádory, endokrinopatie, infekce- ψ faktory – bolest, deprese, cizí prostředí- léčba – agresivní medikace, vyšetřovací program, operace…- věk- ekonomické a sociální – alkoholismus, chudoba, drogová závislost

KO - marasmus – prosté hladovění, nedostatek E a bílkovin- proporcionální ↓ úbytek tuku i svalů- laboratorně v normě- hypometabolismus, může probíhat měsíce i roky- vyhublý vzhled (stáří, anorexie)

- kwashiorkor – stresové hladovění, proteinová malnutrice- ↓ bílkovin, tuk téměř nedotčen- těžká hypoalbuminémie (<30 g/l), ↓ Ig a transportních proteinů, hypermetabolismus (sepse,

trauma), retence extracelulární tekutiny, otoky- vzhled dobře živený, otoky, padání vlasů

- protein-energetická malnutrice – kombinace obou- ↓ svalů → hypoventilace → hypoxie, bronchopneumonie

Dg - nutriční screening – rychlé zhodnocení rizika dotazníkem- objektivní hodnocení – anamnéza, fyzikální nález (i antropometrie), sérové bílkoviny, lymfocyty, krevní obraz,

vit B12, dusíkaté metabolity v séru a moči, mtb Fe, fční testy (síla stisku, FEV1), tělesné složení (bioimpedance)

73.B SEKUNDÁRNÍ ARTERIÁLNÍ HYPERTENZE

- 5-10 %- důsledek jiného přesně def. patologického stavu- pomýšlet při těžké hypertenzi/ hypertenzi rezistentní na léčbu (3kombinace včetně diuretik), epizodické HT,

+DM, +Cuchingoidní znaky, + morfologicky změněné nadledviny…Renální HT – 8 %, viz ot. 32.B

- renoparenchymatózní HT – Th: ACE inhibitory, blokátory rec. pro angiotenzin II

125

Page 126: lf1.czlf1.cz/wp-content/uploads/228-vypracovane_otazky_interna... · Web viewprognoza – 50 % dosáhne kompletního uzdravení do 6 měsíců, 20 % má prodloužený průběh –

Vypracované otázky na INTERNU made by Edita Homolková, Monika Ambroziová, Adam Král, Petr Vermach a Vítek Křehnáč

- renovaskulární HT - stenóza a. renalis, tumory ledvin, ledvina na postižené straně je menší, US, renální arteriografie

Th: katetrizační/chir. odstranění stenózyEndokrinní HT - <1 %, nadprodukce některého z presorických hormonů (nádor/ hyperplazie)

- primární hyperaldosteronismus – autonomní produkce aldosteronu, ↓ reninKO – HT, neuromuskulární příznaky z hypokalémie, hypokalémie, mtb

acidóza, mírná polyurieDg - ↓ aktivita reninu, ↑ aldosteron, poměr aldosteron/ renin, CT/ MR

nadledvin (adenom, hyperplazie)Th – laparoskopick adrenalektomie u 1stranné nadprodukce, spironolakton/

eplerenon- feochromocytom – nádor z chromafinních bb. z dřeně nadledvin/ paraaortálních symp. Ganglií

- (hlavně) noradrenalin/ adrenalin/ dopamin, většinou benigníKO - často hypertenze záchvatovitá (bledost, palpitace, opocení) s norm.tlakem mimo

záchvaty, flush, zblednutí, úbytek hmotnostiDg - v moči kys. vanilmandlová, metanefrin, normetanefrinTh – chir odstranění po přípravě α (doxazosin) a β blokátory (prevence vyplavení

katecholaminů)- SAS – epizod apnoe/hypopnoe trvajících >10 sec + denní hypersomnolence- neurogenní HT - ↑ nitrolební tlak, opakovaná hypoperfuze mozku…- vyvolaná léky – NaCl, kofein, kokain, efedrin, glukokortikoidy, cyklosporin, EPO, alkohol, kontraceptiva,

ktamin, 3cyklická antidepresiva,Pb, Hg…- primární hyperreninismus – benigní nádor juxtaglomerulárního aparátu, T : chir - Cushingův sy - akromegalie

74.A MYELODYSPLASTICKÝ SYNDROM

- mutace pluripotentní myeloidní b.- dřeň je normo i hypercelulární, ale v periferii různě velká cytopenie (neefektivní hematopoesa, defektní imunitní

reakce indukuje apoptózu zralejších forem jednotlivých krevních řad)- genová nestabilita klonu vede k deregulaci a rozvoji AML- dříve jako „ doutnající leukémie“, 2-3/100 000, nárůst s věkemklasifikace FAB (fran.-brit.-amer.) - dělení dle počtu blastů v k. dřeni, WHO vyčleňuje CMML (1-20% blastů) mimo

MDS a stavy s > 20% blastů v k. dřeni řadí mezi akutní leukémie - časná fáze - lepší prognóza, hyperplastická dysplastická hematopoéza v k. dřeni kontrastující

s pancytopenií v perif. krvi; změny v buň. stromatu→ neoangiogeneze, spíše chronický průběh

- RA = refrakterní anémie (< 5% blastů), RAS = refrakterní anémie s prstenčitými sideroblasty (< 5% blastů)

- pokročilá fáze - horší prognóza, progresivní nárůst CD34+ bb. s velmi nízkým stupněm apoptózy, nadbytek blastů, často přechod do akutní leukémie; rychlý průběh

- RAEB = refrakterní anémie s excesem blastů (5-20%), RAEB-t = refrakterní anémie s excesem blastů v transformaci (21-30 %)

E - primární MDS - prim. příčina?, expo tox. látkám, predispozicí je deficit enzymů v detox. procesech- sekundární MDS - pozdní následek chemoterapie (alkylační l.), důsledek ozařování, terapeut.

podávání radioizotopůKO - záleží, kt. z hematopet. řad je převážně poškozena; únava, dušnost, srdeč. selhávání při těžké anémii,

recidivující infekce při granulocytopenii, krvácení při trombocytopenii- IPSS = mezinárodní prognostický skórovací systém: % blastů ve dřeni, cytopenie v perif. krvi

Lab - krevní nátěr perif. krve a sternálního punktátu - často makrocyt. anémie (makroovalocyty, poikilocyty, schistocyty, Howellova-Jollyho tělíska), mohou být méně zralé elementy, prstenčité sideroblasty (barvení na železo, prstenec z mitochondrií, nemožnost využít Fe v dýchacím řetězci)

- trepanobiopsie k. dřeně – dysplazie u min. 10 % buněk, architektonika..- průtoková cytometrie - nezralé prekurzory CD34+

126

Page 127: lf1.czlf1.cz/wp-content/uploads/228-vypracovane_otazky_interna... · Web viewprognoza – 50 % dosáhne kompletního uzdravení do 6 měsíců, 20 % má prodloužený průběh –

Vypracované otázky na INTERNU made by Edita Homolková, Monika Ambroziová, Adam Král, Petr Vermach a Vítek Křehnáč

Th – konzervativní – u nízkého a středního rizika- transfúze ery/ trombocytů, vitamíny B, chelatační léčba u přetížení Fe, rHuEPO, G-CSF- imunosuprese u hyponastické formy, kortikosteroidy a cyklosporin A

- chemoterapie – jako u AML- transplantace krvetvorných bb. - jediná kurativní th

74.B NÁDORY TRÁVICÍHO TRAKTU VČETNĚ ENDOKRINNÍCH

Tumory jícnu- viz 54.B

Tumory žaludku a duodena- viz 75.B

Tumory tenkého střeva- vzácné, méně než 5 % tumorů GITLymfomy – většinou extranodulární non-Hodgkinské lymfomy vycházející z lymfatické tkáně (MALT)

- hlavně B-lymfomy z marginální vrstvy terminálních center- T-lymfomy jsou spojeny s dlouhotrvající celiakieKO – křčovité bolesti, prljmy, masivní krvácení, hubnutí, teploty

Adenokarcinom – vzácné, riziková je dlouhodobě trvající celiakie/ Crohnova ch./ vrozené polypózní syndromyKO – recidivující subileózní stavy, chronické krvácení, intususcepce, volvulus- nepříznivá prognóza (pozdní dg, 5ti leté přežití 20-35 %)

Neuroendokrinní nádory TS – karcinoid TS – téměř vždy maligní, 30 % ze zhoubných novotvarů v TSKO – neurčité bolesti břicha (ischemizace střeva prorůstáním nádoru

do cév), hubnutí, obstrukce při intraluminálním růstu, proliferace vaziva v dutině břišní

- karcinomová syn. (syn. serotoninismu) = flush, hypermobilita GIT, bronchospasmus, hypotenze (u 10 %, při metastatickém postižení jater)

Dg – rtg enteroklýza, staging dle CT/ MR, hladina chromograninu AKolorektální karcinom - viz 31.AKarcinom slinivky - viz 77.BTumory jater - viz 53.B Neuroendokrinní nádory GIT

- v žaludku nejčastějí karcinoid, v duodenu a proximální jejunu gastrinom…NET pankreatu – inzulinom – většinou benigní

- Whippleova triáda – spontánní hypoglykémie, poruchy CNS, dobrá odpověď na podání glc

Dg – provokační test lačněním, MR- gastrinom – nejčastěji v pankreatu 80 %, maligní, 50 % v době dg již metastázy

- sporadicky/ v rámci MEN-IKO – Zollinger-Ellison syn. (hyperchlorhydrie s tvorbou vředů), průjmy (inaktivace

lipázy HCl)- Werner-Morrisonův syndrom, VIPom – vzácné, produkce VIP

KO – vodnaté průjmy, hypokalémie, hypochlorhydrie, diabetes, váhový úbytek, stavy zmatenosti

- glukagonom – maligní nádor z A-bb.KO – hyperglykémie, diabetes

- MEN – AR dědičné syndromy mnohočetné endokrinní neoplazie- MEN 1 – primární hyperparatyreoidismus, tu pankreatu (gastrinom, inzulinom), tu

hypofýzy- MEN 2a – medulární ca štítné žlázy, feochromocytom, primární hyperparatyreoidismus- MEN 2b - + nádory nervových ganglií, marfanoidní vzrůst

75.A ANÉMIE SIDEROPENICKÁ; ANÉMIE CHRONICKÝCH ONEMOCNĚNÍ

127

Page 128: lf1.czlf1.cz/wp-content/uploads/228-vypracovane_otazky_interna... · Web viewprognoza – 50 % dosáhne kompletního uzdravení do 6 měsíců, 20 % má prodloužený průběh –

Vypracované otázky na INTERNU made by Edita Homolková, Monika Ambroziová, Adam Král, Petr Vermach a Vítek Křehnáč

Anémie sideropenická = z nedostatku Fe, prevalence 5 % mužů a 8-20 % žen- omezení tvorby hemoglobinu, myoglobinu, cytochromů, peroxidázy, katalázy…- snížená konc. nitrobuň. Fe inhibuje zásobní formu Fe - feritinu, nedostatek Fe se rozvíjí pomalu- Etio - ↑ ztráty Fe – urogenitální trakt (menoragie, menometroragie, z ledvin a moč. cest), GIT (peptický vřed,

brániční hernie, jíc. varixy, hemoroidy, polypy, divertikly, Crohnova choroba, ulcerózní kolitida, paraziti, salicyláty, antikoagulancia), dýchací ústr. (recidiv. hemoptýza, Goodpastureův sy, idiopat. plic. hemosideróza), arteficiální ztráty (hemodialýza, odběry krve, dárcovství krve - ztráta 150-200 mg Fe)

- ženy obvykle ztratí menstruací 40-60 ml krve (cca 20 mg Fe) → zvýšená denní resorpce na 20-25 % přijaté dávky

- ↓ příjem Fe – malnutrice - nedostatek v potravě, nadbytek inhibitorů resorpce (fosfáty, tanáty, fytáty, oxaláty)

- maldigesce - nedostatečná redukce Fe (atrofická gastritis, st. p. resekci žaludku)- malabsorpce - celiakie, ulcerózní kolitis, m. Crohn, st.p. resekci střeva

- ↑ nároky na přívod Fe - těhotenství (3-4 mg/den), kojení, růst (potřeba 15 mg/den)- 3 stádia: 1. prelatentní - postupně vyčerpány zásoby Fe při dostatečné dodávce pro erytropoézu, bez klin.př.

2. latentní - zásoby Fe zcela vyčerpány, ↓ dodávka do k. dřeně, ale nelimituje proliferaci bb3. manifestní - nedostatek Fe již tlumí proliferaci a vzniká sideropen. anémie

- KO - sider. anémie vzniká pomalu a nemocní jsou na ni dobře adaptováni- projevy jen při hlubším deficitu Fe: únava, slabost, hučení v uších, palpitace, dušnost, bolesti hlavy,

podrážděnost, nekoncentrovanost, snížená fyz. výkonnost i kardiopulmon. selhávání- poruchy epiteliálních struktur: pálení jazyka (hladký červený lesklý jazyk), angulární stomatitida („koutky“),

koilonychie, předčasné šedivění vlasů, dominující s bledostí- zvýšený sklon k infekcím

- Lab - včasná dg. latentní f. - ↓ konc. ferritinu v séru pod 12 μg/l, ↓ konc. Fe v séru pod 6 μmol/l, ↑ celk. vazebná kapacita transferinu pro Fe, saturace transferinu klesá pod 16%, ↑ solubilní transferinové receptory, ↑ volného protoporfyrinu v ery, ↓ sideroblastů pod 30 %

- manifestní f. - hypochromní mikrocytární anémie- Th - léčba příčiny krvácení!!! + substituce Fe po dostatečně dlouhou dobu (vzestup Hb o 2g/l/den → 180-200

mg prvkového Fe p.o.) rovnoměrně během dne, nejlépe nalačno- optimální zásoba Fe v organismu nejméně 300-500 mg (1μg/l feritinu = 8 mg zásobního Fe)

Anémie chronických onemocnění- doprovází chronické infekce a záněty, systémová onemocnění a nádory, incidence 25-30 %- důsledek poruchy distribuce Fe → retence v monocyto-makrofágovém sys.- Etio - chronické infekce (TBC, osteomyelitida, infekční endokarditida, AIDS, mykotické inf., chron. uroinfekce)

- chron. zánětlivá onem. (ulcerózní kolitis, regionální enteritida)- systémová onem. (SLE, sklerodermie, vaskulitidy, revmatoidní arthritis)- nádory (maligní lymfomy, solidní nádory, leukemie)- hepatopatie při alkoholismu- aktivace imunit. systému → omezení nabídky Fe invadujícím patogenům a nádor. bb. (Fe-růst. faktor),

udržení konc. Fe pro cytotoxickou fci efektorových bb. imunit. syst.- zvýšená sekrece cytokinů → stimulace tvorby ferritinu + blokáda uvolňování Fe ze zásobárny v monocyto-

makrofágovém systému cestou stimulace hepcidinu (inhibuje ferroportin, Fe nemůže z b ven) → pokles konc. Fe v séru, ↓nabídka pro utilizaci v k. dřeni

- ↓ tvorba erytropoetinu je hlavním činitelem u anémie při chron. nedostatečnosti ledvin- KO - rozvoj anémie až po 1-2 měsících trvání choroby, kombinace příznaků anémie + zákl. choroby- Lab - zpočátku normocytární normochromní anémie, poději rozvoj mikrocytózy a hypochromie

- ↓ konc. Fe(S), ↓ syntézy transferinu - saturace není snížena, konc. transf. rec. v normě, feritin v normě/↑

- při barvení punktátu k. dřeně (zvýšený obs. Fe v siderofázích kontrastuje se sníženým počtem sideroblastů a siderocytů)

- Th - léčba zákl. onem., popř. transfúze erytrocytární masy

128

Page 129: lf1.czlf1.cz/wp-content/uploads/228-vypracovane_otazky_interna... · Web viewprognoza – 50 % dosáhne kompletního uzdravení do 6 měsíců, 20 % má prodloužený průběh –

Vypracované otázky na INTERNU made by Edita Homolková, Monika Ambroziová, Adam Král, Petr Vermach a Vítek Křehnáč

75.B NÁDORY ŽALUDKU A DUODENA

Polypy a benigní nádory žaludku- polyp je makroskopicko-morfologický termín, histo rozhoduje- polypy nádorové (adenom/ ca)/ hamartomy/ zánětlivé- polypy epitelové (90 %)/ mezenchymovéEtio – fokální foveolární hyperplazie – často souvislost s žaludeční aftou, velikost hrachu

- hyperplaziogenní polypy – specifický pro žaludek, hyperplazie s adenomatózní komponentou- vypovídají o ↑ riziku ca (schopnost atypického růstu sliznice)

- adenomy – maligní potenciál- mezenchymální polypy – 10 %, leiomyom, neurofibrom

KO – náhodný nález, dyspepsieDg – endoskopie s biopsií – polypektomie, u submukózně uložených mezenchymálních endoUSG

Maligní nádory žaludku- 95 % karcinom, lymfom, leiomyosarkomEtio – kancerogeny v potravě (nitrosaminy, PAU, benzpyren, aflatoxin), genetika, H. pyloriKlasifikace – o prognóze rozhoduje hloubka invaze (od svaloviny hlouběji je to blbé)

- intestinální typ – v terénu atrofické gastritidy s intestinální metaplazií, většinou ve vyšším věku- difuzní typ – bez výrazné gastritidy, u mladších, horší prognóza

KO – nejsou typické obtíže, většinou mírné → pozdní/ náhodné zachycení- Virchowova uzlina – metastaticky zvětšenád levým klíčkem, zlé

Dg – gastroskopie s biopsií (každý žaludeční vřed!)- USG břicha prokáže metastázy v játrech, CT, ednoUSG

Prognóza – 5-10 leté přežití: do sliznice 80-90%, u pokročilých ca je jen 50 % operabilních a z toho přežije necelých 20 %

Lymfom žaludku- nehodgkinský lymfom, lymfogranulom- málokdy primární lokalizaceMALT – nízký stupeň malignity, často ke zmnožení lymfoidní tkáně sliznice vede infekce H. pylori

76.A ANEMIE MEGALOBLASTOVÉ, PERNICIÓZNÍ ANEMIE

Megaloblastové anémie= anémie z nedost. vit. B12 a kys. listové, dle morfologie ery- nedostatečný přívod vit. B12 a kys. listové, nedostatek vnitřního faktoru, celiakální sprue,

Crohnova choroba, resekce terminálního ilea, striktury, ileokolické píštěle, divertikly, parazité, inhibitory resorpce (fenylhydantoin, PAS, pirimidin, neomycin), selektivní malabsorpce B12

s proteinurií, poruchy transportu při nedostatku transkobalaminu, gravidita, růst, jaterní choroby (↑ ztráty),dialýza, resekce žaludku, inhibitory dihydrofolát reduktázy (MTX), antagonisté pyrimidinů a purinů

Perniciózní anémie = zhoubná chudokrevnost (m. Addison-Biermer), autoimunní onemocnění, typ megaloblastové anemie, postihuje starší osoby

- tvorba protilátek proti parietálním bb. žaludeční sliznice → atrofie, achlorhydrie, ↓ produkce vnitřního faktoru (nutný pro vstřebání B12 v ileu – hlavně terminální ileum)

- 2 typy protilátek - brání vazbě vit.B12 na vnitřní f./ inhibují vazbu komplexu vit. B12 + vnitřní f. na receptor

P - vit.B12 - tvoří prostetickou sk. homocysteinmethyltransferázy (demethyluje tetrahydrofolát a přenáší methylovou sk. z homocysteinu na metionin, je donorem methylu pro vznik thyminu

- podílí se na demethylizaci L-methylmalonyl-CoA na sukcinyl-CoA → hromadící se L-methylmalonyl-CoA je toxický pro myelin → neurologické příznaky

- kys. listová - je zdrojem tetrahydrofolátu- nedostatek obou → porucha syntézy thyminových bází a tvorby DNA, prodloužení S-fáze

- syntéza RNA a tvorba bílkovin i Hb není porušena → velký objem buňky, ve kt. je Hb v normální konc. a ve větším množství, buň. jádro méně zralé s malou kondenzací chromatinu = vzniká megaloblast

129

Page 130: lf1.czlf1.cz/wp-content/uploads/228-vypracovane_otazky_interna... · Web viewprognoza – 50 % dosáhne kompletního uzdravení do 6 měsíců, 20 % má prodloužený průběh –

Vypracované otázky na INTERNU made by Edita Homolková, Monika Ambroziová, Adam Král, Petr Vermach a Vítek Křehnáč

KO - hluboká anémie s makrocytózou MCV >100 fl, makroovalocyty v nátěru (ery s průměrem >10-12 μm), ↓ retikulocyty, leukopenie s posuvem doleva, hypersegmentace neutrofilů, ↓ trombocytů

- dřeň je hyperplastická-zmnožení megaloblastické erytropoézy- ↓ vit.B12 v séru, Fe a feritin v normě, u těžké anémie ↑ přímý i nepřímý bilirubin- protilátky proti parietal. bb, atrofická gastritida, histaminrezistentní achlorhydrie- specifické příznaky: pálení jazyka, neurologická symptomatologie (parestezie v prstech, poruchy hlubokého

čití, areflexie až ataxie), malinově červený jazyk s vyhlazenou špičkou- Schillingův test - měření stupně resorpce vit. B12 značeného radioizotopem celotělovým měřením s detekcí

množství aktivity vyloučené močíTh - substituce vit. B12 (300μg i.m/den) → po 5-7 dnech vzestup retikulocytů na hodnoty 10-30 % = retikulocytární

krize + doplnit Fe, po úpravě 300μg B12 1x/ měs.- substituce kys. listové (10-20 mg/den)

76.B AKUTNÍ ZÁNĚTLIVÁ ONEMOCNĚNÍ DOLNÍCH DÝCHACÍCH CEST

- k akutním onemocněním DC patří akutní tracheobronchitida a akutní exacerbace chronických stavů (astma, CHOPN)

- akutní tracheobronchitida je zánět postihující tracheu a bronchy a obvykle nasedá na akutní zánět horních cest, někdy se vyskytuje jenom bronchitida.

Etio - Infekce - respirační viry (rhinoviry, virus influenzy a parainfluenzy), mykoplazmata, chlamydie, bakterie (haemophilus, streptokoky, stafylokoky)

– průběh závisí na virulenci patogenu a stavu nemocného- neinfekční – inhalace iritačních látek

KO - na počátku je bolest v krku, dysfonie, dráždivý kašel, únava a slabost- v pozdější fázi je kašel produktivní s expektorací bělavého hlenu- dušnost bývá spíše mírná- tracheitida se projevuje tlakem a pálením za sternem, které se zhoršuje při kašli a hlubším dýchání- chrapot + bolest v krku – současná laryngitida - stridor + inspirační dušnost – epiglotitida, dětský pseudokrup nebo závažná stenóza- kašel přetrvávající déle než 3 týdny – subakutní postinfekční kašel, kašel přetrvávající déle než 8 týdnů –

syndrom chronického kašle (nutno vyloučit astma, CHOPN, GER )Dg - podle typu zánětu různě vysoká teplota + projevy únavy

- fyzikální vyšetření – suché i vlhké bronchitické fenomény- mikrobiologické vyšetření sputa – jen při produktivním kašli- skiagram hrudníku – obvykle bpn, indikován při kompl- bronchoskopie – při trvání potíží

Th - antipyretika, inhalace a šetřící režim (základ), antitusika, expektorancia, mukolytika (bromhexin), ATB při přetrvávání hnisavého sputa a průkazu bakt. etiologie (aminopeniciliny, makrolidy, tetracyklin, cefalosporiny II.)

77.A CUSHINGŮV SYNDROM

Cushingova choroba= Cushingův syn. způsobený ↑ tvorbou ACTH, adenom hypofýzy/ ektopická paraneoplastická produkce

(malobuněčný plicní ca, bronchiální ca aj.)Cushingův syndrom

= syn. z dlouhodobé expozice ↑ konc. kortizolu- unilaterální adenom/ ca kůry nadledvin, bilaterální hyperplazie/ tumoryKO – centrální obezita, bíčí šíje, měsíčkovitý obličej, tenká a křehká kůže, purpurové strie, vypadávání vlasů,

hirsutismus, arteriální hypertenze, porucha glc tolerance až DM, osteopenie/ osteoporosa, steroidní myopatie (slabost, atrofie), ψ poruchy (koncetrace, spánku, paměti, deprese), „fční“ hypogonadismus, nefrolitiasa, polyurie, leukocytóza, hyperkoagulační stav

Dg – vyšetření cirkadiánního rytmu sekrece (chybí noční pokles), volný kortizol v moči, dexametazonový supresní test

- plasmatické konc. ACTH ráno,

130

Page 131: lf1.czlf1.cz/wp-content/uploads/228-vypracovane_otazky_interna... · Web viewprognoza – 50 % dosáhne kompletního uzdravení do 6 měsíců, 20 % má prodloužený průběh –

Vypracované otázky na INTERNU made by Edita Homolková, Monika Ambroziová, Adam Král, Petr Vermach a Vítek Křehnáč

- MR hypofýzy, CT krku a hrudníku při podezření na ektopickou produkci, nadledviny orientačně SONO + MR/ CT

Th – neurochirurgické odstranění adenomu hypofýzy, stereotaktická radiační léčba- inhibitory steroidogeneze – ketokonazol- bilaterální adrenektomie + suplementace- ektopie – odstranění tumoru- mitotan u primární CS (inhibitor steroidogeneze, selektivně cytotoxický pro bb nadledvin)

77.B KARCINOM SLINIVKY BŘIŠNÍ

- těžké, první příznaky až při plném rozvinují nemoci s tvorbou metastáz- incidence 11/ 100 000 ob, 5ti leté přežití závisí na resekabilitě 5 % (u neresekability 1 %)- výrazně agresivní, časně zakládá metastázy (játra, peritoneum, plíce)Etio – neznámé

- kouření, nadměrný přívod tuků, dlouhodobý diabetes, chronická pankreatitida (nad 10 let, hl. hereditární), dehet, benzidin

- 90 % vyrůstá z vývodů (duktální ca), v 80 % v hlavě pankreatuKO – symptomy často nevýrazné, závisí na lokalizaci

- v hlavě – bezbolestný obstrukční ikterus, obstrukce duodena- v kaudě – bolest bederní páteře- progresivní váhový úbytek, hepatomegalie, Courvouisierovo znamení (velký hmatný nebolestivý žlučník),

ikterus- objevení se diabetu, hmatná nádorová masa- komplikace: krváceni do GIT, ascites, splenomegalie, migrující tromboflebitida

Dg – SONO - zvětšení pankreatu, metastázy v játrech - CT - expanze, zvěšení abdominálních uzlin, metastázy v játrech, vztah novotvarů k cévám- endoSONO – bezpečný bioptický odběr- MR, PET (dif dg maligní a benigní léze)- nádorové markery s vysokou specifitou CA19-9, CA 125 (ale ne senzitivitou)

Th – jen 20-30 % je resekabilních, jen chirurgie je kurabilní- často jen paliativní (endoskopická drenáž žluč. cest, choledochojejuno-anastomóza, radio- a chemoterapie)- Whippleova operace – resekce žaludku, duodenektomie a hemipankreatekromií s wirsungo-jejuno-

anastomózou- chemoterapie – prodloužení přežití, zlepšení kvality života

- gemcitabin (antagonista pirimidinů)- analgezie

78.A HYPOTYREÓZA (MYXEDÉM)

- způsobeno nedostatečnou sekrecí/ působením hormonů štítné žlázy- 1 % populace (u lidí >60 let 5 %), spíše ženyPrimární (periferní) hypotyreóza

- nejčastějšíEtio – chronická lymfocytární tyreoiditida = Hashimotova (autoimunní), destrukce (tyreodektomie, terapie

radiojódem, ozáření), vrozené defekty hormonů, deficit jódu, poléková (lithium, amiodaron, INFα), novorozenecká hypotyreóza (transplacentární přenos protilátek blokující TSH receptor)

Sekundární (hypofyzární) hypotyreózaEtio – hypopituitarismus izolovaný/ kombinovaný

KO – v dětství – zpomalení růstu a psychomotorického vývoje, - v dospělosti - ↓ metabolismu, únavnost, zimomřivost, mírný ↑ hmotnosti, hypercholesterolémie, suchost

kůže, ztluštění kůže (depozice glykosaminoglykanů), nažloutnutí kůže (zpomalení přeměny karotenů na vit A), zácpa, hypermenorea, metroragie, ↓ frekvence, výdeje a kontraktility myokardu, ↑ periferní vaskulární rezistence, mělké a zpomalené dýchání (až myxedémové koma), ↓ glomerulární filtrace, anémie (většinou normocytární), svalová slabost, ↓ psychomotorické tempo, zhrubění hlasu

131

Page 132: lf1.czlf1.cz/wp-content/uploads/228-vypracovane_otazky_interna... · Web viewprognoza – 50 % dosáhne kompletního uzdravení do 6 měsíců, 20 % má prodloužený průběh –

Vypracované otázky na INTERNU made by Edita Homolková, Monika Ambroziová, Adam Král, Petr Vermach a Vítek Křehnáč

Dg – sérové konc. TSH a free-T4 (norma 9-22 pmol/l)- sérové protilátky proti tyreoidální peroxidáze (TPOAb) a proti tyreoglobulinu (TgAb), protilátky blokující TSH-R- USG štítnice, CT a MR jen u velkých strum s retrosternální propagací

Th – hormonální substituce – T4/ T3/ kombinované preparáty (1,7 μg/kg/den)- pomalu navyšovat dávku při těžké hypotyreóze (při tyreodektomii se nemusí)

Subklinická hypotyreóza – primární hypotyreóza s žádnými/ mírnými příznaky, s normal free-T4, ale se ↑ TSHOligosymptomatické a monosymptomatické formy – u geriatrických pacientůMyxedémové koma – vzácné, mortalita 50 %

- progresivní slabost, letargie, koma, hypoventilace, hypoglykémie, hypotermie, hyponatrémie, anémie, hypercholesterolémie, respirační insuficience s acidózou,

78.B HORNÍ A DOLNÍ FUNKČNÍ DYSPEPTICKÝ SYNDROM

Dyspepsie - GIT diskomfort, soubor trávicích obtíží, může být organického nebo neorganického původu (funkční)- horního typu – projev poruchy trávení v horní části trubice (jícen, žaludek, duodenum), hl. žaludeční symptomy

- dyskomfort v epigastriu, pocit plnosti, tlaku, žíhání, pálení žáhy, návrat šťáv do úst- dolního typu – porucha trávení na úrovni tenkého a tlustého střeva

- nadýmání, plynatost, přelévání obsahu ve střevecj, pocit dyskomfortu, pocit plnosti a tlaku kolem pupku nebo v podbřišku

- dyspeptický syndrom (neurodigestivní astenie) - kombinace obou předchozích, např. tlak a pocit plnosti, nauzea, anorexie, pocit nadmutí s flatulencí nebo bez ní, kyselý nebo hořký pocit v ústech, říhán

- akutní – dietní chyba → horní i dolní DS (přejedení, kořeněný pokrm, alkohol)- chronická – organický/ fční onemocnění GIT (dráždivý žaludek a tračník), potravinová alergie- v průběhu jídla (jícen, žaludek)/ do 2 hod po jídle tenké a tluské střevo/ za 3-5 hod po jídle onemocnění

hepato-pankreato-biliárního sys.- bez vazby na jídlo příčina mimo GIT/ viscerální hypersenzitivita

Dyspepsie z organické nebo fční choroby trávicí trubiceFční žaludeční dyspepsie – horrní DS bez závažných symptomů u mladého pacienta

- déletrvající stres, hodně kávy a alkoholu, špatná životospráva, i rexluční choroba jícnu, možná kombinace se solárním syndromem

Peptický vřed žaludku a duodena – horní DS a bolestivý syndrom (do 30 min po jídle a 2-3 hod po jídle)Refluxní choroba jícnu – vyjímečne DS, většinou dysfagie/ odynofagieZhoubné novotvary žaludku – hl. u seniorů, pocit plnosti a sytosti, nechutenství a váhový úbytek, často bolestivý

syn.Zánětlivé onemocnění tenkého střeva – celiakie – dolní DS atd.

- divertikulóza tenkého střeva/ syn. bakteriálního přerůstání – enormní nadýmání a plynatost, makrocytová anémie z nedostatku B12, bolest hl. v levém mesogastriu

- Crohnova choroba – dolní DS a bolest (viscerální, kolem pupku)Dráždivý tračník – dlouhodobé obtíže a negativní vyšetření, porucha vyprazdňování, střídání zácpy a průjmůGIT infekce – H. pylori, Giardia lamblialisStavy po operacích – resekce žaludku/ střeva

Dyspepsie při onemocnění jater a pankreato-biliární oblastiPortální hypertenze a cirkulační poruchy v portálním systému – žaludeční a střevní DS, nadýmání a plynatost,

předchází často ascituOnemocnění žlučníku a žlučových cest – horní i dolní DS, díky nedostatečná evakuace žlučníku/ transport žlučeRakovina podjaterní krajinyChronická pankreatitida - střevní dyspepsie až v pokročilé fázi, kdy je porušena zevně sekretorická fce

(sekundární diabetes, maldigesce tuků)Dyspepsie jako nežádoucí účinek léků/ toxinů

NSA – NÚ až u 30 %, porucha motility žaludku a duodenaKortikosteroidy – v imunosupresivních dávkách (cca 30 mg/ den), žaludeční dyskomfort, pyróza, říháníDigitalisové preparáty – hypoperfuze splanchniku?Atb – narušení mikroflóry

132

Page 133: lf1.czlf1.cz/wp-content/uploads/228-vypracovane_otazky_interna... · Web viewprognoza – 50 % dosáhne kompletního uzdravení do 6 měsíců, 20 % má prodloužený průběh –

Vypracované otázky na INTERNU made by Edita Homolková, Monika Ambroziová, Adam Král, Petr Vermach a Vítek Křehnáč

Onkologická terapieAlkohol, dráživé porkmy, kávaDyspepsie při celkových/ systémových chorobách

Srdeční selhání – hepatomegalieMetabolická onemocnění – DM…Endokrinopatie – hypertyreóza adrenální insuficience, hyperparatyreózaRenální insuficience – horní i dolní DS při hyperurémii (Treithzova uremická kolitida), také díky imunodeficitního

stavu

79.B SUPRAVENTRIKULÁRNÍ ARYTMIE

- arytmie, které vznikají a udržují se v síních a ke svému vlastnímu udržení nepotřebují AV uzel- ze síní se převádějí na komory, jejich převod na komory závisí na převodní kapacitě AV uzlu- pokud je převodní kapacita AV uzlu snížená, mohou mít i velmi rychlé supraventrikulární tachykardie poměrně

klidnou odpověď komor- zpravidla tedy dochází k depolarizaci komor přes normální převodní systém a QRS je štíhlý nebo je široký při

ramínkové blokádě- je pravidelná až na fibrilaci síní

Síňové extrasystoly- díky předčasné depolarizaci v ektopické,m fokusu v myokardu síní- vlna P je předčasná/ skrytá ve vlně T/ aberuje tvar vlny T- častý nález (v malém počtu až u 50 % zdravé populace)- často asymptomatické (neléčí se)Th – malé dávky β-blokátorů nebo anxiolytik

Typický flutter síní- časté- pravidelná tachykardii s velkým reentry okruhem- původ v pravé síní, elektrický impuls při ní krouží v pravé síní kolem anulu trikuspidální chlopně s frekvencí 200

– 350/min- ve svém okruhu proběhne vždy oblastí mezi vyústěním dolní duté žíly a anulem trikuspidální chlopně (tzv.

kavotrikuspidální isthmus) – ukončení arytmie ablací isthmu- na komory se převede nejčastěji pravidelně každý 2 či 3 impuls (mluvíme o flutteru blokovaném 2/1, 3/1

apod.), vzácně je neblokovaný (převod 1:1)EKG - vlnky přirovnávané k zubům pily a absencí izoelektrické linie (nejlépe patrné ve svodech II, III a aVF)KO – perzistující arytmie, častěji u organického srdečního onemocnění

- palpitace, dušnost při převodu 1:1- tachykardií indukovaná kardiomyopatie po týdnech a měsících u převodu 2:1- jinak s minimálními symptomy

Th – vyšší účinnosti elektrické kardioverze než u fibrilace síní- ke kontrolo komorové frekvence – β-blokátory, verapamil, dioxin- ke kardioverzi – elektrická, ablace isthmu- profylaxe amiodaronem/ propafenonem, ASA

Atypický flutter síní- síová makro re-entry tachykardie- flutterový okruh je lokalizován kdekoliv v pravé či levé síni- u lidí se strukturálním onemocněním srdce s postižením síní, po IM, u chlopenních vad, po kardiochirurgické

operaci (okolo jizev)…EKG – pravidelné flutterové vlnky (jiné než u typického)Th – elektrická kardioverze, profylaktická antiarytmická léčba

AV nodální centry tachykardie- nejčastější pravidelná paroxyzmální tachykardie (140-220/ min)- reentry okruh v oblasti AV uzlu – je funkční dualita (do AV vedou 2 vstupy)- pomalu vedoucí dráha ze zdola s rychlým zotavením a rychle vedoucí dráha ze shora s pomalým zotavením +

vhodná extrasystola → převede se přes pomalý uzel a vrátí se do již zotaveného rychle vedoucího a dokola…

133

Page 134: lf1.czlf1.cz/wp-content/uploads/228-vypracovane_otazky_interna... · Web viewprognoza – 50 % dosáhne kompletního uzdravení do 6 měsíců, 20 % má prodloužený průběh –

Vypracované otázky na INTERNU made by Edita Homolková, Monika Ambroziová, Adam Král, Petr Vermach a Vítek Křehnáč

- zpravidla současná aktivace síní i komorEKG – P vlny schovány v QRSKO – kolem 30. Let, náhlý vznik, pravidelní bušení srdce, slanost, tlak na prsou, presynkopy…

- náhle skončí (do několika minut)Th – adenosin i.v. – bolus, vytvoří se krátká AV blokáda a obnoví se sinusový rytmus

- verapamil- katétrová ablace pomalé dráhy

AV reentry tachykardie- reentry okruh využívající AV uzel a přídatnou síňokomorovou spojku- spouští ho extrasystola- impuls při arytmii může běžet ze síně do komory přes AV uzel, dále přes svalovinu komor k oblasti akcesorní

spojky, spojkou zpět do síně a svalovinou síní k AV uzlu- při tomto směru šíření impulsu se arytmie označuje jako orthodromní AVRT, na EKG se projeví jako pravidelná

tachykardie s frekvencí 150 – 200/min a QRS komplex je štíhlý (aktivace komor probíhá přes AV uzel)- vzácněji impuls postupuje šířit obráceně: ze síně na komory akcesorní spojkou, přes svalovinu komor k AV uzlu

a jím retrográdně na síně atd. - se označuje jako antidromní AVRT, na EKG se projeví jako pravidelná tachykardie s podobnou frekvencí, ovšem s širokým QRS komplexem (aktivace myokardu komor nezačíná od AV uzlu, ale od komorového úponu akcesorní spojky)

KO – jako AVNRTTh – jako AVNRT

Fokální síňová tachykardie- pravidelná depolarizace síní z ektopického síňového ložiska- krátké epizody = slavy síňových extrasystol- asymptomatické a klinicky nevýznamné- k akutnímu ukončení a profylaxi varapamil a betablokátory

Fibrilace síní – ot. 8.A

80.B AKUTNÍ SRDEČNÍ SELHÁNÍ, PLICNÍ EDÉM

= náhle vzniklá poruchu srdeční funkce (nebo její náhlé zhoršení), kdy srdce není schopno dostatečně přečerpávat krev z žilního řečiště do plic a/nebo z plic do tepenného řečiště

- důsledkem této poruchy je městnání krve v plicích a dalších orgánech a nedostatečné zásobení orgánů okysličenou krví

- ASS může být projevem řady kardiovaskulárních onemocnění, může však být způsobeno i nekardiálními onemocněními, jindy se mohou příčiny kombinovat

Etio - náhlé zhoršení chronického srdečního selhání- akutní koronární syndromy (Akutní infarkt myokardu a/nebo jeho mechanické komplikace, nestabilní angina

pectoris)- vysoký krevní tlak- poruchy srdečního rytmu: komorová tachykardie, fibrilace a flutter síní…- náhle vzniklá chlopenní nedomykavost při: infekční endokarditidě, ruptuře šlašinky mitrální chlopně,

zhoršení již známé nedomykavosti- závažné zúžení srdečních chlopní- myokarditida (tj. zánět srdečního svalu)- srdeční tamponáda (tj. útlak srdce tekutinou v osrdečníku např. při jeho zánětu nebo poranění)- dissekce aorty (částečná prasklina stěny aorty)- kardiomyopatie po porodu (tj. onemocnění srdečního svalu v souvislosti s těhotenstvím)- plicní embolie- nekardiální příčiny a zhoršující faktory - špatná spolupráce pacienta (neužívání doporučených léků,

nadměrný příjem tekutin u pacientů s chronickým srdečním selháním, závažná infekce, závažná onemocnění mozku, rozsáhlá operace, selhání ledvin, asthma bronchiale, otrava léky nebo alkoholem, feochromocytom, poruchy funkce štítné žlázy, anemie…

KO - odvíjejí se od příčiny, převahy postižení levé či pravé komory a pokročilosti stavu- nejzávažnější formou ASS je kardiogenní šok

134

Page 135: lf1.czlf1.cz/wp-content/uploads/228-vypracovane_otazky_interna... · Web viewprognoza – 50 % dosáhne kompletního uzdravení do 6 měsíců, 20 % má prodloužený průběh –

Vypracované otázky na INTERNU made by Edita Homolková, Monika Ambroziová, Adam Král, Petr Vermach a Vítek Křehnáč

- nejčastějším projevem ASS je dušnost v klidu nebo při malé zátěži (obvykle náhle vzniklá nebo náhle zhoršená)

- v případě klidové dušnosti bývají obtíže horší vleže (záchvaty noční dušnosti), vsedě se zmírňují nebo ustupují- kašel (někdy i s vykašláváním malého množství krve), únava, otoky (zejména dolních končetin), tlak v

nadbřišku, nechutenství, nevolnost (až zvracení), tahypnoe a tachykardie(>90/min), krevní tlak může být vysoký i normální, v případě kardiogenního šoku je krevní tlak nízký (systolický TK = vyšší hodnota < 90 mmHg), může být přítomno modravé zbarvení rtů, kůže prstů, nosu a ušních lalůčků

Dg - zhodnocení subjektivních obtíží, objektivních příznaků a výsledků vyšetřovacích metod (EKG, RTG hrudníku, laboratorní vyšetření, echokardiografie

81.A DYSLIPIDÉMIE = 49.A

81.B AKUTNÍ A CHRONICKÁ GASTRITIDA A GASTROPATIE

Akutní gastritis- stavy se zřetelnou žaludeční (a střevní) symptomatologií, zánět je často neprokázaný, většinou reflexní porucha

motility s krátkým trvánímEtio – akutní, hemoragická (erozivní) – difúzní zánět, různé noxy (alkohol, aspirin, NSA, glukokortikoidy)

- komplikuje celkové těžké stavy (šok, sepse, polytraumata)- aktutní korozivní – louhy, kyseliny- akutní neutrofilní – akutní fáze infekce H. pylori

KO – akutní dyspepsie (indigesce), tlak v epigastriu, nechutenství, odpor k jídlu, říhání, nauzea, zvracení, flatulence, křeče v břiše, průjem

- břicho lokalozovaně i difúzně citlivé- akutní obtíže trvají většinou krátce, může být subfebrilie

Th – klid, první 2 dny jen čaj a suchary, pak pozvolná realimentace, plná strava za cca týden- spasmolytika při kolikovitých bolestech, prokinetika při nauzee, při průjmu carbo animalit/ smecta/

endiaron- hydratace!

Chronická gastritis- histologická, ne klinická diagnóza- hodnocení zánětu, atrofie, přítomnost H. pylori, intestinální metaplazie (kompletní/ inkompletní)- v 80 % původ u H. pyloriDifuzní astrální gastritida – nestrofická, díky H. pyloriDifuzní korporální gastritida, autoimunní – destrukce žlázek fundu, achlorhydrie, hyperplazie G-bb.Multifokální atrofická gastritida – 80 % případů, díky H. pylori, může být prekancerózouDg – histologická, není korelace mezi makoskopií a histoTh – eradikace Helicobactera

- asymptomatická se neléčíGastropatie

- choroby žaludku bez prolázaného zánětlivého původu, různé etio Hemoragická gastropatie – jeden z nejčastějších důvodů krvácení do GIT

- projev gastropatie indukované NSA/ kongestivní gastropatie díky portální hypertenziKO – hemateméza a meléna různé intenzity

- může být bolest, pocit plnosti, pyrózaDg – endoskopie nejlépe do 48 h ve fázi intrahemoragické (patrné petechie, hemoragické erozeTh- prevence příčin

Gastropatie indukovaná nesteroidními antirevmatiky – potižení nejen žaludku, ale i systémové- lokálně NSA prolamují obrannou bariéru, dochází k nekróze epitelu, ke zpětné difuzi H+ a lézi

submukózních cév- systémově blokují COX → ↓ prostaglandinů → ↓ produkci hlenu, hydrogenkabonátu a prokrvení- změny hlavně v antruKO – není korelace mezi symptomy a endoskopickým nálezem

- až u ½ případů se poškození projeví komplikacemi (krvácení, perforace)- dyspepsie, vředové symptomy, refluxní symptomy, anémie při chronických ztrátách krve

135

Page 136: lf1.czlf1.cz/wp-content/uploads/228-vypracovane_otazky_interna... · Web viewprognoza – 50 % dosáhne kompletního uzdravení do 6 měsíců, 20 % má prodloužený průběh –

Vypracované otázky na INTERNU made by Edita Homolková, Monika Ambroziová, Adam Král, Petr Vermach a Vítek Křehnáč

Th – vysazení NSA (nejhorší je kys. acetylosalicylová, nejšetrnější jsou coxiby)- kys. acetylsalicylovou v enterosolventní formě, jinak NSA užívat s jídlem- náhrada v analgetické indikaci je paracetamol- inhibitory H+ pumpy

Kongestivní gastropatie – díky městnání v povodí v. portae (městnavá srdeční slabost, selhání pravého srdce, portální hypertenze

- edematózní, zarudlá sliznice, náchylná ke krváceníKO – asymptomaticky, anorexie, dyspepsie

Ménétrierova nemoc, gastropatia gigantea – obrovité řasy hlavně v těle žaludku (foveolární hyperplazie)- může docházet ke ztrátám bílkovin žaludeční sliznicí → hypoproteinémie (všechny třída

imunoglobulinů), otoky- častější výskyt ca

82.A TROMBOFILNÍ STAVY = 51.A

82.B NAUZEA, ZVRACENÍ, SINGULTUS

Nauzea – nevolnost, pocit na zvracení- objevuje se při mořské nemoci a jiných kinetózách, zažívacích obtížích různého původu, metabolických

poruchách, na počátku těhotenství, při závrati aj.Vomitus, zvracení

- reflexní vyprázdnění žaludečního obsahu- bývá vyvoláno poruchami přímo v trávicím ústrojí nadměrné rozepětí žaludku, požití dráždivých látek, infekce

apod. nebo vyprovokováno z mozku, event. z oblasti vnitřního ucha, kde je sídlo rovnováhy mořská nemoc a jiné kinetózy, těhotenství, některé otravy včetně alkoholu, selhání jater či ledvin, mozkové choroby či úrazy aj.

- může nebo nemusí být provázeno nevolností- velké zvracení vede ke ztrátám tekutin, iontů a narušuje vnitřní prostředí- ztráta HCl ze žaludku způsobuje metabolickou alkalózu, delší zvracení zhoršuje příjem živin apod.Th - antiemetika - ondansetron (5-HT3-R-antagonista), skopolamin (antimuskarinergní látka), moxastin+kofein/

promethazin (sedativní antihistaminika), chlorpromazin (sedativní neuroleptika)Singultus, škytavka

- náhlé, prudké a mimovolní stahy bránice vyvolané drážděním bráničního nervu nebo podrážděním mozkových center

- stahy bránice, která je hlavním dýchacím svalem, vedou k prudkému nasávání vzduchu do plic a ten při průchodu hrtanem způsobí prudké uzavření epiglottis, čímž vzniká typický zvuk

- častými provokujícími faktory jsou chladný nápoj s obsahem oxidu uhličitého, alkohol, přeplnění žaludku pivo, vzácněji některé nemoci mezihrudí, nádory, brániční kýla, plicní onemocnění apod.

- při dlouho trvající škytavce lze na krku provést obstřik, popř. i přetětí bráničního nervu nerv později regeneruje

83.A OBEZITA = 68.A

83.B ONEMOCNĚNÍ CHLOPNÍ PRAVÉHO SRDCE

Stenóza pulmonální chlopně- v dospělosti vzácné- porevmaticky/ díky karcinoidu (vysoké hladiny serotoninu)KO – mírnýDg- echoTh – balónková valvuloplastika

Pulmonální regurgitace- díky plicní hypertenzi/ stav po valvuloplastice

Trikuspidální stenóza - vzácná- porevmatická

136

Page 137: lf1.czlf1.cz/wp-content/uploads/228-vypracovane_otazky_interna... · Web viewprognoza – 50 % dosáhne kompletního uzdravení do 6 měsíců, 20 % má prodloužený průběh –

Vypracované otázky na INTERNU made by Edita Homolková, Monika Ambroziová, Adam Král, Petr Vermach a Vítek Křehnáč

KO - většinou jsou dominantní příznaky z vad levostranných chlopní, jinak mohou být příznaky pravostranné srd. insuf. (zvýšená náplň krčních žil, hepatomegalie, otoky)

Dg - askultace - diastolický šelest, nejlépe slyšet parasternálně vlevo/vpravo doleTh - katetrizační/operační valvuloplastika, bioprotéza

Trikuspidální insufícience- při dilataci pravé komory z přenesení chlopenní vady z levého srdce (relativní insuf. z trikuspidalizace mitrálni

stenózy/insuf.), bakteriální endokarditida u toxikomanůKO - viz trikuspidální stenózaDg - askultace (holosystolický šelest parasternálně vlevo dole, systolická pulzace jater)Th - většinou nevyžaduje náhradu, stačí vyléčit příčinu (mitrálni stenóza/insuf.), anuloplastika

84.A REVMATOIDNÍ ARTRITIDA A JUVENILNÍ IDIOPATICKÁ ARTRITIDA = 66.A

84.B PLICNÍ HYPERTENZE, COR PULMONALE

= zvýšení tlaku v plicnici nad hranici normy: systolický tlak > 35 mm Hgstřední tlak > 25 mm Hgdiastolický tlak > 12 mm Hg

- pro určení diagnózy jsou signifikantní hodnoty středního tlaku v plicniciPatologická klasifikace - dle hodnoty středního tlaku v plicnici rozlišujeme plicní hypertenzi lehkou, středně

těžkou a těžkou: střední tlak (mm Hg) systolický tlak (mm Hg)

norma < 25 < 35lehká 26 – 35 36 – 45středně těžká 36 – 45 46 – 60těžká > 45 > 60

- hodnoty systolického tlaku sice nejsou signifikantní pro diagnózu plicní hypertenze, ale tyto hodnoty lze zjistit při ECHO vyšetření a mohou při určení diagnózy významně pomoci.

Primární plicní hypertenze- se rozvíjí bez známé příčiny (hlavně mladší ženy) s postižením malých arterií (typické plexiformní léze), příčinou

je zřejmě primární poškození endotelu plicního řečiště- onemocnění má špatnou prognózu (většinou zmírají do 3 let na pravostranné srdeční selhání)KO – dominuje progredující dušnost s periferní cyanózou, bolesti na hrudi, únavnost

Sekundární plicní hypertenze- vzniká na základě jiného onemocněníPrekapilární plicní hypertenze - charakterizována normálními hodnotami tlaku v zaklínění (ten je roven tlaku v

levé síni)- vyskytuje se typicky u plicních chorob (CHOPN, plicní fibróza, sarkoidóza,

pneumokoniózy), chronické plicní embolii primárního postižení plicních arterií a po resekci plic, nebo také při hypoventilaci plic, vaskulitidy při systémových onemocněních pojiva

Postkapilární plicní hypertenze - bývá vždy ↑ tlak v zaklínění- příčina je v levostranném srdečním postižení (levostranné srdeční selhávání,

mitrální stenóza, hypertrofická kardiomyopatie), konstriktivní perikarditidaHyperkinetická plicní hypertenze - díkyí levo-pravým srdečním zkratům (perzistující ductus arteriosus, defekty

síňového nebo komorového septa, nebo stavy s vysokým minutovým srdečním výdejem (například hypertyreóza))

Diagnostická klasifikace - z hlediska příčin, prognózy a terapie onemocnění (tzv. Benátská klasifikace) odlišujeme:plicní arteriální hypertenze (PAH)plicní hypertenze při onemocnění levého srdceplicní hypertenze u respiračních chorob a nebo při hypoxémiiplicní hypertenze u chronické trombotické nebo embolické nemociplicní hypertenze z jiných příčin

Plicní arteriální hypertenze

137

Page 138: lf1.czlf1.cz/wp-content/uploads/228-vypracovane_otazky_interna... · Web viewprognoza – 50 % dosáhne kompletního uzdravení do 6 měsíců, 20 % má prodloužený průběh –

Vypracované otázky na INTERNU made by Edita Homolková, Monika Ambroziová, Adam Král, Petr Vermach a Vítek Křehnáč

Etio – přestavba stěn plicních arterií, především hypertrofii hladké svaloviny medie, což vede ke ↑ rezistence plicních arterií a tím i ke ↑ tlaku v plicním řečišti

- tlak v zaklínění má normální hodnoty (do 12 mm Hg)- u plicní arteriální hypertenze bývají obvyklé velmi ↑↑ hodnoty středního tlaku v plicnici okolo 60 mm Hg,

mohou se blížit až hodnotám v systémovém řečišti (špatná prognóza onemocnění)- výskyt tohoto onemocnění je malý (asi 10 na 1 milión obyvatel- patří sem především: primární (idiopatická) plicní arteriální hypertenze, hereditární plicní arteriální hypertenze

Plicní hypertenze při onemocnění levého srdce- nejčastější typ plicní hypertenze -při levostranném srdečním selhávání se ↑ tlak krve propaguje z levé síně do plicního řečiště- ↑ je tedy jak střední tlak v plicnici, tak i tlak v zaklínění (tlak v levé síni)- při mitrální stenóze, levostranném srdečním selhání- rozvoj plicní hypertenze zvyšuje mortalitu pacientů se srdečním selhávánímTh -léčba srdečního selhání

Plicní hypertenze u respiračních chorob- 2. nejčastější typ plicní hypertenze, primární onemocnění je v plicích, nikoli v plicním arteriálním řečišti- tlak v zaklínění je normální- CHOPN, intersticiální plicní fibróza, sarkoidóza- zvláštní léčba neexistuje, léčí se primární onemocnění

Chronická tromboembolická plicní hypertenze (CTEPH)- stav, kdy po plicní embolii nedojde ani přes adekvátní terapii k dostatečné rekanalizaci trombů, naopak dochází

k jejich reorganizaci, fibrotizaci, trvalému přichycení ke stěně arterií a možnému následnému narůstání (toto se děje u 2 - 4 % pacientů po proběhlé plicní embolii)

- tlak v zaklínění (pokud jsme schopni jej přes přítomnost trombů katetrizačně změřit) je normálníTh - chirurgická - odstranění reorgenizovaného trombu endarterektomií plicnice (PEA)

Plicní hypertenze z jiných příčin- sem patří všechny jinam nezařaditelné příčiny plicní hypertenze jako sarkoidóza, histiocytóza X,

lymfadenopatie a nádorová onemocnění.KO - neexistují symptomy, které by byly typické čistě pro plicní hypertenzi

- námahová dušnost, rychlá unavitelnost, bolest na hrudi, synkopy, hemoptýza, otoky dolních končetin ( a jiné příznaky pravostranné srdeční selhávání)

Dg -EKG – odhalí známky hypertrofie pravé komory (hrotnaté vlny P ve svodu II a III, vertikální sklon elektrické osy srdeční, vysoký kmit R ve svodu V1, invertované T ve svodech V1 - V3, prodloužení aktivační doby pravé komory)

- RTG hrudníku – může odhalit zvětšení pravostranných srdečních oddílů, zvětšení plicnice- ECHO – zjistí hypertrofii pravé komory, na základě dopplerovského vyšetření tlakového gradientu na

trikuspidální chlopni zjistíme orientačně systolický tlak v kmeni plicnice, insuficienci pulmonální nebo trikuspidální chlopně

- HRCT hrudníku – detailnější zobrazení srdečních oddílů, typicky zvětšení pravé komory, zbytnění její svaloviny ve srovnání s levou komorou (ta je utlačovaná a má tvar písmene D)

- spirometrie – odhalí funkční poruchu plic- celotělová pletysmografie – odhalí nebo vyloučí plicní embolii- ventilačně perfuzní scintigrafie – odhalí nesouhru mezi ventilací a perfuzí plic- Pravostranná srdeční katetrizace definitivně určí diagnózu! – měří se: střední tlak v plicnici (> 25 mm Hg)

tlak v zaklínění (> 12 mm Hg)- lab – markery srdečního selhávání (natriuretické peptidy, srdeční troponiny)

Th – farm - antikoagulační léčba – warfarin- blokátory kalciových kanálů (BCC) – vazodilatace (účinné asi u 5 % pacientů)

- specifická – terapie, která se snaží zabránit vazokonstrikci nebo podporuje vazodilataci v cévním řečišti - antagoniské receptoru pro endotelin – zabránění vazokonstrikci- inhibitory fosfodiesterázy 5 (PDE5I) – inhibicí enzymu zvyšují vazodilatační účinek NO- analoga prostacyklinu – vazodilatační a antiproliferační účinek, kvůli krátkému biologickému

poločasu musí mít pacient infuzní pumpu- transplantace plic – indikováni jsou pacienti s NYHA IV

138

Page 139: lf1.czlf1.cz/wp-content/uploads/228-vypracovane_otazky_interna... · Web viewprognoza – 50 % dosáhne kompletního uzdravení do 6 měsíců, 20 % má prodloužený průběh –

Vypracované otázky na INTERNU made by Edita Homolková, Monika Ambroziová, Adam Král, Petr Vermach a Vítek Křehnáč

- balónková septostomie (PBAS) – umělé otevření mezisíňového septa (vytvoření pravo-levého zkratu), část krve neprotéká plícemi, důsledkem je zvýšení srdečního výdeje levé komory ale i desaturace krve

- Určení správné terapie - Obvykle podle stupně dušnosti:NYHA I–II – konvenční terapieNYHA II – perorální léčba (antagonisté receptoru pro endotelin nebo inhibitory

PDE5), bráníme progresi do NYHA III nebo IVNYHA III – antagonisté receptoru pro endotelin, inhibitory PDE5, případně

prostaoidy, bráníme progresi a snažíme se o zlepšeníNYHA IV – prostanoidy podávané kontinuálně infuzí, zároveň zařazení do

transplantačního programu, snažíme se o zlepšení nebo přemostění k transpantaci

Cor pulmonale= horobné zvětšení pravé srdeční komory, které je způsobeno plicní hypertenzí v důsledku plicních onemocnění - později se projevují známky selhání funkce pravé komory Chronické - je hypertrofie pravé komory srdeční, která se rozvíjí jakožto následek onemocnění plic a progredující

prekapilární plicní hypertenze- střední tlak v a. pulmonalis roste nad 20 mmHg, pravá komora je nucena pumpovat krev proti

většímu odporu a reaktivně tak hypertrofujeAkutní - je dilatace pravé komory vzniklá jako důsledek akutní prekapilární plicní hypertenze

- nejčastější příčinou náhle vzniklé prekapilární plicní hypertenze je masivní plicní embolie- při náhlé masivní plicní embolizaci dojde ke zvýšení plicního odporu (ucpáním průsvitu cév a zároveň

reflexní vazokonstrikcí zbylého plicního řečiště)- kvůli větší plicní rezistenci je pro pravou komoru těžší pumpovat krev, ↑ se její práce, zároveň

městnáním venózní krve roste tlak v pravé síni → vyústí v dilataci pravé komory a jeji dysfunkci

85.B AKUTNÍ FORMY TEPENNÉ ISCHEMIE

- nejčastějšími příčinami akutních tepenných uzávěrů je embolie, trombóza (včetně trombózy cévních rekonstrukcí), trauma

Ptg - náhlý uzávěr vede ke vzniku nasedajícího trombu, který se šíří proximálě i distálně- stupeň ischémie záleží na možnosti kolaterálního oběhu – větší schopnost kolateralizace bude mít zdravé

řečiště ucpané embolem než chronicky zúžené řečiště, v němž vznikne trombóza0- ischémii toleruje nejlépe kůže a podkoží (až 12 hodin), svaly a kost podléhají nekróze po 6–8 hodinách, nervy

po 2–4 hodinách- účinky ischémie na buňky: tvorba laktátu, exprese xanthinoxidázy, vznik reperfúzního poškození, edému

buněk (může být způsobeno kompartment syndromem po obnově cirkulace)- myonefropatický metabolický syndrom je systémovým důsledkem reperfúze → zaplavení organismu kyselými

produkty a draslíkem, případně i myoglobinem vede k renální a respirační insuficienci s vysokou úmrtností- zdroje embolizace: nejčastěji srdce (po transmurálním IM), fibrilace síní, aneurysma, chlopenní vady, vzácně

myxom, iatrogenně, paradoxní embolizace- embolus se uchytí nejčastěji v místě větvení tepen. Na DK v odstupu arteria profunda femoris, ve větvení

popliteální tepnyKO - embolus v bifurkaci ilických arterií → náhle vzniklá bolest, chybění periferních pulzací, zblednutí a chlad kůže,

porucha citlivosti a hybnosti, příznaky mají punčochovitý nebo rukavičkový charakter

- pro embolický uzávěr svědčí negativní anamnéza klaudikací, normální pulzace na druhé končetině, arytmie- trombóza se projevuje klaudikacemi, pulzovým deficitem i na druhé končetině- při klinicky jasné embolii není většinou třeba další vyšetření, při trombóze se provádí arteriografie.

Th - celková terapie - okamžitě heparin – bolus 10 000 j i.v. (zabraňuje progresi nasedajícího trombu, který by mohl uzavřít kolaterální řečiště)

- maximální pozornost věnujeme celkovému stavu nemocného (hlavně kardiálně)- prevence myonefropatického syndromu – infúze glukózy s inzulinem (podpora ukládání

kalia do buněk), zlepšení diurézy hydratací a podáváním manitolu, PGE1

139

Page 140: lf1.czlf1.cz/wp-content/uploads/228-vypracovane_otazky_interna... · Web viewprognoza – 50 % dosáhne kompletního uzdravení do 6 měsíců, 20 % má prodloužený průběh –

Vypracované otázky na INTERNU made by Edita Homolková, Monika Ambroziová, Adam Král, Petr Vermach a Vítek Křehnáč

- místní terapie - nejefektivnější chirurgický výkon je embolektomie Fogartyho katétrem (lze provést v lokální anestézii, z nářezu zavedeme do tepny distálně od uzávěru katetr s nafukovacím balónkem na konci a uvolníme embolus)

- výjimečně je třeba přímá embolektomie (např. v bifurkaci arteria poplitea)- po dokonalé embolektomii můžeme přerušit heparinizaci (pokud ji nevyžaduje základní

onemocnění)- jen odstranění trombu dlouhodobě nestačí, je možno použít i další způsoby – trombektomie doplněná

revaskularizací, TEA nebo bypassem, event. PTA- farmakologická trombolýza (fibrinolýza) - absolutní kontraindikace: fluoridní vředová choroba, CMP před

méně než 3 měsíci, intrakraniální nádor, cévní malformace, bakteriální endokarditida- relativní kontraindikace: nedávné operace, trauma, hypertenze, srdeční vady, koagulopatie,

těhotenství;, jaterní nemoc, alergie na streptokinázu- urokináza, tPA (tkáňový aktivátor plazminogenu)- ortográdně při periferní trombóze nebo z druhé strany při trombóze v stehenní nebo pánevní

tepně, začínáme s instilací vysoké dávky, kterou po počáteční rekanalizaci snížíme

86.A CHRONICKÁ RENÁLNÍ INSUFICIENCE = 70.A

86.B MALABSRORPČNÍ SYNDROM

- soubor příznaků doprovázející choroby, v jejichž důsledku dochází hl. v tenkém střevě k poruše fcí zajišťující absorpci živin

- Porucha intraluminálního trávení (maldigesce), porucha vstřebávání živin (malabsorpce), abnormální sekrece trávicích šťáv, závažné narušení motility trávicí trubice, nedostatečný příjem potravy

Porucha intraluminálního trávení (maldigesce)Intra- a extrahepatální cholestáza – blokáda tvorby/ odtoku žluče do duodena → ↓ emulgace tuků

- primární biliární cirhóza, primární sklerotizující cholestáza, sekundární biliární cirhózaKO – plynatost, meteorismus, řídká stolice, bolesti břicha, statorea (mastná, šedá,

pronikavý zápach), projevy hypovitaminózy A/ D/ E/ KZevně sekretorická insuficience pankreatu – chronická etylická pankreatitida, raritně hereditární chronická

pankreatitida a pankreatitida při cystické fibróze, manifestace cystického adenomu pankratu

- destrukce musí postihnout více jak 90 %, aby se objevila klinika (steatorea)Žaludeční hypersekrece – Zollinger-Ellisonův syn., po operacích pro vředovou chorobu (syndrom retinovaného

žaludečního antra)- abnormálně ↓ pH → inaktivace pankreatické lipázy, zrychlená pasáž tenkým střevemKO – vředová symptomatologie (krvácení, perforace)

Porucha absorpce na úrovni enterocytůPolékové poškození enterocytů - toxické poškození NSA, cytostatiky (MTX), antibiotiky (druhotně při

enterokolitidě z přemnožení C. difficile)GIT infekce – Giardia lamblia (komtaminovaná voda, do duodena a žlučovách cest), při AIDS/ imunosupresi a

chemoterapii se může objevit enteropatie s atrofií sliznice- chronická kryptokokóza, salmonelóza, yersinióza

Autoimunní enteropatie – vzácné, těžká atrofie klků a sliznice v proximální části tenkého střeva- protilátky proti enterocytům

Celiakie – nejdůležitější jednotka, častěji v dospělostiWhippleova choroba (intestinální lipodystrofie) – infekce Tropheryma whipplei (ic parazit, ve fagocytech),

blokace lymfatikKO – průjmy, recidivující artalgie, teploty, váhový úbytekTh – co-trimoxazol

A-betalipoproteinemie – genetický defekt s poruchou systézy β-lipoproteinů → váznutí intracytosolické syntézy micel v enterocytech → jejich mobilizace do lymfatik → kumulace tuků ve sliznici střeva

KO – steatorea, hubnutí, akantocytóza ery, cerebelární ataxiePost-enterocytární příčiny

140

Page 141: lf1.czlf1.cz/wp-content/uploads/228-vypracovane_otazky_interna... · Web viewprognoza – 50 % dosáhne kompletního uzdravení do 6 měsíců, 20 % má prodloužený průběh –

Vypracované otázky na INTERNU made by Edita Homolková, Monika Ambroziová, Adam Král, Petr Vermach a Vítek Křehnáč

Vrozené a získané lymfangiektázie – sekundárně při obstrukci mezenteriálních uzlin chronickým zánětem (Crohnova ch., tbc), metastázami

- vzácně vrozené (kombinace s Ménétrierovou ch. žaludku a hypoplazií lymfatické drenáže tenkého střeva) → hyproteinemické otoky (protein loosing enteropathy)

Amyloidóza – kumulace amyloidu v submukóze, progresivní, přežití v rámci měsícůT-lymfom tenkého střeva – velmi maligní, spontánní perforace, nejčastěji muži kolem 60. Roku

- bývá komplikací refrakterní celiakieStřevní tbc – typicky v terminální ileu

Komplexní příčinySyndrom bakteriálního přerůstání – masivní kontaminace střevního obsahu koliformními bakteriemi

- poškození enterocytů toxiny a porucha inraluminálního trávení- steatorea, průjmy, střevní dyspepsieEtio – slepá klička (vytvořená pro bypas střeva), stenóza, těžká primární porucha motility

(diabetická neuropatie, autoimunitní myopatie)Syndrom krátkého střeva – po extenzivní resekci (více jak 100 cm, v 90 % pro mezenteriální ischemii), kdy zbude

méně než 80 cm tenkého střeva (hraniční je zbytek o 90-100 cm), horší je ztráta ilea- dobré je zachovat Bauhinskou chlopeň – nedochází ke kontaminaci tenkého střeva

koliformními bakteriemi- podíl na malabsorpci má i zrychlená pasáž a hypersekrece žaludečních šťáv (hypergastrémie

jako adaptační mechanismus)KO – průjem, hubnutí - často nutná parenterální výživa

Endokrinní a metabolické nemoci – hypertyreóza, hypokorikalismus, endokrinně aktivní tu GIT (gastrinom, karcinoid)

Selektivní malabsorpceLaktázová intolerance – nejčastější, vrozená/ získaná (po infekcích, zánětech, resekci žaludku)

- defekt disacharidázy-laktázy v kartáčovém lemu → laktóza je bakteriemi rozložena na kys. mléčnou → průjem osmotický

KO – bolest břicha, páchnoucí a imperativní stolice- vyloučení laktózy z diety

Malabsorpce žlučových kyselin – získaná/ vrozená porucha transportu v distálním ileu (specifický transport pro konjugované žluč. kys.)

- resekce pro Crohnovu ch- kŽK indukují v tlustém střevě mohutnou peristaltiku, ↑ sekreci hlenu a vodyKO – impreativní průjmy časně postprandiálně, málo objemná a řídká stolice, zpěněná a

žluté barvy- ↑ cholesterol v plasmě

Th – cholecystramin a vlákninaMalabsorpce B12 – aktivní vstřebávání v distálním ileu

KO – makrocytová anémie, megaloblastová přestavba kostní dřeně, porucha syntézy myelinu (syn. zadních míšních provazců)

- resekce pro Crohnonu ch., infekce Diphylobothrium latum, syn. bakteriálního přerůstání, chronická atrofická gastritida (nedostatek vnitřního faktoru z parietálních bb.)

87.A PLAZMOCYTOM = 60.A

87.B ONEMOCNĚNÍ MOZKOVÝCH CÉV, CÉVNÍ MOZKOVÉ PŘÍHODY

= onemocnění cévného původu s rychle se rozvíjejícími ložiskovými/ celkovými příznaky poruchy mozkové fce- 300/ 100 000 obyvatelKO - porucha hybnosti: parézy, plegie (hemi), poruchy citlivosti, poruchy vyšší nervové činnosti (porucha řeči -

dysfázie a afázie), poruchy vědomí (náhlý vznik bez prodromů - krvácení), poruchy zraku a zrakového pole, mozečkové a vestibulární poruchy, kmenové poruchy, bolesti hlavy, vegetat. příznaky (nauzea, zvracení)

Dg - anamnéza: charakter začátku (náhlý, postupný), předchozí příhody- interně: KVS onem., arytmie, srdeční selhání, metabol. příčina

141

Page 142: lf1.czlf1.cz/wp-content/uploads/228-vypracovane_otazky_interna... · Web viewprognoza – 50 % dosáhne kompletního uzdravení do 6 měsíců, 20 % má prodloužený průběh –

Vypracované otázky na INTERNU made by Edita Homolková, Monika Ambroziová, Adam Král, Petr Vermach a Vítek Křehnáč

- neurologické: stanovení rozsahu postižení- biochem. a hematolog. - doplňující vyš.: oční pozadí, CT a MR, doppler, angiografie, lumbální punkce (průkaz krvácení)

Mozková ischémie- důsledek poruchy prokrvení určité oblasti nebo celého mozku, s následnou hypoxií Etio - lokální - ložisková symptomatologie

- vaskulární (ateroskleróza, trombóza (nejčastější), zánětlivé postižení cév)- srdeční (embolie při chlopenní vadě, arytmie)- hematologické (abnormality koagulace)- celkové - difúzní hypoxické poškození mozku

- rozdělení podle průběhu: 1) tranzitorní CMP - TIA (tranzitomí ischemická ataka) - epizoda fokální mozkové, dysfunkce, která odezní do 24 h

- ateroskleróza, drobné embolizace, cévní spasmus2) reverzibilní CMP - RIND (reversible ischémie neurologie deficit) - trvá déle než

24 h a odezní do 14 dnů, někdy drobné trvalé následky 3) progredující CMP - SE (široké in evolution) - postupně narůstající fokální

mozková hypoxie s progresí klinických příznaků4) ireverzibilní CMP — CS (completed široké) - dokončená příhoda – ložisková

hypoxie mozku s trvalým funkčním deficitem- komplikace: 1) místní: edém mozku, mozkové krvácení

2) celkové: bpn, hluboká žilní trombóza, embolie plícnice, moč. infekce, dekubity, kontrakturyTh - zajištění vitálních fcí (podání kyslíku), odstranění vyvolávající příčiny (srdeční, metabolické), zajištění

adekvátního průtoku mozkovým řečištěm: zajištění dostatečného tlaku (170-150/ 100-85 torr), doplnění tekutin, ionotropní podpora

- zprůchodnění uzavřené cévy: trombolýza (rt PA) v časných fázích do 6h, angioplastika - farmakoterapie - antiagregační – ASA

- antikoagulační - heparin (dle APTT), warfarin (dle INR jen u embolie)- antiedematózní léčba - manitol, furosemid u SS

- operační řešení: u významných stenóz karotid (nad 70%), angioplastika.- všeobecná péče o bezvědomého, adekvátní výživa, časná rehabilitace a fyzioterapie

Mozková hemoragieEtio - arteriální hypertenze 80%, jiný původ (aneuryzmata, arter. malformace, venózní angiomy)

- epidurální krvácení - následek natržené důrami artérie (a. meningea media), rychlá progrese, vede v krátké době k poškození mozku a ke smrti, léčba chirurgicky

- subdurální - u KCP, natržené přemosťující žíly, špatná prognóza - subarachnoideální krvácení (SAK) - krvácení do subarachnoideálního prostoru, častá příčina – aneuryzma;

meningeámí příznaky - intracerebrální, intraventrikulární

Th - zajištění vitálních fcí (podání kyslíku), odstranění vyvolávající příčiny (cévní, hematologické), udržení přiměřeného nitrolebního tlaku, tlumení bolesti

- operační řešení - nutné u expanzivního hematomu, časné odstranění putaminálního krvácení (do 6 h) zlepšuje prognózu

- farmakoterapie - antiedematózní léčba - manitol, furosemid u SS, zábrana spazmů mozkových cév – nimodipin

- klid na lůžku a obecná podpůrná péče o bezvědomého pacienta, adekvátní výživa- časná rehabilitace a fyzioterapie

88.A DIABETES MELLITUS II. TYPU = 50.A

88.B KOMOROVÉ ARYTMIE

Komorová tachykardie- mechanismus reentry/ ↑ automacie v pracovním myokardu komor/ v převodním systému distálně od Hisova

svazku

142

Page 143: lf1.czlf1.cz/wp-content/uploads/228-vypracovane_otazky_interna... · Web viewprognoza – 50 % dosáhne kompletního uzdravení do 6 měsíců, 20 % má prodloužený průběh –

Vypracované otázky na INTERNU made by Edita Homolková, Monika Ambroziová, Adam Král, Petr Vermach a Vítek Křehnáč

= 3 a více širokokomplexové QRS (>120 ms) komorového půvosu o frekvenci 100/ min (triplet komorových extrasystol)

- monomorfní/ polymorfní- setrvalé (déle než 30 s)/ nesetrvalé (hemodynamicky závažné)- přítomnost/ nepřítomnost strukturální vady srdceEKG - široké QRS >140 ms, šíňookomorová disociaceTh – prekordiální úder – 1. Pomoc

- elektrická kardioverze – v katodobé anestezii- amiodaron – po recidivě po elektrické kardioverzi- suplementace MG a K- prevence – betablokátory- kardioverter-defibrilátor

Akcelerovaný idiovetrikulární rytmus – díky urychlené automacii komor o frekvenci 50-110/ min- u aktuního IM při reperfuzi tepny

Idiopatická komorová tachykardie – zvýšená automacie v ektopickém fokusu ve výtokovém traktu PKU ICHS – vyžaduje většinou rychlou léčbuU dilatační kardiomyopatie – fibróza umožňující vznik reentry tachykardiíU hypertrofické kardiomyopatie – příčina náhlé smrtiTorsade de pointes – díky dlouhému QT intervalu (norma do 44 ms), synkopy, náhlá smrt

Fibrilace komor - 300/ min, neuspořádaná elektrická aktivita vedoucí k oběhové zástavě- často v průběhu aktuního IM, často jí předchází setrvalá komorová tachykardieTh – elektrická defibrilace a KPR

89.A ANÉMIE SIDEROPENICKÁ; ANÉMIE CHRONICKÝCH ONEMOCNĚNÍ = 75.A

89. B GRANULOMATÓZNÍ PLICNÍ PROCESY

Sarkoidóza- systémové granulomatózní onemocnění neznámé etio postihující hlavně nitrohrudní uzliny a plíce- obvykle mladí/ střední věkPtg – tvorba epiteloidních granulomů, granulomy se buď zresorbují nebo jsou nahrazeny jizevnatou tkáníKO – často jako oboustranná hilová lymfadenopatie, plicní infiltrace, oční a kožní léze (játra, slezina, uzliny, svaly,

kosti, nerv. sys.)- může začít horečkami, nodózním erytémem, artralgiemi, myalgiemi, hubnutí, únava, dráždivý chronický

kašel, dušnost- akutní - Löfgrenův sy = trias: oboustranné ↑nitrohrudních uzlin + nodózní erytém (bérce, kolem kotníků) +

negativní tuberkulinový test- chronická – trvá alespoň 2 roky

- mimohrudní projevy: sarkoidóza očí (fotofobie, slzení, seloda…), myokardu (synkopy, arytmie, poruchy vedení), CNS (parézy hlav. Nervů, bolesi hlavy, parestézie…), artralgie, dysfagie, bolesti břicha, nefrokalcinóza, nefrolitiáza

- Stadium I (symetrické zvětšení stínů plicních hilů)- Stádium II (polycyklické zvětšení hilových uzlin + mikronodulární stíny ve středních plicních polích)- Stádium III (pouze plicní změny)

Dg – ↑ FW, ↑ CRP, hyperklacémie, hyperkalciurie (tvorba vit D v uzlech), aktivita ACE, ↑ cirkulujících imunokomplexů

- RTG, HRCT –(stanovení dg, aktivity ), MR, PET-CT- fční vyšetření často v normě/ restriktivní porucha, BAL (20-40 % lymfocytů), biopsie

Th – zahájíme při progresi restrikčního onemocnění nebo mimohrudních formách, možné spontánní uzdravení- NSA na polyartralgie- glukokortikoidy (Prednison 40mg a postupně snižovat, měla by trvat aspoň 6 měsíců), při oční a kožní formě

podáváme kortikoidy i lokálně- při kontraindikaci kortikoidů dáváme antimalarika

143

Page 144: lf1.czlf1.cz/wp-content/uploads/228-vypracovane_otazky_interna... · Web viewprognoza – 50 % dosáhne kompletního uzdravení do 6 měsíců, 20 % má prodloužený průběh –

Vypracované otázky na INTERNU made by Edita Homolková, Monika Ambroziová, Adam Král, Petr Vermach a Vítek Křehnáč

Wegenerova granulomatóza - chronická granulomatózní nektrotizující vaskulitida postihující plíce a ledviny (i kůže klouby, oči, srdce,

nervovou soustavu)- nejčastěji mezi 40.-60. rokemEtio - není jasná, uplatňuje se genetika i faktory prostředí (resp. infekce, alergie)KO - onemocnění se vyskytuje ve 3 formách – lokalizovaná forma s izolovaným postižením HCD nebo DCD,

systémová forma a generalizovaná forma- postižení HCD (u 90 %) se projeví chronickou rýmou a sinusitidou, epistaxemi a poruchou sluch- postižení DCD se projeví kašlem, hemoptýzou a bolestí na hrudi s dušností

Dg - detekce ANCA – protilátek (cANCA, pANCA)- RTG – noduly/ nehomogenní infiltráty/ ložiskové stíny často bilaterálně s kavitacemi, pleurálním výpotkem a

stenózou trachey- charakteristické jsou známky krvácení do alveolů s obrazem DAH (difuzní alveolární hemoragie)

- přesněji HRCT nebo BAL- Histologie - průkaz epiteloidních granulomů s nekrózou a vaskulárními změnami

Th - cyklofosfamid + kortikoidy, kotrimoxazol pro prevenci recidiv zánětů HCDChurg-Strauss syndrom

- vzácné onemocnění postihující především mužeKO - probíhá ve 3 fázích: 1. Fáze – alergické projevy (rinitida, nosní polypy, asthma bronchiale)

2. Fáze – eozinofilie v krvi a ve tkáních (dominuje plicní postižení, výpotky)3. Fáze – systémová vaskulitida s projevy postižení orgánů (↑ FW a IgE…)

Dg –lab - ↑ FW, eozinofílie, ↑ IgE, pozitivní revmatický faktor, většinou pANCA- histologie - nekrotizující vaskulitida malých a středních cév, granulomy a eozinofilní infiltrace tkání

Th - Prednisonem 40 – 60 mg + imunosupresivyGranulomatóza z Langerhansových buněk (plicní histiocytóza)

- vzácné onemocnění na podkladě bronchiolocentrického granulomatózního zánětu s tvorbou cystických útvarů a uzlů

- vyskytuje se u kuřáků – imunopatologická reakce na složky tabákového kouře- klinicky je nápadná námahou dušnostDg - HRCT, biopsie s průkazem X tělísek a pozitivitou S100 proteinu

- v BAL jsou CD1+ buňky- průběh variabilní (po zanechání kouření se stav může zlepšit nebo stabilizovat) v pokročilých stádiích je

indikována transplantace plic

90.A PORUCHY METABOLISMU SODÍKU A VODY = 54.A

90.B OBĚHOVÁ ZÁSTAVA A KARDIOPULMONÁLNÍ RESUSCITACE

Primární zástava oběhu – během několika vteřiv vede k bezvědomí a zástavě dechu díky zástavě toku krve do mozku

– akutní IM, primární srdeční arytmie, plicní embolie, CMP, mozkolebeční poranění, hypoxie z různých příčin, velká ztráta krve, úraz elektrickým proudem, tonutí, anafylaxe…

Primární zástava dechu – vede k zástavě objehu do 2 minut- po cca 5 minutách ireverzibilní hypoxické poškození bb. mozku- aspirace cizího tělesa, zhoršení plicního onem., broncho-/laryngospasmus, edém laryngu,

transverální léze krřní míchy, nervosvalové poruchy…Základní KPR dospělých

- záklon hlavy → kontrola dechu → telefonovat → 30 kopresí hrudníku (100/ min, 4-5 cm) → 2 dechy → do vyčerpání/ příjezdu záchranářů

- pokud je k dispozici defibrilátor → defibrilace (automaticky vyhodnotí přístroj) → 2 min KPR → opakovat defibrilaci

Udržení průchodnosti dýchacích cest – záklon hlavy, předsuneme spodní čelist a pootevřeme pacientovi ústa (pozor na poranění krční páteře, poté nezakláníme hlavu)- zapadlý jazyk u bezvědomých, vyčištění dutiny ústní

144

Page 145: lf1.czlf1.cz/wp-content/uploads/228-vypracovane_otazky_interna... · Web viewprognoza – 50 % dosáhne kompletního uzdravení do 6 měsíců, 20 % má prodloužený průběh –

Vypracované otázky na INTERNU made by Edita Homolková, Monika Ambroziová, Adam Král, Petr Vermach a Vítek Křehnáč

- cizí těleso zapadlé hlouběji v dýchacích cestách - vyzvání ke kašli (účinnější než manévry)- Heimlichův manévr - dospělý pacient - několik kompresí nadbřišku dorsálním a

proximálním směrem proti bránici- těhotná pacientka - komprimujeme laterální partie

hrudníku- novorozenci - obrátíme je „sendvičovým manévrem", dítě

pak leží hrudníkem a nadbřiškem na naší dlani a předloktí, hlavu má níže, poklepem na záda se pokusíme pomoci vypuzení cizího tělesa

- jsou-li tyto manévry neúspěšné, můžeme ještě zkusit hlubokým umělých vdechem „zatlačit" cizí těleso do jednoho z bronchů a uvolnit tak alespoň ventilaci jedné plíce

Dýchání - po uvolnění dýchacích cest zjišťujeme přítomnost spontánního dýchání, pakliže není přítomno, zahájíme neprodleně umělé dýchání z plic do plic

- po nádechu pevně přiložíme ústa na ústa pacienta, 2 prsty uzavřeme jeho nos a vydechneme do jeho úst, pozorujeme, zda se zvedá pacientův hrudník, výdech je pasivní děj

- vdech má mít objem cca 700-1000 ml a má trvat cca 1,5-2 sekundy, 2 dechyCirkulace - pacient musí lěžet na zádech na tvrdé podložce, palpačně se zjišťuje přítomnost pulsu na velkých

tepnách (a. carotis - palpační místo je vzdáleno 2 prsty laterálně od prominence štítné chrupavky do prohlubně mezi hrtanem a kývačem hlavy), palpace má trvat 5 sekund, není-li přítomen pulz, zahájíme neprodleně nepřímou zevní srdeční masáž

- místo komprese hrudníku - dva prsty nad mečovitým výběžkem ve střední čáře sterna, hrudník komprimujeme tak, že se nakloníme nad pacienta a pres natažené ruce přenášíme váhu celého těla tak, aby se sternum klesalo o 3-5 cm

- dýchání se kombinuje se srdeční masáží v poměru 2:30, nejnověji ve stejném poměru i při 2 zachráncích, u malých dětí v poměru 1:5

Stabilizovaná poloha – po obnově normálního dýcháníRozšířená neodkladná resuscitace

- její postup se neliší od laické resuscitace, pouze díky zkušenostem zdravotnického personálu jsou všechny kroky prováděny účinněji, včetně podávání léků a použití defibrilátoru (při včasném zastižení fibrilace se postup mění z pravidla ABC na pravidlo CBA)

Zajištění dýchacích cest – vzduchovod (brání jen zapadání jazyka)/ endotracheální intubace (brání i aspiraci, umožňuje použití ventilátoru)

- ambuvak (samorozpínací vak s obličejovou maskou)- po zajištění se pokračuje masáží 100/ min bez pauz a vdechy jsou prováděny 10/ min

Defibrilace – maximální výboj při fibrilaci komor a komorové tachykardii bez pulzové aktivity- silný prekoriální úder v prvních vteřinách komorové tachykardie

Léky - je zapotřebí zajistit žilní vstup, nejlépe do centrální žíly nejblíže srdci (vena jugularis externa), nepodaří li se žilní vstup zajistit, mohou se léky podávat do trachey a bronchiálního stromu, odkud se mohou vstřebat do krevního řečiště, dávka musí být ale 3-5x násobek dávky IV

- Adrenalin - v bolusové dávce 1 mg i.v. , při asystolii, bezpulzové srdeční aktivitě, při přetrvávající fibirliaci/ tachykardii komor po 2. výboji

- každých 2-5 minut do obnovy spontánní cirkulace- Amiodaron - při přetrvávající fibrilaci komor rezistentní na elektrickou defibrilaci, a to bolusovou dávkou

300 mg i.v.- Atropin - parasympatolytikum pro léčbu bradykardie, podává se ve stejné dávce a frekvenci jako adrenalin,

výhodný, protože nezpůsobuje vazokonstrikci v koronárním a mozkovém řečišti, také napomáhá změně jemné terminální fibrilace komor ve fibrilaci hrubovlnnou, která se lépe ovlivní defibrilací

- Lidokain, trimekain - lokální anestetikum s antiarytmickým účinkem (zvyšuje práh pro komorovou fibrilaci), podává se v dávce 1 mg / 1kg hmotnosti i.v., může se podávat i opakovaně a po úspěšném zvládnutí fibrilace komor se podává kontinuální infuzí rychlostí 1-2 mg / min, většinou po dobu 48-72 hodin

- 10 ml 10 % MgSO4 při podezření na hypomagnezémiiElektrická defibrilace

145

Page 146: lf1.czlf1.cz/wp-content/uploads/228-vypracovane_otazky_interna... · Web viewprognoza – 50 % dosáhne kompletního uzdravení do 6 měsíců, 20 % má prodloužený průběh –

Vypracované otázky na INTERNU made by Edita Homolková, Monika Ambroziová, Adam Král, Petr Vermach a Vítek Křehnáč

- zásady elektroimpulzoterapie - předpokladem úspěšnosti léčby je okysličený myokard, lékař provádějíc! léčbu elektrickým výbojem je zodpovědný za bezpečnost druhých

- kardioverze - depolarizace stejnosměrným proudem, který je synchronizován s EKG nemocného tak, aby nezapadl do vulnerabilní fáze a nezpůsobil fibrilaci komor

- indikace - komorová tachykardie, supraventrikulární tachykardie se známkami oběhového selháváni

- KO - pomalá odpověď komor na fibrilaci síní, arytmie vyvolané léky (intoxikace digitalis)- pod celkovou i.v. anestézií přikládáme jednu elektrodu pod pravou kliční kost a druhou nad

srdeční hrot, aplikujeme počáteční výboj v místě dostatečné vlny R o síle 50 J (u dětí 3,5 J na kg tělesné hmotnosti), při neúspěchu zvýšíme sílu na dvojnásobek, dojde-li k fibrilaci komor, okamžitě defibrilujeme

- defibrilace - depolarizace stejnosměrným proudem, který není synchronizován s EKG- indikace -fibrilace komor, komorová tachykardie s nehmatným pulzem na velkých cévách- elektrody se přikládají stejně jako u kardioverze, provede se série 3 elektrických výbojů 200 J, 200

J a 360 J (u dětí 50 J, maximálně 150 J), poté se provádí KPR a po dvou minutách se opakuje série 3 elektrických výbojů 360 J, 360 J a 360 J

- v průběhu resuscitace se podává adrenalin a lidokain- přetrvává-li i přes opakované defibrilace fibrilace komor, podává se amiodaron

Odlišnosti v KPR u novorozenců a malých dětí (do 8 let)- poměr umělých dechů a kompresí hrudníku je u novorozenců 1:3, u malých dětí 1:5- u novorozenců se zahajuje nepřímá srdeční masáž při poklesu tepové frekvence pod 60 za minutu- dýchání z plic do plic se provádí metodou z plic do úst a nosu, u novorozenců je frekvence dýchání vyšší (30 za

minutu)- tep se hmatá na brachiální tepně na paži- kompresní místo na hrudníku pro nepřímou srdeční masáž se určuje přiložením tří prstů pod pomyslnou

spojnici obou prsních bradavek na sternum, prst nejblíže této spojnice se zvedne a hrudník se komprimuje zbylými dvěma prsty, frekvence kompresí je 100 -120 za minutu

- léky lze podávat i intraoseálně a to pod tuberozitu tibie 1 cm mediálně od střední čáry- záchranný systém se aktivuje po 1 minutě resuscitace

91.A HYPOTYREÓZA = 78.A

91.B ANEURYZMA A DISEKCE AORTY

Aneuryzma aorty- abnormální dilatace aortální stěny (vřetenovité při rozšíření celého obvodu, vakovité při rozšíření jen části

obvodu)- 4 cm v hrudní arotě, 3 cm v břišní aortě- častěji u mužů, kuřáků, starších 50 let, hypertenze, hypercholesterolémie…Etio - ateroskleróza (většinou subrenálně, méně často suprarenálně nebo torakálně) + degenerativní proces

- Marfanův sy (arachnodactylie + hypermobilita kloubů + skolióza -*- dislokace čočky + aneryzma aorty, mitrální insuf.), aortitis (syfilis, Takayasuova choroba, Reitrův sy)

KO – většinou asymptomatické, při ruptuře 90 % úmrtnost- abdominální forma - častěji muži (> 65 let), většinou asymptomatický průběh (náhodný nález), jinak bolest

kolem pupku s propagací do třísel, tromboembolické komplikace, komprese okolí (dyspepsie, tromóza hlubokých žil), ruptura (retroperitoneálně, intraperitoneálně, aortokavální fistula)

- torakálnl forma - může být bolest na hrudi, komprese dýchacích cest, horní duté žíly apod.Th - operace - otevřenou cestou, už moc ne

- implantace stentu endovaskulárním výkonem- u asymptomatických – aneurysma >5,5 cm- ACEI, statiny

Disekce aorty= podélní rozštěpení stěny, katastrofální- častěji muži 60-80 let

146

Page 147: lf1.czlf1.cz/wp-content/uploads/228-vypracovane_otazky_interna... · Web viewprognoza – 50 % dosáhne kompletního uzdravení do 6 měsíců, 20 % má prodloužený průběh –

Vypracované otázky na INTERNU made by Edita Homolková, Monika Ambroziová, Adam Král, Petr Vermach a Vítek Křehnáč

- trhlína v intimě aorty → krev se přes defekt dostává z pravého lumen do oblasti médie aorty a tlakem vytváří falešné Iumen (většinou se distálně vytvoří další defekt, kterým se krev vrací do pravého Iumen)

- defekt se může šířit jak po směru toku i proti směru toku krveEtio - AHT (80%), ateroskleróza, aneuryzma aorty, koarktace aorty, kolagenní poruchy (Marfanův sy), trauma,

iatrogenní příčina (katetrizace), bikuspidální aortální chlopeň s poststenotickou dilatací, těhotenstvíKlasifikace - typ A (60%) - postižení ascendentní aorty, často přechází i na descendentní aortu

- typ B - postižení descendentní aorty distálně od a. subclavia sin.KO - náhlá krutá trhavá bolest na hrudi, často propagující se mezi lopatky, často s kolapsem, hypertenze (pokud

není ruptura), může být asymetrie v pulzech na pažích, karotidách nebo stehenních tepnáchDg - dopplerECHO - nejlépe transezofageální, může být aortální insuf., někdy je vidět míhání membrány direkce

- CT, MRI, angiografieKomplikace - krvácení (hemothorax, hemoperitoneum)

- aortální insuficience- obstrukce větví aorty (IM, paraplegie, cévní ileus, selhání ledvin, akutní ischemie končetin)

Th - musí být rychlá (každou hodinu letalita stoupá o 1 %)- akutní terapie - nitroprusid sodný/beta-blokátor (hlavní je snížit sTK pod 100 mm Hg), analgetika- typ A - okamžitá operace (Dacronový štěp)- typ B - konzervativní farmakoterapie, operace pouze pokud je ruptura nebo známky ischemie orgánů/

končetin- další možností terapie je endovaskulárnl katetrizační zákrok na aortě, kdy se vytvoří distálně umělá

komunikace mezi falešným a pravým luminem nebo se implantuje stent

92.A LEDVINY A DIABETES = 69.A

92.B LÉČBA ARTERIÁLNÍ HYPERTENZE – ZÁKLADNÍ TŘÍDY ANTIHYPERTENZIV, HLAVNÍ VLASTNOSTI

- cíl : TK <140/90 mm Hg u všech, <130-135/80-85 mm Hg u DM/ metab. skóre, SCORE >5 %, renální dysfce a proteinurie/ CMP

Nefarmakologická - pokud po 3 měsících není zlepšení → farmakoterapie (u těžké HT a orgán. změn od začátku)Kauzální - odstranění stenózy (aorta, a. renalis), léčba endokrinologické příčiny…Farmakologická

- monoterapie - zahájení malými dávkami, postupně zvyšovat-úspěšná u 30-45 %

- kombinační léčba - zahájení u těžké hypertenze s TK >180/110 mm Hg, u středně závažné hypertenze (TK >160/100 mm Hg) s vysokým KV rizikem

- u mírné hypertenze s DM- při neúspěchu monoterapie

- Hypertrofie LK - nejvhodnější ACEI, ARB, BKK, β-blokátor, méně vhodná diuretika mimo indapamidu- ICHS - stav po AIM – β-blokátor lékem volby, ACEI nebo ARB při srdečním selhání (NYHA II-IV) nebo

asymptomat. dysfunkci LK- angina pectoris – β-blokátor

- Srdeční selhání - ACEI, ARB, diuretika, malé dávky β-blokátoru (NYHA II-IV)- Diabetes mellitus - ACEI, ARB, BKK, β-blokátory jen selektivní, alfa-adrenergní blokátory- HT starších osob včetně ISH - diuretika, β-blokátory, BKK(dihydropyridiny vyššígenerace), ACEI- Parenchymatózní onemocnění ledvin - ACEI (ev. kombinace s non-dihydropyridinovými BKK), ARB

- diuretika – u osob starších 65let, při současném srdečním selhání, při retenci sodíku a vody -thiazidová (hydrochlorthiazid 12,5-25 mg 1x denně), jejich účinek ↓s klesající funkcí ledvin, pak

místo nich furosemid- kličková (furosemid 40mg 1xdenně)- s vazodilat. účinkem (metipamid, indapamid)- kalium šetřící - spironolakton 25-50 mg 2-3x denně, amilorid 5-10 mg 2-3x denně, eplerenon 50-

100 mg/ den- β-blokátory (sympatolytika) – nejčastější lék k lehké - střední HT v monoterapii a těžké v kombinaci s ACEI

nebo diuretiky

147

Page 148: lf1.czlf1.cz/wp-content/uploads/228-vypracovane_otazky_interna... · Web viewprognoza – 50 % dosáhne kompletního uzdravení do 6 měsíců, 20 % má prodloužený průběh –

Vypracované otázky na INTERNU made by Edita Homolková, Monika Ambroziová, Adam Král, Petr Vermach a Vítek Křehnáč

- neselektivní (metipranol 10-40 mg 3/ den)- neselektivní s ISA (bopindolol, pindolol)- selektivní (metoprolol 100-200 mg 2/ den)- selektivní s ISA (acebutol, celiprolol)

- blokátory α-receptorů - ACEI - mají antihypertenzní účinek, kardioprotektivní (prevence dilatace), vazoprotektivní (regrese

hypertrofické cévní stěny), renoprotektivní (↓ hyperfiltraci a proteinurii), vhodná při orgánovém postižení

- ramipril, captopril, trandolapril - blokátory Ca kanálů – tlak ovlivňují syst. vazodilatací, vhodné i při astmatu

- nifedipin, amlodipin,verapamil, diltiazem- inhibitory rec pro angiotenzin II - losartan, valsartan, bez NÚ, indikace jako u ACEI

Hypertenzní krize - život ohrožující stav s nutností hospitalisace na JIP- hypertenzní encefalopatie, HT se srdečním selháním, HT u akutních koronárních syndromů, HT u

subarachnoidálního krvácení a CMP, u feochromocytomu, po požití drog, preeklamsie, eklampsie…- cíl je v 1. hodině ↓ o 20-25 % původní hodnoty/ dosažení 160-150/110-100 mm HgIschemická CMP – v prvních dnech se autoregulací zvyšuje tlak a pak spontánně klesá

- nezasahovat pokud není vyšší než 200/120 nebo nejsou další závažné stavy- BB

Krvácení do mozku – podobné, začínáme snižovat TK od 160/110Hypertenzní encefalopatie – BB i.v.Levostranné srdeční selhání – nitráty, furosemidDisekce aorty – nitroprusid sodný, BBRenální insuficience – clonidin, furosemid

93.A CUSHINGŮV SYNDROM = 77.A

93.B LÉČBA CHRONICKÉ ICHS - viz 11.A

94.A ANÉMIE SIDEROPENICKÁ; ANÉMIE CHRONICKÝCH ONEMOCNĚNÍ = 75.A

94.B LÉČBA AKUTNÍCH KORONÁRNÍCH SYDROMŮ

= nestabilní angina pectoris + akutní infarkt myokardu + náhlá srdeční smrtEtio – ruptura nestabilního aterosklerotického plátu → nasedající intraluminální trombóza věnčité tepny

→ částečný uzávěr → nestabilní AP (nedojde k nekróze, nejsou kardiomarkery – troponin a kreatinkináza)→ IM s ST depresemi (NSTEMI)

→ úplný uzávěr → IM s ST elevacemi (STEMI)Th aktuního infarktu myokardu

- jestliže STEMI, pak patří okamžitě do kardiocentra k perkutánní koronární intervenci (u NSTEMI je oněco víc času)

- antiagregancia – ASA - 500 mg p.o.- clopidogrel – 600 mg p.o. (inhibice ACP rec.), pak 150 mg/ 24 h po 7 dní a dále jen 75 g/ 24 h

-antikoagulace – nefrakcinovaný heparin – i.v.- oxygenoterapie- anodyna – fentanyl (100 μg), morfin (10 g) – i.v., úleva a potlačení sympatoadrenální stimulace- β-blokátory – hl. působení na β1 rec. (↓ spotřeby O2), hl. u tachykardie a hypertenze

- metoprolol 5 mg i.v.- KI: bradykardie, AV blokády, hypotenze, asthma bronchiale

- nitráty – nitroglycerin, pro potlačení acinózní bolesti (KI hypotenze)- statiny - sekundární prvence, vede ke stabilizaci plátú a zlepšuje fci endotelu- antiarytmika? – pokud jsou komorové tachykardie

- amiodaron – 150 mg i.v.

148

Page 149: lf1.czlf1.cz/wp-content/uploads/228-vypracovane_otazky_interna... · Web viewprognoza – 50 % dosáhne kompletního uzdravení do 6 měsíců, 20 % má prodloužený průběh –

Vypracované otázky na INTERNU made by Edita Homolková, Monika Ambroziová, Adam Král, Petr Vermach a Vítek Křehnáč

- nemocniční léčba – systémová trombolýza – pokud není kardiocentrum v dosahu 60 min- u STEMI a stavy s bolestí na hrudi s „nediagnostickým“ EKG- tPA 20 mg i.v.

- perkutánní koronární intervence – klasická selektivní koronarografie

95.A CHRONICKÁ MYELOIDNÍ LEUKÉMIE = 65.A

95.B LÉČBA CHRONICKÉHO SRDEČNÍHO SELHÁNÍ

Kauzální - léčba AHT, ICHS, myokarditidy, perikarditidy, arytmie, srdeční vady, transplantace srdce

Nefarmakologická- monitorace tělesné hmnotnosti a při její ↑ o 2 kg/ 3 dny vyhledat lékaře- při retenci tekutin omezit NaCl na <4 g/ den- redukce váhy při nadváze a obezitě, nekouřit, omezení alkoholu, přiměřená tělesná aktivita

Farmakoterapie- oxygenoterapie s pozitivním endexspiračním tlakem- odstranit periferní vazokonstrikci => ulehčení srdci - ACE inhibitory/ATH-antagonisté, nitráty (s převážně žilním

účinkem - nitráty, nitropusid sodný, s převážně arteriálním účinkem – hydralaziny)- odstranění retence Na a vody – diuretika (furosemid)- ↑ kontraktility – digoxin- normalizace SF - digoxin, antiarytmika- ochrana před škodlivou neuroendokrinní aktivací – ACEI/ AT II - antagonisté, β-blokátoryACE inhibitory – 1. volba u systolické dysfce a u asymptomatické systolické dysfce LK, i u diastolického selhání

- brání přeměně angiotenzinu I na vysoce vasokonstrikční angiotenzin II a současně blokují degradaci vazodilatačně působícího bradykininu → vazodilatace → ↓ rezistence; inhibují sekreci aldosteronu (mohou vést k hyperkalémii, !! proto pozor na kombinaci s kalium šetřícími diuretiky), ↓ účinek sympatoadrenální stimulace a brání remodelaci srdce

- zabraňují/ oddalují vznik srdečního selhání, ↓ riziko IM a náhlé srdeční smrti- KI při oboustranné stenóze renálních tepen a anamnéze angioedému při předchozí léčbě- ramipril – 1 x 10 mg/ den- perindopril - 1 x 4-8 mg/ den- trandolapril – 1x 4 gm/ den

Blokátory receptorů AT1 pro angiotenzin II - účinky jako ACE inh. akorád nekumulují bradykinin který je zodpovědný za NÚ – dráždivý kašel

- losartan, valsartanβ – blokátory - 1. volba u systolické dysfce a u asymptomatické systolické dysfce LK po IM

- ↓ mortalitu, riziko náhlé smrti, prodlužují přežití, ↓ nutnost hospitalizace, zlepšují NYHA- začínáme nízkou dávkou, kterou titrujeme k optimu (z2násobení po 2 týdnech)- bisoprolol, meroprolol, carvediol, nebivolol

Diuretika - při retenci tekutin (plicní městnání, otoky končetin), vždy kombinace s ACEI a β-blokátorem, neovlivňují mortalitu

- kličková – furosemid (brání reabsorbci Na v Henleově kličce, rychlý účinek; při střední a velké retenci, možná kombinace s hydrochlorthiazidem)

- thiazidová – hydrochlorthiazid (pomalejší nástup, ale delší účinek → vhodné k udržovací léčbě), užíváme při mírných retencích (při těžkém renálním selhání nefungují, cl kreatininu <30 ml/ min)

- kalium šetřící – když hypokalémie při léčbě ACEI/ ATI, KI při K >5 mmol/l, max 50 mg/ den- eplerenon – po nedávném IM se systolickou dysfcí LK nebo srdečním selháním

+ ionotropní léky - digitalisové glykosidy – na membránách kardiomyocytů blokují Na+ pumpu → ↑ Na+ v buňce → ↑ výměna Na+/Ca2+ a ↑í intracelulárního Ca2+ vedoucí k ↑ kontraktility

- +inotropní, - chronotropní, - dromotropní

149

Page 150: lf1.czlf1.cz/wp-content/uploads/228-vypracovane_otazky_interna... · Web viewprognoza – 50 % dosáhne kompletního uzdravení do 6 měsíců, 20 % má prodloužený průběh –

Vypracované otázky na INTERNU made by Edita Homolková, Monika Ambroziová, Adam Král, Petr Vermach a Vítek Křehnáč

- digoxin – hl. jako antiarytmikum a vagotinikum, indikací je chronické srdeční selhávání při systolické dysfci LK a fibrilaci síní s rychloou komorovou odpovědí, lepší v kombinaci s β-blokátory

- KI: bradykardie, AV blokády 2. a 3. stupně, sick sinus syndrom, hypertroficko-obstrukční kardiomyopatie, hypokalémie, hyperkalcémie

- 0,125-0,25 mg/ den- sympatomimetika – vazbou na β1 rec. myokardiální membrány aktivují adenylátcyklázu → ↑

cAMP a následně i Ca2+, jsou účinné pouze i.v- dopamin - β1-agonista, malé dávky (2-5 ug/kg/den přes dopaminergní rec. →

vazodilataci v koronárním, renálním, mesenterickém a CNS systému; větší dávky (5-10 ug/kg/den, přes β1 → ↑ frekvevce, ↑ tlak, ↑ výdej)

- dobutamin – stimulace β1 a β2 a α1 rec a neovlivňuje dopaminergní rec.- inhibitory fosfodiesterázy – blokují degradaci cAMP v buňkách, ↑ cAMP v bb. hladké svaloviny

cév, vyvolá vazodilataci, !!jediný lék s pozitivně ionotropním účinkem a vazodilatačním současně!!

- amrinon, milrinon- calcium-sensiters - ↑ citlivost myokardu na ca2+ a ↑ jeho vazbu na troponin C

- levosimendan – i.v., krátkodobá léčba akutních stavůAntiagregační léčba – kys. acetylosalicylová je spornáAntikoagulační léčba – warfarin u srdečního selhání a fibrilace síní, anamnézy embolie, intrakardiálním

trombem, rozsáhlém Q-IM přední stěny, výrazné dilatace LK neischemické etioHypolipidemika – rozporné

- akutní srdeční insuf. - kauzální (snížení TK, reperfuzní terapie, chirurgie chlopenních vad, punkce perikardu při tamponádě, léčba arytmie), farmakoterapie (poloha vsedě, sedace, oxygenoterapie, nitroglycerin + furosemid, případně betasympatomimetika (dopamin)

- chronická srdeční insuf. - již od NYHA I - ACE/AT II (zlepšují prognózu), od NYHA II trojkombinace ACEi + diuretikum + betablokátory (případně digoxin)

Chirurgie- srdeční selhání důsledkem korigovatelkné poruchy srdeční struktury- revaskularizace aortokoronárním bypassem, aneuryzmektomie, plastiky mitrální chlopně, náhrada aortální

chlopně, transplantace srdcePřístojová léčba

- srdeční desynchronizační léčba (biventrikulární stimulace) – u NYHA III a IV, EF <35 % a širokým QRS >150 ms- implantace kardioverteru/ defibrilátoru – u rizika náhlé smrt

96.A ONEMOCNĚNÍ ŠTÍTNÉ ŽLÁZY = 67.A

96.B PRINCIPY LÉČBY ARYTMIÍ

Režimová a dietní opatření- u někoho může arytmie vznikout po požití kofeinu/ alkoholu, po fyzické zátěži

Vagové manévry - ↑ parasympatikus → zpomalení vedení a prodlouží refrakteritu AV uzlu; užíváme u tachyarytmií na podkladě

reentry nejčastěji parox. suprav. Tachy- Valsalvův manévr – nádech a usilovný výdech proti zavřené glotis- masáž karotického sinu – u ležícího při monitoraci EKG- ponoření obličeje do studené vody

Farmakoterapie- je nutno stanovit typ arytmie → parametr jehož ovlivnění odstraní arytmii → volba iontového kanálu nebo rec.,

kterým parametr ovlivním → volba antiarytmikaTřída I (blokátory Na+ kanálů) – propafenon – kardioverze a profylaxe fibrilace síní, profylaxe AV reentry

tachykardieTřída II (β blokátory) – zlepšují prognózu

150

Page 151: lf1.czlf1.cz/wp-content/uploads/228-vypracovane_otazky_interna... · Web viewprognoza – 50 % dosáhne kompletního uzdravení do 6 měsíců, 20 % má prodloužený průběh –

Vypracované otázky na INTERNU made by Edita Homolková, Monika Ambroziová, Adam Král, Petr Vermach a Vítek Křehnáč

- metoprolol, atenolol – kontrola frekvence komor u fibrilace síní, profylaxe supraventrikulární tachykardie

Třída III (blokátory K+ kanálů) - prodlužují trvání akčního potenciálu- amiodaron – kardioverze FS, profylaxe FS/ AVRT/ KT

Třída IV (blokátory Ca kanálu) – verapamil – kardioverze a profylaxe SVT, kontrola frekvence komor u FSdigoxin – zpomaluje vedení AV uzlem (užíváme ke snížení chronické tachyfibrilace síní)adenosin – selektivně blokuje vedení v AV uzlu, velmi krátký biolog. poločas 3-5 s

- i.v.bolus → přechodný blok AV uzlu→zástava supraventrikulární tachykardie způsobené reentryKardiostimulace

- u symptomatických bradykardií,, desynchronizační terapie při srdečním selhání, kardiovetr-defibrilátor u tachykardií

- 1. Znak = stimulovaná dutina (A/ V/ D), 2. Znak =snímaná dutina (A/ V/ D/ 0), 3. Znak = reakce kardiostimulátoru (T/ I/ D)- trvalá stimulace – implantace stimulátoru do podkoží a impulzy vedeny do hrotu pravé komory (v případě

dvoudutinového i do ouška síně)- implantabilní kardioverter/defibrilátor – při maligních arytmiích

katetrizační ablace- po přesném zmapovaní arytmického ložiska se provede radiofrekvenční ablace

97.A ASTMA BRONCHIALE = 18.A

97.B LÉČBA DIABETU

- v době dg u DM I bez komplikací, DM II pravděpodobně s komplikacemi metabolického syn. (↑ Tk, ateroskleróza, dislipidémie, obezita)

- kompenzace glób do 4,5% (norma je 4 %)Léčebný plán u DM II – dieta, životní styl, stanovení individuálních cílů, edukace pacienta a rodiny, farmakoterapie

DM a komplikací, ψ-sociální péče- hyperglykémie- obezita- hypertenze – ACEI, sartany (zlepšují citlivost ke glc), blokátory Ca2+ kanálů u těhotných- dyslipidémie – statiny, fibráty- prokoagulační stav – kys. acetylosalicylová

Biguanidy – metformin, 500-3000 mg/den- zpomaluje absorpci glc ve střevě, ↓ glykémii, ↓ BMI- u všech bez komplikací, ↓ nádory, ↓ inzulinorezistenci (játra, sval)KI: renální insuficience (sérový kreatini u mužů >135 μg/l a u žen >110 μg/l), závažné srdeční selhání

(NYHA III-IV)- když meteorismus, nevolnost, průjem → retardovaná forma, podávat během jídla- vysadit 48 h pře operačním výkonem/ podáním jodové kontrastní látky

Sekretagoga – deriváty sulfonylurey - ↓ gly, antiagregační a antioxidační účinkyNÚ: hypoglykémi, ↑ hmotnosti, lékové interakce- glimepirid (30-120 mg/den), gliklazid (1-5 mg/den)- přidávat do kombinace po nasazení metforminu a dosažení jeho střední až

vysoké dávky- glinidy – kratší poločas, fyziologičtější působení

Glitazony - ↑ inzulinovou senzitivitu (aktivují nukleární rec PARP-γ), ↓glykémii, ↓ TGL a VMK, ↑ HDL- v kombinaci s metforminem, deriváty sulfonylurey i s inzulinem- plný efekt za 3-6 měsícůNÚ: otoky, srdeční selhání, fraktury u postmenopauzálních žen, ↑ hmotnosti

Inkretiny – GLP-1 (glukagon like peptide 1), ↑ inzulin a ↓ glukagon- stimuluje sekreci inzulinu jen při hyperglykémii (není riziko hypogly)- kombinační léčba s metforminem a/ nebo deriváty sulfonylurey- u obézních dochází ke snížení hmotnosti

Inzulin – u 70 kg cca 40 UI/den, ½ bazální sekrece a ½ po přijmu potravy

151

Page 152: lf1.czlf1.cz/wp-content/uploads/228-vypracovane_otazky_interna... · Web viewprognoza – 50 % dosáhne kompletního uzdravení do 6 měsíců, 20 % má prodloužený průběh –

Vypracované otázky na INTERNU made by Edita Homolková, Monika Ambroziová, Adam Král, Petr Vermach a Vítek Křehnáč

– u DM II – selhání p.o. antidiabetik, operace, těhotenství, těžké interkurentní stavy (CMP, infekce, sepse), nutnost těsnější kompenzace (diabetická noha, neuropatie)

- konvenční režim – dlouhodobý inzulin večer – ke ↓ hypergly nalačno- premixovaný inzulin – 2x/ den, ovlivnění hypergly nalačno i postprandiální

- intenzifikovaný režim – systém bazál-bolus- u DM I – intenzifikovaný režim – napodobění fyzi sekrece

- sys bazál-bolus, inzulinová pumpa- inzulinová analoga (rychlejší vstřebávání)

CHYBÍ: 4.B DNAVÁ ARTRITIDA 10.B LÉČBA ZÁNĚTLIVÝCH REVMATICKÝCH ONEMOCNĚNÍ; BIOLOGICKÁ LÉČBA 16.B PRIMÁRNÍ A SEKUNDÁRNÍ IMUNODEFICIENCE 20. B INFEKČNÍ ARTRITIDY 29.A CHRONICKÁ OBSTRUKČNÍ PLICNÍ NEMOC 37.B PORUCHY FOSFOKALCIOVÉHO METABOLISMU46 . B ALERGICKÁ ONEMOCNĚNÍ; LÉKOVÉ ALERGIE 48. B MIMOKLOUBNÍ REVMATISMUS 58.A SYSTÉMOVÉ VASKULITIDY 59.A PORUCHY ACIDOBAZICKÉ ROVNOVÁHY64.A OSTEOARTRÓZA A SPONDYLARTRÓZA 67.B ONEMOCNĚNÍ POVRCHOVÉHO ŽILNÍHO SYSTÉMU79.A PORUCHY ACIDOBAZICKÉ ROVNOVÁHY80.A INFEKCE MOČOVÝCH CEST A AKUTNÍ PYELONEFRITIDA85.A OSTEOARTRÓZA A SPONDYLARTRÓZA

152


Recommended